Sei sulla pagina 1di 162

M AY O C L I N I C

INTERNAL MEDICINE
B OARD REVIEW
QUESTIONS AND ANSWER S
Mayo Clinic Scientific Press
Mayo Clinic Atlas of Regional Anesthesia and Ultrasound-Guided Nerve Blockade
Edited by James R. Hebl, MD, and Robert L. Lennon, DO
Mayo Clinic Preventive Medicine and Public Health Board Review
Edited by Prathibha Varkey, MBBS, MPH, MHPE
Mayo Clinic Challenging Images for Pulmonary Board Review
Edited by Edward C. Rosenow III, MD
Mayo Clinic Gastroenterology and Hepatology Board Review, Fourth Edition
Edited by Stephen C. Hauser, MD
Mayo Clinic Infectious Diseases Board Review
Edited by Zelalem Temesgen, MD
Mayo Clinic Antimicrobial Handbook: Quick Guide, Second Edition
Edited by John W. Wilson, MD, and Lynn L. Estes, PharmD
Just Enough Physiology
By James R. Munis, MD, PhD
Mayo Clinic Cardiology: Concise Textbook, Fourth Edition
Edited by Joseph G. Murphy, MD, and Margaret A. Lloyd, MD
Mayo Clinic Internal Medicine Board Review, Tenth Edition
Edited by Robert D. Ficalora, MD
MAYO CLINIC
INTERNAL MEDICINE
B OARD REVIEW
QUESTIONS AND ANSWER S

EDITOR-IN-CHIEF

Robert D. Ficalora , MD
C O N S U LTA N T, D I V I S I O N O F G E N E R A L I N T E R N A L M E D I C I N E

M AYO C L IN I C , R O C HE S T E R , M IN N E S OTA

A S S O C I AT E P R O F E S S O R O F M E D I C I N E

C O L L E G E O F M E D I C IN E , M AYO C L IN I C

EDITOR

Paul S. Mueller, MD

A S S O C I AT E E D I T O R S

Thomas J. Beckman, MD Nicole P. Sandhu, MD, PhD


Margaret Beliveau, MD Amy T. Wang , MD
Mark C. Lee, MD Christopher M. Wittich, MD

MAYO CLINIC SCIENTIFIC PRESS OXFORD UNIVERSITY PRESS


The triple-shield Mayo logo and the words MAYO, MAYO CLINIC, and MAYO CLINIC
SCIENTIFIC PRESS are marks of Mayo Foundation for Medical Education and Research.

3
Oxford University Press is a department of the University of Oxford.
It furthers the Universitys objective of excellence in research, scholarship,
and education by publishing worldwide.

Oxford New York


Auckland Cape Town Dar es Salaam Hong Kong Karachi
Kuala Lumpur Madrid Melbourne Mexico City Nairobi
New Delhi Shanghai Taipei Toronto

With offices in
Argentina Austria Brazil Chile Czech Republic France Greece
Guatemala Hungary Italy Japan Poland Portugal Singapore
South Korea Switzerland Thailand Turkey Ukraine Vietnam

Oxford is a registered trademark of Oxford University Press in the UK and certain other
countries.

Published in the United States of America by


Oxford University Press
198 Madison Avenue, New York, NY 10016

Mayo Foundation for Medical Education and Research 2013

All rights reserved. No part of this publication may be reproduced, stored in a retrieval system, or transmitted, in any form or by
any means, electronic, mechanical, photocopying, recording, or otherwise, without the prior permission of Mayo Foundation for
Medical Education and Research. Inquiries should be addressed to Scientific Publications, Plummer 10, Mayo Clinic, 200 First St SW,
Rochester, MN 55905.

Library of Congress Cataloging-in-Publication Data


Mayo Clinic internal medicine board review : questions and answers / editor-in-chief Robert D. Ficalora ; editor, Paul S. Mueller ;
associate editors, Thomas J. Beckman ... [et al.].10th ed.
p. ; cm.(Mayo Clinic scientific press)
Internal medicine board review : questions and answers
Companion volume to: Mayo Clinic internal medicine board review. 10th ed. c2013.
Includes bibliographical references and index.
ISBN 9780199985876 (alk. paper)ISBN 9780199322039 (alk. paper)ISBN 9780199322046 (alk. paper)
I. Ficalora, Robert D. II. Mayo Clinic. III. Mayo Foundation for Medical Education and Research. IV. Mayo Clinic
internal medicine board review. V. Title: Internal medicine board review : questions and answers. VI. Series: Mayo Clinic
scientific press (Series)
[DNLM: 1. Internal MedicineExamination Questions. WB 18.2]

616.0076dc23 2013001640

Mayo Foundation does not endorse any particular products or services, and the reference to any products or services in this book is
for informational purposes only and should not be taken as an endorsement by the authors or Mayo Foundation. Care has been taken
to confirm the accuracy of the information presented and to describe generally accepted practices. However, the authors, editors, and
publisher are not responsible for errors or omissions or for any consequences from application of the information in this book and
make no warranty, express or implied, with respect to the contents of the publication. This book should not be relied on apart from the
advice of a qualified health care provider.

The authors, editors, and publisher have exerted efforts to ensure that drug selection and dosage set forth in this text are in accordance
with current recommendations and practice at the time of publication. However, in view of ongoing research, changes in government
regulations, and the constant flow of information relating to drug therapy and drug reactions, readers are urged to check the package
insert for each drug for any change in indications and dosage and for added wordings and precautions. This is particularly important
when the recommended agent is a new or infrequently employed drug.

Some drugs and medical devices presented in this publication have US Food and Drug Administration (FDA) clearance for limited
use in restricted research settings. It is the responsibility of the health care providers to ascertain the FDA status of each drug or device
planned for use in their clinical practice.

9 8 7 6 5 4 3 2 1
Printed in China on acid-free paper
Dedicated to all the patients who help us, as internists,
learn, practice, and master internal medicine.
Robert D. Ficalora, MD
This page intentionally left blank
FOREWORD

One of the Department of Medicines strategic goals is to volume make both volumes more portable and user-friendly.
provide premier education in the science and art of medicine. New questions and answers simulate the types of questions
Established goals include leading the nation in the develop- included on the American Board of Internal Medicine exami-
ment of lifelong learning programs and educating physician nation. The editors and associate editors added their depth of
and nonphysician learners at all levels and along all points of experience to ensure that this edition is the finest in the long
the education continuum. These goals are attained by provid- history of this book. The text is not only informational but
ing state-of-the-art graduate medical education. The rapid also of great assistance in preparing for board certification and
pace at which medical knowledge is being discovered neces- recertification, and it allows for the practical application of
sitates frequent updates. Mayo Clinic Internal Medicine Board knowledge to serve our patients.
Review: Questions and Answers reflects changes in the science
of medicine and contains features that facilitate retention of Morie A. Gertz, MD
the knowledge imparted. The chapters in the companion text- Chair, Department of Internal Medicine
book have been completely revised to correspond to American Mayo Clinic, Rochester, Minnesota
Board of Internal Medicine objectives and include evidence- Professor of Medicine
based recommendations. Bulleted points allow easy access to College of Medicine
key points. The questions and answers placed in this companion Mayo Clinic

vii
This page intentionally left blank
PREFACE

Mayo Clinic Internal Medicine Board Review: Questions and without the dedication of the associate editors who labored
Answers is the result of the combined efforts of Mayo Clinic in isolation over the chapter drafts. We are all indebted to
physicians who practice in all the various subspecialties of staffs of the Department of Medicine; Section of Scientific
Internal Medicine. Many have achieved certificates in medical Publications, Joseph G. Murphy, MD, Chair; LeAnn M.
education and thus understand how to communicate informa- Stee and Randall J. Fritz, DVM (editors), Kenna Atherton
tion to our readersphysicians who are in training and prac- (manager), Jane M. Craig (editorial assistant), and Alissa K.
ticing clinicians who are preparing for the American Board of Baumgartner (proofreader); and Section of Illustration and
Internal Medicine (ABIM) certification and maintenance-of- Design, Deb Veerkamp and Ryan Ledebuhr, at Mayo Clinic
certification examinations in internal medicine. for their contributions to this edition. I gratefully acknowl-
Our annual Mayo Clinic Board Review Course, now in its edge the support and cooperation of the publisher, Oxford
27th year, gives the authors and editors the unique opportu- University Press. In particular, I am indebted to my admin-
nity to interact with our readers and tailor our approach to istrative partner, Michael OBrien, whose unfailing support
the way the current generation of learners prepares for a high helped me through some very difficult times.
stakes examination. In response to learner feedback, we have In the spirit of the previous editions, I trust that Mayo
placed the questions and answers in this volume separate from Clinic Internal Medicine Board Review: Questions and Answers
the specialty-area content. More than 300 ABIM-format will serve our readers well in preparation for the primary certi-
multiple-choice questions with a single answer and explana- fication or maintenance-of-certification examination.
tion are keyed to each chapter in the content textbook.
I am grateful to the current and past authors for their Robert D. Ficalora, MD
careful attention and hard work. This book would not exist Editor-in-Chief

ix
This page intentionally left blank
CONTENTS

Contributors xiii

1. Preparing for the ABIM Examination 1


2. Cardiology Questions and Answers 9
3. Gastroenterology and Hepatology Questions and Answers 27
4. Pulmonary Diseases Questions and Answers 35
5. Infectious Diseases Questions and Answers 45
6. Rheumatology Questions and Answers 59
7. Endocrinology Questions and Answers 67
8. Oncology Questions and Answers 79
9. Hematology Questions and Answers 83
10. Nephrology Questions and Answers 93
11. Allergy Questions and Answers 105
12. Psychiatry Questions and Answers 109
13. Neurology Questions and Answers 113
14. Dermatology Questions and Answers 117
15. Cross-Content Area Questions and Answers 123

Index 141

xi
This page intentionally left blank
CONTRIBUTOR S

Charles F. Abboud, MB, ChB Robert D. Ficalora, MD


Consultant, Division of Endocrinology, Diabetes, Consultant, Division of General Internal Medicine
Metabolism, & Nutrition Mayo Clinic, Rochester, Minnesota; and
Mayo Clinic, Rochester, Minnesota; and Associate Professor of Medicine
Associate Professor of Medicine College of Medicine, Mayo Clinic
College of Medicine, Mayo Clinic
William W. Ginsburg, MD
Thomas J. Beckman, MD Consultant, Division of Rheumatology
Consultant, Division of General Internal Medicine Mayo Clinic, Jacksonville, Florida; and
Mayo Clinic, Rochester, Minnesota; and Associate Professor of Medicine
Professor of Medicine and of Medical Education College of Medicine, Mayo Clinic
College of Medicine, Mayo Clinic
Lyell K. Jones Jr, MD
Margaret Beliveau, MD Consultant, Department of Neurology
Consultant, Division of General Internal Medicine Mayo Clinic, Rochester, Minnesota; and
Mayo Clinic, Rochester, Minnesota; and Assistant Professor of Neurology
Assistant Professor of Medicine College of Medicine, Mayo Clinic
College of Medicine, Mayo Clinic
Mary J. Kasten, MD
Elie F. Berbari, MD Consultant, Divisions of General Internal Medicine and
Consultant, Division of Infectious Diseases Infectious Diseases
Mayo Clinic, Rochester, Minnesota; and Mayo Clinic, Rochester, Minnesota; and
Associate Professor of Medicine Assistant Professor of Medicine
College of Medicine, Mayo Clinic College of Medicine, Mayo Clinic
Peter A. Brady, MB, ChB, MD Kyle W. Klarich, MD
Consultant, Division of Cardiovascular Diseases Consultant, Division of Cardiovascular Diseases
Mayo Clinic, Rochester, Minnesota; and Mayo Clinic, Rochester, Minnesota; and
Associate Professor of Medicine Associate Professor of Medicine
College of Medicine, Mayo Clinic College of Medicine, Mayo Clinic
Maria L. Collazo-Clavell, MD Mark C. Lee, MD
Consultant, Division of Endocrinology, Diabetes, Consultant, Division of General Internal Medicine
Metabolism, & Nutrition Mayo Clinic, Rochester, Minnesota; and
Mayo Clinic, Rochester, Minnesota; and Assistant Professor of Medicine
Associate Professor of Medicine College of Medicine, Mayo Clinic
College of Medicine, Mayo Clinic
Scott C. Litin, MD
Lisa A. Drage, MD Consultant, Division of General Internal Medicine
Consultant, Department of Dermatology Mayo Clinic, Rochester, Minnesota; and
Mayo Clinic, Rochester, Minnesota; and Professor of Medicine
Assistant Professor of Dermatology College of Medicine, Mayo Clinic
College of Medicine, Mayo Clinic
Conor G. Loftus, MD
J. Christopher Farmer, MD Consultant, Division of Gastroenterology and Hepatology
Consultant, Division of Pulmonary and Critical Mayo Clinic, Rochester, Minnesota; and
Care Medicine Assistant Professor of Medicine
Mayo Clinic, Rochester, Minnesota; and College of Medicine, Mayo Clinic
Professor of Medicine
College of Medicine, Mayo Clinic

xiii
Fabien Maldonado, MD John J. Poterucha, MD
Consultant, Division of Pulmonary and Critical Care Consultant, Division of Gastroenterology and Hepatology
Medicine Mayo Clinic, Rochester, Minnesota; and
Mayo Clinic, Rochester, Minnesota; and Professor of Medicine
Assistant Professor of Medicine College of Medicine, Mayo Clinic
College of Medicine, Mayo Clinic
Abhiram Prasad, MD
Robert D. McBane, MD Consultant, Division of Cardiovascular Diseases
Consultant, Division of Cardiovascular Diseases Mayo Clinic, Rochester, Minnesota; and
Mayo Clinic, Rochester, Minnesota; and Professor of Medicine
Professor of Medicine College of Medicine, Mayo Clinic
College of Medicine, Mayo Clinic
Rajiv K. Pruthi, MBBS
Kevin G. Moder, MD Consultant, Division of Hematology
Consultant, Division of Rheumatology Mayo Clinic, Rochester, Minnesota; and
Mayo Clinic, Rochester, Minnesota; and Associate Professor of Medicine
Associate Professor of Medicine College of Medicine, Mayo Clinic
College of Medicine, Mayo Clinic
Qi Qian, MD
Timothy J. Moynihan, MD Consultant, Division of Nephrology & Hypertension
Consultant, Division of Medical Oncology Mayo Clinic, Rochester, Minnesota; and
Mayo Clinic, Rochester, Minnesota; and Associate Professor of Medicine and of Physiology
Associate Professor of Oncology College of Medicine, Mayo Clinic
College of Medicine, Mayo Clinic
Nicole P. Sandhu, MD, PhD
Paul S. Mueller, MD Consultant, Division of General Internal Medicine
Chair, Division of General Internal Medicine Mayo Clinic, Rochester, Minnesota; and
Mayo Clinic, Rochester, Minnesota; and Assistant Professor of Medicine
Professor of Biomedical Ethics and of Medicine College of Medicine, Mayo Clinic
College of Medicine, Mayo Clinic
M. Rizwan Sohail, MD
Suzanne M. Norby, MD Consultant, Division of Infectious Diseases
Consultant, Division of Nephrology & Hypertension Mayo Clinic, Rochester, Minnesota; and
Mayo Clinic, Rochester, Minnesota; and Assistant Professor of Medicine
Assistant Professor of Medicine College of Medicine, Mayo Clinic
College of Medicine, Mayo Clinic
Marius N. Stan, MD
Amy S. Oxentenko, MD Consultant, Division of Endocrinology, Diabetes,
Consultant, Division of Gastroenterology and Hepatology Metabolism, & Nutrition
Mayo Clinic, Rochester, Minnesota; and Mayo Clinic, Rochester, Minnesota; and
Associate Professor of Medicine Assistant Professor of Medicine
College of Medicine, Mayo Clinic College of Medicine, Mayo Clinic
John G. Park, MD Bruce Sutor, MD
Consultant, Division of Pulmonary and Critical Care Consultant, Division of Psychiatry and Psychology
Medicine Mayo Clinic, Rochester, Minnesota; and
Mayo Clinic, Rochester, Minnesota; and Assistant Professor of Psychiatry
Assistant Professor of Medicine College of Medicine, Mayo Clinic
College of Medicine, Mayo Clinic
Karen L. Swanson, DO
Naveen L. Pereira, MD Consultant, Division of Pulmonary and Critical Care
Consultant, Division of Cardiovascular Diseases Medicine
Mayo Clinic, Rochester, Minnesota; and Mayo Clinic, Rochester, Minnesota; and
Assistant Professor of Medicine Associate Professor of Medicine
College of Medicine, Mayo Clinic College of Medicine, Mayo Clinic
Axel Pflueger, MD, PhD Seth R. Sweetser, MD
Consultant, Division of Nephrology & Hypertension Consultant, Division of Gastroenterology and Hepatology
Mayo Clinic, Rochester, Minnesota; and Mayo Clinic, Rochester, Minnesota; and
Professor of Medicine Assistant Professor of Medicine
College of Medicine, Mayo Clinic College of Medicine, Mayo Clinic

xiv C O N T R I B U TO R S
Keith M. Swetz, MD Amy T. Wang, MD
Consultant, Division of General Internal Medicine Senior Associate Consultant, Division of General Internal
Mayo Clinic, Rochester, Minnesota; and Medicine
Assistant Professor of Medicine Mayo Clinic, Rochester, Minnesota; and
College of Medicine, Mayo Clinic Assistant Professor of Medicine
College of Medicine, Mayo Clinic
Carrie A. Thompson, MD
Consultant, Division of Hematology Christopher M. Wittich, MD
Mayo Clinic, Rochester, Minnesota; and Consultant, Division of General Internal Medicine
Assistant Professor of Medicine Mayo Clinic, Rochester, Minnesota; and
College of Medicine, Mayo Clinic Assistant Professor of Medicine
College of Medicine, Mayo Clinic
Pritish K. Tosh, MD
Mayo Clinic Scholar in Infectious Diseases Alexandra P. Wolanskyj, MD
Mayo Clinic, Rochester, Minnesota; and Consultant, Division of Hematology
Assistant Professor of Medicine Mayo Clinic, Rochester, Minnesota; and
College of Medicine, Mayo Clinic Associate Professor of Medicine
College of Medicine, Mayo Clinic
Gerald W. Volcheck, MD
Consultant, Division of Allergic Diseases
Mayo Clinic, Rochester, Minnesota; and
Associate Professor of Medicine
College of Medicine, Mayo Clinic

C O N T R I B U TO R S xv
This page intentionally left blank
1.
PREPARING FOR THE ABIM EXAMINATIONa
Robert D. Ficalora, MD

O VE RVI EW E X A M I N AT I O N F O R M AT

Since 2006, more than 7,000 individuals per year have taken Almost all of the questions are clinical and based on cor-
the American Board of Internal Medicine (ABIM) initial rect diagnosis and management. Because there is no penalty
certification examination, and between 3,000 and 5,000 indi- for guessing, candidates should answer every question. Most
viduals per year have taken the Maintenance of Certification questions are based on clinical cases. Among these, 75% are
(MOC) examination. Pass rates have ranged from 79% to related to the outpatient and emergency department settings,
94%. Pass rates for first-time takers on both examinations and the remainder are related to the inpatient setting, includ-
exceed those of repeat takers. There is no doubt that careful ing the critical care unit and nursing home. Increasing empha-
and serious preparation for the examination is valuable and sis is placed on patient safety and evidence-based quality of
necessary. Although some individuals can take and pass the care. Selecting the correct answer to these questions requires
examination with minimal preparation, most takers need integration of information provided from several sources (eg,
rigorous preparation. In recent years, board certification has history, physical examination, laboratory test results, and
assumed greater importance in the minds of patients. In a consultations), prioritization of alternatives, or use of clinical
2003 Gallup poll of 1,001 US adults aged 18 years or older, judgment. Up to one-third of questions are experimental and
98% wanted their physicians to be board-certified, 79% included to test question quality. They are not scored and can-
thought that the recertification process was very important, not be identified during the examination. Patient management
and 54% would choose a new internist if their physicians with a cost-effective, evidence-based approach is stressed. Very
board certification had expired. few questions require simple recall of medical facts. There are
no intentional trick questions.

E X A M I N AT I O N : B A S I C I N F O R M AT I O N The ABIM examination has a uniform question approach


that stresses clinical reasoning over simple recall.
The ABIM website (www.abim.org) has a wealth of informa-
tion for test takers. No one should approach the examina-
tion without reading the ABIM Information and Statistics E X A M I N AT I O N C O N T E N T
(http://www.abim.org/exam/prepare.aspx) and the ABIM
Certification and Recertification Exam Guide (http://www. The questions in the examination cover a broad area of inter-
abim.org/exam/default.aspx). The Exam Day: What to nal medicine. They are divided into primary and cross-content
Expect section, http://www.abim.org/exam/exam-day.aspx, groups (http://www.abim.org/pdf/blueprint/im_cert.pdf ).
has up-to-date information about changes to and navigation Each session (4 for initial certification and 3 for mainte-
of the ABIM approach to computer-based testing, such as the nance of certification) contains 60 multiple-choice questions.
following: The question may include a case history, a brief statement, a
radiograph, a graph, or a photograph (such as a blood smear
How do I answer questions? or Gram stain). Each question has 5 possible answers, and the
How do I change answers? candidates should identify the single-best answer. More than
1 answer may appear correct or partially correct for a question.
How do I make notes? Sample questions are included in the ABIM tutorial, http://
How do I mark questions for review? www.abim.org/exam/prepare.aspx.

Review the ABIM website materials to understand the


testing approach. C O M P U T E R -B A S E D T E S T I N G

a
Candidates currently take a computer-based certification
Reprinted from Ficalora RD, editor. Mayo Clinic internal medicine board review.
10th ed. Rochester (MN): Mayo Clinic Scientific Press and New York (NY): examination that has been designed to provide a flexible,
Oxford University Press; c2013. quiet, and professional environment for examination. The

1
computer-based test is administered by about 200 centers in The self-evaluation modules evaluate performance in clini-
the United States. Candidates schedule their examination cal skills, preventive services, practice performance, fund of
date according to the updated instructions on the ABIM medical knowledge, and feedback from patients and colleagues.
website, http://www.abim.org/exam/. Candidates are well Successfully completed self-evaluation modules are valid for
advised to access the online tutorial at http://www.abim. 10 years. Candidates may apply to begin the MOC process
org/exam/prepare.aspx. This tutorial allows the candidate any time after initial certification. The ABIM recommends
to become familiar with answering questions, changing that completion of the self-evaluation modules be spread out
answers, making notes electronically, accessing the table over the certification period. A candidate should complete
of normal laboratory values, and marking questions for 1 self-assessment module every 1 to 2 years. The ABIM encour-
review. ages candidates to enroll within 4 years of certification in order
to have adequate time to complete the program.
Candidates are advised to familiarize themselves with the
computer-based testing format by accessing the online Candidates who passed the ABIM certification
tutorial. examination in internal medicine in 1990 and thereafter
have a certificate that is valid for 10 years.
The MOC process is called continuous professional
M A I N T E N A N C E O F C E RT I F I C AT I O N development and consists of a 3-step process.

The diplomate certificates issued to candidates who have


passed the ABIM examination in internal medicine since 10 T I P S F O R E F F E C T I VE E X A M I N AT I O N
1990 are valid for 10 years. The total number of candidates P R E PA R AT I O N
who took the ABIM MOC examination for the first time in
2007 was 3,837. Of these, 83% passed. 1. H AV E A S T U DY P L A N
We all have busy lives. Successful candidates stress that the most
E N H A N C E M E N T S TO M O C P R O G R A M valuable preparation strategies must include scheduling a time
to study. Preparing in small, discrete pieces improves recall, facil-
In January 2006, the ABIM enhanced the MOC pro- itates review, and makes the overall task less onerous. Spending
gram to increase flexibility and assess performance in 3 or 4 hours a week, using various approaches such as directed
clinical practice. The 3 retained general components (creden- reading, practice questions, and group review, is enough to stay
tialing, self-evaluation, and secure examination) and the added focused. Simply reading by itself is usually a bad strategy. You
self-evaluation module each have a point value. may not be able to retain much of the ABIM material by read-
Every candidate must complete a total of 100 points in ing without focus. Start with a question, a problem to solve, or
self-evaluation modules. Unlike the previous system, renewal a patient scenario in mind. This approach to a study session will
of more than 1 certificate does not necessitate taking addi- help you and your group understand what you are studying, the
tional self-evaluation modules (ie, the same number of points, clinical context, pathophysiology, and management and the rea-
100, satisfies the requirement to sit for these examinations). sons for it. Keep asking yourself why? and why not?
Candidates must complete at least 20 points in medical
knowledge and at least 20 points in practice performance. How can I study such a large mass of material?
The remaining 60 points may be obtained from comple- Plan a pace of no more than 3 major topics per hour.
tion of modules developed by ABIM or other organizations
that meet the ABIM standards. Thus, one could combine an Survey the material.
ABIM knowledge module (20 points) and an ABIM practice Consider the major subsections as potential questions.
improvement module (20 points) with the American College
of Physicians Medical Knowledge Self-assessment Program Review the material in each subsection carefully to answer
(MKSAP) (3 modules, 60 points), or one could combine the question.
an ABIM practice improvement module (20 points) with Recite in your own words.
6 annual-update ABIM knowledge modules (60 points) and
the ABIM peer and patient feedback module (20 points). All Revise in your notes.
points are valid for 10 years. Further refinements to this pro-
cess are likely. Thus, candidates should check for updates on Take notes! Even if you never look at them again, the act of
the ABIM website. synthesizing the information in writing will help you retain it.

MOC is a multistep process in addition to the To study, use active learning approaches to maximize
examination. efficiency.
Always check the ABIM website for information and Simply reading, no matter how much, is generally an
updates. ineffective preparation strategy.

2 M AYO C L I N I C I N T E R N A L M E D I C I N E B OA R D R E VI EW: Q U E S T I O N S A N D A N S W E R S
2. F O R M A S T U DY G RO U P MOC candidates should use practice questions to guide
your study choices. Do as many questions as you can, and
If possible, form a study group. You will be more likely to
monitor your performance by the ABIM blueprint section,
make and stay on a schedule if individuals feel a responsibility
http://www.abim.org/pdf/blueprint/im_cert.pdf.
to the groups progress. A group will boost everyones morale
Serious preparation for the examination actually starts
and give a common sense of purpose. A group size of only 2
at the beginning of residency training. In addition to daily
to 5 candidates permits study of different textbooks, board
reading and achieving subspecialty-based proficiency, most
review materials, and review articles in journals. Make sure
candidates require a minimum of 6 to 8 months of intense
that you have a committed, available group of study partners.
preparation for the examination. Cramming before the exami-
Individuals who push ahead on their own and those who dont
nation, whether by yourself or at a review course, is unlikely to
keep their commitments can sabotage an otherwise produc-
be successful.
tive group. Schedule regular meetings and assign individuals
Use a standard textbook of internal medicine. Ideally,
specific topics. This approach saves time, covers more top-
you should use one good textbook and not jump from one to
ics per session, and allows everyone to retain more from the
another. Although online, just-in-time resources may be use-
discussion. Take turns acting as group moderator, to keep to
ful for fact checking, they rarely give an inclusive, case-based
the topic and schedule. The moderator should be responsible,
review. The most effective way to use the textbook is with
congratulating productive members and offering a friendly
patient-centered reading. Read the descriptions of the symp-
word to someone who might be slacking off. If everyone has
toms and signs carefully because often they are part of the ques-
a turn, no one person has to be the bad guy. Selected review
tions in the examination. Table 1.1 provides several examples
articles on common and important topics, such as represented
of the common descriptions of symptoms and signs that could
by the ABIM objectives, should be included in every session.
be part of the examination. Rather than reading chapters at
Avoid indiscriminate reading of articles from many journals.
random, read the literature in a structured manner to assist in
Remember that questions are tested for several examina-
future recall of facts. This book and similar books are excellent
tion cycles before they are included in the examination. It is
tools for brushing up on important board-relevant informa-
unlikely that new information or current controversies will be
tion several weeks to months before the examination. They,
represented on your examination. Notes and other materials
however, cannot take the place of comprehensive textbooks
the candidates have gathered during residency training can
of internal medicine. This book is designed as a study guide
be good sources of information. Finding the justification for
rather than a comprehensive textbook of medicine. Therefore,
these pearls can cement ones command of a particular topic.
it should not be used as the sole source of medical information
These clinical pearls gathered from mentors will be of help
for the examination.
in remembering certain important points. Always save some
time each session to review questions and discuss the answers
and their rationales. Dont forget to discuss each of the options Study first with a standard textbook of internal medicine.
in detail. This will develop your thought process and sharpen This book is designed as a study guide and should not be
your test-taking skills. used as the sole source of information for preparation for
the examination.
Keep your study group small and stay focused.
Pay attention to the descriptions of signs and symptoms.
Make a schedule and read ahead of the discussion.
Discuss study material.
4. C R E AT E A N D D EVOT E T I M E
Do multiple-choice questions in groups.
Board preparation should be part of your daily routine, like
Indiscriminate reading of articles from many journals
exercising, showering, or brushing your teeth. If you dont
should be avoided.
regularly do some preparation, it will fall off your routine, and
Information in recent journals is unlikely to be included in your preparation just wont happen. You can spend as little or
the examination. as much time as you want on a particular activity. Often you
can review familiar topics in small discrete time periods (eg,
before or after lunch). Less familiar topics may require an hour
in a quiet room, and this time may be best reserved for early
3. D ET E R M I N E WH AT YO U N E E D TO S T U DY
in the morning or on a weekend. Keep in mind that the more
For recent graduates attempting primary certification, let time and energy you spend actively learning a topic, the bet-
your in-training examination subsection score results guide ter your command, and the less dependent you will be on rote
your study choices. In general, if your score in a given area memorization.
was below the fifth decile, or fiftieth percentile, you should Some people can study effectively while on a treadmill,
consider that an area for intensive review and preparation. on a train, or in a car. If you can do this, you can incorpo-
Use the section called educational objectives, which gives rate this into your studying routine. How much time it takes
your performance by content area, to guide your choice of to perform these tasks varies from person to person and will
preparation topics. improve as you solidify your study habits. Regardless, every

1. P R E PA R I N G F O R T H E A B I M E X A M I N AT I O N 3
Table 1.1. COMMON DESCRIPTIONS OF SIGNS AND SYMPTOMS IN EXAMINATION QUESTIONS
HISTORY (SYMPTOMS) PHYSICAL FINDINGS (SIGNS) LIKELY DIAGNOSIS

Cardiology
Shortness of breath or asymptomatic Late peaking systolic murmur, intensity decreases with Hypertrophic obstructive cardiomyopathy
handgrip & increases with squatting Coarctation of aorta
Asymptomatic, headache Hypertension, diminished or absent lower extremity
pulses, systolic murmur, bruit over chest wall

Neurology
Gait impairment, falls, dysphagia, dysarthria Inability to look up & side to side Progressive supranuclear palsy
Diplopia, oscillating images, reading fatigue, loss of depth perception Impaired adduction on lateral gaze, with nystagmus in Internuclear ophthalmoplegia (consider multiple sclerosis,
the contralateral abducting eye cerebrovascular disease)
Fluctuating memory, confusion, visual hallucinations Mild parkinsonism, dementia Lewy body dementia
Inappropriate behavior, dementia, poor social skills Dementia Frontotemporal dementia

Paroxysmal pain affecting the side of the face Usually normal Trigeminal neuralgia affecting 1 of the branches of cranial nerve V
Muscle stiffness, clumsiness, occasional emotional lability Brisk reflexes, spasticity (upper motor neuron signs), Amyotrophic lateral sclerosis
atrophy, fasciculation (lower motor neuron signs)
Altered mental status, fever, headache Flaccid paralysis, neck rigidity , altered mental status West Nile virus encephalitis

Infectious disease
Recurrent sinusitis, skin, or pulmonary infections due to Sinus tenderness, abnormal lung sounds Chronic granulomatous disorder
Staphylococcus aureus
Recurrent Neisseria infections Neck rigidity , altered mental status Inherited deficiencies of complement (C5, 6, 7, 8, 9), factor D, or
properdin
Recurrent episodes of bacterial pneumonia, sinusitis, diarrhea due to Malnourished, abnormal lung sounds Common variable immunodeficiency
Streptococcus pneumoniae

Gastroenterology
Cirrhosis of liver, ingestion of raw oysters Fever, hypotension, hemorrhagic bullae, signs of Vibrio vulnificus
cirrhosis of liver
Diarrhea Pruritus, grouped vesicles over the elbow, knee, scalp, Dermatitis herpetiformis due to celiac sprue
or back of neck
Hepatitis C, photosensitivity Skin fragility, erosions, blisters on dorsum of hand, Porphyria cutanea tarda
hyperpigmentation
Dermatology
Facial rash, photosensitivity Papules & pustules on bridge of nose & face, Rosacea
telangiectasia
Rash Sharply demarcated erythematous papules, silvery Psoriasis
white scales over scalp, extensor surfaces of
extremities, & nails
Cough with sore throat Tender, erythematous pretibial nodules Erythema nodosum
Ulcerative colitis Irregular, undermined ulcer with violaceous border or Pyoderma gangrenosum
scarring in lower extremities
Flushing, diarrhea, rapid heart rate Brown-red macules, urticaria on stroking skin Systemic mastocytosis
learning task takes time, and you must budget for that time. range from 5 to 10 with some estimated to have 0 to 2. Pace
Only you can decide how much time you want to spend in your preparation by subject:
solitary study, in groups, or in summary objective review. You
can make great plans, but life and work arent predictable, Subjects with a large estimated number of questions are
so you should build in some catch-up time for unexpected very likely to be there. Master them.
distractions.
You also have to consider how much time it takes to Subjects with a medium to low estimated number of
organize your studying. Review of cardiology may go questions will be there in some form. Review them.
quickly, whereas glomerulopathies may take a pad and pen- Subjects with a very low estimated number of questions
cil to figure out. You may have to travel to study sessions may be tangential or favorite board zebras. Depending on
or spend time looking for information to ensure your com- your available study time, it may be worthwhile for you to
mand of a given objective. Many candidates try to set aside consider them only for last-minute review.
large blocks of time. With our busy lives, that may be laud-
able but impractical. Many shorter sessions not only allow Plot out your objectives review on a calendar, mixing more
for study and catch-up but also can be worked in around and less complex objectives. Leave time for discussion, lit-
standing commitments more effectively than large blocks, erature and online searches, and follow-up for problem items
and thus a missed session wont be a major setback for that from previous study sessions. Always plan to cover new mate-
week. The time you spend will come back to you when you rial and to periodically cycle back to previously reviewed diffi-
pass, and a failure only means you have to devote the time cult or detailed information. Imagine cases that might go with
all over again. the material at hand.
Certifying or recertifying board examinations can be stress- The most effective way to manage your study time is to
ful. The sheer mass of information can be overwhelming to periodically assess your progress through practice tests and
some. The press of occupational and personal responsibilities test questions. It is impossible to overemphasize the impor-
makes finding the time to study very difficult, so many opt to tance of this point. Boardsmanship is a real skill, and there
take a review course. A review course should be the final inte- is no substitute for familiarity with the form and content of
grating activity once you have completed your own primary the ABIM-type questions. Therefore, you are strongly encour-
preparation. To be successful you should go to the review aged to take at least a few mock examinations and simulate
course prepared and ready to fill in objectives you may have the actual testing environment (ie, no breaks, snacks, music,
missed or to learn from experts objectives that you couldnt phones, pagers). Taking mock questions is an effective use of
understand on your own. Dont expect a course to substitute a board review or board questions book. Practice material at
for primary preparation. Attending a highly focused, no-fluff intervals during your long-term test preparation schedule.
course that delivers the information in a concentrated, Your schedule should be the most intense in the 3 months
high-yield manner right before the examination may seem like before the test. After that, focus on review, consolidating key
an easy way out, but it is unlikely to be the difference between points, and resolving previously difficult problems.
success and failure. Once you or your group has a schedule, stick to it. Add ses-
sions, but never delete any. Stick to your start and stop times.
Residents: Prepare for the boards during residency.
You will not find that kind of quality time after your Schedule your progress and build in assessment sessions.
residency. Make changes, but no deletions.
Once residency is over and you start fellowship or a job, Plan to review material that you have chosen several times
you will not find time to study. (minimum twice).
MOC: Schedule the time. Stay focused throughout the months before the
Schedule multiple short preparation sessions rather than examination.
fewer long ones. Board review preparation must be at its peak by 3 months
Do not rely solely on a review course; they are not a before the examination. If you have not yet formed a study
substitute for primary preparation. group, now is your last chance.

6. A NSWE R Q U E S T I O NS S I M I L A R TO
5. P R E PA R E A P L A N A N D S C H E D U L E I T E X A M I NAT I O N C O N T E N T
All ABIM objectives are not equal. Review the relative per- The purpose of standardized testing is to measure a candi-
centage of the contents of the examination and the number dates command of the material so that scores from different
of questions per objective (http://www.abim.org/pdf/blue- test dates can be reliably compared with one another. The
print/im_cert.pdf ). Note that some areas may have an esti- results must correlate statistically with the results of all the
mated number of questions of 11 to 15, whereas many will test-takers who have answered the same questions. Persons

1. P R E PA R I N G F O R T H E A B I M E X A M I N AT I O N 5
who construct board review materials and questions go to Familiarize yourself with the teaching principle and the
great pains to build them for the same content and content testing objective, which may give you insight into the
level as on the actual board examination. The good ones are questions and the possible responses.
validated and have been tested to make sure that they perform
in a reliable, predictable manner and that they adequately test
the content they purport to test. Any questions prepared for
7. D O N T F O RG ET A B O U T I M AG E S
other courses, local residency, fellowship rotations, or other
venues that cover similar material likely wont test the mate- Every image-based question will also include text or a clinical
rial in a way that predicts your performance on the board case or both. Dont simply focus on the image without reading
examination. Read a board review book, go to a board review the text. Familiarize yourself with the image and its details after
course, and always practice answering questions. When you you have read the case, and then read it again. Photographs
practice answering questions, do it as you would during the of skin disorders, radiographs, electrocardiograms, and other
examination; just reading the book and reading the answers images given in board questions are generally easy ways to
likely will not prepare you for the actual test. For every ques- score points. Reading the ABIM question stem last helps
tion, identify in your own mind the concept being tested. put the pieces of the puzzle in place. Methodically review a
Make sure you read all the wrong answers and make sure you radiograph as you would in a patient encounter. Immediately
understand why they werent the best answer for the question focusing on an obvious abnormality can distract you from a
asked. more subtle finding that may alert you to the correct answer.
Sometimes candidates try to prepare by studying mate- You may miss the pneumothorax as the cause of the dyspnea if
rials that are harder than the real test, such as subspecialty you focus only on the heart size and the small pleural effusion.
boardlevel review courses and practice tests. The idea is that Likewise for skin findings; use your clinical skills to interpret
becoming familiar with something harder will make the real the finding. Is it flat or raised, erythematous or pigmented?
thing easier by comparison. Preparing with something more
challenging can be a good idea in some types of athletic or Approach an image question as you would a patient.
endurance preparation, but it is a bad idea for the ABIM
Methodically examine the image.
examination. Because the objectives are specific and pub-
lic, preparing by using objectives for another examination Use the text and the stem to focus your inquiry.
may cause you to misinterpret or overinterpret what is being
asked. Reliably finding the easiest approach to a test question
requires being aware that the test cant require you to use a cer-
8. S P E N D S O M E T I M E B EC O M I N G E X P E RT I N
tain higher-level data set or decision tree. Because the harder
B OA R D S M A NS H I P
material is testing a different skill set, reviewing this may lead
to incorrect answers. Some candidates fail the examination despite intense prepara-
As silly as this piece of advice may seem, read the ques- tion and the clinical competence necessary to pass the exami-
tions carefully! Doing so can make a big difference in your nation. Failing usually happens because they dont understand
score. If you read questions hastily, there is a high likelihood or interpret the questions properly. The ability to understand
that you will misinterpret them. Some questions offer incor- the nuances of the question format is sometimes referred to as
rect choices that are designed to answer a common misinter- boardsmanship. Intelligent interpretation of the questions
pretation of the actual question. Be particularly careful with is very important for candidates who are not well versed in
answers that have more than one part. Only one part may be the format of multiple-choice questions. Answer the ques-
correct. Other distraction techniques include 2 responses that tions whose answers you know first, making sure you under-
are similar except for a word or phrase. Watch for responses stand what is being asked to ensure that they are answered
that contradict others; usually, both of these can be ruled out. correctly. It is easy to become overconfident with such ques-
What if you read a question and the traditional correct answer tions, and thus you may fail to read the questions or the
isnt an option? What if more than one answer could be cor- answer options carefully. Make sure you never make mistakes
rect? Then select the best option available. Be very careful on easy questions. Read the final sentence (that appears just
of responses that are the longest or the unique answer. They before the multiple answers) several times to understand how
are no more likely or unlikely to be correct despite prevailing an answer should be selected. Recheck the question format
wisdom. before selecting the correct answer. Read each answer option
completely.
Dont try to read the board review material from cover to Occasionally, a response may be only partially correct. At
cover. times, the traditionally correct answer is not listed. In these
The best way to prepare is to review and always practice situations, select the best alternative listed. Watch for qualifiers
answering questions. such as next, immediately, or initially. Avoid answers that con-
tain absolute or very restrictive words such as always, never, or
To improve your understanding, read the explanation, must. Answer options that contain absolutes are likely incor-
and look up additional information related to each of the rect. Try to think of the correct answer to the question before
choicesboth correct and incorrect. looking at the list of potential answers. Assume you have

6 M AYO C L I N I C I N T E R N A L M E D I C I N E B OA R D R E VI EW: Q U E S T I O N S A N D A N S W E R S
been given all the necessary information to answer the ques- For young patients, aim for aggressive management.
tion. If the answer you had formulated is not among the list
of answers provided, you may have interpreted the question For elderly patients, aim for less aggressive alternatives,
incorrectly. When a patients case is presented, think of the especially in those with multisystem disease.
diagnosis before looking at the list of answers. If you do not Beware of adverse medication effects and polypharmacy.
know the answer to a question, very often you are able to rule
out one or several answer options. Determine whether your For asymptomatic healthy patients, do nothing and
diagnosis is supported by any of the answers. If you can elimi- observe.
nate any answers as clearly wrong, you will improve your odds
at guessing. Occasionally, you can use information presented Use your existing fund of knowledge of internal medicine
in one question to help you answer other, difficult questions. and your previous clinical experience.
Many questions are on the test for trial or validation purposes
and are not scored. If a question seems to you to be a bad or Approach each question as a real-life patient encounter.
confusing question, it may be in this category. It is best not to There are no trick questions.
spend an inordinate amount of time trying to second guess
this type of question. Come back to it after you have finished,
if you still have time. 10. P L A N F O R T H E DAY O F T H E E X A M I NAT I O N

When reading long multiple-choice cases: You should have adequate time to read and answer all the ques-
tions; therefore, there is no need to rush or become anxious.
First read the actual lead line of the question Watch the time to ensure that you are at least halfway through
Once you understand what the question is asking: the examination when half of the time has elapsed. Start by
answering the first question and continue sequentially. Almost
Stay focused and look for clues in the long stem of the all of the questions follow a case-presentation format. At times,
question. subsequent questions will give you information that may help
As you read through the questions: you answer a previous question. Do not be alarmed by lengthy
questions; look for the questions salient points. When faced
Note the key facts and abnormal findings with a confusing question, do not become distracted by that
Skip questions about which you have no idea, and come question. Mark it so you can find it later, then go to the next
back after a complete first pass question and come back to the unanswered ones at the end.
Extremely lengthy stem statements or case presentations are
intended to test the candidates ability to separate the essen-
tial from the unnecessary or unimportant information. You
9. US E YO U R R E F L E X E S may want to highlight important information presented in
Associations, causes, complications, and other relationships the question in order to review this information after reading
between a phenomenon or disease and clinical features are the entire question and the answer options. There is no pen-
important to remember and recognize. Each subspecialty has alty for guessing, so you should never leave an answer blank.
many common connections, and candidates for the ABIM Every time you can eliminate just one choice you increase your
and other examinations may want to prepare lists like this chance of choosing a correct answer by 20%, so its best to
for different areas. For example, a case that presents a patient guess among the remaining choices. If you truly have no idea
with health careassociated pneumonia should immediately about any of the choices, the B answer has been statistically
bring to mind antipseudomonal antibiotics, not antibiot- more likely to be correct. It is better to choose B if you truly
ics traditionally used for community-acquired pneumonia. dont know the answer.
Combined knee and hip pain should have you consider-
ing a gait abnormality rather than abnormality in 2 joints Look for the salient points in each question.
simultaneously. If a question is confusing, mark it to find it and come back
Use the basic fund of knowledge accumulated from to the unanswered questions at the end.
clinical experience and reading to solve the questions.
Approaching the questions as real-life encounters with If you must guess, choose B; statistically, it is more likely
patients is far better than trying to second-guess the exam- to be correct.
iners or trying to analyze whether the question is tricky. As
indicated above, the questions are never tricky, and there is Its really not productive to discuss the questions or answers
no reason for the ABIM to trick the candidates into choos- after the examination with other candidates. Such discussions
ing wrong answers. usually cause more consternation, although some candidates
Use examination techniques to your advantage. Look for may derive a false sense of having performed well on the exam-
target populations in questions. Start with a basic premise in ination. In any case, the candidates are bound by their oath to
mind, then modify it as the information warrants. Examples the ABIM not to discuss or disseminate the questions. Do not
are as follows: study between examination sessions. To minimize stress, stick

1. P R E PA R I N G F O R T H E A B I M E X A M I N AT I O N 7
to your daily routine; dont start or stop exercising or using S U M M A RY
caffeine, and dont skip meals or load up on carbohydrates. Be
as rested and refreshed as you can be. Forget about your elec- Preparation for the ABIM examination requires a serious and
tronic devices such as pagers and cell phones. organized approach. Devote adequate time. Familiarize your-
self with the examination format and objectives. Use com-
Dont study the day before the examination or between the monsense test-taking strategies, including practice tests and
examination sessions. question analysis. Treat the examination day as you would for
any competitive event by preparing physically.
Discussing the examination questions with others raises
anxiety and can adversely affect your performance in the
next session.
Maintain your normal routine.

8 M AYO C L I N I C I N T E R N A L M E D I C I N E B OA R D R E VI EW: Q U E S T I O N S A N D A N S W E R S
2.
CARDIOLOGY QUESTIONS AND ANSWER S

QUESTIONS 2. A 78-year-old woman comes to your office with irregu-


lar palpitations, shortness of breath, and fatigue. She
Multiple Choice (choose the best answer) has type 2 diabetes mellitus and hypertension, both of
which she has kept under excellent control since she had
a minor stroke at age 69. She has no other known medi-
A R R H Y T H M I A S A N D C L I N I C A L SY N D RO M E S
cal problems of note. Examination findings are normal
except that she has a soft, barely audible right carotid
1. You are asked to see a 22-year-old man who under-
bruit. The electrocardiogram (ECG) from a week ago is
went a routine electrocardiographic (ECG) examina-
shown (Figure 2.Q2). A transthoracic echocardiogram
tion as part of a preemployment physical examination
shows normal left ventricular function and moderate
for a large manufacturing company. Other than having
left atrial enlargement. You order another ECG today
a fairly sedentary lifestyle, he is healthy and reports
since her regular heart rate confirms normal sinus
no symptoms. Cardiac examination findings are nor-
rhythm. What anticoagulation therapy should you rec-
mal. You view the 12-lead ECG shown in Figure 2.Q1.
ommend for this patient at this time?
Which of the following would be the best management
a. Warfarin (goal international normalized ratio, 2.03.0)
strategy for this patient at this time?
b. Aspirin 325 mg
a. Perform an electrophysiology study and catheter ablation.
c. Catheter ablation of this arrhythmia to eliminate stroke
b. Observe.
risk
c. Start therapy with metoprolol 25 mg twice daily.
d. Aspirin 325 mg and clopidogrel 75 mg
d. Start therapy with procainamide 150 mg twice daily.
e. No anticoagulation since she is now in normal sinus rhythm
e. Start therapy with digoxin 0.125 mg daily.
and at low risk

I aVR V1 V4

II aVL V2 V5

III aVF V3 V6

II

Figure 2.Q1

9
I aVR V1 V4

II aVL V2 V5

III aVF V3 V6

II

V1

V5

Figure 2.Q2

3. An 85-year-old man has had 2 separate syncopal epi- On examination, the patient was fully orientated
sodes that occurred suddenly and without warning. On and appeared younger than his age. His heart rate and
the first occasion, he was talking to his daughter on the blood pressure were satisfactory, but his systolic pres-
phonethen he was lying on the floor. Recovery was sure decreased by 10 mm Hg on standing. Bilateral
immediate, but he injured his left hand. On the second carotid bruits and a harsh 3/6 ejection murmur were
occasion, he was in the kitchen making a sandwich. His present. The murmur was loudest at the left sternal
daughter was present and was able to break his fall. She border, with some radiation into the neck. Reversed
noted that both arms made jerking movements, which splitting of the second heart sound (separation during
resolved when he was placed into a recovery position expiration rather than inspiration) was also present, but
on his left side. On both occasions, a full recovery no diastolic murmur was heard. There was no evidence
occurred within a few minutes without sequelae. of jugular venous distention, and his lungs were clear,

I aVR V1 V4

II aVL V2 V5

III aVF V3 V6

V1

II

V5

Figure 2.Q3

10 M AYO C L I N I C I N T E R N A L M E D I C I N E B OA R D R E VI EW: Q U E S T I O N S A N D A N S W E R S
but a small amount of peripheral edema was present. a. Add sotalol.
Gait was normal and there were no neurologic signs of b. Add dronedarone.
note. The electrocardiogram is shown in Figure 2.Q3. c. Refer for catheter ablation.
This patients presentation is most consistent with d. Continue the current regimen.
which of the following?
a. Cardiogenic syncope C A R D I AC P H Y S I C A L E X A M I NAT I O N
b. A possible seizure disorder
c. Stroke or transient ischemic attack 6. A 30-year-old woman is referred for a pre-insurance
d. Neurocardiogenic syncope examination. She has no complaints. She is active
e. Orthostatic response and runs with her large dog about 1 hour daily (45
4. A 55-year-old woman with nonischemic dilated car- miles). She has no exercise limitations and was hiking
diomyopathy and exertional dyspnea (New York Heart at an elevation of 3,030 m in the Rockies last summer.
Association [NYHA] functional class III) and 2-pillow On examination, her pulse rate is 50 beats per minute,
orthopnea underwent placement of an implantable blood pressure is 120/80 mm Hg, and she is thin. On
cardioverter-defibrillator 4 years ago for sudden cardiac cardiac examination, there is a normal first heart sound
arrest prophylaxis. Now she returns and asks whether and normal splitting; an early low-pitched diastolic
cardiac resynchronization therapy may be appropri- sound is consistent with a third heart sound (S3). There
ate. Her current medications are lisinopril 5 mg daily, is a grade 1 to 2 midsystolic murmur at the left upper
carvedilol 6.25 mg twice daily, spironolactone 25 mg sternal border. The murmur disappears with the Valsalva
daily, and furosemide 40 mg daily. maneuver. Carotid upstrokes are normal, and the jugu-
Physical examination and diagnostic study findings lar venous pressure is normal. No systolic clicks can be
were as follows: heart rate 97 beats per minute, blood appreciated at rest or with the squat-to-stand maneuver.
pressure 142/70 mm Hg, jugular venous pressure 12 cm The electrocardiogram shows sinus bradycardia (heart
water, lungs clear, lateral displacement of the cardiac rate 48 beats per minute) but is otherwise normal. What
impulse, audible third heart sound, and no murmurs; further cardiac testing needs to be done?
a. Cardiac catheterization to measure a gradient across the
electrocardiography showed left bundle branch block
mitral valve
(LBBB) with QRS duration of 130 ms, and transtho-
b. Stress echocardiography
racic echocardiography showed left ventricular ejection
c. Treadmill ECG
fraction (LVEF) of 20%. What is the most appropriate
d. No additional testing
management for this patient at this point?
e. Holter monitoring
a. There is no indication for cardiac resynchronization therapy
(CRT)the symptoms are not severe enough. 7. Which of the following statements about shunts in the
b. There is no indication for CRTthe QRS complex is not atrial septum is true?
wide enough. a. The auscultatory findings associated with atrial septal defect
c. Upgrade to CRT now. (ASD) are paradoxical splitting of the second heart sound
d. Up-titrate the dosages of lisinopril and carvedilol. (S2) and a diastolic flow rumble heard midsternum.
b. Patent foramen ovale (PFO) is uncommon in healthy adults
5. A 32-year-old woman who received a diagnosis of peri-
and is found in approximately 2.5% of adults.
partum cardiomyopathy (New York Heart Association
c. A fixed splitting of the S2 and a right ventricular (RV) lift are
class II) after the birth of her second child presents
found in ASD with significant shunting.
with symptomatic paroxysmal atrial fibrillation (AF).
d. Secundum ASD and PFO require subacute bacterial endo-
Typically, she experiences approximately 4 episodes of
carditis prophylaxis.
AF per year, each lasting 2 to 4 hours. During episodes,
she notes palpitations, shortness of breath, and fatigue, 8. A 30-year-old woman comes to your office for evalua-
with heart rates around 90 beats per minute, but she tion. She appears healthy and is not limited by physi-
is able to continue working in her home. Although cal activity. She says that she has occasional chest pains
she has generally done well from a heart failure (HF) that occur at rest, often with a change in position. Her
standpoint, she was hospitalized 3 weeks ago with blood pressure is 110/70 mm Hg, and her heart rate is
acute decompensated HF. Current medications include 76 beats per minute and regular. Auscultatory findings
carvedilol 25 mg twice daily, lisinopril 20 mg daily, are a midsystolic click followed by a 2/6 apical systolic
furosemide 40 mg daily, and digoxin 0.125 mg daily. murmur that radiates to her axilla. Which of the fol-
Physical examination and diagnostic study findings lowing dynamic physical examination findings would
were as follows: heart rate 60 beats per minute (regular confirm your suspicion of mitral valve prolapse?
rhythm), blood pressure 100/70 mm Hg, jugular pres- a. There is no change in the click with squatting or standing.
sure not elevated, and lungs clear; electrocardiography b. A passive leg raise makes the click-murmur occur earlier.
showed normal sinus rhythm and left bundle branch c. A squat maneuver makes the click-murmur occur later.
block. Which of the following would you recommend d. The mitral regurgitant murmur does not change with posi-
at this time? tional maneuvers.

2 . C A R D I O L O GY Q U E S T I O N S A N D A N S W E R S 11
9. A 25-year-old man, who is slender and athletic, comes C O N G E S T I VE H E A RT FA I LU R E
to your office because of an episode of syncope imme-
diately after playing basketball. He exercises diligently 13. A 72-year-old man presents with progressively increas-
but says that he has experienced mild dyspnea on exer- ing dyspnea and edema of the lower extremities. He
tion over the past 2 months. On physical examination, denies having chest discomfort or palpitations. He
his blood pressure is 175/90 mm Hg, his carotid pulse experiences dyspnea on minimal exertion, orthopnea,
is bounding, and there is an early systolic click imme- and occasional episodes of paroxysmal nocturnal dysp-
diately followed by a late peaking 3/6 systolic murmur nea. He had a myocardial infarction 10 years ago and
heard best at the right second intercostal space and underwent primary percutaneous coronary interven-
radiating to the carotids. There is no dynamic increase tion then. His residual left ventricular ejection fraction
in murmur intensity with the Valsalva maneuver and (LVEF) after that event was 30% by echocardiography.
no change in the timing of the click with squatting. He has adhered to his medical regimen. He has been
The apical impulse is sustained. A radial femoral delay hospitalized for decompensated heart failure repeat-
is suspected on examination. What should be the next edly over the past 3 years. He received an implantable
step in the evaluation or management of this patient? cardioverter-defibrillator with cardiac resynchroniza-
a. Coronary angiogram tion therapy 2 years before the current presentation.
b. Use of a 24-hour Holter monitor He has a history of hypertension and hyperlipidemia.
c. Computed tomographic (CT) scan of the chest His medical regimen consists of enalapril 10 mg twice
d. Reassurance with recommendations for warm-up and daily, carvedilol 6.25 mg twice daily, simvastatin 20 mg
cooldown routines with exertion daily, aspirin 81 mg daily, spironolactone 25 mg daily,
e. Use of a 24-hour blood pressure monitor and furosemide 80 mg daily.
Examination findings were as follows: afebrile, heart
10. A 65-year-old patient with a history of coronary artery rate 62 beats per minute, blood pressure 90/50 mm Hg,
bypass graft surgery 1 year ago presents with fatigue, significantly elevated jugular venous pressures with
dyspnea, and progressive lower extremity edema. prominent v waves, lungs clear, a 3/6 pansystolic murmur
Examination reveals a blood pressure of 120/70 mm at the apex, a third heart sound (S3), and pitting edema
Hg and a pulse of 77 beats per minute. The lungs are (3+). Laboratory test results were as follows: hemoglo-
clear. The heart is quiet, with normal first and second bin 10 g/dL, sodium 126 mEq/L, potassium 4.5 mEq/L,
heart sounds and no murmurs. The jugular venous pres- serum urea nitrogen 42 mg/dL, and creatinine 1.6 mg/
sure ( JVP) is elevated at midneck (approximately 20 cm dL. The electrocardiogram showed a paced rhythm. A
water). There is an increase in the JVP with inspiration recent exercise stress test done with sestamibi imaging
with a rapid descent. What does this combination of showed a large fixed defect in the anteroseptal wall with
findings strongly suggest? no evidence of reversible ischemia and an LVEF of 20%.
a. Heart failure due to biventricular dysfunction The patient was admitted to the hospital for intravenous
b. Superior vena cava syndrome diuretic therapy. The edema improved significantly, but
c. Constrictive pericarditis he continued to experience dyspnea on minimal exertion.
d. Failure of the bypass grafts and recurrent coronary artery His creatinine increased to 2.0 mg/dL. What should be
disease the next step in this patients management?
11. A patient comes to you for evaluation of peripheral a. Increase the carvedilol dosage to 12.5 mg twice daily.
edema and palpitations. On examination, a loud grade b. Optimize cardiac resynchronization therapy.
3/6 holosystolic murmur is heard at the left lower ster- c. Add metolazone 5 mg daily.
nal border, and an intermittent third heart sound (S3) d. Perform a positron emission tomographic viability scan.
is heard that seems to be present only with inspiration. e. Refer for implantation of a left ventricular assist device
A large v wave is noted in the jugular venous pressure (LVAD).
( JVP). What is your diagnosis?
a. Superior vena cava syndrome 14. Which statement is false regarding serum brain natri-
b. Atrial fibrillation uretic peptide (BNP) levels?
c. Constrictive pericarditis a. Serum BNP levels may be in the reference range in patients
d. Significant pulmonary stenosis with advanced heart failure.
e. Significant tricuspid regurgitation b. A persistently elevated serum BNP level despite optimal
medical therapy is a marker of poor prognosis in heart
12. Which of the following is true about the ausculta- failure.
tory finding of a fourth heart sound (S4) on physical c. Optimization of medical therapy based on serum BNP lev-
examination? els decreases hospitalizations in heart failure.
a. Often associated with hypertension d. Serum N-terminal pro-BNP has a longer half-life than
b. Normal in young children serum BNP.
c. Normal in athletes e. The serum BNP level is frequently elevated in idiopathic
d. A hallmark of the onset of atrial fibrillation constrictive pericarditis.

12 M AYO C L I N I C I N T E R N A L M E D I C I N E B OA R D R E VI EW: Q U E S T I O N S A N D A N S W E R S
15. A 63-year-old woman presents with dyspnea on exer- hypokinesis, ejection fraction 11%, and severe mitral
tion and tingling numbness in both lower extremities. regurgitation. Coronary angiography is normal. Which
On physical examination, she is afebrile, her heart rate of the following treatment options would not prolong
is 92 beats per minute, and her blood pressure is 110/70 this patients survival?
mm Hg in a sitting position and 90/60 mm Hg while a. -Blockers
standing. She has jugular venous distention accentu- b. Biventricular pacing
ated with inspiration and prominent x and y descents. d. Isosorbide dinitrate in combination with hydralazine
Bibasilar crackles are present posteriorly. A fourth heart e. Spironolactone
sound is present. She has mild bilateral pitting edema e. Digoxin
of the lower extremities. Sensation is diminished in
18. A 28-year-old man presents with syncope. He has a
both feet, but reflexes and motor power are preserved.
family history of sudden cardiac death. On physical
Laboratory test results were as follows: hemoglobin 10
examination, his blood pressure is 120/70 mm Hg and
g/dL, sodium 135 mEq/L, potassium 4.5 mEq/L, serum
his heart rate is 68 beats per minute. Jugular venous
urea nitrogen 16 mg/dL, and creatinine 1.0 mg/dL. The
pressures show a prominent a wave. Carotid upstroke
urinanalysis is remarkable for mild proteinuria. The
is rapid; left ventricular impulse is sustained and dis-
electrocardiogram shows sinus rhythm, low voltage,
placed laterally. He has a prominent fourth heart sound
and an old anteroseptal infarct pattern. An echocardio-
and a 3/6 systolic ejection murmur at the left sternal
gram shows a left ventricular ejection fraction of 60%,
border radiating to the right second intercostal space
severely thickened left ventricular walls with a speckled
and apex. He has a 2/6 pansystolic murmur at the apex.
appearance, and mild mitral regurgitation. What is the
The murmur at the left sternal border is accentuated
best treatment option for this patient?
with the squat-to-stand maneuver. His lungs are clear,
a. Stem cell transplant
he has no hepatomegaly, and his extremities are not
b. Angiotensin-converting enzyme inhibitors
edematous. Which of the following statements regard-
c. Combined heart and liver transplant
ing this patients disease state is false?
d. Diuretic therapy
a. A majority of patients are asymptomatic.
e. Hospice care
b. The pathophysiology of the disease is characterized by
16. A 57-year-old man with a history of diabetes mellitus and abnormalities in myocardial energetics.
hypertension presents to the hospital with new orthop- c. Unexplained syncope is an indication for an implantable
nea and edema. His blood pressure is 95/60 mm Hg, cardioverter-defibrillator (ICD).
and his heart rate is 96 beats per minute. He has clear d. Patients with heart failure symptoms should be offered
lungs, a displaced cardiac apex with a third heart sound surgery.
but no cardiac murmurs, and no lower extremity edema. e. All first-degree relatives should undergo screening for this
Laboratory test results were as follows: hemoglobin 14 disorder.
g/dL, sodium 137 mEq/L, potassium 4.4 mEq/L, and
19. A 74-year-old woman presents with progressive dysp-
serum creatinine 1.7 mg/dL. Chest radiography shows
nea on exertion. She denies having chest pain. She has a
pulmonary congestion with cardiomegaly. The elec-
long-standing history of hypertension and chronic atrial
trocardiogram shows sinus rhythm, nonspecific ST-T
fibrillation. Her medications include warfarin 3 mg
changes, and frequent premature ventricular complexes
daily, enalapril 5 mg daily, and digoxin 0.125 mg daily.
(PVCs). Echocardiography shows a left ventricular ejec-
On physical examination, her blood pressure is 140/70
tion fraction of 20% and a dilated left ventricle. What
mm Hg and her heart rate is 82 beats per minute and
should be the next step in evaluating this patient?
irregular. Jugular venous distention is present. A soft
a. Endomyocardial biopsy
2/6 pansystolic murmur was heard at the left sternal
b. Stress testing
border. Her lungs were clear. Her extremities had pit-
c. Coronary angiography
ting edema (2+). The electrocardiogram showed atrial
d. Holter monitoring
fibrillation with left ventricular hypertrophy. Chest
e. Sleep study
radiography showed cardiomegaly with mild pulmo-
17. A 46-year-old African American woman with pulmo- nary venous hypertension. Echocardiography showed
nary edema is transferred to your hospital. Her heart severe biatrial enlargement and left ventricular hyper-
rate is 105 beats per minute and regular; her blood pres- trophy with normal left ventricular cavity dimensions
sure is 130/70 mm Hg. Her jugular venous pressure is and an ejection fraction of 65%. Which of the following
normal. Her lungs are clear. She has a left ventricular would not be included in the differential diagnosis for
lift and a 3/6 pansystolic murmur at the apex. A sum- this patients symptoms?
mation gallop is present. She has no hepatosplenomeg- a. Constrictive pericarditis
aly, ascites, or edema. Chest radiography is consistent b. Arteriovenous fistula
with pulmonary congestion and cardiomegaly. The c. Amyloidosis
electrocardiogram shows sinus rhythm with a left bun- d. Coronary artery disease
dle branch block. The echocardiogram shows global e. Hypertensive heart disease

2 . C A R D I O L O GY Q U E S T I O N S A N D A N S W E R S 13
C O RO NA RY A RT E RY D I S E A S E A N D 22. A 69-year-old woman presents with chest pain on exer-
MYO C A R D I A L I N FA RC T I O N tion. Past medical history is remarkable for hyper-
lipidemia, hypertension, abdominal aortic aneurysm
20. A 64-year-old woman presents to a community hospi- repair, and a stroke from which she has made a good
tal with a 5-day history of intermittent chest pressure neurologic recovery. Current treatment includes aspi-
at rest. The pain is persistent on the day of admission. rin 81 mg daily, lisinopril 20 mg daily, and atorvastatin
She has a history of chronic stable angina treated with 40 mg daily. On physical examination, she weighs 59
aspirin 325 mg daily, metoprolol 25 mg twice daily, and kg, her pulse is regular at 68 beats per minute, and her
isosorbide mononitrate 60 mg daily. Her past medi- blood pressure is 140/84 mm Hg. Jugular venous pres-
cal history is significant for hypertension and diabetes sure is normal, and all peripheral pulses are normal. On
mellitus. Additional medications include hydrochlo- auscultation, heart sounds are normal and the lungs are
rothiazide 25 mg daily. On physical examination, her clear. There is no peripheral edema. A stress test is per-
pulse is regular at 50 beats per minute, and her blood formed: she exercises for 6 minutes in a Bruce protocol,
pressure is 158/88 mm Hg. Her jugular venous pressure and typical symptoms develop with 2-mm ST-segment
and all peripheral pulses are normal. On auscultation, depression in multiple leads. A coronary angiogram
the heart sounds are normal and the lungs are clear. shows a 90% stenosis in the midsection of the left ante-
There is trace peripheral edema. The electrocardiogram rior descending coronary artery. A drug-eluting stent
(ECG) shows a 1- to 2-mm ST-segment depression in is deployed without complications, and the patient is
the inferior leads and in leads III, aVF, and V6. The tro- discharged with clopidogrel 75 mg daily in addition to
ponin T level is elevated (0.23 ng/mL). After treatment her usual medications. Five days later, she calls and says
with intravenous heparin and nitroglycerin, the patient that an intense pruritic rash has developed on her trunk
becomes asymptomatic with complete resolution of the and limbs. She believes that it is due to the clopidogrel.
ECG abnormalities. Later that day, recurrent, transient Which of the following is an absolute contraindication
chest pressure and ST-segment depression develop. In for the use of prasugrel as an alternative to clopidogrel
addition to arranging transfer to a hospital with facili- in this patient?
ties for coronary angiography, which of the following is a. Female sex
the most appropriate next step? b. Age older than 65 years
a. Increase the dosage of metoprolol to 50 mg twice daily. c. Weight less than 60 kg
b. Perform exercise sestamibi testing. d. Percutaneous coronary intervention
c. Administer morphine intravenously. e. History of stroke
d. Administer reteplase 10 units intravenously over 2 minutes.
23. A 68-year-old woman wants your advice about a recent
e. Initiate an infusion of eptifibatide, a glycoprotein IIb/IIIa
cardiology evaluation. Having had angina for 5 years
inhibitor.
with increasingly limiting symptoms, she was advised
to have coronary angiography, which showed severe
21. A 72-year-old woman presents to the emergency depart-
3-vessel disease. An echocardiogram showed that the
ment with dyspnea and retrosternal chest pain that
left ventricular ejection fraction was 40%. Current med-
developed 3 hours ago, soon after she had an intense
ications include aspirin 81 mg daily, simvastatin 40 mg
argument with her daughter. Her past medical history
daily, atenolol 50 mg daily, and long-acting isosorbide
is remarkable for hypertension and diabetes mellitus.
dinitrate 60 mg daily. Physical examination findings
The chest discomfort is not relieved by sublingual nitro-
include a pulse of 68 beats per minute, blood pressure
glycerin. Her heart rate is 100 beats per minute, and her
of 138/76 mm Hg, and normal jugular venous pressure
blood pressure is 156/92 mm Hg. Her jugular venous
and heart sounds. There are no murmurs, and the lungs
pressure, carotid pulse, and peripheral pulses are nor-
are clear on auscultation. Which of the following is the
mal. On auscultation, heart sounds are normal and the
most appropriate recommendation for this patient to
lungs are clear. The cardiac troponin T level is 0.12 ng/
treat her symptoms and improve the prognosis?
mL (reference range 0.01 ng/mL). The 12-lead electro-
a. Increase the dosage of atenolol to 75 mg daily.
cardiogram shows a 0.5- to 1-mm ST-segment elevation
b. Add ranolazine to her treatment and reassess in 1 month.
in precordial leads V2 through V4. An emergency coro-
c. Coronary artery bypass graft (CABG) surgery is the optimal
nary angiogram shows mild coronary atherosclerosis. A
strategy.
left ventriculogram shows severe hypokinesis of the api-
d. Percutaneous coronary intervention (PCI) is the optimal
cal and midsegments of the heart with normal function
strategy.
at the base. Which of the following is the most likely
e. PCI and CABG are equivalent strategies.
diagnosis?
a. Myocarditis 24. A 52-year-old man returns for a clinic visit after an ante-
b. Apical ballooning syndrome (takotsubo cardiomyopathy) rior myocardial infarction 2 months ago. He was treated
c. Acute coronary syndrome successfully with primary percutanteous coronary
d. Dilated cardiomyopathy intervention. He has made an uncomplicated recov-
e. Pericarditis ery, completed cardiac rehabilitation, and returned to

14 M AYO C L I N I C I N T E R N A L M E D I C I N E B OA R D R E VI EW: Q U E S T I O N S A N D A N S W E R S
work as a salesman. His risk factors for vascular disease protocol; toward the end of the test, typical symptoms
include smoking and hypertension. His current medi- and 1-mm ST-segment depression develop. A coronary
cations are aspirin 81 mg daily, metoprolol 50 mg twice angiogram shows a solitary 70% stenosis in the mid-
daily, clopidogrel 75 mg daily, simvastatin 40 mg daily, section of the right coronary artery. The patient elects
and lisinopril 20 mg daily. On examination, his body medical therapy and returns to your clinic to discuss
mass index is 29. His pulse rate is 64 beats per minute, whether he should have percutaneous coronary inter-
his blood pressure is 126/78 mm Hg, no murmurs or vention (PCI). What is the most appropriate advice on
gallops are noted on heart examination, and the lungs the merits of medical therapy compared with PCI for
are clear. Complete blood cell count and serum chem- this patient?
istry results are all within the reference ranges. Results a. PCI will reduce the need for optimal risk factor
of a fasting lipid panel are as follows: low-density lipo- management.
protein cholesterol (LDL-C) 68 mg/dL, high-density b. PCI will reduce the risk of myocardial infarction.
lipoprotein cholesterol (HDL-C) 34 mg/dL, and tri- c. PCI will result in much less angina compared with medical
glycerides 186 mg/dL. Fasting blood glucose is 114 mg/ therapy.
dL. For secondary prevention, in addition to smoking d. Both medical therapy and PCI will result in similar
cessation, what is the most important next step? survival.
a. Intensify weight management and physical activity. e. The patient will eventually require PCI if medical therapy is
b. Increase the dosage of simvastatin to 80 mg daily. the initial treatment strategy.
c. Measure the plasma high-sensitivity C-reactive protein
(hs-CRP) level.
VA S CU L A R M E D I C I N E
d. Continue the cardiovascular rehabilitation program
long-term.
27. A 60-year-old man is evaluated for a 1-year history of
e. Measure the plasma lipoprotein (a) level.
progressive right leg pain. He has a history of hyperten-
25. A 66-year-old woman presents with progressive sion but not diabetes mellitus. His medications include
angina. Her past medical history is significant for hydrochlorothiazide and aspirin. On examination,
hyperlipidemia, hypertension, and diabetes mellitus. his blood pressure is 150/70 mm Hg and his pulse is
On physical examination, her heart rate is 72 beats 70 beats per minute and regular. Cardiac examination
per minute and her blood pressure is 144/88 mm findings are normal. Vascular examination identifies
Hg. The heart sounds are normal, and a fourth heart palpable but diminished pulses in the popliteal and
sound is present. The lungs are clear on auscultation, posterior tibial arteries bilaterally with an absent dor-
and the peripheral pulses are normal. Current daily salis pedis pulse on the left. Which of the following fea-
medications include aspirin 325 mg, atenolol 25 mg, tures would be most useful to distinguish intermittent
lisinopril 10 mg, atorvastatin 20 mg, insulin, and met- claudication from pseudoclaudication as the primary
formin. Laboratory test results are a normal complete cause of his symptoms?
blood cell count and creatinine 1.6 mg/dL. A stress a. History of nicotine addiction
test is markedly positive for ischemia. The cardiologist b. Symptoms brought on by exertion
has recommended proceeding with coronary angiog- c. Soft bruit over the right common femoral artery
raphy. In addition to discontinuing use of metformin, d. Ankle-brachial index of 0.80 on the right and 0.85 on the
which of the following steps would be most important left
before angiography? e. Symptom relief with sitting only
a. Increase the dosage of atenolol to 50 mg daily.
28. A 70-year-old man presents to the emergency depart-
b. Ensure that the patient is not dehydrated before the
ment with a 1-hour history of back pain radiating to
procedure.
the anterior chest and neck. He describes the pain as
c. Add slow-release isosorbide mononitrate 60 mg daily.
tearing and severe. His pain came on suddenly while
d. Decrease the dosage of aspirin to 81 mg daily.
watching a football game on the television. His past
e. Start clopidogrel therapy with 75 mg daily.
medical history includes hypertension and ongoing
26. A 62-year-old man presents with chest pain on exertion. tobacco use. On examination, he is uncomfortable.
He can walk approximately one-half mile before symp- His blood pressure is 190/100 mm Hg in the right arm
toms develop. He has a history of hypertension and and 170/90 mm Hg in the left arm with a regular pulse
hyperlipidemia. Current treatment includes lisinopril of 110 beats per minute. His chest is clear on ausculta-
40 mg daily and atorvastatin 20 mg daily. On physical tion. On cardiac examination, there is a 2/6 holosys-
examination, his pulse is regular at 66 beats per minute tolic murmur at the apex and a soft fourth heart sound.
and his blood pressure is 134/86 mm Hg; his jugular Vascular examination identifies a slight delay in the left
venous pressure is normal, and all peripheral pulses radial and ulnar pulses with a bruit over the left sub-
are normal. On auscultation, heart sounds are normal clavian artery. Femoral, popliteal, and posterior tibial
and the lungs are clear. There is no peripheral edema. pulses are symmetrical but diminished. On computed
During a stress test, he exercises for 8 minutes in a Bruce tomographic (CT) imaging, a type B aortic dissection

2 . C A R D I O L O GY Q U E S T I O N S A N D A N S W E R S 15
extends to the aortic bifurcation. Which of the follow- c. Right popliteal deep vein thrombosis, heterozygous factor V
ing is an indication for surgical intervention? Leiden, and right fibular fracture 5 days prior
a. Type B aortic dissection d. Symptomatic PE 10 days after a 5-hour flight
b. Nonperfused left kidney identified on the CT scan e. Left ovarian vein thrombosis after vaginal hysterectomy
c. Severe hypertension
32. A 75-year-old right-handed woman is being evaluated
d. Discordant brachial blood pressures, indicating compromise
for a transient ischemic attack (TIA) that she experi-
of the left subclavian artery by the dissection
enced earlier this morning. While clearing the break-
e. Presence of the murmur, indicating disruption of the aortic
fast dishes, she noted a 2-minute episode of slurred
valve annulus by the dissection
speech and right hand and facial numbness. She has a
29. A 75-year-old woman is evaluated for an abdominal history of hypertension but has been healthy otherwise
aortic aneurysm (AAA) found at a vascular screening with no additional diagnoses. On examination, her
fair at her local supermarket. Her risk factors for ath- blood pressure is 150/80 mm Hg with a regular pulse of
erosclerosis include hypertension and hyperlipidemia. 72 beats per minute. Her chest is clear on auscultation.
She has not smoked in the past and has no history of Cardiac examination identifies a normal jugular venous
diabetes mellitus. On examination, her blood pressure pulse and apical impulse. On auscultation, there is no
is 140/70 mm Hg in both arms with a regular pulse of murmur or gallop. A bruit is heard over both carotid
72 beats per minute. Her lungs are clear. The carotid arteries. Carotid upstrokes are normal. The electrocar-
upstrokes are normal. On abdominal examination, an diogram shows normal sinus rhythm with normal inter-
enlarged pulsatile mass is superior to the umbilicus. vals and axes. Which of the following statements is true
Femoral, popliteal, and posterior tibial pulses are nor- for symptomatic carotid disease?
mal. Which of the following findings should prompt a. After a TIA in a patient with an ipsilateral internal carotid
surgical referral for intervention? artery stenosis greater than 70%, the combined outcomes of
a. A 5.1-cm infrarenal AAA stroke or death are improved with carotid endarterectomy
b. Annual growth rate of 0.3 cm surgery compared with medical management.
c. Accompanying left iliac artery aneurysm measuring 2.8 cm b. Most TIAs result from a symptomatic ipsilateral carotid
d. Father who died of a ruptured AAA lesion.
e. Tender aneurysm on examination c. For an ipsilateral symptomatic carotid stenosis greater than
70%, surgery and medical therapy result in similar 2-year
30. A 45-year-old man presents for evaluation in the emer-
stroke-free survival rates.
gency department with acute onset of dyspnea and
d. For the patient presented, carotid artery stenting with a distal
severe cough. He has had no recent travel, trauma, or
protection device will provide superior short- and long-term
surgery. He notes that his left leg has been painful for
stroke-free survival compared with carotid endarterectomy.
2 days, and this morning he noted some swelling. He
e. At 2 years, the rate of recurrent stenosis (>70%) is equal for
is dyspneic. His blood pressure is 86/60 mm Hg with a
carotid stenting and endarterectomy.
regular pulse of 117 beats per minute. His chest is clear.
Cardiac examination identifies mild jugular venous dis- 33. A 46-year-old man with diabetes mellitus, ongo-
tention with a subtle right ventricular lift. His left leg is ing tobacco use, hypertension, and hyperlipidemia
slightly edematous. Computed tomographic angiogra- complains of 2-block claudication in both legs. His
phy of the chest identifies bilateral pulmonary emboli. symptoms improve with standing and are consistent
Duplex ultrasonography identifies an extensive left from day to day. He has no rest pain or ulceration. His
femoral-popliteal deep vein thrombosis (DVT). Which symptoms have been stable for the past 2 years. His
of the following findings should prompt inferior vena blood pressure is 150/70 mm Hg with a regular pulse
cava (IVC) filter placement? of 80 beats per minute. His chest is clear on ausculta-
a. Hemodynamic instability with coexistent DVT and a large tion. His carotid upstrokes are normal without bruit.
pulmonary embolism (PE) Cardiac examination identifies a normal jugular venous
b. Mobile thrombus in the left femoral vein identified on ultra- pressure and apical impulse. Findings on auscultation
sonographic imaging are normal. He has normal pulses in the femoral and
c. Family history of PE popliteal arteries bilaterally without bruit. The pedal
d. History of PE 3 years ago pulses are not palpable. His extremities do not have
e. Retroperitoneal hemorrhage after initiation of heparin ulcers, ischemic fissures, dependent rubor, or elevation
therapy pallor. The ankle-brachial index is 0.55 on the right and
0.52 on the left. An angiogram performed at his local
31. Which of the following conditions should prompt pro-
medical facility 1 year ago showed severe infrapopliteal
longed secondary prophylaxis with warfarin?
arterial occlusive disease with diseased but patent prox-
a. Portal vein thrombosis 3 weeks after colonic resection for
imal arteries bilaterally. What is the most appropriate
ulcerative colitis
treatment regimen for this patient with intermittent
b. Bilateral pulmonary embolism (PE) in a 23-year-old woman
claudication?
who uses oral contraception

16 M AYO C L I N I C I N T E R N A L M E D I C I N E B OA R D R E VI EW: Q U E S T I O N S A N D A N S W E R S
a. Computed tomographic angiography (CTA) of the legs 37. A 54-year-old man has an elevated blood pressure (BP)
b. Magnetic resonance angiography (MRA) of the legs (150/94 mm Hg) that has been confirmed on several
c. Risk factor modification and the Canadian walking office visits. He is a smoker with stable, mild claudica-
program tion. He has hyperlipidemia, which is controlled with
d. Angioplasty and stenting diet and statin therapy. Lifestyle modification and drug
e. Bypass surgery therapy are initiated for BP control. What is the recom-
mended BP goal for this patient?
a. Less than 140/90 mm Hg measured in the office
H Y P E RT E NS I O N
b. Less than 135/85 mm Hg measured at home
c. Less than 130/80 mm Hg measured in the office and at
34. A 45-year-old man with no complaints comes to you for
home
a general medical examination. His body mass index is
d. Less than 125/75 mm Hg measured in the office
26, his blood pressure is 145/95 mm Hg, and his pulse
e. Less than 125/75 mm Hg measured at home
is 65 beats per minute. The rest of the examination find-
ings are normal. He takes acetaminophen for occasional 38. A 72-year-old woman has long-standing hypertension.
arthritis pain. He has no other significant past medical Two days after total hip arthroplasty, her blood pressure
or surgical history. What would be your next step in is 220/110 mm Hg, which is confirmed on a subsequent
management? measurement. She reports having substernal chest pres-
a. Recheck blood pressure in 2 years. sure and mild dyspnea. An electrocardiogram shows
b. Recheck blood pressure in 1 year. ST-segment depression in the inferior leads. What is
c. Recheck blood pressure in 6 months. the most appropriate parenteral antihypertensive drug
d. Recheck blood pressure in 2 months. to consider for this patient?
e. Evaluate and treat blood pressure now. a. Sodium nitroprusside
b. Hydralazine
35. A 34-year-old sexually active woman has a blood pres-
c. Labetalol
sure of 150/94 mm Hg that has been confirmed on
d. Nitroglycerin
several office visits and with self-monitoring despite a
e. Nicardipine
6-month trial of lifestyle modifications. Which of the
following drugs would be most appropriate for this 39. A 34-year-old woman has had episodes of headache
patient? associated with diaphoresis and nausea. These episodes
a. Lisinopril 10 mg once daily begin suddenly and vary in duration from 15 to 30 min-
b. Doxazosin 2 mg once daily utes. Recently, a blood pressure of 210/140 mm Hg
c. Losartan 50 mg once daily was measured during an episode. Her family history is
d. Hydrochlorothiazide 12.5 mg once daily significant for pheochromocytoma in her mother. On
e. Aliskiren 150 mg once daily examination, her blood pressure is 140/94 mm Hg.
Results of routine laboratory tests were normal. What
36. A 40-year-old woman is referred for further evaluation
is the most appropriate next step in her evaluation?
of an elevated office blood pressure (BP) of 150/96 mm
a. Measure plasma free metanephrines.
Hg. Her friend, who is a nurse, has measured her BP
b. Begin drug treatment with metoprolol.
several times at home, where it ranged from 130/80 to
c. Obtain a computed tomogram of the abdomen.
136/88 mm Hg. The patient eats a high-salt diet and
d. Obtain a duplex ultrasonogram of the renal arteries.
does not exercise. Her mother has hypertension. On
e. Measure plasma and urine catecholamines.
examination, the patients BP is 146/94 mm Hg; exam-
ination findings are otherwise normal. Routine labora- 40. A 42-year-old man with a history of metabolically
tory test results are within the reference ranges. What is active calcium oxalate nephrolithiasis has hypertension
the most appropriate next step in evaluating or treating that was recently diagnosed and is not controlled with
this patient? lifestyle modifications. Which of the following drugs
a. Discuss lifestyle modifications and begin therapy with would be the most appropriate initial choice for treat-
atenolol. ing his hypertension?
b. Begin therapy with a low dose of hydrochlorothiazide. a. Lisinopril
c. Obtain a duplex ultrasonogram of the renal arteries. b. Furosemide
d. Begin therapy with a low dose of hydrochlorothiazide in c. Losartan
combination with lisinopril. d. Atenolol
e. Obtain a 24-hour ambulatory BP recording. e. Chlorthalidone

2 . C A R D I O L O GY Q U E S T I O N S A N D A N S W E R S 17
ANSWER S

1. Answer b. be permanent pacemaker implantation. The other causes of


syncope are most unlikely in this patient.
The ECG is typical for Wolff-Parkinson-White syndrome.
Since the patient is asymptomatic, there is no indication 4. Answer d.
for drug therapy or an electrophysiology study unless he
is in a high-risk occupation (eg, pilot, military member). CRT is indicated for patients who have LVEF less than 35%,
With the onset of symptoms (palpitations, class I indica- LBBB with QRS duration of more than 120 ms, and conges-
tion), the preferred approach would be an electrophysiol- tive heart failure in NYHA class III or IV despite receiving
ogy study and catheter ablation (>95% success rate with a the maximally tolerated medical therapy. The most appro-
low risk of complications). priate management is to up-titrate the dosages of lisinopril
and carvedilol now and reevaluate the patient over the next
2. Answer a. 3 to 6 months to determine whether CRT is indicated.
The clinical decision on how best to treat the patients 5. Answer d.
atrial fibrillation (rate control or rhythm control) should
be based on symptom burden and adequacy of rate control, The patient is well compensated from an HF standpoint.
independently of stroke risk, as determined by calculating Her episodes of AF are infrequent and not particularly long
her CHADS2 (congestive heart failure, hypertension, in duration. Therefore, use of a daily antiarrhythmic drug
age >75 years, diabetes mellitus, and previous stroke) risk (eg, sotalol) is not warranted (toxicity could occur over the
score. Her CHADS2 score is greater than 2. According to long term with limited benefit). Dronedarone is contrain-
current guidelines, warfarin is indicated. dicated in patients with HF or recent decompensation of
HF. (See Kber et al in the Suggested Reading list.)
3. Answer a.
6. Answer d.
The symptoms and presentation are consistent with car-
diogenic syncope and conduction system disease (there is This patient is asymptomatic and presents with bradycardia
electrocardiographic evidence of high-grade atrioventricu- that may be readily explained by her athletic conditioning.
lar block and left bundle branch block). Treatment would Her murmur is systolic, soft (less than grade 2), and heard

Presence of cardiac murmur

Systolic murmur Diastolic or continuous murmur

Grade 3 or higher
Grade 1 or 2 &
holosystolic or late
midsystolic
systolic

Asymptomatic Other signs or


& no associated symptoms of Echocardiography
findings cardiac disease

Catheterization &
No further workup
angiography if required

Figure 2.A6. (Adapted from Bonow RO, Carabello B, De Leon AC Jr, Edmunds LH Jr, Fedderly BJ, Freed MD, et al. ACC/AHA guidelines for the
management of patients with valvular heart disease: a report of the American College of Cardiology/American Heart Association. Task Force on Practice
Guidelines [Committee on Management of Patients with Valvular Heart Disease]. J Am Coll Cardiol. 1998 Nov;32[5]:1486588. Used with permission.)

18
in the pulmonary position, and it disappears with the is not indicated in ASD or PFO. Chest radiography may
Valsalva maneuver. If you were worried about hypertrophic be helpful. Classically, with large shunts radiography shows
cardiomyopathy, the response to this maneuver fairly well enlargement of the right atrium (bulging of the right heart
argues against it. There are no associated systolic clicks of border), RV (decreased retrosternal clear space), and pul-
mitral valve prolapse or bicuspid aortic valve, no diastolic monary artery and increased pulmonary vascular markings.
snaps to suggest mitral stenosis, and the electrocardiogram Common anatomical types of ASD are ostium secundum
is normal with the exception of the slow heart rate. Thus, ASD, ostium primum ASD, and sinus venosus ASD. Sinus
the murmur is most likely a benign flow murmur due to a venosus ASD is commonly accompanied by anomalous
thin chest wall, and you are likely hearing pulmonary out- pulmonary venous return. Ostium primum ASD is com-
flow, anatomically located just below the left upper sternal monly associated with cleft mitral valve and other complex
border. An S3 is a normal finding in a young athlete. There congenital heart diseases. (See Levin et al, OToole et al,
is no need for further testing (Figure 2.A6). and Wilson et al in the Suggested Reading list.)
7. Answer c. 8. Answer c.
Several clinical findings in ASD provide clues to the condi- This 30-year-old patient presents with atypical chest pain,
tion: 1) RV lift or sternal lift; 2) palpable pulmonary artery which may be associated with mitral valve prolapse. Mitral
pulse consistent with pulmonary artery volume and pres- valve prolapse is essentially a mismatch of the left ven-
sure overload of the RV (associated with significant shunt- tricular cavity size and the mitral valve leaflets, which are
ing); 3) fixed splitting of the S2 (the hallmark of ASD); 4) redundant. The classic bedside maneuver is the squat-to-
accentuation of the tricuspid valve closure leading to split- stand maneuver; however, any maneuver that increases the
ting of the first heart sound; 5) midsystolic pulmonary ejec- left ventricular cavity size will delay midsystolic clicks and
tion murmur due to increased volume of flow through the the mitral regurgitant murmur (Figure 2.A8). The Valsalva
RV outflow tract; and 6) if a large shunt is present, a mid- to maneuver decreases venous return, which would result in
late-diastolic rumble may also be appreciated. PFO is usu- a smaller left ventricle and thus an earlier mitral valve pro-
ally not associated with any significant physical examination lapse. A postextrasystolic beat (due to the compensatory
findings, and it is a relatively common finding in autopsy pause), the passive leg raise, and squat (all would improve
studies; on average, PFO is seen in 25% of otherwise left ventricular filling) should actually delay midsystolic
healthy people. Subacute bacterial endocarditis prophylaxis clicks and murmur.

Supine S1
S2
C

Standing S1
S2
C

Squatting S1
S2
C

Figure 2.A8. C indicates click; S1, first heart sound; S2, second heart sound. (Adapted from Shaver JA, Leonard JJ, Leon DF. Examination of the heart.
Part 4: Auscultation of the heart. Dallas [TX]: American Heart Association; c1990. p. 13. Used with permission.)

2 . C A R D I O L O GY Q U E S T I O N S A N D A N S W E R S 19
9. Answer c. 13. Answer e.
This patient, with a systolic click murmur in the absence The patient presents with stage D congestive heart fail-
of dynamic change, likely has bicuspid aortic valve (AV) ure. He has several poor prognostic indicators, including
stenosis. This diagnosis is more commonly associated with recurrent hospitalizations for heart failure exacerbations
a coarctation of the aorta whose flow murmur is best heard despite optimal medical therapy, low blood pressure, ane-
over the posterior aspect between the shoulder blades, mia, renal insufficiency, and a decreasing LVEF. His blood
slightly to the left of the midline. In the presence of hyper- pressure would not allow an increase in -blocker dose.
tension and radial-femoral delay, there is enough evidence Optimization of cardiac resynchronization therapy is
on examination to support imaging of the aorta to rule out unlikely to help patients with stage D heart failure and a
coarctation. A CT scan (or magnetic resonance imaging) low output state. Adding a thiazide diuretic when edema
of the chest would show the coarctation. Another appro- has improved and renal insufficiency has worsened would
priate test would be transthoracic echocardiography to not be an appropriate next step. The patient had a remote
confirm the diagnosis of a bicuspid AV and to assess the history of myocardial infarction and had no active angina
degree of aortic stenosis. Echocardiography is often used to pectoris; hence, determining myocardial viability with a
view the descending thoracic aorta to determine the degree view toward revascularization is unlikely to provide much
of coarctation; however, if the aorta is not well visualized benefit toward improving the patients symptoms of heart
by echocardiography, other imaging is indicated to rule failure or survival. Patients with dilated cardiomyopathy,
out the coarctation. A coronary angiogram is unlikely to an LVEF of 25% or less, and New York Heart Association
be helpful unless it is coupled with an aortogram, which (NYHA) class IV symptoms should be considered for
might provide the diagnosis, but there may be an increased LVAD implantation after optimization of medical and
danger due to the potential for coarctation, which could be electrical therapy. LVADs have been shown to prolong sur-
difficult to cross with the catheter. This patient most likely vival, improve NYHA functional class, and improve qual-
does not have coronary artery disease. The murmur and ity of life.
clinical presentation are not benign, and a thorough evalu-
ation is warranted. This patients syncopal event is related 14. Answer e.
to aortic stenosis or coarctation with significant stenosis Although serum BNP levels correlate with clinical sever-
and the inability to adequately increase cardiac output, ity, some patients with advanced heart failure have normal
especially with postexercise vasodilatation. BNP levels. Several large clinical trials have shown that
a persistently elevated BNP level despite ongoing medi-
10. Answer c.
cal therapy is a marker of poor prognosis. A BNP-guided
Constrictive pericarditis is a relatively infrequent compli- approach to titrate medical therapy has been shown to
cation of open heart surgery, yet cardiac surgery is now the decrease hospitalizations compared with standard therapy,
number one cause of constrictive pericarditis causing heart but the benefit may result from more intense medical sur-
failure. The heart failure often begins slowly, with symptoms veillance and subsequent up-titration of medical therapy;
of dyspnea, peripheral edema, and signs of elevated neck veins therefore, its routine use for this purpose cannot be rec-
and the Kussmaul signelevation of JVP with inspiration. ommended. Although the serum BNP level is frequently
Another finding often reported on the JVP is rapid descent. elevated in constrictive pericarditis due to cardiac surgery
or radiotherapy, it is frequently within the normal range in
11. Answer e. patients with idiopathic constrictive pericarditis.
The presence of a prominent v wave in the JVP profile is 15. Answer c.
consistent with significant tricuspid regurgitation. The
right-sided S3 may be intermittent and more prominent The patient has severe heart failure with preserved ejection
with the inspiratory phase of respiration. Mitral regurgita- fraction. The cause of heart failure in this patient is cardiac
tion will produce a v wave into the pulmonary circulation, amyloidosis, which is consistent with the low voltage on
which can be seen with Swan-Ganz catheter tracings, but the the electrocardiogram and the thickened left ventricular
v wave is generally not transmitted to the JVP. Constrictive walls with a classic speckled pattern on the echocardio-
pericarditis has very rapid y descents. Pulmonary stenosis is gram. The concomitant presence of peripheral neuropathy
associated with a giant a wave due to increased atrial con- is consistent with a diagnosis of familial amyloidosis. In
traction caused by right ventricular pressure overload. In familial amyloidosis, the source of the amyloidogenic vari-
superior vena cava (SVC) syndrome, the pulsatile waves are ant of the transthyretin protein is the liver. Hence, the best
often lost because of extra-cardiac obstruction of the SVC. treatment option for this patient is combined heart and
liver transplant.
12. Answer a.
16. Answer c.
An S4 indicates increased left ventricular stiffness and is
commonly heard in adults with hypertension. It is caused The patient has dilated cardiomyopathy. The most com-
by the increased filling in late diastole during atrial contrac- mon cause of dilated cardiomyopathy, especially in a person
tion. An S4 is never normal. with atherosclerotic risk factors, is coronary artery disease.

20 M AYO C L I N I C I N T E R N A L M E D I C I N E B OA R D R E VI EW: Q U E S T I O N S A N D A N S W E R S
The most appropriate next step for this patient would be hypertensive heart disease. However, she should be evalu-
to perform coronary angiography, the best diagnostic test ated for possible constrictive pericarditis and amyloidosis.
in this circumstance. Stress testing may have false-positive Rarely, arteriovenous fistulae can also cause high-output
or false-negative results in patients with dilated cardio- heart failure. Coronary artery disease can be present in
myopathy. Endomyocardial biopsy should not be routinely patients with heart failure and normal ejection fraction and
performed in the evaluation of patients with heart failure. frequently needs to be excluded, but it would not account
Holter monitoring is useful in detecting the PVC burden for the patients symptoms and signs of heart failure.
over a 24-hour period, and if the PVC burden were espe-
20. Answer e.
cially high, it could be implicated as a rare cause of heart
failure. However, Holter minitoring would not be the next The patient presents with a nonST-elevation myocardial
step in evaluation and should not be performed without infarction. Her age, ECG changes, and elevated levels of
initiating medical therapy because indicated medical treat- biomarkers give her a high-risk profile. An initially conser-
ment (eg, -blockers) can attenuate PVCs. Sleep apnea vative medical management strategy is reasonable because
could be a frequent accompaniment of decompensated she presented to a community hospital. However, recur-
heart failure, but a sleep study should be performed only rence of chest pain is a clear indication for transferring her
after patients are optimally treated. to a facility with a cardiac catheterization laboratory. Several
studies have shown that glycoprotein IIb/IIIa inhibitors
17. Answer e.
are effective in the management of nonST-segment eleva-
-Blockers, spironolactone, isosorbide dinitrate in com- tion acute coronary syndromes, particularly in high-risk
bination with hydralazine, and biventricular pacing have patients who require percutaneous coronary intervention.
improved survival among patients with severe heart fail- Initiating therapy with a glycoprotein IIb/IIIa inhibi-
ure and idiopathic dilated cardiomyopathy. However, the tor such as eptifibatide is effective in reducing recurrent
Digitalis Investigation Group trial did not show a survival ischemia and recurrent myocardial infarction before the
benefit with the use of digoxin in heart failure. The lack coronary intervention and improves outcomes after percu-
of benefit may have been related to a higher serum digi- taneous coronary intervention. Increasing the dose of the
talis level resulting in a higher mortality presumably due to -blocker would not be advisable in view of the bradycar-
a proarrhythmic effect. Digitalis retains a role in treating dia. Stress testing is absolutely contraindicated for patients
patients who have persistently symptomatic heart failure with acute ischemic symptoms. Morphine may be adminis-
and patients who have atrial fibrillation with rapid ventric- tered to relieve symptoms but is not a definitive therapy for
ular response and heart failure. severe ischemia. Thrombolytics such as reteplase are not
indicated in nonST-segment elevation acute coronary
18. Answer d.
syndromes.
The diagnosis is hypertrophic cardiomyopathy (HCM) and
21. Answer b.
is made primarily by the physical findings. Although hos-
pitalized patients present with syncope, chest pain, heart The patient presents with symptoms consistent with an
failure, or sudden cardiac death, most patients with HCM acute coronary syndrome with ST-segment elevation on the
are asymptomatic. The pathophysiology of this autosomal electrocardiogram. Therefore, it was reasonable to perform
dominant disease has been linked to genetic mutations, emergency angiography, which demonstrated normal cor-
predominantly in the sarcomere and mitochondria, that onary arteries. The differential diagnosis includes coronary
result in either increased energy use or decreased energy spasm, coronary embolism, pericarditis, and myocarditis.
production, which ultimately promotes myocyte growth. However, this patient had a very characteristic regional
Hence, all first-degree relatives should be evaluated with wall motion abnormality on the left ventriculogram, which
electrocardiography and echocardiography. Patients with a involved the mid and apical segments of the left ventricle
personal or family history of sudden cardiac death, unex- with sparing of the basal segments. This entity has recently
plained syncope, nonsustained ventricular tachycardia on been recognized as the apical ballooning syndrome (tako-
Holter monitoring, severe left ventricular hypertrophy, or tsubo cardiomyopathy). Apical ballooning syndrome
an abnormal blood pressure response to exercise are at the occurs predominantly in postmenopausal women and is
highest risk for sudden cardiac death and should be consid- frequently preceded by mental or physical stress. Acute
ered for ICD implantation. Treatment of the patient with coronary syndrome was ruled out by coronary angiography.
HCM and heart failure is initially medical, with -blockers Myocarditis is a differential diagnosis, although it typically
or calcium channel blockers (or both), but if symptoms are produces global left ventricular dysfunction. Pericarditis is
progressive despite medical therapy, myomectomy or alco- not associated with systolic dysfunction.
hol septal ablation should be considered.
22. Answer e.
19. Answer d.
Prasugrel is a new antiplatelet agent that acts by irrevers-
This patient presents with evidence of heart failure with ibly blocking adenosine diphosphate receptors on plate-
a normal left ventricular ejection fraction. She has the lets, preventing their activation and aggregation. It has
classic presentation of diastolic heart failure likely due to been approved for use in patients with unstable angina,

2 . C A R D I O L O GY Q U E S T I O N S A N D A N S W E R S 21
nonST-segment elevation myocardial infarction, or prophylactic measures include administering intravenous
ST-segment elevation myocardial infarction managed with fluid and N-acetylcysteine before the procedure. Initiating
percutaneous coronary intervention. Its use is absolutely the use of clopidogrel before angiography is not routine
contraindicated in patients with active pathologic bleeding practice. The most appropriate time for adjusting the medi-
or a history of transient ischemia attack or stroke. It is gen- cal therapy would be after coronary angiography as part of
erally not recommended (but may be used) in patients who the overall management strategy. In the presence of abnor-
are 75 years or older or who weigh less than 60 kg. It is not mal renal function, it is important to discontinue the use
contraindicated in women, but women have an increased of metformin before coronary angiography. There is an
risk of bleeding. increased risk of lactic acidosis among diabetic patients
who have renal impairment and contrast nephropathy and
23. Answer c.
are receiving metformin. Therefore, it is best practice to
The patient has severe 3-vessel disease with reduced ejec- discontinue the use of metformin before elective proce-
tion fraction and significant angina despite medical ther- dures and to resume it 48 hours after the administration of
apy. CABG surgery is the treatment of choice to relieve radiologic contrast material if the patient has an uncompli-
symptoms and to improve long-term prognosis. PCI is cated recovery.
increasingly being performed for multivessel disease but is
26. Answer d.
not the standard of care for revascularization in patients
with severe 3-vessel disease. Medical therapy alone in This patient has moderate symptoms with single-vessel coro-
patients with 3-vessel disease and decreased ejection frac- nary artery disease. PCI is an established, effective therapy
tion is associated with worse outcomes compared with sur- for coronary heart disease; however, it has not been shown
gery followed by optimal medical therapy. to reduce the risk of myocardial infarction or improve sur-
vival among patients with stable angina. Previous studies
24. Answer a.
have suggested that PCI reduces the amount of medica-
This patient has premature coronary artery disease. tions required and provides better symptomatic relief.
Aggressive risk factor management is required. Smoking The Clinical Outcomes Utilizing Revascularization and
cessation and adequate treatment of hypertension are Aggressive Drug Evaluation (COURAGE) trial confirmed
essential. The patients blood pressure appears to be below that with optimal medical therapy, PCI does not offer signif-
the recommended goal of 130/80 mm Hg. The Adult icant advantage. (See Boden et al in the Suggested Reading
Treatment Panel III (ATP III) guidelines for managing list.) However, PCI was associated with slightly lower use of
hyperlipidemia recommend an optimal LDL-C goal of less antianginal medications and slightly better symptom con-
than 70 mg/dL. This patient has achieved that goal. The trol. Thus, for this particular patient both treatment strate-
ATP III guidelines also recommend screening and treat- gies are equal for survival and risk of myocardial infarction.
ing for the metabolic syndrome as a secondary therapeu- PCI is likely to lead to less antianginal medication use and
tic goal for managing hyperlipidemia. This patient meets better control of angina in the short term.
criteria for metabolic syndrome: the level of triglycerides
27. Answer e.
is high (150 mg/dL), HDL-C is low (40 mg/dL), and
fasting blood glucose is high (110 mg/dL). The guidelines Distinguishing true claudication from pseudoclaudica-
state that the initial step in managing metabolic syndrome tion can be accomplished with a good history and physi-
is to treat underlying causes, such as overweight or obesity cal examination. Patients with true claudication typically
and physical inactivity. If these measures are inadequate, describe cramping pain with muscle fatigue occurring in
pharmacologic therapy for low HDL-C or elevated tri- the calf and progressing to the thigh and buttock with con-
glycerides (or both) is indicated. Increasing the dose of tinued exercise. The classic description includes exertional
simvastatin from 40 to 80 mg is not indicated since the pain involving the calf that impedes walking, resolves
patient has achieved the LDL-C goal and an increase in within 10 minutes of rest, and neither begins at rest nor
the dose is unlikely to substantially increase the HDL-C resolves on walking. In contrast, pseudoclaudication
level. Elevated levels of hs-CRP are associated with an occurs both with walking and with prolonged standing
increased risk of future events, but the precise clinical or any activity associated with spinal extension. Patients
utility of measuring hs-CRP has not been established. with pseudoclaudication must sit (with spinal flexion) to
Long-term cardiovascular rehabilitation therapy has not relieve the pinching of the involved nerve roots. Sorting
been shown to improve outcome. Elevated plasma lipopro- out the cause of patients symptoms can be difficult since
tein (a) levels are associated with an increased risk of car- many have both true claudication and pseudoclaudication.
diovascular events; however, no therapeutic intervention The patient described in this question has peripheral artery
has been shown to improve outcomes among patients who disease (PAD), but from the data provided, his symptoms
have elevated lipoprotein (a) levels. cannot be definitely attributed to PAD. Only 33% of
patients with PAD have classic symptoms of claudication.
25. Answer b.
Indeed, more than half of patients with documented PAD
A patient with diabetes mellitus and chronic renal failure have no symptoms of claudication. (See McDermott et al
has an increased risk of contrast nephropathy. Reasonable in Suggested Reading list.)

22 M AYO C L I N I C I N T E R N A L M E D I C I N E B OA R D R E VI EW: Q U E S T I O N S A N D A N S W E R S
28. Answer b. warfarin therapy. The family history of PE is not relevant
to this decision. For cancer patients with recurrent throm-
This patient has a descending aortic dissection. By defini-
bosis despite adequate warfarin therapy, the proper treat-
tion, the proximal tear occurs distal to the left subclavian
ment is conversion to low-molecular-weight heparin rather
artery orifice and adjacent to the ligamentum arteriosum. In
than IVC filter placement. (See Baglin et al in Suggested
contrast to type A (ascending) thoracic aortic dissections,
Reading list.)
which are managed surgically (mortality is 1% per hour
without surgery), type B dissections are typically managed 31. Answer d.
medically. Indications for surgical intervention for type B
When defining appropriate venous thromboembolism
dissections include dissection progression despite medical
treatment (ie, the first 3 months of therapy) and dura-
management, persistent or recurrent pain despite medical
tion of secondary prophylaxis (ie, the time beyond the
management, and end-organ ischemia due to branch lumi-
initial 3 months of therapy), it is important to distinguish
nal compromise from the dissection flap. In this patient,
provoked from unprovoked or spontaneous thrombotic
the dissection does not involve the left subclavian artery
events. In general, events provoked by a transient risk
by definition. The discordant brachial blood pressures are
factor (as in answer choices a-c and e) are treated with 3
a result of prolonged tobacco exposure and associated sub-
months of warfarin therapy. While heterozygous factor V
clavian arterial atherosclerosis. The murmur described is
Leiden is a congenital risk factor, it carries a rather weak
from mitral insufficiency, not from aortic valve disruption.
propensity for venous thrombosis, with a hazard ratio of
(See Hiratzka et al in the Suggested Reading list.)
approximately 3. Therefore, neither heterozygous factor V
29. Answer e. Leiden nor heterozygous prothrombin G20210A muta-
tions require prolonged secondary prophylaxis in and of
Timing of AAA repair (either surgically or by endograft) themselves. One might argue that a 5-hour flight 10 days
is important and requires balancing the risk of rupture before the event was a provocation, but the hazard asso-
with the risk associated with the intervention. Currently, ciated with such an exposure is low. A stronger argument
AAA repair is recommended if the diameter is 5.5 cm. This is that the PE was unrelated to the air travel and was an
measurement should be anteroposterior and perpendicu- unprovoked event. Idiopathic or unprovoked events carry
lar to the axis, which can be difficult with tortuous aorta. a higher rate of recurrence. Furthermore, if this patient has
Other indications for repair include annual growth rates a recurrent event, it is twice as likely to be another PE. PE,
exceeding 0.5 cm or any symptoms suggesting instability, and especially recurrent PE, carries a high mortality rate.
such as tenderness of the aorta on examination. The iliac Severe symptomatic PE in the period immediately after
artery aneurysm is small and would not require surgery. travel is extremely rare after flights of less than 8 hours. In
The typical cutoff for iliac artery aneurysms is 3.5 cm. The flights longer than 12 hours, the rate is 5 per million. (See
family history of ruptured AAA is not a criterion for sur- Kearon et al and see Watson and Baglin in the Suggested
gery. The guidelines support a one-time screening for AAA Reading list.)
in men aged 65 to 75 who have never smoked but who have
a first-degree relative who required repair of an AAA or 32. Answer a.
died of a ruptured AAA (grade 2C). A one-time screening Nearly 800,000 strokes occur each year in the United
for AAA with abdominal ultrasonography is also recom- States, and stroke is the third leading cause of death and
mended for men aged 65 to 75 who have smoked (grade the leading cause of long-term disability. Most strokes are
1A). Screening for asymptomatic women is not recom- ischemic and thromboembolic in nature. The prevalence of
mended; however, Medicare allows for a one-time screen- moderate to severe carotid disease in the United States is
ing for AAA in women with a family history of AAA. (See 5% to 9% in persons older than 65, accounting for 1.3 to 2.4
Hirsch et al in the Suggested Reading list.) million Americans. The annual prevalence of TIA is nearly
30. Answer e. 5 million. Of these, one-third occur in patients with mod-
erate to severe carotid stenosis. Both the Asymptomatic
More IVC filters are placed in medical institutions in the Carotid Atherosclerosis Study (ACAS) and the North
United States than in any other country. Current indica- American Symptomatic Carotid Endarterectomy Trial
tions for IVC filter placement include inability to provide (NASCET) showed that surgery improved stroke-free
anticoagulation because of active bleeding or circumstances survival at 2 years compared with medical management
in which the risk of initiating major bleeding is high (eg, in symptomatic patients. The Endarterectomy Versus
recent surgery). The proper treatment of hemodynamic Angioplasty in Patients With Symptomatic Severe Carotid
instability with DVT and a PE would not be placement Stenosis (EVA-3S) trial and the International Carotid
of an IVC filter but rather consideration of thrombolytic Stenting Study showed that carotid endarterectomy was
therapy. The finding of mobile thrombi (rather than immo- the procedure of choice for symptomatic carotid stenosis
bile thrombi) on cross-sectional imaging does not increase compared with carotid stenting (with embolic protection
the risk of embolism. A prior personal history of PE would devices). The Stent-Protected Angioplasty Versus Carotid
not justify IVC filter placement; however, it would sug- Endarterectomy (SPACE) study found that carotid stent-
gest the need for prolonged secondary prophylaxis with ing resulted in a significantly greater rate of restenosis

2 . C A R D I O L O GY Q U E S T I O N S A N D A N S W E R S 23
(>70%) at 2 years (11.1% vs 4.6%) compared with endar- hypertension may evolve into sustained hypertension
terectomy. (See Ederle et al, European Carotid Surgery over time. Thus, when hypertension is diagnosed, patients
Trialists Collaborative Group, Mas et al, North American should be advised on lifestyle modifications associated
Symptomatic Carotid Endarterectomy Trial Collaborators, with lowering BP and be followed. Self-measurement of
and Ringleb et al in the Suggested Reading list.) BP should be encouraged.
33. Answer c. 37. Answer c.
This patient has a classic presentation of infrapopliteal arte- In general, the office BP goal for persons with hyperten-
rial occlusive disease with long-standing diabetes mellitus. sion is less than 140/90 mm Hg and the home BP goal
Both the latest physical examination and the previous con- is less than 135/85 mm Hg. Current guidelines recom-
ventional angiogram support this impression. It would not mend a more aggressive BP goal of less than 130/80 mm
be helpful to repeat either MRA or CTA now. Angioplasty Hg for persons who have diabetes mellitus, chronic kid-
and stenting of these distal arteries would have poor dura- ney disease, known vascular disease, or a high risk of vas-
bility. There is no indication for surgical bypass since cular disease. This same goal applies for both office and
the patient has neither rest pain nor ulceration. The best self-measured readings. A goal of less than 125/75 mm Hg
option for this patient is risk factor modification (smoking is appropriate for patients who have renal disease associ-
cessation, statin therapy, hypertension control, and diabe- ated with proteinuria.
tes therapy). (See Hirsch et al in the Suggested Reading
38. Answer d.
list.)
This case should be considered a hypertensive emergency
34. Answer d.
because the patient has evidence of organ injury (coronary
The Seventh Report of the Joint National Committee on ischemia and infarction). Therefore, an immediate reduc-
Prevention, Detection, Evaluation and Treatment of High tion in blood pressure with a parenteral agent is indicated.
Blood Pressure ( JNC 7) provides follow-up recommenda- Of the available drugs, nitroglycerin is preferred with myo-
tions based on the classification of blood pressure. Patients cardial ischemia. It is a balanced arterial and venous dilator
with normal blood pressure should be rechecked in 2 years. and lessens myocardial oxygen demand by reducing both
Prehypertensive patients should be rechecked in 1 year. preload and afterload. Hydralazine is a direct arterial vaso-
Those with stage 1 hypertension should be rechecked in 2 dilator and may worsen myocardial ischemia. The other
months. Those with stage 2 hypertension should have more medications listed would be acceptable second-line agents
urgent follow-up. The patient in this question has stage 1 for this patient. Of these, sodium nitroprusside is the best
hypertension and should therefore be rechecked within 2 studied.
months.
39. Answer a.
35. Answer d.
The presentation is highly suggestive of familial pheochro-
Angiotensin-converting enzyme inhibitors, angiotensin mocytoma. The first step is to establish the diagnosis by
receptor blockers, and direct renin inhibitors are con- measurement of free metanephrines in the plasma, which
traindicated in pregnancy because their use has been is the screening test of choice, especially if a familial dis-
associated with serious fetal abnormalities (limb defects, order is suspected. Plasma and urine catecholamines lack
lung hypoplasia, craniofacial deformities, and renal dys- diagnostic accuracy. Computed tomography or magnetic
plasia). Traditionally, fetal abnormalities were thought resonance imaging of the abdomen is appropriate as the
to develop after the first trimester; however, the latest initial test to locate the tumor since 90% are located in 1
information suggests that they can occur at any time or both adrenal glands and 98% are located in the abdo-
during pregnancy. Thus, it is recommended that these men. A search for the tumor should follow biochemical
drugs be avoided in hypertensive women who are sexu- confirmation of the diagnosis. Then an -blocker should
ally active and can become pregnant. Doxazosin is not be used as initial therapy. -Blocker monotherapy can be
recommended as an initial drug for the treatment of associated with a paradoxical increase in blood pressure,
hypertension. but a -blocker may be used to treat tachycardia that may
occur with adequate -blocker therapy.
36. Answer e.
40. Answer e.
Approximately 10% to 20% of patients with elevated office
BP have normal readings outside the clinic environment. Thiazide diuretics are associated with decreased urinary
This is referred to as office or white coat hypertension. excretion of calcium and are often used to treat calcium
These persons are at low risk and do not require drug ther- nephrolithiasis to reduce the risk of recurrent stone for-
apy. Self-measured BP consistently less than 130/80 mm mation. Thus, for this patient, chlorthalidone would be an
Hg confirms the diagnosis. If the self-measured systolic BP appropriate first agent to treat hypertension. Furosemide
is 130 to 140 mm Hg and the self-measured diastolic BP is associated with an increase in urinary calcium excretion
is 80 to 90 mm Hg, the diagnosis is best confirmed with and should not be considered if a patient has concomitant
noninvasive 24-hour ambulatory BP monitoring. Office calcium stone disease.

24 M AYO C L I N I C I N T E R N A L M E D I C I N E B OA R D R E VI EW: Q U E S T I O N S A N D A N S W E R S
SUGGESTED RE ADING Institute; Society for Vascular Nursing; TransAtlantic Inter-Society
Consensus; and Vascular Disease Foundation. Circulation. 2006 Mar
Baglin TP, Brush J, Streiff M; British Committee for Standards in 21;113(11):e463654.
Haematology Writing Group. Guidelines on use of vena cava filters. Br Kearon C, Kahn SR, Agnelli G, Goldhaber S, Raskob GE, Comerota
J Haematol. 2006 Sep;134(6):5905. Epub 2006 Jul 26. AJ; American College of Chest Physicians. Antithrombotic therapy
Boden WE, ORourke RA, Teo KK, Hartigan PM, Maron DJ, Kostuk for venous thromboembolic disease: American College of Chest
WJ, et al; COURAGE Trial Research Group. Optimal medical ther- Physicians Evidence-Based Clinical Practice Guidelines (8th Edition).
apy with or without PCI for stable coronary disease. N Engl J Med. Chest. 2008 Jun;133(6 Suppl):454S-545S. Erratum in: Chest. 2008
2007 Apr 12;356(15):150316. Epub 2007 Mar 26. Oct;134(4):892.
Ederle J, Dobson J, Featherstone RL, Bonati LH, van der Worp HB, de Kber L, Torp-Pedersen C, McMurray JJ, Gtzsche O, Levy S, Crijns
Borst GJ, et al; International Carotid Stenting Study investigators. H, et al; Dronedarone Study Group. Increased mortality after
Carotid artery stenting compared with endarterectomy in patients dronedarone therapy for severe heart failure. N Engl J Med. 2008
with symptomatic carotid stenosis (International Carotid Stenting Jun 19;358(25):267887. Erratum in: N Engl J Med. 2010 Sep
Study): an interim analysis of a randomised controlled trial. Lancet. 30;363(14):1384.
2010 Mar 20;375(9719):98597. Epub 2010 Feb 25. Erratum in: Levin AR, Spach MS, Boineau JP, Canent RV Jr, Capp MP, Jewett PH.
Lancet. 2010 Jul 10;376(9735):90. Nasser, H-C [corrected to Nahser, Atrial pressure-flow dynamics in atrial septal defects (secundum type).
H-C]. Circulation. 1968 Apr;37(4):47688.
European Carotid Surgery Trialists Collaborative Group. MRC European Mas JL, Chatellier G, Beyssen B, Branchereau A, Moulin T, Becquemin
Carotid Surgery Trial: interim results for symptomatic patients with JP, et al; EVA-3S Investigators. Endarterectomy versus stenting in
severe (7099%) or with mild (029%) carotid stenosis. Lancet. 1991 patients with symptomatic severe carotid stenosis. N Engl J Med. 2006
May 25;337(8752):123543. Oct 19;355(16):166071.
Hiratzka LF, Bakris GL, Beckman JA, Bersin RM, Carr VF, Casey McDermott MM, Greenland P, Liu K, Guralnik JM, Criqui MH, Dolan
DE Jr, et al; American College of Cardiology Foundation/ NC, et al. Leg symptoms in peripheral arterial disease: associated
American Heart Association Task Force on Practice Guidelines; clinical characteristics and functional impairment. JAMA. 2001 Oct
American Association for Thoracic Surgery; American College of 3;286(13):1599606.
Radiology; American Stroke Association; Society of Cardiovascular North American Symptomatic Carotid Endarterectomy Trial
Anesthesiologists; Society for Cardiovascular Angiography and Collaborators. Beneficial effect of carotid endarterectomy in symp-
Interventions; et al. 2010 ACCF/AHA/AATS/ACR/ASA/SCA/ tomatic patients with high-grade carotid stenosis. N Engl J Med. 1991
SCAI/SIR/STS/SVM guidelines for the diagnosis and management Aug 15;325(7):44553.
of patients with Thoracic Aortic Disease: a report of the American OToole JD, Reddy PS, Curtiss EI, Shaver JA. The mechanism of split-
College of Cardiology Foundation/American Heart Association ting of the second heart sound in atrial septal defect. Circulation. 1977
Task Force on Practice Guidelines, American Association for Dec;56(6):104753.
Thoracic Surgery, American College of Radiology, American Stroke Ringleb PA, Allenberg J, Bruckmann H, Eckstein HH, Fraedrich G,
Association, Society of Cardiovascular Anesthesiologists, Society Hartmann M, et al; SPACE Collaborative Group. 30 day results from
for Cardiovascular Angiography and Interventions, Society of the SPACE trial of stent-protected angioplasty versus carotid endart-
Interventional Radiology, Society of Thoracic Surgeons, and Society erectomy in symptomatic patients: a randomised non-inferiority trial.
for Vascular Medicine. Circulation. 2010 Apr 6;121(13):e266 Lancet. 2006 Oct 7;368(9543):123947. Erratum in: Lancet. 2006
369. Epub 2010 Mar 16. Erratum in: Circulation. 2010 Jul Oct 7;368(9543):1238.
27;122(4):e410. Watson HG, Baglin TP. Guidelines on travel-related venous thrombosis.
Hirsch AT, Haskal ZJ, Hertzer NR, Bakal CW, Creager MA, Halperin JL, Br J Haematol. 2011 Jan;152(1):314. Epub 2010 Nov 18.
et al; American Association for Vascular Surgery; Society for Vascular Wilson W, Taubert KA, Gewitz M, Lockhart PB, Baddour LM, Levison
Surgery; Society for Cardiovascular Angiography and Interventions; M, et al; American Heart Association Rheumatic Fever, Endocarditis,
Society for Vascular Medicine and Biology; Society of Interventional and Kawasaki Disease Committee; American Heart Association
Radiology; ACC/AHA Task Force on Practice Guidelines Writing Council on Cardiovascular Disease in the Young; American Heart
Committee to Develop Guidelines for the Management of Patients Association Council on Clinical Cardiology; American Heart
With Peripheral Arterial Disease; et al. ACC/AHA 2005 Practice Association Council on Cardiovascular Surgery and Anesthesia;
Guidelines for the management of patients with peripheral arterial Quality of Care and Outcomes Research Interdisciplinary Working
disease (lower extremity, renal, mesenteric, and abdominal aortic): Group. Prevention of infective endocarditis: guidelines from the
a collaborative report from the American Association for Vascular American Heart Association: a guideline from the American Heart
Surgery/Society for Vascular Surgery, Society for Cardiovascular Association Rheumatic Fever, Endocarditis, and Kawasaki Disease
Angiography and Interventions, Society for Vascular Medicine and Committee, Council on Cardiovascular Disease in the Young, and
Biology, Society of Interventional Radiology, and the ACC/AHA the Council on Clinical Cardiology, Council on Cardiovascular
Task Force on Practice Guidelines (Writing Committee to Develop Surgery and Anesthesia, and the Quality of Care and Outcomes
Guidelines for the Management of Patients With Peripheral Arterial Research Interdisciplinary Working Group. Circulation. 2007 Oct
Disease): endorsed by the American Association of Cardiovascular 9;116(15):173654. Epub 2007 Apr 19. Erratum in: Circulation.
and Pulmonary Rehabilitation; National Heart, Lung, and Blood 2007 Oct 9;116(15):e3767.

2 . C A R D I O L O GY Q U E S T I O N S A N D A N S W E R S 25
This page intentionally left blank
3.
GASTROENTEROLOGY AND HEPATOLOGY
QUESTIONS AND ANSWER S

QUESTIONS a. Oral prednisone


b. Bismuth subsalicylate
Multiple Choice (choose the best answer) c. Budesonide
d. Ciprofloxacin
e. Metronidazole
C O L O N A N D PA N C R E A S
4. A 55-year-old woman presented to the emergency
1. A 52-year-old man presents for colon cancer screening.
department with a 6-hour history of epigastric pain,
He is asymptomatic, and he states that he is not aware of
nausea, and vomiting. She has a history of hyperten-
any family member with a prior history of either colon
sion and hyperlipidemia. Her medications are aspirin
cancer or colon polyps. The patient undergoes colonos-
81 mg daily, lisinopril 10 mg daily, and simvastatin
copy and is found to have the following polyps and histo-
20 mg daily. Her heart rate is 105 beats per minute,
logic diagnoses: 2-mm cecal polyp and 4-mm transverse
her blood pressure is 100/60 mm Hg, and her tem-
polyp (tubular adenomas, low-grade dysplasia), 6-mm
perature is 36.4C. On clinical examination, the patient
sigmoid polyp (tubulovillous adenoma, low-grade dys-
has moderate epigastric tenderness and reduced bowel
plasia), and 3-mm rectal polyp (hyperplasia). If those 4
sounds. Laboratory test results include the following:
lesions were completely removed, what should you rec-
hemoglobin 13 g/dL, white blood cell count 18109/L,
ommend for ongoing colon surveillance?
amylase 2,563 U/L, lipase 5,637 U/L, aspartate amin-
a. Colonoscopy in 1 year
otransferase (AST) 350 U/L, alanine aminotransferase
b. Colonoscopy in 3 years
(ALT) 250 U/L, and bilirubin 1.1 mg/dL. What should
c. Colonoscopy in 5 years
you recommend next?
d. Yearly fecal occult blood test and colonoscopy in 5 years
a. Computed tomographic (CT) scan of the abdomen with
e. Computed tomographic colonography in 5 years
intravenous (IV) contrast medium
2. A 35-year-old woman with ulcerative colitis of 5 years b. Ultrasonography of the abdomen
duration presents with a red eye. She has no pain or c. Emergent endoscopic retrograde cholangiopancreatography
headache, and her vision is normal. Her stools have (ERCP)
been somewhat looser over recent weeks. She has been d. Plain abdominal radiography
taking mesalamine at her usual dosage of 2.4 g daily, e. IV fluid, bowel rest, and observation
but she has missed doses occasionally. She recently quit
5. A 50-year-old man presents to the emergency depart-
smoking. What is the most likely cause of her red eye?
ment with left lower abdominal pain. He has not had
a. Uveitis
fever or a change in bowel habit. He is eating with-
b. Giant cell arteritis
out difficulty. He has never had similar symptoms in
c. Episcleritis
the past. He has not undergone colon cancer screen-
d. Nicotine withdrawal and insomnia
ing. He has no comorbid conditions. On examina-
e. Viral conjunctivitis
tion, he has mild tenderness in the left lower abdomen
3. A 60-year-old woman presents with a 6-week history of without peritoneal signs. The white blood cell count
diarrhea. She has had 6 to 8 large-volume watery stools is 12.510 9/L. Computed tomography (CT) shows
daily. There has been no blood in the stool. She has not changes consistent with diverticulitis without abscess.
had fever or other systemic symptoms. Recently, she has What should you recommend as the next step for this
not taken antibiotics or changed medication. Clinical patient?
examination findings were normal. Laboratory test a. Hospital admission, bowel rest, and intravenous antibiotics
results were unremarkable. Stool examination for fecal b. Outpatient antibiotics
leukocytes was negative. On colonoscopy, the colon c. Colonoscopy
appeared normal. On biopsy, a thickened subepithe- d. Surgical consultation
lial collagen band was apparent. What initial therapy e. CT colonography
should you recommend?

27
E S O P H AGUS A N D S TO M AC H swallow, and aperistalsis. Which of the following is the
best next step?
6. A 63-year-old man undergoes esophagogastroduo-
a. Perform another EGD now.
denoscopy (EGD) and colonoscopy as part of an evalu-
b. Inject botulinum toxin into the lower esophageal sphincter.
ation for mild iron deficiency anemia. He denies having
c. Refer the patient to a surgeon for myotomy.
melena or hematochezia, and he has not lost weight.
d. Perform computed tomography of the chest.
The only medication he takes is ibuprofen for intermit-
e. Test for anticentromere antibodies.
tent joint aches, and he is otherwise healthy. His fam-
ily history is unremarkable. The EGD showed multiple
linear antral erosions and a 1.2-cm polyp in the body S M A L L B OWE L A N D I N T E S T I N E
of the stomach. Biopsies from throughout the stomach 9. A 49-year-old man presents with a 1-month history of
showed a chemical gastritis with no Helicobacter pylori, diarrhea. He has approximately 10 watery bowel move-
and biopsies of the polyp identified a tubular adenoma ments daily, and he has lost 4.5 kg while he has had diar-
with low-grade dysplasia. Small bowel biopsy findings rhea. Physical examination, complete blood cell count,
were normal. The colonoscopy showed only scattered and chemistry panel results were normal. A 72-hour
sigmoid diverticula. Which of the following is the best stool collection showed 2,000 g of stool with 10 g of fat
next step? per 24 hours. Stool electrolyte concentrations were as
a. No further testing follows: sodium 80 mEq/L and potassium 60 mEq/L.
b. Helicobacter pylori stool antigen test From these findings, what is the most likely cause of this
c. Endoscopic ultrasonography patients diarrhea?
d. EGD now with full removal of the polyp a. Whipple disease
e. EGD in 1 year b. Vasoactive intestinal peptide tumor
c. Celiac sprue
7. A 42-year-old man presents with hoarseness and inter-
d. Chronic pancreatitis
mittent heartburn symptoms. He notes that in the
e. Lactase deficiency
past he had heartburn caused by triggering foods sev-
eral times per month; symptoms now occur several 10. A 40-year-old woman who has iron deficiency anemia
times per week without obvious precipitants. He also began receiving oral iron therapy without response. She
notes that his voice is hoarse, which has affected his reports no gastrointestinal tract complaints or heavy
singing in the church choir. He reports no dysphagia, menses. She has a normal appetite and reports no weight
odynophagia, nausea, vomiting, early satiety, or weight loss. There is no family history of colon cancer or inflam-
loss. Proton pump inhibitor (PPI) therapy twice daily matory bowel disease. Fecal occult blood testing of the
before meals has provided only partial relief of symp- stool is negative. Which test should be performed next?
toms. Esophagogastroduodenoscopy (EGD) showed a. Measurement of serum IgA and IgG tissue transglutaminase
several cystic fundic gland polyps but was otherwise antibodies
normal. Which of the following is the best next step? b. Upper endoscopy with small bowel biopsies
a. An H2 receptor blocker at bedtime c. Capsule endoscopy
b. Gastric scintigraphy with a solid meal d. Small bowel follow-through
c. A 24-hour ambulatory pH probe e. Stool evaluation for ova and parasites
d. Video fluoroscopic swallowing test
e. Esophageal manometry
11. A 55-year-old white man presents with a 6-month
history of weight loss and arthralgias. He reports no
8. A 78-year-old man presents for evaluation of new swal- anorexia but has had diarrhea with up to 4 loose stools
lowing problems. He states that for the past 3 months, daily. He has migratory pain involving the shoulders,
food gets stuck when he swallows, although the food elbows, and knees. On physical examination, he has
eventually passes spontaneously. He also notes dif- skin hyperpigmentation and oculomasticatory myo-
ficulty swallowing liquids and senses fullness in his rhythmia. A 72-hour stool collection shows 32 g of fat
chest for a prolonged period after drinking any bever- per 24 hours. Which test is most likely to be positive or
age. He reports regurgitation of fluid into the back of diagnostic for this patient?
his throat. He has a 40pack-year smoking history, and a. Measurement of serum IgA and IgG tissue transglutaminase
a long-standing history of reflux, well-controlled with antibodies
PPI therapy as needed. He has lost 6.8 kg over the past b. Measurement of serum IgA and IgG deamidated gliadin
few months. He had esophagogastroduodenoscopy antibodies
(EGD) 1 year ago to screen for Barrett esophagus; c. Small bowel biopsy showing enlarged villi with periodic
results were negative. With his new symptoms, he now acid-Schiff (PAS)-positive macrophages
undergoes a barium esophagram, which shows a birds d. Small bowel biopsy showing villous atrophy and crypt
beak narrowing at the distal esophagus; esophageal hyperplasia
manometry shows an elevated pressure in the lower e. Duodenal aspirates showing more than 100,000
esophageal sphincter, which does not relax after a colony-forming units (CFU)/mL

28 M AYO C L I N I C I N T E R N A L M E D I C I N E B OA R D R E VI EW: Q U E S T I O N S A N D A N S W E R S
12. A 45-year-old woman presents with abdominal discom- a. Bacterial overgrowth frequently develops in patients with
fort and diarrhea. Almost every day, she has variable ileal resections.
abdominal discomfort with up to 3 or 4 watery stools. b. Bile acidbinding resins cause diarrhea.
She has associated abdominal bloating and flatulence. c. The bile acid binder has further depleted the bile acid pool.
Eating and stress aggravate symptoms, and abdomi- d. Bile acid binders may cause vitamin deficiencies.
nal discomfort is relieved by defecation. She reports e. The patient most likely increased her consumption of poorly
no anorexia, weight loss, or blood in the stool. You digestible carbohydrates.
suspect irritable bowel syndrome (IBS). Laboratory
study results were normal for complete blood cell
count, erythrocyte sedimentation rate, and C-reactive
LIVER
protein. Fecal leukocytes are present, but culture for
enteric pathogens and testing for Clostridium difficile 16. A 35-year-old man who comes for a preoperative assess-
are negative. Which intervention is appropriate at this ment before hernia repair is mildly jaundiced. Other
time? than a symptomatic inguinal hernia, he feels well. Ten
a. Loperamide years ago, the patient had a brief period of illicit drug
b. Rifaximin use. He currently drinks 3 alcoholic beverages daily. He
c. Colonoscopy with biopsies takes niacin for hyperlipidemia. Physical examination is
d. Reassuring and counseling the patient notable only for mild jaundice. Laboratory test results
e. Stool-bulking agents include the following: hemoglobin 15 g/dL, platelet
count 213109/L, aspartate aminotransferase (AST)
13. A 70-year-old woman with a history of mitral valve 23 U/L, alanine aminotransferase (ALT) 22 U/L, alka-
disease and atrial fibrillation awakens from sleep with line phosphatase 83 U/L, total bilirubin 2.4 mg/dL,
a sudden onset of sharp central abdominal pain and and direct bilirubin 0.2 mg/dL. Which of the following
the sudden urge to have a bowel movement. An explo- is the most likely diagnosis?
sive stool is passed without blood; however, the pain a. Hepatitis C
persists. On examination, her abdomen is soft, with b. Hepatitis B
minimal tenderness and no rebound or guarding. The c. Niacin-induced liver disease
physician notes that her pain is out of proportion to d. Alcoholic hepatitis
the findings on physical examination. Laboratory test e. Gilbert syndrome
data are normal, including results for a complete blood
cell count, blood chemistry panel, amylase, lipase, and 17. A 60-year-old woman presents with increasing values
serum lactate. An electrocardiogram shows normal on liver tests. Three months ago, at a general assess-
sinus rhythm. A computed tomographic (CT) scan of ment, the level of her alanine aminotransferase (ALT)
the abdomen shows a nonspecific gas pattern. What was 63 U/L and she had hyperlipidemia. Evaluation
should be the next step in management? of the elevated liver test results was negative except for
a. Observation steatosis noted on ultrasonography. She began tak-
b. A vascular imaging study ing simvastatin. One month after starting simvastatin,
c. Laparotomy her ALT is 85 U/L. She continues to feel well and her
d. Laparoscopy examination is unremarkable except for obesity. Her
e. Stool studies only other medication is metformin. She drinks 1 glass
of wine every 6 months. Which of the following would
14. A 55-year-old woman who had extensive abdominal you advise at this time?
radiotherapy 5 years earlier for endometrial cancer has a. Stop all alcohol intake.
chronic symptoms of postprandial bloating and peri- b. Undergo liver biopsy.
umbilical discomfort. Over the past several weeks she c. Stop use of simvastatin.
has had persistent diarrhea. Results of routine stool d. Stop use of metformin.
studies for infectious agents are negative. Which of the e. Follow with serial monitoring of ALT.
following treatments would most likely result in sus-
tained improvement in her diarrhea? 18. A 49-year-old woman is referred because of an elevated
a. Oral mesalamine alkaline phosphatase level discovered during evaluation
b. Bile acidbinding resin for fatigue. She has had no previous blood tests. Physical
c. Ciprofloxacin examination is notable for xanthelasmas. Laboratory
d. A proton pump inhibitor study results include the following: alkaline phosphatase
e. Low-fat diet 500 U/L, alanine aminotransferase 73 U/L, and total
cholesterol 300 mg/dL; bilirubin, international nor-
15. A 35-year-old woman with Crohn disease and a prior malized ratio, and albumin are all normal. Which of the
ileal resection receives a bile acidbinding agent for following tests is most likely to establish a diagnosis?
treatment of chronic diarrhea. However, the diarrhea a. Antimitochondrial antibody testing
worsens. What is the most likely explanation? b. Magnetic resonance cholangiopancreatography

3. G A S T R O E N T E R O L O GY A N D H E PATO L O GY Q U E S T I O N S A N D A N S W E R S 29
c. Antinuclear antibody testing 21. A 53-year-old man is admitted from the emergency
d. Antismooth muscle antibody testing department with upper gastrointestinal tract bleeding.
e. Angiotensin-converting enzyme level measurement He has a history of chronic hepatitis C and cirrhosis but
felt well until early this morning when he had hematem-
19. A 43-year-old alcoholic man is referred with a 3-month
esis. Esophagogastroduodenoscopy in the emergency
history of abdominal distention, leg edema, and dysp-
department showed large esophageal varices, which were
nea. Physical examination is notable for mild jugular
ligated. He currently feels weak but has had no other
venous distention, distant heart sounds, marked ascites,
complaints. Physical examination findings include the
and leg edema. Chest radiography shows mild cardio-
following: blood pressure 120/60 mm Hg, heart rate
megaly. Laboratory test results are as follows: total bili-
82 beats per minute, alert and oriented, splenomegaly,
rubin 1.5 mg/dL, albumin 3.5 g/dL, and international
and no ascites or edema. Laboratory study results
normalized ratio 1.2. Liver Doppler ultrasonography
are as follows: hemoglobin 9.2 g/L, platelet count
shows ascites, a coarse echotexture of the liver, and pat-
65109/L, aspartate aminotransferase 84 U/L, alanine
ent hepatic and portal veins. In the abdominal fluid, the
aminotransferase 75 U/L, total bilirubin 1.2 mg/dL,
protein level is 3.9 g/dL and the albumin level is 2.1 g/dL.
albumin 3.0 g/dL, and international normalized ratio
Which of the following would you advise next?
(INR) 1.2. Abdominal ultrasonography shows no liver
a. Hepatic venography
mass, a patent portal vein, splenomegaly, and no ascites.
b. Transjugular intrahepatic portosystemic shunt
Which of the following would you advise now?
c. Liver biopsy
a. Transjugular intrahepatic portosystemic shunt
d. Echocardiography
b. Norfloxacin
e. Laparoscopy
c. Transfusion of 2 units of packed red blood cells
20. A 48-year-old woman is referred with a 3-week history d. Transfusion of 2 units of fresh frozen plasma
of fatigue and vague right upper quadrant pain. She e. Pegylated interferon and ribavirin
drinks 2 glasses of wine daily and occasionally more
22. A 33-year-old Asian woman receives a diagnosis of
on weekends. Her medical history is notable for hypo-
non-Hodgkin lymphoma, and chemotherapy is advised.
thyroidism. She reports no prior blood transfusions,
She has a history of hepatitis B without complications.
illegal drug use, or a family history of liver disease. Her
Her mother also had hepatitis B. On examination,
only medications are ibuprofen 400 mg up to 3 times
the patient has cervical adenopathy consistent with
daily, acetaminophen 500 mg up to 4 times daily, and
lymphoma and no stigmata of chronic liver disease.
thyroxine. Physical examination findings are normal.
Laboratory test results are as follows: platelet count
Laboratory study results are as follows: aspartate ami-
348109/L, alanine aminotransferase 17 U/L, total bil-
notransferase (AST) 740 U/L, alanine aminotrans-
irubin 0.6 mg/dL, hepatitis B surface antigen positive,
ferase (ALT) 900 U/L, alkaline phosphatase 115 U/L,
hepatitis B e antigen (HBeAg) negative, antibody to
and -globulin 5.4 g/dL. Bilirubin and albumin levels
HBeAg positive, IgG antibody to hepatitis B core anti-
and the international normalized ratio (INR) are nor-
gen positive, and hepatitis B virus (HBV) DNA unde-
mal. Serologic studies are negative for hepatitis A, B,
tectable. Which of the following should you advise at
and C. The patient is observed, and 2 weeks later her
this time?
laboratory test results are as follows: AST 756 U/L,
a. Hepatitis B vaccination
ALT 945 U/L, total bilirubin 1.4 mg/dL, and INR
b. Surveillance for hepatocellular carcinoma
1.2. Ultrasonography shows a coarse echotexture to the
c. Lamivudine
liver, borderline splenomegaly, and gallbladder stones
d. Pegylated interferon
without bile duct dilatation. Two weeks later the ALT
e. Nothing further at this time except chemotherapy
is unchanged, but the bilirubin is 2.0 mg/dL. Which of
the following is the most likely diagnosis?
a. Autoimmune hepatitis
b. Ibuprofen hepatotoxicity
c. Acetaminophen hepatotoxicity
d. Alcoholic hepatitis
e. Cholelithiasis

30 M AYO C L I N I C I N T E R N A L M E D I C I N E B OA R D R E VI EW: Q U E S T I O N S A N D A N S W E R S
ANSWER S

1. Answer b. without difficulty, outpatient management may be pur-


sued. If there were evidence of a complication (eg, abscess),
Hyperplastic polyps do not infer an increased risk of colon
or if the patient could not tolerate oral intake, hospital-
cancer. Therefore, the rectal lesion is of no clinical significance.
ization would be necessary. Since this is the patients first
The patient had 3 clinically significant polyps. Although each
episode of diverticulitis, and it is uncomplicated, surgery is
lesion was smaller than 1 cm, the interval to the next colonos-
not indicated. Colonoscopy is contraindicated with acute
copy would be 3 years because there were 3 or more polyps
diverticulitis, but since this patient has not undergone
and because 1 of the lesions had a villous component.
colon cancer screening, it would be reasonable to perform
2. Answer c. colonoscopy 2 to 4 weeks after the acute symptoms have
resolved.
A patient with inflammatory bowel disease who has a red
eye most likely has either episcleritis or uveitis. Uveitis is 6. Answer d.
associated with pain in the eye. Episcleritis is typically pain-
less. Giant cell arteritis typically does not manifest with red This patient has iron deficiency anemia that is likely due to
eye and is usually associated with headache or visual loss. the antral erosions that appear to have been induced by the
nonsteroidal anti-inflammatory drug. However, he also has
3. Answer b. a gastric polyp that is a gastric adenoma; similar to colonic
The clinical presentation and the colonoscopic and his- adenomas, gastric adenomas are deemed premalignant and
tologic findings are typical of microscopic colitis (in this require full endoscopic removal. The patient should have
case, collagenous colitis). Initial therapy for mild to mod- another EGD now with polypectomy since the polyp was
erate disease (36 bowel movements daily) is usually with simply biopsied and not fully removed during his previ-
either an antidiarrheal, such as loperamide hydrochloride, ous EGD. To do no further testing would be inadequate
or bismuth subsalicylate. In more severe cases (>6 bowel because this polyp could continue to grow and progress
movements daily), budesonide may be considered for ini- to gastric cancer. While gastric erosions can be caused by
tial therapy. Prednisone would be used only in cases of H pylori, this patients histologic examination was negative
microscopic colitis that did not respond to the aforemen- for H pylori, and he was not taking any medications that
tioned therapies. could lead to false-negative testing for H pylori (proton
pump inhibitor, antibiotics, etc); therefore, further test-
4. Answer e. ing for H pylori would not be needed. If the gastric polyp
The patient has acute pancreatitis as evidenced by the had been malignant, endoscopic ultrasonography would
clinical presentation and the elevated levels of amylase and be needed to assess the depth of invasion and to complete
lipase. The cause is most likely gallstone disease (AST and locoregional staging, but that is not needed for an ade-
ALT were moderately elevated). The most important initial noma of this size. To wait to repeat the EGD for 1 year is
step in the management of patients with acute pancreatitis not recommended since the polyp could continue to grow,
is to ensure excellent hydration to optimize pancreatic per- progress to cancer, or cause bleeding, all of which could be
fusion and thereby decrease the risk of pancreatic necrosis. prevented by removal now.
The patient does not have evidence of cholangitis (absence
7. Answer c.
of fever, pain localizing to the right upper quadrant, and
jaundice), so there is no indication for emergent ERCP. This patient has features suggestive of gastroesophageal
CT scan of the abdomen with IV contrast medium may be disease. He has not only progressive heartburn but also the
indicated later in the clinical course to evaluate for pancre- extra-esophageal feature of hoarseness. Since he has not
atic necrosis, but it is not indicated at initial presentation. fully improved with PPI therapy and the EGD was negative
Ultrasonography of the abdomen is a reasonable step but for esophagitis, a 24-hour ambulatory pH probe would be
only after IV fluid resuscitation has been initiated. helpful to document whether he truly has acid reflux and
whether there is symptom correlation. When this test incor-
5. Answer b.
porates impedance testing, symptoms can be correlated with
This patient is presenting with a first episode of uncom- acid-mediated reflux or with nonacid-mediated reflux (eg,
plicated diverticulitis. Since he is tolerating oral intake seen with someone receiving PPI therapy). At this point,

31
establishing a diagnosis would be helpful to direct future chronic pancreatitis, and lactase deficiency) are causes of
therapy rather than adding additional acid suppression with osmotic diarrhea and are therefore incorrect.
an H2 receptor blocker at bedtime. While patients with
10. Answer b.
delayed gastric emptying can present with worsening acid
reflux, this patient does not report having any of the other This woman has iron deficiency without evidence of gas-
symptoms that commonly occur with gastroparesis (nausea, trointestinal tract or menstrual blood loss, which suggests
vomiting, early satiety), making it less likely at this point. malabsorption of iron. The most common manifestation
Although hoarseness may result from neuromuscular weak- of celiac disease is iron deficiency anemia. Iron is mainly
ness, which can cause oropharyngeal dysphagia, this patient absorbed in the duodenum. Celiac disease preferentially
does not report swallowing difficulties that would suggest affects the proximal small bowel, interfering with iron
that a video swallowing test would be of value. Esophageal uptake. Therefore, upper endoscopy with small bowel
manometry can be useful for evaluating motility disorders biopsies should be performed to evaluate for celiac disease.
of the esophagus, such as achalasia, but would not have a role A small bowel series or capsule endoscopy may suggest the
here in the evaluation of persistent gastroesophageal reflux diagnosis of celiac disease but does not provide tissue for
diseaserelated symptoms in the absence of dysphagia. diagnosis. Positive serologic testing (tissue transglutami-
nase antibodies) supports the diagnosis of celiac disease
8. Answer a.
but, if negative, does not exclude the diagnosis in this
This patient has clinical, radiographic, and manometric fea- patient with a high pretest probability of celiac disease. In
tures consistent with achalasia; however, given his age, the a patient with iron deficiency and no gastrointestinal tract
rapid onset of his symptoms, and the weight loss, pseudo- symptoms, stool evaluation for ova and parasites would be
achalasia due to malignancy needs to be considered and low yield. Furthermore, a parasitic infection (eg, strongy-
ruled out. An EGD should be performed now to rule out loidiasis) would likely be detected on small bowel biopsy.
esophageal or gastric cardia malignancy because his most
11. Answer c.
recent EGD was 1 year ago (before the onset of his current
symptoms), and an early lesion could have been missed. The diagnosis of Whipple disease should be suspected with
To refer this patient for any therapy targeted at achalasia, the combination of steatorrhea, weight loss, and migratory
such as botulinum toxin injection into the lower esophageal arthralgias in a middle-aged white man. Whipple disease
sphincter or myotomy, would be premature until EGD has may cause central nervous system involvement manifesting
been performed to rule out cancer. If a patient with clinical with the finding of oculomasticatory myorhythmia in 20%
features of pseudoachalasia has negative findings on EGD, of patients. Oculomasticatory myorhythmia is pathog-
imaging of the chest may then be considered, especially nomonic for Whipple disease and consists of continuous
with a smoking history. A pulmonary or mediastinal malig- rhythmic jaw contractions that are synchronous with dis-
nancy can infiltrate the lower esophageal sphincter complex sociated pendular vergence oscillations. Whipple disease
and cause pseudoachalasia symptoms; however, this testing occurs predominantly in middle-aged white men and is
should not take place before another EGD is performed, caused by chronic infection with Tropheryma whipplei. In
allowing direct mucosal inspection. Anticentromere anti- most patients with Whipple disease, the intestinal tract is
bodies can be seen in CREST syndrome associated with involved regardless of the presence or absence of gastro-
scleroderma; similar to patients with achalasia, these intestinal tract symptoms. Thus, the primary diagnostic
patients may also have dysphagia to solids and liquids and approach to a patient with clinically suspected Whipple
are at increased risk of esophageal cancer. However, patients disease is upper endoscopy with mucosal biopsy. Intestinal
with esophageal involvement with scleroderma typically biopsies show the characteristic findings of macrophages
have a decreased lower esophageal sphincter tone, which is with PAS-staining particles, which indicate the presence
the opposite of what is seen in this case. of T whipplei bacilli. Polymerase chain reaction assays may
assist in the detection of T whipplei DNA in the intestinal
9. Answer b.
mucosa. Tissue transglutaminase antibodies, deamidated
This patient has a stool osmotic gap (290 2 [80 + 60]) gliadin antibodies, and small bowel biopsy with villous
of less than 50 mOsm/kg, suggesting a secretory cause of atrophy and crypt hyperplasia are characteristic of celiac
diarrhea. Causes of secretory diarrhea include toxins from disease and do not fit this clinical scenario. Duodenal aspi-
cholera and enterotoxigenic Escherichia coli and peptides rates showing more than 100,000 CFU/mL are diagnostic
produced from endocrine tumors (vasoactive intestinal of small intestinal bacterial overgrowth, which can cause
peptide). The distinction between secretory and osmotic steatorrhea and arthralgias but would not manifest with
diarrhea helps in the differential diagnosis and evaluation oculomasticatory myorhythmia.
of patients with chronic diarrhea. The 2 main methods to
12. Answer c.
help distinguish between secretory and osmotic diarrhea
are by calculating the stool osmotic gap and assessing the This patient has signs and symptoms consistent with
response to fasting. Secretory diarrhea will not decrease IBS. The only test required for patients who have typical
substantially during a fast, whereas osmotic diarrhea will. diarrhea-predominant IBS symptoms and no alarm fea-
The other answer choices (Whipple disease, celiac sprue, tures is serologic testing for celiac disease. However, this

32 M AYO C L I N I C I N T E R N A L M E D I C I N E B OA R D R E VI EW: Q U E S T I O N S A N D A N S W E R S
patient has fecal leukocytes. This finding suggests colonic 17. Answer e.
inflammation and warrants further investigation with
The patient presents with a mildly increasing ALT 1 month
colonoscopy and biopsy to evaluate for inflammatory bowel
after she began taking simvastatin. She almost certainly has
disease or microscopic colitis. Reassurance, antidiarrheals,
nonalcoholic fatty liver disease. It is now well documented
and stool-bulking agents are therapies to consider for the
that patients with nonalcoholic fatty liver disease do not
patient with IBS. Rifaximin is a nonabsorbable antibiotic
have an increased risk of liver toxicity with statins. Mild
used to treat travelers diarrhea, recurrent hepatic encephal-
increases in aminotransferases after starting a statin are
opathy, and small intestinal bacterial overgrowth. Although
common but are nearly always transient; consequently,
rifaximin was recently found to alleviate IBS symptoms, it
serial monitoring of ALT is the correct answer. Patients
is not approved by the US Food and Drug Administration
with nonalcoholic fatty liver disease have an increased
for this indication, and this patient with fecal leukocytes
risk of death, although much of that increase in mortality
requires further evaluation with colonoscopy.
is due to cardiovascular disease rather than liver disease.
13. Answer b. Therefore, control of cardiovascular risk factors is an impor-
tant management issue for these patients. The amount of
This patient has the classic clinical history for superior
alcohol consumed by the patient is irrelevant. Unless there
mesenteric artery embolus. Since the CT scan of the abdo-
would be other changes in the patients clinical condition,
men was negative, selective mesenteric angiography should
liver biopsy is not required but could be considered if there
be the next step in management. Mesenteric ischemia
are further increases in ALT. Metformin is a rare cause of
should be diagnosed promptly. The mortality rate exceeds
elevated liver test results; therefore, metformin may be
80%, even if the embolus is removed and all infarcted
continued. Simvastatin should be discontinued only if the
bowel is resected, because decreased splanchnic blood
ALT level increases to 3 to 5 times the upper limit of the
flow may initiate persistent and irreversible mesenteric
reference range. (See Torres and Harrison, and Chalasani
vasoconstriction.
in the Suggested Reading list.)
14. Answer c.
18. Answer a.
Long-term radiotherapy injury to the small bowel may man-
ifest as impaired motility or stricture, as suggested by the The presence of xanthelasmas and a cholestatic liver pro-
patients chronic stable symptoms. Patients with impaired file in a woman of this age is highly suggestive of primary
motility or stricture are predisposed to small bowel bacte- biliary cirrhosis (PBC), and antimitochondrial antibody
rial overgrowth due to stasis. Treatment with a course of testing should be done. Magnetic resonance cholang-
antibiotics, such as ciprofloxacin, would be the therapy most iopancreatography would be useful if biliary obstruction
likely to result in sustained improvement of the diarrhea. were likely. The absence of pain, history of ulcerative coli-
tis (associated with primary sclerosing cholangitis), or an
15. Answer c. elevation in the bilirubin level makes biliary obstruction
Diarrhea after an ileal resection is usually caused either less likely than PBC. Antinuclear and antismooth muscle
by bile acid malabsorption or bile saltinduced colonic antibodies are used to diagnose autoimmune hepatitis,
secretion (resection 100 cm) or by bile salt malabsorp- which elevates predominantly aminotransferases rather
tion with bile salt pool depletion and fatty acidinduced than alkaline phosphatase. An angiotensin-converting
colonic secretion (resection >100 cm). The clinical finding enzyme level is not necessary in the absence of other clini-
of worsening diarrhea with treatment with binding resins cal features of sarcoidosis. (See Kaplan and Gershwin in
suggests further bile salt depletion. the Suggested Reading list.)
16. Answer e. 19. Answer d.
This patient has an indirect hyperbilirubinemia with The serum-ascites albumin gradient is 1.4, which suggests
normal levels of liver enzymes and without evidence portal hypertension. The fluid protein level greater than
of hemolysis. The most likely diagnosis is Gilbert syn- 2.5 g/dL is suggestive of hepatic venous outflow obstruc-
drome, which occurs in about 5% of the general popula- tion. The hepatic veins are patent, and the most likely cause
tion. Further diagnostic workup is unnecessary. The prior for the ascites and the other symptoms is heart failure, per-
history of illicit drug use puts the patient at risk for hepa- haps due to alcoholic cardiomyopathy. Echocardiography
titis C and hepatitis B. He should be tested, although should be the next test. Transjugular intrahepatic porto-
it would be unusual for him to have viral hepatitis with systemic shunt is used to treat refractory ascites associated
normal liver enzyme levels, and Gilbert syndrome is with cirrhosis. Liver biopsy might be useful later, but it is
much more common. Patients with alcoholic hepatitis invasive and should not be used unless noninvasive tests
can have hyperbilirubinemia, but they generally have do not lead to a diagnosis. Laparoscopy should be reserved
abnormal liver tests, with the AST being higher than for a suspicion of peritoneal carcinomatosis or infection,
the ALT. Niacin-induced liver disease is accompanied by both of which would have a serum-ascites albumin gra-
elevated liver enzymes. (See Kamath in the Suggested dient less than 1.1. (See Runyon et al in the Suggested
Reading list.) Reading list.)

3. G A S T R O E N T E R O L O GY A N D H E PATO L O GY Q U E S T I O N S A N D A N S W E R S 33
20. Answer a. 22. Answer c.
The combination of ALT that is persistently abnormal to Patients with hepatitis B who need immunosuppressive
this degree and hypergammaglobulinemia in a woman therapy are at risk for reactivation of disease and should
with a history of thyroid disease and no other identifiable receive hepatitis B treatment. An oral nucleoside or nucle-
cause for liver disease is most suggestive of autoimmune otide analogue, such as lamivudine, is preferred because
hepatitis. Determination of antinuclear and antismooth of the reliable antiviral effect and lack of toxicity. Ideally,
muscle antibodies and a liver biopsy would be the next hepatitis B treatment is started 2 weeks before initiation
steps in management. Toxicity from ibuprofen is rare, and of chemotherapy and continued for several months after
acetaminophen causes an acute, usually marked, increase completion of the lymphoma treatment. Hepatitis B vac-
in aminotransferases. Alcoholic hepatitis almost never cination is not useful if the patient already has hepatitis B.
increases aminotransferases to more than 400 U/mL. Surveillance for hepatocellular carcinoma (HCC) is
Gallbladder stones can cause abnormalities in liver ducts advised in the following hepatitis B patients: patients who
either through severe inflammation or through passage have cirrhosis, Asian women older than 50 years, Asian
of a common bile duct stone. The patient does not have men older than 40 years, Africans older than 20 years,
clinical features of inflammation, and a common bile duct patients with a family history of HCC, and patients with
stone causes transient aminotransferase elevations and persistently elevated liver test results and high HBV DNA
severe abdominal pain, neither of which are present in this levels. This patient does not meet any of those criteria. (See
patient. (See Czaja et al in the Suggested Reading list.) Lok et al in the Suggested Reading list.)
21. Answer b.
Patients with cirrhosis who are admitted with gastro-
intestinal tract bleeding should receive prophylactic SUGGESTED RE ADING
antibiotics even if there is no ascites. Oral norfloxacin is
Chalasani N. Statins and hepatotoxicity: focus on patients with fatty liver.
probably sufficient, although a recent trial suggested that Hepatology. 2005 Apr;41(4):6905.
a third-generation cephalosporin may be more effective if Czaja AJ, Freese DK; American Association for the Study of Liver Disease.
there is a high prevalence of quinolone resistance. Much of Diagnosis and treatment of autoimmune hepatitis. Hepatology. 2002
the recent reduction in mortality related to variceal bleed- Aug;36(2):47997.
ing is probably due to the increasing use of prophylactic Kamath PS. Clinical approach to the patient with abnormal liver test
results. Mayo Clin Proc. 1996 Nov;71(11):108994; quiz 10945.
antibiotics in patients with cirrhosis and gastrointestinal Kaplan MM, Gershwin ME. Primary biliary cirrhosis. N Engl J Med.
tract bleeding. Transjugular intrahepatic portal systemic 2005 Sep 22;353(12):126173. Erratum in: N Engl J Med. 2006 Jan
shunt is not necessary unless bleeding cannot be controlled 19;354(3):313.
with endoscopy or medical therapy. Although the hemo- Lok AS, McMahon BJ; Practice Guidelines Committee, American
globin level should be carefully monitored, the patient is Association for the Study of Liver Diseases (AASLD). Chronic
hepatitis B: update of recommendations. Hepatology. 2004
hemodynamically stable and does not need transfusion Mar;39(3):85761.
now. Overtransfusion, when hemoglobin is greater than 8 Runyon BA; Practice Guidelines Committee, American Association
to 9 g/dL, can precipitate recurrence of bleeding and fluid for the Study of Liver Diseases (AASLD). Management of
overload. Fresh frozen plasma is also not necessary since adult patients with ascites due to cirrhosis. Hepatology. 2004
the INR is relatively normal. A decision about pegylated Mar;39(3):84156.
Sanyal AJ, Bosch J, Blei A, Arroyo V. Portal hypertension and its complica-
interferon and ribavirin treatment of hepatitis C should be tions. Gastroenterology. 2008 May;134(6):171528.
deferred until the patient has recovered from this bleeding Torres DM, Harrison SA. Diagnosis and therapy of nonalcoholic steato-
episode. (See Sanyal et al in the Suggested Reading list.) hepatitis. Gastroenterology. 2008 May;134(6):168298.

34 M AYO C L I N I C I N T E R N A L M E D I C I N E B OA R D R E VI EW: Q U E S T I O N S A N D A N S W E R S
4.
PULMONARY DISEASES QUESTIONS AND ANSWER S

QUESTIONS a. Haloperidol
b. Lorazepam
Multiple Choice (choose the best answer) c. Hydromorphone
d. Midazolam
e. Fentanyl
CRITICAL CARE MEDICINE
3. A 58-year-old man comes to the emergency depart-
1. A 59-year-old man underwent a radical prostatectomy
ment with clumsiness of his right (dominant) hand that
for prostate cancer 24 hours ago. Since hospital admis-
began abruptly 3 hours earlier. He has hypertension,
sion, he has received a total of 3.5 L of intravenous iso-
for which he takes a -blocker. His temperature is 37C,
tonic crystalloid and has had a total urine output of 2.5 L.
his heart rate is 90 beats per minute, his respirations are
The hospital physician on duty examined him earlier
16 per minute, and his blood pressure is 145/90 mm Hg.
this evening for evaluation of substernal chest pain.
Oxygen saturation as measured by pulse oximetry is
The patients electrocardiogram (ECG) was normal. He
97% with room air. Physical examination reveals an
received intravenous furosemide and oral nitroglycerin
intact sensorium, no obvious cranial nerve deficits, 3/5
and had 1.0 L of urine output. He became symptomati-
motor strength in the right upper arm with inability
cally worse, so he was transferred to the intensive care
to perform repetitive hand movements (ataxia), and
unit for further evaluation. When you initially examine
4/5 motor strength in the right lower leg. No sensory
the patient, his blood pressure is 150/100 mm Hg, his
deficits are noted. Emergent computed tomography of
heart rate is 115 beats per minute, his respiration rate
the head does not show a stroke. All screening labora-
is 24 breaths per minute, and his temperature is 37C.
tory study results are normal except for a serum choles-
He reports chest pain and shortness of breath. There is
terol level of 250 mg/dL. It is now 4 hours after onset
no jugular vein distention. You hear bibasilar crackles
of symptoms. Which of the following is the definitive
posteriorly and a harsh systolic murmur across the pre-
treatment?
cordium. His extremities are well perfused with good
a. Alteplase
pulses. An ECG shows sinus tachycardia and delayed
b. Aspirin
precordial transition, but no other changes are noted.
c. Clopidogrel
While you wait for results of additional diagnostic
d. Continuous-infusion eptifibatide
studies, which of the following should you administer
e. Continuous-infusion heparin
now?
a. Furosemide
4. A 52-year-old man is admitted to the hospital because
b. Metoprolol
of acute dyspnea and a syncopal episode during which
c. Nitroglycerin
he fell. He is hypotensive and requires intubation and
d. Aspirin
norepinephrine to maintain adequate blood pressure
e. Heparin
and oxygenation. A computed tomographic (CT) scan
2. A 32-year-old man underwent surgical repair and fixa- of the chest shows a saddle pulmonary embolism, and
tion of multiple traumatic orthopedic injuries 3 days the patient is administered tissue plasminogen activa-
ago. He also has bilateral lung contusions and has tor (tPA) in the emergency department. An echocar-
required mechanical ventilation. He did not have any diogram shows right ventricular dilatation and failure.
head injuries. His condition is improving, and physi- Ten hours later, the norepinephrine dosage has been
ologically he is ready to begin being weaned from the incrementally increased from 25 to 55 mcg per min-
ventilator. His oxygenation is good. However, when the ute. Oxygen saturation as measured by pulse oximetry
propofol infusion is decreased, he manifests a severe, is 98% with a fraction of inspired oxygen of 0.80. His
agitated delirium with ventilator dyssynchrony and an heart rate is 125 beats per minute, his blood pressure is
inability to follow commands. Neurologic examination 100/50 mm Hg, and his central venous pressure is 1 to
findings are otherwise normal. Laboratory test results 2 mm Hg. The hemoglobin level has decreased from
are normal, computed tomography of the head is nor- 13 g/dL on admission to 5 g/dL. Activated partial
mal, and the electrocardiogram is normal. Which of the thromboplastin time is 65 seconds. A formal reading
following should you order now? of the admission CT scan shows left rib fractures in

35
addition to pulmonary embolism. A subsequent chest D I FF US E LU N G D I S E A S E A N D O C CU PAT I O NA L
radiograph is clear. A subsequent echocardiogram LU N G D I S E A S E
shows a poorly contracting right ventricle. Gastric aspi-
8. A 25-year-old female nonsmoker presents with a 1-week
rate is clear, and the stool is negative for occult blood. In
history of mild cough and dyspnea after a flulike illness
addition to transfusion, which of the following should
with fever, arthralgias, and tender erythematous lesions
you recommend?
on the anterior aspects of the legs. She has no history
a. Perform a CT scan of the abdomen.
of asthma or significant medical illnesses. No environ-
b. Repeat the dose of tPA.
mental or occupational high-risk exposures are noted.
c. Stop the heparin and place an inferior vena cava (IVC)
Examination reveals clear lung fields and no other
filter.
abnormalities. A chest radiograph shows prominent
d. Continue heparin and place an IVC filter.
bilateral hilar lymphadenopathy without parenchymal
e. Perform a surgical embolectomy.
infiltrates. What should you do next?
5. A 25-year-old man is admitted to the intensive care unit a. Set up blood cultures.
(ICU) with decelerating injury after a motor vehicle col- b. Perform human immunodeficiency virus serology testing.
lision. The patient required on-scene mechanical extrica- c. Determine the erythrocyte sedimentation rate.
tion from the vehicle. He has bilateral lower extremity d. Perform Lyme serology testing.
fractures that required surgical intervention before he e. Observe and repeat the chest radiograph in 12 weeks.
arrived in the ICU. Thus far, he has received 10 L of crys-
9. A 62-year-old man, a former smoker, presents with a
talloid and 2 units of packed red blood cells. Over the
2-year history of progressive dry cough and dyspnea.
first 4 hours, his blood pressure and urine output decrease
He has no extrapulmonary symptoms. No occupational
and partially respond to an additional 4 L of crystal-
or environmental exposures are noted. Findings on
loid. Currently, his blood pressure is 80/50 mm Hg, his
examination include bibasilar coarse rales and digital
heart rate is 110 beats per minute, his respiratory rate is
clubbing. A chest radiograph shows prominent inter-
18 breaths per minute, and he is normothermic. Other
stitial infiltrates in the middle and lower lung fields.
than his lower extremity injuries, no abnormal findings
The antinuclear antibody titer is borderline elevated
are noted on his examination. His hemoglobin is 9 g/dL
at 1:40. Serum protein electrophoresis shows a poly-
and his coagulation values are normal. His total creatine
clonal gammopathy. The rheumatoid factor titer is also
kinase (CK) is 800 U/L. A computed tomographic scan
borderline elevated at 1:40. A high-resolution com-
of the abdomen from the emergency department is nor-
puted tomographic (CT) scan of the chest shows sub-
mal. What should you do next?
pleural honeycombing with thickened alveolar septa
a. Administer a colloid fluid bolus.
in both lower lobes with bilateral mediastinal 1.5-cm
b. Obtain an echocardiogram.
lymph nodes. No ground-glass opacities are present.
c. Administer methylprednisolone.
Which treatment is most likely to result in clinical
d. Perform a focused assessment with sonography for trauma
improvement?
(FAST).
a. Azathioprine
e. Begin a bicarbonate infusion.
b. Systemic corticosteroids
6. The incidence of transfusion-related acute lung injury c. Cyclophosphamide
(TRALI) is greatest with transfusion of which of the d. Systemic corticosteroids with azathioprine
following? e. No treatment
a. Fresh frozen plasma
10. A 52-year-old man, a current smoker (75 pack-years),
b. Packed red blood cells
is examined for acute dyspnea and right-sided chest
c. Pooled platelets
pain. He denies having fever, chills, sweats, cough,
d. Single-donor platelets
sputum production, or hemoptysis. On auscultation
e. Salt-poor albumin
of the lungs, diminished breath sounds are heard
7. Which of the following best describes the effects of throughout, with more on the right than the left. The
critical illness on physiologic sleep? chest radiograph and computed tomographic scan
a. Opiate-benzodiazepine combinations promote physiologic of the chest show scattered interstitial changes with
sleep in patients receiving mechanical ventilatory support. cystic and nodular abnormalities, which are more
b. In critical illness, the proportion of rapid eye movement prominent in the mid and upper lung zones, and a
(REM) sleep is decreased. right-sided pneumothorax. What is the most likely
c. In critical illness, sleep has predominant waveforms consis- diagnosis?
tent with deep sleep. a. Lymphangioleiomyomatosis
d. The total duration of sleep during a 24-hour period is b. Pulmonary Langerhans cell histiocytosis (histiocytosis X)
increased. c. Cystic fibrosis
e. The total duration of sleep during a 24-hour period is mark- d. Aspiration pneumonia
edly decreased. e. Idiopathic pulmonary fibrosis

36 M AYO C L I N I C I N T E R N A L M E D I C I N E B OA R D R E VI EW: Q U E S T I O N S A N D A N S W E R S
11. A 68-year-old man, a former smoker, presents with a a. Continue ceftriaxone and levofloxacin for severe
6- to 12-month history of dry cough and dyspnea. He community-acquired pneumonia.
denies fever, chest pain, or hemoptysis. No reflux or b. Add vancomycin or linezolid for community-acquired
dysphagia is reported. His past history is significant for methicillin-resistant Staphylococcus aureus infection.
diabetes mellitus, hyperlipidemia, and benign prostatic c. Check stool samples for ova and parasites.
hypertrophy. No known significant hobby, travel, or d. Repeat bronchoscopy with transbronchial lung biopsies.
environmental exposures are noted. Past occupational e. Start treatment with corticosteroids.
history includes ship refurbishing while in the armed
services. Medications include pravastatin, glyburide, 13. A 52-year-old man who has never smoked and who has
and prazosin. His vital signs are stable, and he is afe- a past medical history of gastroesophageal reflux dis-
brile. No adenopathy is noted. Bibasilar crackles are ease and hypertension presented for recurrent pneu-
present. The heart rhythm is regular. His abdomen is monias. Approximately 6 months before presentation,
normal on palpation and auscultation, and no edema is he received a diagnosis of community-acquired pneu-
noted. Clubbing is present. Blood test results are nor- monia and was treated with macrolide antibiotics with
mal. Pulmonary function testing indicates mild restric- good resolution of his symptoms. He has since had
tion and a low diffusing capacity. A chest radiograph recurrent similar episodes of flulike illness character-
and high-resolution computed tomographic scan of the ized by the acute onset of fever, cough, and occasional
chest show bilateral lower lobe infiltrates along with myalgias and arthralgias. These episodes last 1 to 2
some honeycombing, pleural thickening, diaphrag- weeks and generally seem to respond to short courses of
matic calcification, and an area of consolidation in the antibiotics. Results of human immunodeficiency virus
right lower lobe consistent with rounded atelectasis. testing and toxicology screening were negative. Chest
What should you do next? radiographs during those episodes showed multilobar
a. Begin systemic corticosteroids. alveolar opacities, mostly peripheral, without evidence
b. Begin azathioprine. of pleural effusion. The precise location of these infil-
c. Begin methotrexate. trates seems to vary over time without predilection for
d. Begin cyclophosphamide. particular lobes. Findings from all microbiological
e. Observe only. studies have been repeatedly negative, including mul-
tiple blood cultures, urinary antigens for Streptococcus
12. A 24-year-old man who was previously healthy pres- pneumoniae and Legionella pneumophila, and sputum
ents to the emergency department with an acute onset fungal and mycobacterial cultures. A computed tomo-
of shortness of breath. He denies having chest pain but graphic scan of the chest shows triangular-shaped,
reports a significant nonproductive cough and general- pleural-based infiltrates without evidence of an endo-
ized malaise with myalgias that have developed over the bronchial lesion, an abscess, or pleural effusion. Some
past few days. He has been febrile, with temperatures up of these infiltrates are characterized by ground-glass
to 38.9C, but without chills. He takes no medication attenuation surrounded by more consolidated opaci-
(including over-the-counter or herbal medications) ties. Bronchoscopy with biopsies and bronchoalveolar
and has never had lung or heart problems before. A day lavage show lymphocytic predominance without evi-
before the onset of symptoms, he attended a gradua- dence of infection, and biopsies show the presence of
tion party and smoked several cigars with his friends, plugs of granulation tissue within the alveolar spaces.
none of whom got ill afterward. He has severe hypox- Which of the following is true?
emia with increased work of breathing and is admit- a. The location of the infiltrates suggests septic emboli and the
ted to the intensive care unit. Shortly after admission, need for an echocardiogram.
he requires endotracheal intubation, and mechanical b. The presentation is typical of tuberculosis, and the patient
ventilation is initiated. A chest radiograph showed dif- should be treated with antituberculous medications.
fuse alveolar infiltrates. There was no pleural effusion, c. A trial of corticosteroids without histologic confirmation is
and the heart size was normal. Results of the following appropriate.
blood tests were normal: a complete blood cell count d. Aspiration is the likely cause, and a proton pump inhibitor
and differential count, electrolyte levels, and a coagula- should be prescribed.
tion profile. Results of human immunodeficiency virus e. A bronchogenic carcinoma should be excluded with surgical
testing and toxicology screening were negative. He has lung biopsy.
no relevant travel history. Arterial blood gas results
indicated profound hypoxemia with normal pH and 14. An 82-year-old woman who has never smoked is admit-
Paco2. Immediately after intubation (and before initia- ted for the subacute onset of shortness of breath that
tion of broad-spectrum antibiotics), bronchoalveolar limits her daily activities. Her past medical history is
lavage was performed. Results with Gram, fungal, and significant for gastroesophageal reflux disease, osteopo-
microbiological stains were negative. The differential rosis, hypertension, and recurrent urinary tract infec-
leukocyte count showed 55% eosinophils. What is the tions. Her usual treatment includes lisinopril, aspirin,
most appropriate next step? omeprazole, alendronate, and daily nitrofurantoin.

4 . P U L M O N A RY D I S E A S E S Q U E S T I O N S A N D A N S W E R S 37
She denies having cough or sputum production but has and discolored with onycholysis. Laboratory findings
noticed tingling and numbness in the lower extremities. include the following: hematocrit 35%, leukocyte count
She denies having a rash or photosensitivity, arthritis, 10109/L, serum creatinine 1.2 mg/dL, and serum cal-
and Raynaud phenomenon. Her chest radiograph cium 9.4 mg/dL. Chest radiography shows right pleural
shows reticular opacities preferentially located in the effusion and a normal heart size. On diagnostic right
bases of the lungs bilaterally, and a computed tomo- thoracentesis, there is serosanguineous fluid with a lac-
graphic scan of her chest confirms the presence of dense tate dehydrogenase (LDH) level of 500 U/L. What is
fibrotic-appearing changes in both bases without pleu- the most likely diagnosis?
ral effusions. Results of the following blood tests were a. Congestive heart failure
unremarkable: complete blood cell count and differen- b. Malignant pleural effusion
tial count, electrolytes, and coagulation profile. There is c. Connective tissue disease
nothing to suggest aspiration. At this point, what is the d. Yellow nail syndrome
best next step? e. Asbestos-related pleural effusion
a. Proceed with bronchoscopy with bronchoalveolar lavage
17. A 45-year-old man with alcoholic cirrhosis and por-
and transbronchial biopsies.
tal hypertension presents with progressive dyspnea
b. Initiate treatment with broad-spectrum antibiotics.
over the past year. He denies having chest pain, lower
c. Discontinue use of nitrofurantoin.
extremity edema, ascites, or fever. On physical exami-
d. Refer the patient for lung transplant since no treatment has
nation, his pulse rate is 104 beats per minute, his blood
proved effective for this disease.
pressure is 88/44 mm Hg, and his temperature is
e. Start treatment with corticosteroids.
36.3C. Auscultation of the heart and lungs is normal,
and there is no edema. Oxygen saturation in the seated
position is 87% with room air. Laboratory findings
VA S CU L A R D I S E A S E , E M B O L I S M , A N D
include the following: hemoglobin 17 g/dL, leukocyte
H Y P E RT E NS I O N
count 4.1109/L, serum sodium 129 mEq/L, serum
15. A 36-year-old woman who had a stroke 3 months ago potassium 4.2 mEq/L, and serum creatinine 1.6 mg/
presents for evaluation. Her initial symptoms included dL. Computed tomography of the chest is negative for
right upper extremity weakness that has completely pulmonary embolism, and pulmonary function testing
resolved. Magnetic resonance imaging of the brain is normal with the exception of the diffusion capacity,
shows no underlying pathology other than the infarct. which is 54% of the predicted value. Which test should
As part of her evaluation, she underwent carotid ultra- be performed next to help determine the underlying
sonography, which was normal, and transthoracic diagnosis?
echocardiography with a bubble study, which was sug- a. Pulmonary angiography
gestive of a right-to-left shunt. Physical examination b. Transthoracic contrast echocardiography
findings were as follows: temperature 37.2C, pulse rate c. Cardiopulmonary exercise testing
90 beats per minute, respirations 20 per minute, blood d. Cardiac catheterization
pressure 122/60 mm Hg, and oxygen saturation with e. Overnight oximetry
room air 91%. Lip and finger telangiectases were noted.
18. A 50-year-old man with nonalcoholic steatohepatitis
What test should be performed next?
and portal hypertension presents with progressive dys-
a. Cerebral angiography
pnea on exertion, chest tightness, and lower extremity
b. Positron emission tomography or computed tomographic
edema. On physical examination, his pulse rate is 110
(CT) scan
beats per minute, his blood pressure is 94/52 mm Hg,
c. CT of the chest
and his temperature is 35.8C. Auscultation of the lungs
d. Transesophageal echocardiography
is normal. Jugular venous pressure is elevated at 8 cm
e. Electroencephalography
water, and cardiac auscultation identifies an accentuated
16. A 56-year-old woman presents with dyspnea on exer- pulmonic valve component (P2) and a right ventricular
tion and right pleural effusion over the past 8 years. heave. The liver feels pulsatile with positive hepatojugu-
Cardiac catheterization findings were normal. The lar reflux. Laboratory findings include the following:
patient uses diuretics; however, her lower extremities hemoglobin concentration 11.0 g/dL, leukocyte count
have continued to swell. She has had 5 thoracenteses 3.2109/L, serum sodium 131 mEq/L, serum potassium
over the years for symptomatic relief. Physical examina- 4 mEq/L, serum creatinine 1.4 mg/dL, and N-terminal
tion findings are as follows: temperature 38.0C, pulse B-type natriuretic peptide precursor (NT-pro-BNP)
rate 90 beats per minute, respirations 20 per minute, 1,200 pg/mL. Transthoracic echocardiography shows a
and blood pressure 168/92 mm Hg. Dullness to percus- dilated inferior vena cava with no inspiratory collapse
sion and diminished breath sounds are apparent in the and an estimated right ventricular systolic pressure of
right lung base. Heart examination findings and jugu- 84 mm Hg. His right ventricle is moderately enlarged
lar venous pressure are normal. She has pretibial edema with reduced systolic function and a D-shaped left ven-
(2+). There is no clubbing; however, her nails are short tricle. On right heart catheterization, pulmonary artery

38 M AYO C L I N I C I N T E R N A L M E D I C I N E B OA R D R E VI EW: Q U E S T I O N S A N D A N S W E R S
pressure is 88/32 mm Hg, pulmonary artery occlusion 21. A 47-year-old man who had been healthy without
pressure is 14 mm Hg, and cardiac output is 5.6 L/min. significant past medical history is admitted to your
What is the most likely diagnosis? intensive care unit with severe shock. Endotracheal
a. Idiopathic pulmonary artery hypertension intubation was performed before he arrived. The ini-
b. Diastolic dysfunction tial chest radiograph is shown in Figure 4.Q21A. Even
c. Valvular heart disease after receiving several liters of fluid and vasopressors,
d. Portopulmonary hypertension he remains hypotensive. You decide to place a pulmo-
e. Pulmonary embolism nary artery (PA) catheter. The procedure goes smoothly
and the vessel is cannulated at first pass. You request
another chest radiograph to confirm placement (Figure
P U L M O NA RY S I G NS A N D SY M P TO M S A N D
4.Q21B). Which of the following is the cause for the
CHEST RADIOGRAPHY
finding on the second radiograph?
19. An 80-year-old man with moderately severe chronic a. Vessel injury resulting in hemothorax
obstructive pulmonary disease (COPD) presents to the b. Lobar collapse due to mucous plug
emergency department again for progressively worsen- c. Pneumothorax
ing dyspnea and an increasingly productive cough. He d. Hemopneumothorax
lives alone. Most recently, he was dismissed from the e. Pulmonary infarct due to overwedging of the PA catheter
hospital 3 weeks ago after treatment of COPD exac-
erbations. His initial vital signs include oxygen satura-
tion of 87% with room air, respiratory rate 32 breaths
per minute, blood pressure 138/82 mm Hg, and pulse
102 beats per minute. After nebulizer treatment with
albuterol and ipratropium, his respiratory rate improves
to 28 breaths per minute and his oxygen saturation is
89% on room air. Which of the following is true about
further management of COPD for this patient?
a. He should be given oral corticosteroids and dismissed to home
with follow-up with his primary care physician in 2 days.
b. He should begin receiving tiotropium and dismissed to
home with follow-up with his primary care physician in 2
days.
c. He should be hospitalized for further management of his
COPD exacerbation.
d. He should be intubated and admitted to the intensive care
unit.
e. After another treatment with bronchodilators, he should be Figure 4.Q21A
dismissed and receive a home visit by a nurse tomorrow.
20. A 54-year-old woman presents with progressive dyspnea
at rest. Chest radiography shows significant left-sided
effusion. Results of the thoracentesis and blood tests
are shown in Table 4.Q20.

Table 4.Q20

COMPONENT SERUM THORACENTESIS FLUID

Protein, g/dL 6.5 2.5

Lactate dehydrogenase, U/L 155 125


pH 7.1

Which of the following is not a possible cause for the


pleural fluid?
a. Pulmonary embolism
Figure 4.Q21B
b. Empyema
c. Rheumatoid effusion
d. Tuberculosis
e. Malignancy

4 . P U L M O N A RY D I S E A S E S Q U E S T I O N S A N D A N S W E R S 39
22. A 57-year-old man presents with dyspnea. Pulmonary a. Tiotropium
function test (PFT) results are shown in Table 4.Q22. b. Salmeterol
c. Salmeterol with fluticasone
Table 4.Q22 d. Long-term oxygen therapy
e. N-acetylcysteine
PERCENTAGE OF
COMPONENT VALUE PREDICTED VALUE 24. A 19-year-old woman comes to your office with her
infant because she is concerned about her recurrent
Total lung capacity, L 3.51 52
sinusitis. Although she has never been hospitalized,
Residual volume, L 2.57 102 her younger brother was hospitalized for recurrent
pancreatitis. She describes a chronic productive cough
Forced vital capacity (FVC), L 4.29 95
with dyspnea. Examination reveals wheezing and digi-
Forced expiratory volume in the first 3.51 91 tal clubbing. What is the recommended initial test to
second of expiration (FEV1), L diagnose this disorder?
FEV1/FVC 81.8 a. No further testing
b. Sweat chloride testing
Diffusing capacity of lung for carbon 15.5 60 c. Testing for a CFTR genetic mutation
monoxide, mLmin1mm Hg1
d. Computed tomographic (CT) scan of the chest with an
intravenous contrast agent
The results may be suggestive of which of the e. CT scan of the sinuses followed by magnetic resonance
following? imaging of the brain if abnormalities are detected
a. Asthma
25. A 57-year-old man presents for excessive sleepiness and
b. Obesity
difficulty functioning at his work. His wife has com-
c. Chronic obstructive pulmonary disease
plained of very disruptive snoring and has witnessed
d. Severe pulmonary hypertension
frequent apneic episodes. His body mass index is 42.
e. Idiopathic pulmonary fibrosis
His blood pressure is 155/85 mm Hg. His neck size is
23. A 67-year-old man with severe chronic obstructive pul- 48 cm. Which of the following tests should be per-
monary disease (COPD) presents for follow-up after formed next to confirm your suspicion?
hospitalization. He was admitted with respiratory fail- a. Overnight pulse oximetry
ure and COPD exacerbation for the second time in the b. Use of a 24-hour ambulatory blood pressure monitor
past 7 months. He feels that his dyspnea is at baseline. c. Overnight polysomnography
Which of the following has not been shown to reduce d. Carotid duplex ultrasonography
exacerbation or rehospitalization rates? e. Adrenal imaging with computed tomography

40 M AYO C L I N I C I N T E R N A L M E D I C I N E B OA R D R E VI EW: Q U E S T I O N S A N D A N S W E R S
ANSWER S

1. Answer b. mechanism of accident injury and his elevated CK. A bed-


side echocardiogram can quickly confirm this diagnosis.
This patient has hypertrophic cardiomyopathy with outflow
The other answer choices are less plausible.
obstruction that is exacerbated by intravascular volume con-
traction. His diagnosis is suggested by 1) the murmur, 2) 6. Answer c.
tachycardia (shortened left ventricular diastolic filling time),
3) lung crackles, 4) absence of jugular vein distention, and The risk of TRALI is related to the potential number of
5) worsening with furosemide-induced diuresis. His fluid exposures to plasma from different sources. The greatest
output and input are matched since admission, but his surgi- risk occurs when multiple pooled sources are used, as in
cal procedure is associated with increased third-space fluid platelet transfusions. The transfusion of pooled products
losses. Therefore, his intravascular volume is likely depleted. from women, in particular fresh frozen plasma, has also
Of the answer choices given and the various differential been associated with an increased risk of TRALI. (See
diagnostic possibilities (congestive heart failure, myocardial Gajic et al and Toy et al in the Suggested Reading list.)
infarction, pulmonary embolism, and methemoglobinemia), 7. Answer b.
only metoprolol will improve his outflow obstruction.
In critical illness, disrupted and fragmented sleep leads to
2. Answer a. impaired cognitive function. Opiates and benzodiazepines
This patient has an acute, agitated delirium. Benzodia- alter normal sleep architecture, but the total duration of
zepines are not indicated; they may worsen delirium and physiologic sleep during a 24-hour period has been found
merely suppress its manifestations. Similarly, this is not a to be normal. Sleep patterns in critical illness have a pre-
pain syndrome, and opioids will suppress but not treat the dominance of superficial stages N1 and N2 and a low pro-
underlying pathophysiology. portion of slow-wave deep sleep (stage N3). REM sleep is
severely decreased or absent. (See Cooper et al and Friese et
3. Answer a. al in the Suggested Reading list.)
Emerging data suggest that the interval of time from symp-
8. Answer e.
tom onset to administration of thrombolytic therapy can
be prolonged in selected patients with acute ischemic Sarcoidosis is a granulomatous disease most often affecting
stroke, including patients not at increased risk of bleed- the lungs and lymph nodes. It can occur after a flulike illness
ing. These patients had improved survival if they received and may be diagnosed by a specific constellation of symp-
thrombolytic therapy up to 4.5 hours after onset of symp- toms and signs when presenting with Lfgren syndrome
toms. (See Hacke et al in the Suggested Reading list.) (erythematous nodosum, bilateral hilar lymphadenopathy,
fever, and polyarthritis). In most other instances, a diagno-
4. Answer a.
sis of sarcoidosis requires a compatible history, findings of
This patient presents with a massive pulmonary embolism noncaseating granulomas by biopsy, and exclusion of other
and shock. Thrombolysis or embolectomy is indicated. The possible causes of granulomatous inflammation. If there is
lack of improvement reflects either failure of primary throm- systemic involvement, blood work may show abnormali-
bolysis or development of a new complication. The central ties, including hypercalcemia, anemia, and elevated liver
venous pressure is much lower than would be expected in a enzymes. Serum angiotensin-converting enzyme levels are
patient with pulmonary embolism and acute right heart fail- neither specific nor sensitive to use as a diagnostic tool but,
ure. The rib fractures and decreased hemoglobin suggest an when elevated, may be helpful for following disease activ-
occult site of bleeding, potentially in the abdomen or retro- ity. Bronchoscopy can confirm granulomatous disease in
peritoneum, which needs to be identified before a decision over 90% of patients with hilar adenopathy and parenchy-
is made on possible rescue therapy for the pulmonary embo- mal lung involvement. Rales are uncommon in sarcoidosis
lism. (See Meneveau et al in the Suggested Reading list.) even when parenchymal interstitial changes are present.
Incidence, clinical course, and prognosis of sarcoidosis are
5. Answer b.
influenced by ethnic and genetic factors. Computed tomo-
This patient has myocardial contusion after trauma, with graphic scan may show nodular opacities with bronchovas-
possible left ventricular failure. This is suggested by the cular and subpleural distribution, thickened intralobular

41
septa, architectural distortion, or conglomerate masses fibers typically are dormant for decades before pulmonary
(late stage). Tobacco use has not been associated with fibrosis develops. A clinical response to corticosteroids
development of sarcoidosis. Extrapulmonary involvement or other medication is not expected in the treatment of
from sarcoidosis may involve the heart, liver, spleen, eyes, asbestos-related pulmonary fibrosis. Smoking in the pres-
bone, skin, bone marrow, parotid glands, pituitary gland, ence of asbestos-related pulmonary fibrosis increases the
and reproductive organs. This patients presentation is rate of progression of fibrosis and the risk of bronchogenic
most consistent with Lfgren syndrome, which carries carcinoma. Tuberculosis is not a common complication of
a very good prognosis; symptoms resolve without treat- asbestosis. No therapy has been effective in preventing pro-
ment. Thus, observation with follow-up chest radiography gressive pulmonary fibrosis due to asbestos exposure.
is appropriate. If symptoms are more bothersome, symp-
12. Answer e.
tomatic treatment such as nonsteroidal anti-inflammatory
agents may be considered. For progressive pulmonary and Acute eosinophilic pneumonia is a rare but increasingly rec-
extrapulmonary disease, corticosteroids or immunosup- ognized cause of acute respiratory distress syndrome. The
pressive therapy should be considered. pathophysiology is unclear, but a recent onset of smoking
and an increase in smoking patterns have been recognized
9. Answer e.
as common precipitating factors. Clinically, acute eosino-
The combination of interstitial lung infiltrates predomi- philic pneumonia is characterized by the acute onset of dif-
nantly involving the lower lung zones, lack of occupational fuse alveolar infiltrates with pulmonary eosinophilia; unlike
exposure, duration of symptoms, and peripheral honey- in chronic eosinophilic pneumonia, peripheral eosinophilia
combing make the diagnosis of idiopathic pulmonary is rare. Radiologically, the infiltrates are diffuse but nonspe-
fibrosis (IPF) most likely. Favorable prognostic factors in cific, and treatment with corticosteroids generally results
IPFusual interstitial pneumonia include age younger than in dramatic improvement. Bronchoscopic lung biopsies are
50, female sex, shorter duration of symptoms before pre- not generally needed for the diagnosis. Other causes of pul-
sentation, presence of ground-glass opacities on CT scan monary eosinophilia include allergic bronchopulmonary
of the chest, and lymphocytosis on examination of bron- aspergillosis, Churg-Strauss syndrome, parasitic infections,
choalveolar lavage fluid. Pulmonary function tests in IPF some fungal infections, drug-induced lung diseases, chronic
usually indicate restrictive impairment. Patients with IPF eosinophilic pneumonia, and, rarely, connective tissue dis-
generally do not respond to corticosteroids or other immu- easerelated interstitial lung disease and malignancies.
nosuppressive therapies. No clearly effective treatment
13. Answer c.
options are currently available. Oxygen extends survival
among patients with chronic obstructive pulmonary dis- The likely diagnosis is cryptogenic organizing pneumo-
ease, but this benefit has not been shown for IPF patients. nia (formerly known as idiopathic bronchiolitis obliter-
Familial clusters of IPF patients suggest a potential genetic ans with organizing pneumonia, or BOOP). Organizing
predisposition in some cases of IPF. pneumonia is characterized pathologically by the presence
of plugs of fibroblasts and myofibroblasts occupying the
10. Answer b.
distal airspaces without disruption of the underlying archi-
This patients presentation is most consistent with adult tecture of the lung. Cryptogenic organizing pneumonia
pulmonary Langerhans cell histiocytosis, which, in most typically manifests with recurrent or persistent episodes of
cases, is a form of smoking-related interstitial lung dis- pneumonia-like illness that occasionally responds to anti-
ease. Smoking cessation is the primary form of treatment. biotics, particularly macrolides. The treatment consists of a
Stabilization or improvement occurs in up to two-thirds prolonged course of corticosteroids (3 months), and the
of patients with smoking cessation alone. Other therapies absence of response to treatment should suggest an under-
including systemic corticosteroids and immunosuppres- lying cause such as hematologic malignancy, drug-induced
sives have been used with limited success. No role of plas- lung disease, or connective tissue disease (ie, secondary
mapheresis has been described. organizing pneumonia). While usually very effective,
rebound after discontinuation of treatment is frequent.
11. Answer e.
14. Answer c.
Asbestosis typically has a basilar predominance, and
rounded atelectasis is suggestive of asbestos exposure. The possibility of drug-induced lung disease should always
Known or remembered histories of occupational asbestos be considered when evaluating a case of diffuse parenchy-
exposure may not always be present, but they can be help- mal lung disease. Since nitrofurantoin is a common offender
ful. Common findings include pleural plaques or diaphrag- (with bleomycin, methotrexate, and amiodarone), its use
matic calcifications (or both). Malignant mesothelioma is should be carefully investigated because patients may not
strongly associated with asbestos exposure but not smok- always volunteer that information. Nitrofurantoin lung
ing. Pleural surfaces are generally abnormal and involved toxicity can manifest either as an acute form of lung toxicity
with asbestos-related parenchymal lung disease. However, (when it is usually associated with peripheral eosinophilia
pleural and parenchymal abnormalities may, in some and elevated inflammatory markers) or, less frequently
instances, occur independently of each other. Asbestos (as in this case), as a chronic form of fibrotic lung disease.

42 M AYO C L I N I C I N T E R N A L M E D I C I N E B OA R D R E VI EW: Q U E S T I O N S A N D A N S W E R S
Discontinuation of the drug is warranted in both cases and 19. Answer c.
should take precedence over any other intervention. The
This patient has multiple risk factors requiring in-patient
role of corticosteroids, sometimes recommended for the
management of his COPD exacerbation. As noted by the
acute form of the disease, is less clear in chronic cases, which
Global Initiative for Chronic Obstructive Lung Disease
may progress and even be fatal in up to 10% of patients.
(GOLD), indications for hospital admission for COPD
15. Answer c. exacerbation include the following: marked increase in
intensity of symptoms, severe underlying disease, onset
This young patient has lip and finger telangiectases, an
of new physical signs, failure of exacerbation to respond
inappropriately low oxygen saturation, and evidence of
to initial medical management, significant comorbidi-
a right-to-left shunt on bubble echocardiography. Her
ties, newly occurring arrhythmias, diagnostic uncertainty,
stroke was likely caused by paradoxical embolism, and her
frequent exacerbations, older age, and insufficient home
low oxygen saturation suggests a pulmonary arteriovenous
support. Although the patients symptoms improved some-
malformation, so that CT of the chest is the best choice.
what initially, with the persistence of abnormal vital signs
With the telangiectases, the shunt, and the suggestion of
and other risk factors noted above, he should be admitted
a pulmonary arteriovenous malformation, the most likely
for further management. (See Rabe et al in the Suggested
diagnosis is hereditary hemorrhagic telangiectasia. The
Reading list.)
clinical criteria for this diagnosis include telangiectases
(lips, mouth, nose, and fingertips), visceral arteriovenous 20. Answer a.
malformation, epistaxis, and family history of any of the
Pleural fluid analysis and the ratio of pleural fluid lactate
preceding criteria. Stroke in a young person should prompt
dehydrogenase (LDH) to serum LDH (125:155) show
one to think of pulmonary arteriovenous malformation.
that the fluid is an exudate. For fluid to be considered
16. Answer d. an exudate, the fluid needs to meet only 1 of the Light
criteria (ratio of pleural fluid protein to serum protein
This patient presents with dyspnea, recurrent right pleu-
>0.5, ratio of pleural fluid LDH to serum LDH >0.6,
ral effusion, lower extremity edema, and discolored, dys-
pleural fluid LDH greater than two-thirds of the upper
trophic nails. Her thoracentesis fluid is exudative, with an
limit of the reference range for serum LDH). While the
LDH of 500 U/L. The most likely diagnosis is yellow nail
protein ratio is less than 0.5, the LDH ratio is greater
syndrome, which consists of the triad of yellow nails, lym-
than 0.6; thus, this fluid is an exudate. Furthermore,
phedema, and respiratory tract illness (pleural effusions,
the low pH suggests a certain diagnosis. When thora-
bronchiectasis, and recurrent pneumonias). The nails usu-
centesis fluid pH is less than 7.3, diagnostic possibili-
ally do not grow, and patients will wonder why they do not
ties include empyema, esophageal rupture, rheumatoid
have to cut their nails.
arthritis, trauma, tuberculosis, and malignancy. Pleural
17. Answer b. fluid in pulmonary embolism may be either a transudate
or an exudate, but pH of the pleural fluid should not be
This man has alcoholic cirrhosis, portal hypertension, and severely acidic.
hypoxemia. The most likely diagnosis is hepatopulmonary
syndrome. The clinical triad in the diagnosis is 1) pres- 21. Answer b.
ence of liver disease with portal hypertension, 2) intrapul-
While all the answer choices are potential complications
monary shunting, and 3) hypoxemia. The transthoracic
of PA catheter placement, careful inspection of the radio-
contrast echocardiogram will show the presence of intra-
graph shows that lobar collapse is the best answer. Note
pulmonary shunting by the passage of bubbles into the
the elevation of the minor fissure and the elevation of the
left heart after 3 to 5 cardiac cycles. The polycythemia is
right hemidiaphragm. These are characteristic findings of
due to the chronic hypoxemia, which is present at rest and
a lobar collapse. Hemothorax should not be limited to the
worsens with exertion. Liver transplant is the treatment of
upper lobes only; rather, fluid would accumulate in the
choice if the patient is a surgical candidate otherwise.
lower portions of the chest, thereby blunting the costo-
18. Answer d. phrenic angle. Pneumothorax should result in collapse of
the lower lobes as well in this previously healthy patient.
This patient has severe pulmonary artery hypertension with Furthermore, pneumothorax should create an air interface
liver disease and portal hypertension. The most likely diag- and thus appear black, not white, on the radiograph. A
nosis is portopulmonary hypertension because of the follow- hemopneumothorax should create an air-liquid interface
ing criteria: 1) liver disease or portal hypertension, 2) mean along the lateral margin of the lung, which is not appar-
pulmonary artery pressure greater than 25 mm Hg, 3) pul- ent in this radiograph. The region of involvement would be
monary vascular resistance greater than 240 dynesscm5, too large for pulmonary infarct due to overwedging. This
and 4) pulmonary artery occlusion pressure less than patient underwent bronchoscopic clearing of the airways;
15 mm Hg. Liver transplant for this patient is contraindi- the chest radiograph 30 minutes later is shown in Figure
cated, and he should be considered for vasodilator therapy 4.A21.
to try to control the pulmonary artery hypertension.

4 . P U L M O N A RY D I S E A S E S Q U E S T I O N S A N D A N S W E R S 43
is recommended. (See Farrell et al and McMullen et al in
the Suggested Reading list.)
25. Answer c.
This patient has all the risk factors for at least moderately
severe obstructive sleep apnea (OSA). Therefore, overnight
polysomnography should be performed next to confirm
the diagnosis. If OSA is confirmed, treatment should
begin with a continuous positive airway pressure device.
Overnight oximetry is occasionally used as a screening tool,
but the results are not sufficient to establish the diagnosis
even though they may be suggestive of OSA. Other tests
have no role in establishing the diagnosis of OSA.

SUGGESTED RE ADING

Calverley PM, Anderson JA, Celli B, Ferguson GT, Jenkins C, Jones PW,
Figure 4.A21 et al; TORCH investigators. Salmeterol and fluticasone propionate
and survival in chronic obstructive pulmonary disease. N Engl J Med.
22. Answer e. 2007 Feb 22;356(8):77589.
Cooper AB, Thornley KS, Young GB, Slutsky AS, Stewart TE, Hanly
PFT results show a moderately severe restrictive defect with PJ. Sleep in critically ill patients requiring mechanical ventila-
a markedly reduced diffusing capacity of lung for carbon tion. Chest. 2000 Mar;117(3):80918. Erratum in: Chest 2001
monoxide (Dlco). Of all the options, only idiopathic pul- Mar;119(3):993.
monary fibrosis fits the PFT findings. A restrictive defect Farrell PM, Rosenstein BJ, White TB, Accurso FJ, Castellani C, Cutting
GR, et al; Cystic Fibrosis Foundation. Guidelines for diagnosis of cystic
would not be expected in asthma or chronic obstructive fibrosis in newborns through older adults: Cystic Fibrosis Foundation
pulmonary disease. Obesity may result in a restrictive lung consensus report. J Pediatr. 2008 Aug;153(2):S4-S14.
defect, but the Dlco is often normal to elevated. While Friese RS, Diaz-Arrastia R, McBride D, Frankel H, Gentilello LM.
the Dlco may be reduced in severe pulmonary hyperten- Quantity and quality of sleep in the surgical intensive care unit: are our
sion, lung volumes should not be reduced. patients sleeping? J Trauma. 2007 Dec;63(6):12104.
Gajic O, Rana R, Winters JL, Yilmaz M, Mendez JL, Rickman OB,
23. Answer d. et al. Transfusion-related acute lung injury in the critically ill: prospec-
tive nested case-control study. Am J Respir Crit Care Med. 2007 Nov
Several studies have examined readmission rates and mea- 1;176(9):88691. Epub 2007 Jul 12.
sures that mitigate them. Tiotropium reduced exacerba- Gerrits CM, Herings RM, Leufkens HG, Lammers JW. N-acetylcysteine
tions, among other outcomes, during a 4-year trial. Another reduces the risk of re-hospitalisation among patients with
chronic obstructive pulmonary disease. Eur Respir J. 2003
trial compared the effect of salmeterol alone, fluticasone May;21(5):7958.
alone, or the 2 drugs in combination. Exacerbation rates Gonzalez C, Servera E, Marin J. Importance of noninvasively measured
were significantly reduced in all groups compared with respiratory muscle overload among the causes of hospital readmis-
placebo, and combination therapy reduced hospitaliza- sion of COPD patients. Chest. 2008 Apr;133(4):9417. Epub 2008
tion rates. Furthermore, use of N-acetylcysteine reduced Feb 8.
Hacke W, Kaste M, Bluhmki E, Brozman M, Davalos A, Guidetti D, et al;
the risk of readmissions in a study of 1,219 patients. While ECASS Investigators. Thrombolysis with alteplase 3 to 4.5 hours after
long-term oxygen therapy reduces overall mortality, its acute ischemic stroke. N Engl J Med. 2008 Sep 25;359(13):131729.
use was associated with increased risk of readmissions in McMullen AH, Pasta DJ, Frederick PD, Konstan MW, Morgan WJ,
those with moderate-to-severe COPD. (See Calverley Schechter MS, et al. Impact of pregnancy on women with cystic fibro-
et al, Gerrits et al, Gonzalez et al, and Tashkin et al in the sis. Chest. 2006 Mar;129(3):70611.
Meneveau N, Seronde MF, Blonde MC, Legalery P, Didier-Petit K, Briand F,
Suggested Reading list.) et al. Management of unsuccessful thrombolysis in acute massive pul-
24. Answer b. monary embolism. Chest. 2006 Apr;129(4):104350.
Rabe KF, Hurd S, Anzueto A, Barnes PJ, Buist SA, Calverley P, et al;
This patient has recurrent sinusitis, wheezing, digital club- Global Initiative for Chronic Obstructive Lung Disease. Global strat-
bing, and a family member with recurrent pancreatitis. egy for the diagnosis, management, and prevention of chronic obstruc-
tive pulmonary disease: GOLD executive summary. Am J Respir Crit
Cystic fibrosis must be considered as the underlying disor- Care Med. 2007 Sep 15;176(6):53255. Epub 2007 May 16.
der. Women with cystic fibrosis can be fertile if they have Tashkin DP, Celli B, Senn S, Burkhart D, Kesten S, Menjoge S,
adequate nutritional and pulmonary reserve. In contrast, et al; UPLIFT Study Investigators. A 4-year trial of tiotropium in
men often present with azoospermia. The Cystic Fibrosis chronic obstructive pulmonary disease. N Engl J Med. 2008 Oct
Foundation recommends sweat chloride testing as the ini- 9;359(15):154354. Epub 2008 Oct 5.
Toy P, Popovsky MA, Abraham E, Ambruso DR, Holness LG, Kopko PM,
tial diagnostic test. If the sweat chloride concentration is et al; National Heart, Lung and Blood Institute Working Group on
more than 60 mEq/L, the diagnosis of cystic fibrosis is con- TRALI. Transfusion-related acute lung injury: definition and review.
firmed; if it is 30 to 59 mEq/L, testing for CFTR mutations Crit Care Med. 2005 Apr;33(4):7216.

44 M AYO C L I N I C I N T E R N A L M E D I C I N E B OA R D R E VI EW: Q U E S T I O N S A N D A N S W E R S
5.
INFECTIOUS DISEASES QUESTIONS AND ANSWER S

QUESTIONS no clubbing or cyanosis. The total leukocyte count is


14.4109/L. Results of serum chemistry tests are nor-
Multiple Choice (choose the best answer) mal. The chest radiograph shows a new dense consolida-
tion in the left upper lung field. Which of the following
antimicrobial regimens would you initiate?
P N EU M O N I A , Z O O N O S E S , T R AVE L , A N D a. Azithromycin orally
B I OT E R RO R I S M b. Ceftriaxone intravenously and azithromycin orally
c. Ciprofloxacin intravenously
1. A 67-year-old man with uncomplicated type 2 diabe-
d. Ampicillin intravenously and clindamycin orally
tes mellitus and hypertension presents to his primary
e. Meropenem intravenously
care physician with a 3-day history of fever, productive
cough, and shortness of breath. He has been in good 3. A 44-year-old male business executive is admitted to
health otherwise, has never been hospitalized, and the intensive care unit for fevers, diarrhea, cough, and
has good social support at home. Examination find- shortness of breath progressing over the past 48 hours.
ings include the following: temperature 39.1C, blood His past medical history includes cigarette smok-
pressure 110/75 mm Hg, heart rate 78 beats per min- ing and weekly binge drinking but no other chronic
ute with regular rate and rhythm, and respiratory rate medical problems. He is from St Louis, Missouri, but
26 breaths per minute. Oxygen saturation is 94% with was attending a conference in Chicago, Illinois, 2 days
room air. He is awake and alert and oriented to person, before admission. Abdominal pain, diarrhea, cough,
place, and time; he responds to questions appropriately. and shortness of breath developed 2 days before admis-
Inspiratory crackles are audible in the right lower lung sion. He had mentioned that other people from the
field, his abdomen has active bowel sounds and is not convention had similar illnesses. He has had no other
tender, and his extremities have no clubbing or cyano- recent travel and has not had any exposure to animals.
sis. Which of the following should be done next? Antimicrobial therapy has been started and blood
a. Outpatient observation only with follow-up in 2 days and sputum cultures are pending. Examination find-
b. Outpatient consultation with an infectious diseases ings include the following: temperature 39.2C, blood
specialist pressure 85/60 mm Hg, heart rate 72 beats per min-
c. Hospital admission and treatment with levofloxacin ute with regular rate and rhythm, and respiratory rate
d. Intensive care unit admission and treatment with 40 breaths per minute. Oxygen saturation is 90% with
levofloxacin 10 L of oxygen per minute by face mask. He is drowsy and
e. Outpatient treatment with levofloxacin does not reliably follow commands, and he is oriented
to person only. Diffuse bilateral crackles are audible on
2. A 57-year-old woman is admitted to an internal medi-
lung auscultation, his abdomen has active bowel sounds
cine hospital ward with a 3-day history of fever, cough,
and is not tender, and his extremities have no club-
progressively purulent sputum, and shortness of breath.
bing or cyanosis. Remarkable diagnostic test results
She has been well otherwise, has never been hospital-
were leukocytosis (16.2109/L) and hyponatremia
ized, and has a past medical history of well-controlled
(126 mEq/L). A chest radiograph showed bilateral patchy
type 2 diabetes mellitus and hypertension. She has not
infiltrates. Gram staining of a sputum showed scant poly-
had any recent antimicrobial exposure, foreign travel,
morphonuclear cells and no bacteria. What is the most
or animal exposure. She has no known drug allergies.
likely cause of the patients respiratory infection?
Examination findings include the following: tempera-
a. Legionella pneumophila
ture 38.7C, blood pressure 125/75 mm Hg, heart rate
b. Respiratory syncytial virus (RSV)
78 beats per minute with regular rate and rhythm, and
c. Enterovirus
respiratory rate 26 breaths per minute. Oxygen satu-
d. Chlamydophila psittaci
ration is 95% with 2 L of oxygen per minute by nasal
e. Coccidioides immitis
cannula. She is awake and alert and oriented to person,
place, and time. Inspiratory crackles are audible in the 4. Which of the following is a recommended intervention
left upper lung field, her abdomen has active bowel to reduce the risk of ventilator-associated pneumonia
sounds and is not tender, and her extremities have among patients receiving mechanical ventilation?

45
a. Preference for invasive ventilation over noninvasive are growing a gram-positive rod in 2 out of 2 bottles.
ventilation Therapy with ceftriaxone and levofloxacin was started
b. Preference for nasotracheal intubation over orotracheal upon admission to the ICU. What is the most likely
intubation etiologic agent?
c. Keeping patients supine during enteral feeding a. Streptococcus pneumoniae
d. Preference for parenteral nutrition over enteral nutrition b. Avian influenza virus H5N1
e. Maintaining endotracheal tube cuff pressure greater than c. Francisella tularensis
20 cm water d. Bacillus anthracis
e. Listeria monocytogenes
5. For which pair of pathogens should hospital anti-
microbial resistance rates be considered when an
empirical antimicrobial regimen is chosen to treat
S K I N A N D S O F T T I S S U E I N FEC T I O NS ,
hospital-acquired pneumonia?
B O N E A N D J O I N T I N FEC T I O NS , A N D
a. Acinetobacter baumannii and Candida albicans
MYC O BAC T E R I A L I N F E C T I O NS
b. Streptococcus pneumoniae and Haemophilus influenzae
c. Pseudomonas aeruginosa and Staphylococcus aureus 8. A 53-year-old diabetic man underwent abdominal
d. Klebsiella pneumoniae and influenza virus hernia repair 4 days ago after receiving preoperative
e. Serratia marcescens and Enterococcus faecium cefazolin. A nurse calls you to evaluate him for fever,
hypotension, and tachycardia. He has a toxic appear-
6. A previously healthy 32-year-old man presents to his pri- ance; his incision is tender and erythematous with some
mary care physician after his cat bit his arm earlier in the sero-sanguinous drainage. Which group of bacteria
morning. The cats vaccinations are current, and the cat should your therapy target?
has not been attacked by other animals. The patient says a. Gram-positive cocci
that the bite was deep enough to draw blood, although b. Gram-positive cocci and gram-positive bacilli
the bleeding has stopped. You cleanse the wound, apply c. Gram-positive cocci and gram-positive and gram-negative
a bandage, and update his tetanus-diphtheria vaccina- bacilli
tion. Examination findings include the following: tem- d. Gram-negative bacilli
perature 37.2C, blood pressure 125/80 mm Hg, heart e. Gram-positive bacilli
rate 72 beats per minute, and respiratory rate 16 breaths
per minute. A 1.5-cm laceration is evident on the dor- 9. A 31-year-old man underwent septoplasty for a deviated
sal aspect of the left forearm without any erythema or nasal septum and recurrent sinusitis. He presents to an
purulence. Which of the following antimicrobial regi- emergency department 48 hours later with headache,
mens should be prescribed? fever, chills, myalgia, nausea, vomiting, and abdomi-
a. Amoxicillin-clavulanic acid nal cramping. His temperature is 39.9C, his pulse is
b. Dicloxacillin 142 beats per minute, his respiratory rate is 28 breaths
c. Cephalexin per minute, and his blood pressure is 74/30 mm Hg. He
d. Clindamycin has generalized erythroderma. On head and neck exam-
e. No antimicrobials ination, the nasal passages are hyperemic but not puru-
lent. Laboratory test results included the following:
7. A previously healthy 25-year-old man is admitted to an leukocyte count 14.7109/L, hemoglobin 15.0 g/dL,
intensive care unit (ICU) with a 4-hour history of rap- and platelet count 84109/L. The results of liver func-
idly progressive fever, shortness of breath, and cough. tion tests, serum creatinine, and amylase were normal.
He had spent most of the previous 2 days at the county Which pathogen is most likely to cause this syndrome?
fair. Upon arrival at the hospital, the patient underwent a. Pseudomonas aeruginosa
endotracheal intubation, was given fluid resuscitation, b. Haemophilus influenzae
and received vasopressor medications and mechanical c. Moraxella catarrhalis
ventilation. Examination findings include the follow- d. Streptococcus pneumoniae
ing: temperature 40.2C, blood pressure 90/60 mm Hg e. Staphylococcus aureus
with vasopressors, heart rate 102 beats per minute
(tachycardic), and respiratory rate 16 breaths per 10. Which of the following patients is best suited for out-
minute. Oxygen saturation is 95% with assist/control patient parenteral antibiotic therapy (OPAT) for the
ventilation and 50% fraction of inspired oxygen. He infection?
is sedated and does not follow commands. Diffuse a. A 23-year-old injection drug user with Staphylococcus aureus
bilateral crackles are audible on lung auscultation, his tibial osteomyelitis
abdomen has active bowel sounds and is not tender, b. A 76-year-old man with enterococcal prosthetic valve endo-
and his extremities have no clubbing or cyanosis. A carditis with a new Wenckebach heart block
chest radiograph shows diffuse pulmonary infiltrates c. A 46-year-old woman with a diabetic foot ulcer and
and a widened mediastinum. Blood samples drawn in Pseudomonas metatarsal osteomyelitis that requires
the emergency department 2 hours ago for cultures imipenem-cilastatin every 6 hours

46 M AYO C L I N I C I N T E R N A L M E D I C I N E B OA R D R E VI EW: Q U E S T I O N S A N D A N S W E R S
d. A 26-year-old woman with pneumococcal meningitis that is rate and C-reactive protein level have been normal.
improving on day 7 of a 14-day ceftriaxone regimen What is the most likely cause of the change in her blood
e. An 89-year-old man with early Alzheimer disease treated glucose level and blood pressure?
with ceftriaxone for community-acquired pneumonia a. Ongoing infection and need for resection arthroplasty
b. Poor control of diet
11. A 45-year-old man with diabetes mellitus and severe
c. Medication interaction
peripheral vascular disease presents with a 6-week his-
d. Inadequate pain control
tory of erythema and induration surrounding a 3-cm
e. Poor adherence to drug therapy
plantar ulcer. Today he is nauseated, febrile, and tachy-
cardic. You can insert a metallic probe through the 15. Each of the following patients underwent a tuberculin
open wound to the bone surface. There is surround- skin test (TST) for appropriate indications. Which
ing redness and drainage of foul-smelling pus. Which result would be considered a positive TST reaction
of the following would be the next appropriate step in (ie, a positive purified protein derivative skin test
management? [PPD])?
a. Swabbing the patients nose for methicillin-resistant a. A 36-year-old man who is positive for human immunodefi-
Staphylococcus aureus (MRSA) ciency virus (HIV)an 8-mm induration
b. Magnetic resonance imaging (MRI) of the foot b. A 26-year-old Mexican native migrant workera 3-mm
c. Parenteral vancomycin and piperacillin-tazobactam induration last year and a 20-mm induration this year
d. Bone scintigraphy c. A 42-year-old man whose chest radiograph suggests old
e. Plain radiography of the foot tuberculosisa 6-mm induration
12. For which of the following would use of the d. A 56-year-old diabetic womana 15-mm induration
QuantiFERON-TB Gold test for Mycobacterium tuber- e. All of the above
culosis be preferable to a purified protein derivative 16. Which of the following tuberculosis-suspect patients
(PPD) skin test? should be considered infectious?
a. A 23-year-old resident physician from India who received a. A 44-year-old man had a smear positive for acid-fast bacilli
BCG vaccine as a child (AFB) 2 weeks ago; no further specimens were obtained. He
b. A 45-year-old health care worker with recent exposure to has received self-administered antitubercular therapy for the
someone with tuberculosis in the past 3 months past 7 days and continues to cough.
c. A 62-year-old man who recently returned from 1 year of vol- b. A 22-year-old school teacher with pulmonary tubercu-
unteer work in Rwanda losis has been receiving directly observed therapy for 6
d. A 35-year-old Somali woman with a new diagnosis of weeks and is asymptomatic. Three sputum smears have
human immunodeficiency virus infection (CD4 count 478 been evaluated; the first was positive and the rest were
cells/L) negative.
e. A 29-year-old nurse undergoing annual required tuberculo- c. Cough and malaise developed in a 56-year-old Asian
sis screening man visiting his grandchildren. A tuberculin skin test and
13. A 34-year-old Mexican immigrant with human immu- QuantiFERON-TB Gold test were both positive. His chest
nodificiency virus infection (CD4 count 300 cells/L) radiograph was normal, and 3 sputum samples were negative
has been treated with isoniazid, rifampin, pyrazin- for AFB.
amide, ethambutol, and pyridoxine for pulmonary d. A 74-year-old woman with a cough has lost weight. The
tuberculosis for the past 2 months. He is not currently QuantiFERON-TB Gold test was positive, and her chest
taking antiretroviral medications. He now presents radiograph was normal.
with severe pain, swelling, and redness of the left great e. A 54-year-old health care worker had a newly positive
toe. Which of the following medications is most likely purified protein derivative skin test and a normal chest
associated with this condition? radiograph.
a. Isoniazid
17. A 24-year-old male landscaper with human immuno-
b. Rifampin
deficiency virus (HIV), sickle cell anemia, and chronic
c. Pyridoxine
renal failure has a hot, painful, swollen right knee.
d. Ethambutol
Results of an aspiration of the knee joint are as fol-
e. Pyrazinamide
lows: the leukocyte count is 65109/L with 65% poly-
14. A 76-year-old woman with hypertension and type 2 dia- morphonuclear cells, and Gram staining is negative for
betes mellitus is seen for worsening hypertension and microorganisms. Which of the following is the least
poor glycemic control. She is taking verapamil and gly- likely cause of his symptoms?
buride. Recently, a methicillin-resistant Staphylococcus a. Acute sickle cell crisis
aureus infection developed in her left prosthetic hip b. Acute gout
joint, for which she underwent dbridement and com- c. Acute gonococcal arthritis
ponent retention and is at a nursing home receiving van- d. Acute Salmonella arthritis
comycin and rifampin. Her erythrocyte sedimentation e. Acute Staphylococcus aureus arthritis

5. I N F E C T I O U S D I S E A S E S Q U E S T I O N S A N D A N S W E R S 47
H I V I N FEC T I O N pressure is 120/75 mm Hg, her heart rate is 80 beats
per minute, and her respiratory rate is 18 breaths per
18. A 34-year-old man who is positive for human immu-
minute. She has cervical lymphadenopathy and mild
nodeficiency virus (HIV) presents with progressive
diffuse abdominal tenderness. Findings on the rest of
headache, irritability, and low-grade fever over the
the examination, including mouth, lungs, heart, and
past month. A friend who brought him to the emer-
pelvis, are unremarkable. Laboratory evaluation shows
gency department today is worried about his persis-
the following: hemoglobin 12.5 g/dL, leukocyte count
tent complaints. The patient has not been seen in the
8.010 9/L, and platelet count 200109/L; atypical
clinic for 3 years and has not taken antiretrovirals
lymphocytes are noted on the peripheral smear, the
for more than 2 years. At the initial diagnosis 8 years
serum creatinine is normal (1.0 mg/dL), and liver func-
ago, cytomegalovirus (CMV) serology was negative,
tion test values (aspartate aminotransferase, alanine
serum Toxoplasma IgG antibody test results were
aminotransferase, bilirubin, and alkaline phosphatase)
positive, hepatitis serology tests were negative, and
are all within the reference range. Which statement is
the tuberculin skin test was negative. His CD4 count
false?
was 145 cells/L but increased to 340 cells/L with
a. Results of HIV antibody testing will likely be negative.
treatment. He reports that his headache has become
b. If the HIV test results are positive, genotyping should be
progressively disabling. He has not worked as a waiter
performed.
for the past week. His temperature is 37C, his blood
c. Testing for acute HIV, Epstein-Barr virus, and cytomega-
pressure is 120/75 mm Hg, and his heart rate is 88
lovirus should be done if her symptoms persist for several
beats per minute. He has questionable nuchal rigid-
more weeks, but it is not currently indicated.
ity, but no other neurologic findings are noted. The
d. If she has acute HIV, her HIV viral load by polymerase chain
ophthalmoscopic examination is normal, without
reaction should be more than 100,000 copies/mL.
papilledema. His CD4 cell count is 55 cells/L, and
e. Patients with undiagnosed HIV infection often deny being
HIV-1 RNA is 100,000 copies/mL. A computed
at risk.
tomographic (CT) scan is normal. Which of the fol-
lowing is most appropriate? 21. A 43-year-old woman with long-standing human immu-
a. Empirical fluconazole therapy nodeficiency virus (HIV) infection and hepatitis C
b. Empirical treatment with pyrimethamine and sulfadiazine began a salvage program of stavudine/tenofovir/lami-
c. Lumbar puncture and cerebrospinal fluid (CSF) studies vudine/darunavir with boosted ritonavir in combination
d. CMV serology with raltegravir about 4 months ago. She is also taking
e. Magnetic resonance imaging (MRI) of the head pravastatin and fish oil for hyperlipidemia. In a routine
appointment 6 weeks ago, she reported mild nausea and
19. A 36-year-old man has a severe cough and is found to fatigue. A pregnancy test was negative. Her viral load
have lobar pneumonia. He is treated with levofloxacin was suppressed for the first time in 4 years. Her CD4
and improves over the next 2 weeks. He denies having count had increased from 205 to 240 cells/L. Her ala-
risk factors for sexually transmitted and blood-borne nine aminotransferase (ALT) increased slightly from 50
diseases. A human immunodeficiency virus (HIV) to 75 U/L. She reports feeling increasingly unwell, and
test is done, and results of both the enzyme-linked her appetite has been poor. She believes that she may
immunosorbent assay (ELISA) and the Western blot have lost weight. In the past 3 days she has had new dif-
are positive. Which of the following is the most likely fuse abdominal discomfort. Laboratory values are as
explanation? follows: ALT 200 U/L, glucose 115 mg/dL, leukocyte
a. Levofloxacin can interfere with HIV testing, and the test count 8.0109/L, hemoglobin 12.5 g/dL, CD4 count
should be repeated in 1 month. 245 cells/L, HIV viral load nondetectable, and lactate
b. Acute infections can cause a false-positive HIV antibody test 12 mmol/L. Which, if any, of her medications is likely
result, and he can be reassured that it is likely a false-positive responsible for her symptoms and elevated lactate?
result. The test should be repeated in 3 months. a. The protease inhibitor darunavir
c. The result is a true-positive, and he has not been forthcom- b. Nucleoside reverse transcriptase inhibitors (NRTIs), espe-
ing about his risk factors. cially stavudine
d. The result is likely a false-positive from laboratory error. c. None of her HIV medications, but instead an HIV-associated
e. He should be asked whether he is taking any over-the-counter infection
herbal supplements since products containing ginseng can d. Pravastatin combined with the ritonavir
interfere with the test. e. Raltegravir
20. A 30-year-old married woman presents with a his- 22. A 36-year-old surgical nurse was stuck with a suture
tory of low-grade fever, malaise, sore throat, anorexia, needle while assisting with emergent surgery of a patient
abdominal discomfort, and diarrhea of 10 days dura- who is human immunodeficiency virus (HIV) positive.
tion. She denies having risk factors for human immu- She saw blood in her glove and called the employee
nodeficiency virus (HIV) infection. On physical health office for recommendations. Which of the fol-
examination, her temperature is 38.0C, her blood lowing is false?

48 M AYO C L I N I C I N T E R N A L M E D I C I N E B OA R D R E VI EW: Q U E S T I O N S A N D A N S W E R S
a. Initiation of postexposure prophylaxis with zidovudine in enlarged prostate but normal urodynamics. Urinalysis
combination with lamivudine will significantly decrease her results are the following: 10 to 20 leukocytes per
risk of acquiring HIV. high-power field, a positive leukocyte esterase test, a
b. Postexposure prophylaxis should be continued if tolerated negative nitrite test, and numerous gram-negative rods.
for 12 weeks. Which of the following is the best choice for treatment
c. Use of 3 drugs should be considered if the surgical patient of his symptoms?
has known resistance to lamivudine. a. Nitrofurantoin 100 mg daily for 7 days
d. The risk of acquiring HIV from a needlestick injury is b. Amoxicillin 500 mg 3 times daily for 7 days
approximately 3 in 1,000 events. c. Ciprofloxacin 500 mg twice daily for 28 days
e. Postexposure prophylaxis is unlikely to be helpful if not d. Trimethoprim-sulfamethoxazole 1 tablet twice daily for
started within 72 hours. 14 days
e. Levofloxacin 500 mg once daily for 14 days
23. A 45-year-old man with a history of AIDS, including
a past history of pneumocystis pneumonia (PCP) and 26. A 51-year-old diabetic woman from Hyderabad,
cryptococcal meningitis, did well while incarcerated India, has had unrelenting fever and night sweats
and treated with efavirenz, tenofovir, and emtricitabine. for 2 weeks. Blood cultures have been negative and
His CD4 count gradually improved to 420 cells/L, an abdominal computed tomographic scan shows a
and his viral load was suppressed for over 2 years. When 36-cm hypodense lesion in the left lobe of her liver.
released, he was told to continue the 3 drugs and to follow Which of the following should be done next for this
up with a community human immunodeficiency virus patient?
clinic, but he never sought care. His sister convinced a. Ultrasound-guided needle aspiration
him to seek care 3 years after he was incarcerated, and he b. Serology test on blood
seems interested in restarting therapy. He reports feeling c. Stool test for ova and parasites
well except for mild fatigue, diarrhea (3 soft stools daily d. Piperacillin-tazobactam 3.375 g intravenously every 6 hours
accompanied by urgency) and a facial rash suggestive e. Purified protein derivative (PPD) skin test
of seborrheic dermatitis. On examination, he has mild
27. A 19-year-old woman reports that her urine has been
cervical, axillary, and inguinal adenopathy and thrush.
cloudy for the past 3 days. She is asymptomatic but has
Which of the following is not currently indicated?
missed her period for 3 months. A urine pregnancy test
a. Restarting trimethoprim-sulfamethoxazole (TMP-SMX)
is positive and a urine culture shows more than 100,000
prophylaxis against PCP
colonies of a gram-negative rod. Which of the follow-
b. Checking the viral load with genotyping
ing should be recommended for management of her
c. Purified protein derivative skin test or QuantiFERON-TB
bacteriuria?
Gold blood test for tuberculosis
a. Ciprofloxacin 500 mg twice daily for 3 days
d. Azithromycin prophylaxis against Mycobacterium avium-
b. Trimethoprim-sulfamethoxazole 1 tablet twice daily for
intracellulare (MAI) complex
14 days
e. Ketoconazole cream and shampoo
c. Nitrofurantoin 100 mg daily for 3 days
24. A 28-year-old man who is regularly sexually active with d. Ceftriaxone 1 g intravenously daily for 14 days
multiple partners, both male and female, asks what he e. No treatment
can do (other than decreasing his sexual activity) to
28. A 63-year-old woman with acute myelogenous leuke-
lower his risk of acquiring human immunodeficiency
mia (AML) is treated with daunorubicin and cytara-
virus (HIV). Which of the following will not decrease
bine. She has been neutropenic for 3 weeks and febrile
his risk of acquiring HIV?
for 2 weeks despite taking meropenem, vancomycin,
a. Use of latex condoms
and acyclovir. A computed tomographic scan of her
b. Addition of nonoxynol spermicide in addition to condoms
chest shows a large, wedge-shaped peripheral infiltrate
c. Taking tenofovir in combination with emtricitabine on a
with some central clearing in the left lower lobe. Which
preventive basis
of the following antifungal agents should be most effec-
d. Circumcision
tive for treatment?
e. Engaging in only insertive sex
a. Caspofungin
b. Itraconazole
c. Voriconazole
I N F EC T I O US D I S E A S E SY N D RO M E S : U R I NA RY d. Amphotericin B
T R AC T I N FEC T I O N, S E XUA L LY T R A NS M I T T E D e. Fluconazole
I N F E C T I O N, A N D G A S T RO I N T E S T I NA L T R AC T
29. Which of the following is true about Clostridium dif-
I N FEC T I O N
ficile infection?
25. An 87-year-old man presents with pelvic pain and dys- a. Vancomycin is less effective than metronidazole for treat-
uria. He has had 3 urinary tract infections in the past ment of severe disease.
6 months. A urologic evaluation last month showed an b. Most relapses are due to antibiotic resistance.

5. I N F E C T I O U S D I S E A S E S Q U E S T I O N S A N D A N S W E R S 49
c. Vancomycin-resistant enterococcal (VRE) colonization 33. A 60-year-old man with a recently implanted perma-
occurs more frequently with oral vancomycin treatment nent pacemaker (3 months ago) is admitted to the hos-
than with metronidazole. pital because he has had increasing pain, swelling, and
d. A single recurrence of infection increases the likelihood of erythema at the site of his pacemaker pocket. He has
further relapses. a history of type 2 diabetes mellitus, coronary artery
e. A stool sample for Clostridium difficile toxin should be disease, and third-degree heart block. His symptoms
obtained at the end of treatment to document cure. started 3 days ago and are rapidly progressing. He is
febrile on admission, and blood cultures are grow-
30. A 47-year-old man comes to your office with a 4-day
ing gram-positive cocci resembling staphylococci at
history of malaise, a diffuse rash involving his trunk
24 hours. A transesophageal echocardiogram is nega-
and extremities, swollen glands, and headache. His
tive for any evidence of endocarditis. What is the most
symptoms began 3 days after taking ciprofloxacin for
appropriate management for this patient?
gonorrhea. At that time, his syphilis IgM and IgG were
a. Start intravenous vancomycin and delay device explantation
positive (titer 1:16). His rapid plasma reagin (RPR)
until subsequent blood cultures are negative.
titer was 1:128, and his human immunodeficiency virus
b. Start intravenous vancomycin and immediately proceed to
(HIV) quantitative viral load and HIV antibody test
device explantation.
were both negative. Which of the following should you
c. Device removal is not necessary since a pacemaker pocket
also recommend?
infection can be cured with combination therapy with van-
a. Lopinavir-ritonavir plus zidovudine-lamivudine
comycin and rifampin.
b. Doxycycline 100 mg twice daily for 3 days
d. Start intravenous cefazolin and proceed with device removal
c. Benzathine penicillin 2.4 million units intramuscularly
as soon as possible.
d. Lumbar puncture
e. Start intravenous daptomycin and oral rifampin and reassess
e. Azithromycin 2 g orally
in 72 hours.
31. A 43-year-old morbidly obese diabetic woman, recently
34. Which of the following has not been shown to reduce
treated for abdominal wall cellulitis, now presents with
the incidence of central venous catheterrelated blood-
a 3-day history of fever, right flank pain, and dysuria.
stream infections (CRBSIs)?
A urinalysis is esterase positive and a Gram stain shows
a. Chlorhexidine patch applied to the exit site
gram-negative bacilli too numerous to count. Which
b. Routine catheter exchange over a guidewire at 72 hours
of the following treatments should you recommend for
c. Minocycline-rifampincoated central venous catheters
management of her urinary tract infection?
d. Silver sulfadiazinecoated central venous catheters
a. Ceftriaxone 1 g intravenously daily for 10 days
e. A standardized catheter insertion bundle
b. Ampicillin 2 g intravenously every 6 hours for 10 days
c. Ciprofloxacin 500 mg orally twice daily for 7 days 35. A 40-year-old woman with recurrent episodes of
d. Trimethoprim-sulfamethoxazole (TMP-SMX) 160480 mg sinusitis presents to the emergency department with
orally twice daily for 7 days a 1-week history of fever and constant headache. A
e. Nitrofurantoin 100 mg orally twice daily for 10 days computed tomographic scan of the head shows opaci-
fication of the frontal and sphenoid sinuses and an
abscess in the left frontal lobe. Which of the follow-
I N FE C T I O US D I S E A S E SY N D RO M E S :
ing would be the best initial antimicrobial regimen for
C A R D I O VA S C U L A R , B L O O D S T R E A M , A N D
this patient?
C E N T R A L N E RVO US SYS T E M I N FEC T I O NS
a. Vancomycin
32. A 70-year-old woman is undergoing her second cycle of b. Piperacillin-tazobactam
chemotherapy for breast cancer. On her third hospital c. Cefazolin and metronidazole
day, she has a fever (38.9C) and you notice a red track d. Vancomycin, ceftriaxone, and a lipid formulation of
along her Hickman catheter tunnel. Cultures from amphotericin
catheter-drawn blood and from peripheral vein blood e. Vancomycin, ceftriaxone, and metronidazole
are negative at 48 hours. Which of the following should
36. Which of the following patients should receive antibi-
you recommend for management of her febrile illness?
otic prophylaxis for infective endocarditis?
a. Intravenous vancomycin and cefepime for 14 days
a. A 72-year-old man who has a dual chamber permanent pace-
b. Intravenous vancomycin and vancomycin lock therapy for
maker and is undergoing dental extraction
14 days
b. A 55-year-old woman who has a history of aortic valve endo-
c. Removal of the Hickman catheter and intravenous vanco-
carditis and is scheduled for placement of an orthodontic
mycin for 14 days
appliance
d. Transesophageal echocardiography to determine the dura-
c. A 65-year-old man who has severe mitral valve regurgitation
tion of antibiotic therapy
and is undergoing endoscopy for evaluation of epigastric
e. Topical neomycin, polymixin B, and bacitracin to the exit
pain
site and intravenous vancomycin for 7 days

50 M AYO C L I N I C I N T E R N A L M E D I C I N E B OA R D R E VI EW: Q U E S T I O N S A N D A N S W E R S
d. A 60-year-old heart transplant recipient who has cardiac 38. A 57-year-old diabetic man is seen in the emergency
valvulopathy and is undergoing a dental extraction department for fever, productive cough, headache,
e. A 32-year-old woman who has a history of surgically repaired and altered mental status for the past 6 hours. On
congenital heart disease in childhood and is undergoing a examination, he is febrile (39C) and lethargic,
root canal and he has prominent neck stiffness. A chest radio-
graph shows a dense right lobar consolidative infil-
37. A 55-year-old woman who has a history of mitral valve
trate. His leukocyte count is 22.010 9/L, and his
replacement with a mechanical prosthesis is admitted
creatinine level is 1.0 mg/dL. A lumbar puncture
with a 1-week history of fever, malaise, and shortness
shows an opening pressure of 28 mm water, leuko-
of breath. Admission blood cultures are positive for
cyte count 30.010 9/L (88% neutrophils), protein
methicillin-resistant Staphylococcus aureus (MRSA),
168 mg/dL, and glucose 22 mg/dL. Gram staining
and a transesophageal echocardiogram is consistent
shows gram-positive diplococci. Which of the fol-
with a 5-mm vegetation on the anterior leaflet of the
lowing is the most appropriate regimen for empirical
mitral valve. Which of the following is the most appro-
treatment?
priate antibiotic regimen for this patient?
a. Ampicillin, ceftriaxone, and vancomycin
a. Vancomycin for 6 weeks
b. Ceftriaxone and dexamethasone
b. Vancomycin and gentamicin for 6 weeks
c. Ceftriaxone and vancomycin
c. Daptomycin and gentamicin for 6 weeks, with rifampin for
d. Ceftriaxone, vancomycin, and acyclovir
the first 2 weeks of therapy
e. Ceftriaxone, vancomycin, and dexamethasone
d. Vancomycin and rifampin for 6 weeks, with gentamicin for
the first 2 weeks of therapy
e. Vancomycin and rifampin for 4 weeks

5. I N F E C T I O U S D I S E A S E S Q U E S T I O N S A N D A N S W E R S 51
ANSWER S

1. Answer e. Legionella infections can progress rapidly and are often asso-
ciated with diarrhea and other gastrointestinal tract symp-
Initial assessment of the severity of community-acquired
toms. Alcohol abuse is a known epidemiologic risk factor,
pneumonia is important for internal medicine physicians to
and Legionella often causes outbreaks among persons with
reduce unnecessary hospitalization and to identify patients
common-source exposure. Relative bradycardia and mental
who are at higher risk of death or who need more immediate
status changes frequently occur in patients with Legionella
intervention. Illness severity scores such as the pneumonia
infection. Leukocytosis, hyponatremia, and patchy bilat-
severity index (PSI) and CURB-65 (confusion, urea nitro-
eral infiltrates occur frequently in Legionella infections,
gen, respiratory rate, blood pressure, and 65 years or older)
but those features may not be as helpful for discriminating
have been developed to help in the decision for site of care
between other microbiologic causes of community-acquired
for patients with community-acquired pneumonia. In the
pneumonia. RSV is unlikely to cause respiratory illness of
outpatient setting, the modified CRB-65 (confusion, respi-
this severity in an adult without immunocompromising
ratory rate, blood pressure, and 65 years or older) score is
conditions. Enterovirus is a cause of encephalitis but is not
useful since it does not require laboratory or radiographic
a cause of community-acquired pneumonia. When C psit-
evidence to determine the severity score. In this question, the
taci causes atypical community-acquired pneumonia, it is
patient is not confused, does not have a respiratory rate of
generally not rapidly progressive or associated with relative
30 breaths per minute or more, does not have a systolic blood
bradycardia or gastrointestinal tract symptoms. The patient
pressure less than 90 mm Hg or a diastolic blood pressure of
has not been to a region where C immitis is endemic.
60 mm Hg or less, but is 65 years or older. His CRB-65 score
is 1. Patients with scores of 0 or 1 can generally be treated as 4. Answer e.
outpatients if they can reliably take oral antimicrobials and
have outpatient support resources. (See Mandell et al and This question relates to knowledge of modifiable risk fac-
Capelastegui et al in the Suggested Reading list.) tors to reduce the risk of ventilator-associated pneumo-
nia among patients receiving mechanical ventilation. The
2. Answer b. correct answer and incorrect answers (modified to be
This question is related to appropriate initial antimicrobial negative) were taken directly from the guideline recom-
treatment of community-acquired pneumonia in a patient mendations on health careassociated pneumonia. (See
with medical comorbidities who requires hospitalization but American Thoracic Society and Infectious Diseases Society
not intensive care unit admission. Recommended treatment of America in the Suggested Reading list.)
is either a respiratory fluoroquinolone or a -lactam antibiotic
in combination with a macrolide antibiotic. Azithromycin 5. Answer c.
alone is not a recommended regimen in patients with medical This question is based on an understanding of the epidemi-
comorbidities, including diabetes mellitus. Ciprofloxacin does ology of hospital-acquired pneumonia and how it relates
not have sufficient coverage for Streptococcus pneumoniae and is to the choice of empirical antimicrobials for its treatment.
thus not considered a respiratory fluoroquinolone. Although The hospital prevalence of methicillin-resistant S aureus
intravenous ampicillin would be an acceptable -lactam anti- and multidrug-resistant gram-negative organisms, such as
biotic antimicrobial choice, clindamycin is not a macrolide P aeruginosa, A baumannii, and the enterobacteriaceae,
antibiotic and the patient does not have risk factors to war- need to be considered when deciding on an antimicrobial
rant empirical coverage of methicillin-resistant Staphylococcus regimen. The only correct pair of choices is S aureus and
aureus. Meropenem is a very broad-spectrum antimicrobial P aeruginosa. Enterococci and C albicans are respiratory
and is not recommended for routine use for hospitalized pathogens in only select hosts and are very rare causes of
patients with community-acquired pneumonia who do not hospital-acquired pneumonia. Although influenza virus,
have risk factors for infection with Pseudomonas aeruginosa. S pneumoniae, and H influenzae can cause hospital-acquired
(See Mandell et al in the Suggested Reading list.) pneumonia, they generally are involved in a relatively small
3. Answer a. proportion of nosocomial cases and their resistance pro-
files are generally those of the community-acquired strains.
This question involves recognition of Legionella as a cause (See American Thoracic Society and Infectious Diseases
of community-acquired pneumonia. In healthy hosts, Society of America in the Suggested Reading list.)

52
6. Answer a. 11. Answer c.
This question is based on an understanding of the microbiol- This diabetic man has peripheral vascular disease and a fetid
ogy and subsequent antimicrobial prophylaxis ramifications foot ulcer with surrounding cellulitis, which can be probed
of cat bites. Major pathogens isolated from cat bites include to the bone. He is manifesting systemic toxicity. The first
numerous anaerobes, streptococci, and staphylococci but step in management would be initiation of antimicrobial
most commonly Pasteurella multocida. Prophylactic anti- therapy to cover MRSA and a mixed infection. The infec-
microbials are recommended unless the bite wound is very tion will be polymicrobial, and a culture would have limited
superficial, and P multocida is generally resistant to diclox- value. A nasal swab for MRSA does not establish the pres-
acillin, cephalexin, clindamycin, and erythromycin but is ence of MRSA in the wound. An MRI offers no additional
generally susceptible to amoxicillin-clavulanic acid. (See immediate value at this stage, especially since the wound
Oehler et al in the Suggested Reading list.) can be probed to the bone and is likely osteomyelitic.
7. Answer d. 12. Answer a.
This question relates to a syndrome of a potential bioter- The QuantiFERON-TB Gold test can be done in all cir-
rorism agent. The patient was previously healthy, and a cumstances in which a PPD test is indicated. Its principal
life-threatening illness developed quickly, suggesting a highly role may be in sorting out false-positive tuberculin skin test
aggressive pathogen. The widened mediastinum is suggestive results in patients who received BCG vaccine since it does
of inhalational anthrax but could also be seen with diseases not cross-react with nontuberculous mycobacteria.
such as tularemia, histoplasmosis, and tuberculosis. The quick
13. Answer e.
positivity of the blood cultures with gram-positive bacilli in
this clinical scenario strongly suggests anthrax, which should Pyrazinamide can cause hyperuricemia and gout. Uric acid
not be excluded as a contaminant. (See Quintiliani and levels may need to be monitored in patients at risk who
Quintiliani in the Suggested Reading list.) are receiving pyrazinamide. Key side effects of rifampin
are rash, drug interactions due to induction of hepatic
8. Answer c.
microsomal enzymes, hepatotoxicity, and orange secre-
The patient has type 1 necrotizing fasciitis, which is most tions. Key side effects of isoniazid are hepatotoxicity,
often a mixed infection involving aerobic and anaerobic peripheral neuropathy Lupus-like syndrome, and mono-
organisms. Risk factors for this type of infection are gas- amine (histamine-tyramine) poisoning. Generally, pyri-
trointestinal tract procedures, diabetes mellitus, or vascular doxine has no side effects except for remote peripheral
disease. neuropathy. Side effects of ethambutol include retrobulbar
neuritis and decreased red-green color discrimination.
9. Answer e.
14. Answer c.
This patient has staphylococcal toxic shock syndrome, which
may be associated with the retained packing after the nasal There are many drug interactions with rifampin that have
procedure. Often patients receive clindamycin until the pack- potentially serious consequences. Rifampin is an impor-
ing is removed. The features of staphylococcal toxic shock tant inducer of hepatic cytochrome P450 enzymes, which
syndrome are due to toxin-mediated cytokine activation decrease the effects of drugs such as glyburide, verapamil, war-
and can occur 1) after surgical and postpartum procedures; farin, phenytoin, and many others. These interactions should
2) with mastitis, sinusitis, burns, and skin and soft tissue infec- always be considered when the use of rifampin is begun or
tions (especially of the extremities, perianal area, and axillae); stopped since doses may need to be adjusted or medications
and 3) with respiratory infections after influenza. may need to be changed. For example, verapamil may need to
be changed to an alternative antihypertensive medication.
10. Answer d.
15. Answer e.
There are OPAT guidelines for most infectious diseases.
Often OPAT is used for infections (eg, bone and joint These cases focus on the definitions of a positive PPD skin
infections) that require prolonged parenteral therapy. Key test according to the guidelines of the Centers for Disease
tenets of OPAT include the following: 1) the patient or Control and Prevention. A TST result of more than 5 mm
caregiver (or both) is willing to participate and can safely, is positive in a person who meets any of the following cri-
effectively, and reliably deliver OPAT; 2) a physician or teria: is positive for HIV, had recent contact with a person
home care agency is available and accessible for communi- who had pulmonary tuberculosis (ie, a new conversion),
cations about problems and for monitoring; 3) the active has a chest radiograph consistent with old untreated tuber-
infectious diseases are stable and the patient has no active, culosis, or has received an organ transplant or tumor necro-
new problems, is not at increased risk of complications, sis factor inhibitor treatment.
and has received more than 6 days of therapy for bacterial
16. Answer a.
meningitis; and 4) the patient can pay for treatment, has a
safe and adequate home or outpatient environment to sup- This patients treatment was self-administered for only
port care, and is not actively using illicit drugs. 1 week, and he is still symptomatic; thus, he should be

5. I N F E C T I O U S D I S E A S E S Q U E S T I O N S A N D A N S W E R S 53
considered infectious. The other patients are no longer recognized as the most common presentation, but other
considered infectious. symptoms, including prominent gastrointestinal tract
symptoms, frequently occur. Unfortunately, acute HIV is
17. Answer a.
often overlooked as a possibility, even when patients seek
The patient has large joint monoarticular arthritis and inflam- medical care. Symptomatic acute HIV does spontaneously
matory synovial fluid. The leukocyte count of 65109/L with resolve, and, if acute HIV is being considered, observation
65% polymorphonuclear cells suggests inflammation more is not appropriate. HIV antibody testing with the acute
than purulent infection. All the choices except acute sickle retroviral illness should be negative or indeterminate. HIV
cell crisis could give this clinical picture. The differential diag- antibody testing will usually seroconvert by 3 months but
nosis includes crystalline and infectious arthritis. Sickle cell may take up to 6 months after exposure to become positive.
crisis usually causes a hemorrhagic fluid and is not associated Patients should have genotyping done at diagnosis since
with this type of cell count unless coinfected with Salmonella. resistance is becoming more common. HIV often reverts
Gonococcal arthritis may appear inflammatory and in a back to wild-type virus, although the resistant virus is still
patient with another sexually transmitted disease (HIV), this present in the patient. Genotyping is therefore recom-
should be considered. The leukocyte count is usually greater mended as soon as possible after diagnosis. Many patients
than 50109/L (typically >90% polymorphonuclear cells). In who engage in behaviors that place them at risk of acquir-
gonococcal arthritis, intracellular gram-negative diplococci ing HIV deny being at risk because of the stigma attached
are found in less than 25% of synovial fluid aspirates. The syn- to these behaviors and because others do not know that
ovial fluid should be cultured on prewarmed chocolate agar they are at risk since they do not realize that their partner
for highest yield (positive findings in only 50% of patients is engaging in risky behavior or is HIV positive.
with gonococcal arthritis and 25% to 30% of patients with
21. Answer b.
disseminated gonococcal infection). Genital and oral sites
should be cultured for Neisseria gonorrhoeae. NRTIs have been associated with asymptomatic, mild ele-
vations in lactate levels (which can generally be ignored)
18. Answer c.
and with more severe symptomatic lactic acidosis, which
The patient is clinically stable, so empirical fluconazole or can lead to profound illness and death. The most common
ertapenem is not indicated. Since he does not have a mass symptoms of lactic acidosis in HIV-infected patients are
lesion, empirical treatment for toxoplasmosis is inappro- the subacute development of nausea, vomiting, abdominal
priate. His symptoms are consistent with possible central pain, fatigue, weakness, and weight loss. Tachypnea, dys-
nervous system cryptococcosis. His CD4 count is less than pnea with exertion, arrhythmias, and neurologic findings
100 cells/L, placing him at risk. A CSF examination with have also been reported in the absence of gastrointestinal
a cryptococcal antigen test is most appropriate, and a lum- tract symptoms. Liver test results are generally abnormal.
bar puncture should be safe in the absence of papilledema. The combination of stavudine and didanosine, although
If the CSF is normal, an MRI can be reconsidered to look popular in the past, is no longer recommended because it
for small lesions or other abnormalities that may be missed carries an increased risk of lactic acidosis and neuropathy.
on the CT scan. Patients with certain risk factors (preexisting liver disease,
lower CD4 counts, decreased glomerular filtration rate,
19. Answer c.
and being female or pregnant) seem to be at increased risk
Medications and herbs are not known causes of false-positive of symptomatic lactic acidosis. Stavudine is thought to be
HIV antibody tests. Laboratory error can occur but is unusual, the NRTI most strongly associated with lactic acidosis, but
and positive results should be confirmed. Acute infection all NRTIs can probably cause lactic acidosis and the more
can lead to cross-reacting antibodies and, occasionally, to a NRTIs used at a time, the more likely lactic acidosis is to
false-positive ELISA result; however, the Western blot is very occur.
specific, and it should be negative or indeterminate if a patient
22. Answer b.
is not infected with HIV and has not participated in a vac-
cine trial. Patients are often reluctant to speak openly about A case control study has shown a 79% decreased risk
their risk factors for HIV because of the stigma attached to of acquiring HIV from use of zidovudine for 4 weeks
many of these behaviors and a desire to be respected by their promptly after a percutaneous exposure. Zidovudine in
caregivers. The Centers for Disease Control and Prevention combination with lamivudine should be equally or more
recommends that all patients undergo screening at least once, effective. Tenofovir in combination with emtricitabine
regardless of risk, between ages 15 and 65. Pneumonia (par- is an alternative that needs to be taken only once daily.
ticularly pneumococcal pneumonia) is more common in Prophylaxis is recommended for 4 weeks, and there is no
patients with HIV than in uninfected adults even when the information that a longer period will result in fewer infec-
CD4 count is relatively preserved. tions after exposure. The addition of a third drug (gener-
ally a protease inhibitor) should be considered when the
20. Answer c.
patient has known drug resistance, or the risk of transmis-
Symptomatic acute HIV infection occurs in up to 85% sion is felt to be high because of the type of exposure (ie,
of persons with HIV. A mononucleosis-like syndrome is deep puncture wound, hollow needle with visible blood, or

54 M AYO C L I N I C I N T E R N A L M E D I C I N E B OA R D R E VI EW: Q U E S T I O N S A N D A N S W E R S
a very high viral load). The estimated risk of acquiring HIV negative in the absence of antimicrobial therapy, a bacte-
from a needlestick is 3 in 1,000, but, as noted above, some remic seeding of the liver is less likely. Common bacterial
exposures are more risky than others. In animal studies, causes of liver abscess include enteric organisms from the
prompt initiation of treatment (within hours) was associ- biliary tree, viridans group streptococci from the digestive
ated with decreased transmission; initiation after 72 hours tract, and Staphylococcus aureus secondary to bacteremia. In
is unlikely to be of any benefit. India, Entamoeba histolytica infection is common and may
manifest as a febrile illness with a focal liver mass. The best
23. Answer d.
test is blood serology for E histolytica. Ultrasound-guided
Thrush is an indication for PCP prophylaxis. TMP-SMX needle aspiration is useful for many liver lesions but should
prophylaxis needs to be restarted, and the patient should be delayed pending the serology results. A stool test for ova
restart antiretroviral medication as soon as possible. A viral and parasites is not specific, and ameba may not be seen
load with genotype should first be determined to guide at the stage of liver abscess. Empirical antibiotic therapy
therapy. It is likely that his seborrheic dermatitis, fatigue, and should be avoided until the diagnosis is established. A PPD
diarrhea will improve with immune reconstitution. His past skin test is likely to be positive and not helpful in establish-
history of cryptococcal meningitis puts him at high risk of ing the cause of the liver lesion.
a recurrence when his CD4 count is less than 200 cells/L,
27. Answer c.
and fluconazole therapy should be restarted despite his lack
of symptoms if his CD4 count is less than 200 cells/L. This patient is pregnant and has asymptomatic bacteriuria
Azithromycin is not indicated unless his CD4 count is less with a gram-negative rod. She appears to be in the second
than 50 cells/L and symptomatic MAI infection has been trimester. Asymptomatic bacteriuria in pregnancy is defined
excluded. Treatment of his seborrheic dermatitis with keto- as more than 100,000 colony-forming units per milliliter
conazole cream and shampoo would be reasonable therapy. and is associated with worse pregnancy outcomes. Thus, it
is one of the few situations in which asymptomatic bacteri-
24. Answer b.
uria should be treated. Ciprofloxacin and other fluoroqui-
If used correctly, latex condoms decrease the risk of acquir- nolones are contraindicated in pregnancy and should be
ing or transmitting HIV by 87% among heterosexuals who avoided. Two weeks of trimethoprim-sulfamethoxazole is
consistently use a condom. Condom failure may be more excessive for asymptomatic bacteriuria. Ceftriaxone is use-
common with anal intercourse. Nonoxynol-9 spermicide ful for treatment of urinary tract infections in pregnancy
does not decrease the risk of acquiring HIV and can increase but is not needed for asymptomatic bacteriuria with a sus-
the risk in women by causing mucosal irritation. Using a ceptible organism. Nitrofurantoin can be safely used in
condom with nonoxynol-9 is safer than not using a condom. pregnancy and a 3- to 7-day course is adequate for asymp-
A randomized study of men who have sex with men showed tomatic bacteriuria. A urine culture should be repeated
that continuous treatment with tenofovir in combination 2 weeks after completion of the treatment.
with emtricitabine decreased the relative risk of acquiring
28. Answer c.
HIV by 44% overall and by 73% among those who reported
over 90% adherence. Circumcision significantly decreases This woman has AML with prolonged febrile neutropenia
the risk of HIV acquisition by men in heterosexual relation- and a wedge-shaped peripheral pulmonary infiltrate that
ships by 60%. Insertive anal or vaginal sex is less risky for has not resolved with broad-spectrum antibiotics. The
acquiring HIV than receptive anal or vaginal sex. leading infectious cause is invasive fungal infection, partic-
ularly aspergillosis or, less frequently, mucormycosis. The
25. Answer c.
appropriate therapy in the absence of voriconazole prophy-
This elderly man has had recurrent episodes of urinary laxis is the addition of empirical voriconazole to treat pre-
tract infection. In the absence of a neurogenic bladder sumed invasive aspergillosis while attempting to establish
or other secondary cause, this almost always reflects an the diagnosis. Caspofungin is often used as second-line or
infected prostate. The diagnosis of chronic prostatitis is add-on therapy. Amphotericin B is more toxic and may be
a clinical one, based on symptoms and signs of recurrent less effective. Fluconazole has no activity against Aspergillus,
urinary tract infection as seen in this patient. Antibiotics and itraconazole is less effective than voriconazole.
that penetrate well into the prostate include fluoroquino-
29. Answer d.
lones, trimethoprim-sulfamethoxazole, and doxycycline.
Amoxicillin and nitrofurantoin penetrate poorly into the Clostridium difficile is the most common cause of infec-
prostate. The duration of therapy should be prolonged to tious diarrhea in hospitalized patients. Those who have
reduce the rates of recurrence. Thus, a 28-day course of cip- 1 episode are predisposed to recurrences and those with 2
rofloxacin would be the best choice for management of this or more episodes are at highest risk for multiple relapses.
patients chronic prostatitis due to Escherichia coli. Oral vancomycin is more effective than metronidazole
for treatment of severe disease. Most relapses result from
26. Answer b.
reinfection or germination of spores, not from antibiotic
This diabetic woman from India has a large abscess in resistance. VRE colonization often occurs in similar pop-
the left lobe of her liver. Since blood cultures have been ulations of patients who have received broad-spectrum

5. I N F E C T I O U S D I S E A S E S Q U E S T I O N S A N D A N S W E R S 55
antibiotics; however, there is no difference in VRE colo- 2-coated catheters and chlorhexidine patches, have also
nization whether patients receive vancomycin or metron- been effective. Coated catheters are used when other mea-
idazole for treatment of C difficile infection. Because stool sures do not reduce the rate of central venous CRBSIs.
toxin assays may remain positive during and after success- Routine guidewire exchange of catheters does not reduce
ful treatment, follow-up stool toxin assays for test of cure central venous CRBSI rates. (See OGrady et al in the
should be avoided. Suggested Reading list.)
30. Answer d. 35. Answer e.
This 47-year-old man was recently treated for gonorrhea This patient has a brain abscess from a contiguous focus
and has secondary syphilis with multiple systemic fea- originating from the sinuses. The microorganisms to cover
tures, including headache. He has a high positive RPR empirically are those often associated with sinus infec-
titer (>1:32), which should indicate the need to consider a tions, which are usually polymicrobial infections involv-
lumbar puncture to exclude neurosyphilis before initiating ing streptococci, Haemophilus influenzae, staphylococci,
therapy. and anaerobes. Because of increasing methicillin-resistant
Staphylococcus aureus (MRSA) in the community, initial
31. Answer c.
therapy should cover MRSA; thus, cefazolin would not be
This woman has acute uncomplicated pyelonephritis. the best choice. Empirical antifungal therapy is not needed,
She has been recently exposed to antimicrobials, so she nor is anti-Pseudomonas coverage. (See Carpenter et al in
is at increased risk for TMP-SMX and ampicillin resis- the Suggested Reading list.)
tance. Thus, an oral fluoroquinolone is the drug of choice.
36. Answer d.
Intravenous therapy is not necessary. Nitrofurantoin,
although effective for cystitis, is not appropriate for In 2007, the American Heart Association issued revised
pyelonephritis. guidelines for antimicrobial prophylaxis for endocarditis:
The only patients who should receive antibiotic prophy-
32. Answer c.
laxis are those who are scheduled to undergo invasive den-
This patient has tunnel infection of her indwelling catheter. tal procedures and who have a prosthetic heart valve, a prior
She is not bacteremic. Essentially, this is a soft tissue infec- history of endocarditis, surgically uncorrected congenital
tion with a device in place (similar to a pocket infection for heart disease, or cardiac valvulopathy after undergoing
a pacemaker). The most appropriate management is removal cardiac transplant. Invasive dental procedures are defined
of the catheter, along with antimicrobial therapy directed at as those that involve manipulation of gingival tissue or the
the likely cause of the soft tissue infection: coagulase-negative periapical region of teeth or perforation of oral mucosa.
staphylococci, streptococci, or Staphylococcus aureus. Two Administration of antibiotic prophylaxis solely to prevent
weeks of therapy without removal of the catheter is not sat- endocarditis is not recommended for patients undergoing
isfactory, nor is lock therapy, which is useful for intralumi- gastrointestinal tract or genitourinary tract procedures.
nal infections but not tunnel infections. Topical therapy is The presence of a permanent pacemaker with transvenous
effective for only localized exit site infections. (See Mermel cardiac leads is not an indication for antibiotic prophylaxis
et al in the Suggested Reading list.) before invasive dental procedures. (See Wilson et al in the
Suggested Reading list.)
33. Answer b.
37. Answer d.
Complete device removal is necessary in all cases of
pacemaker infections regardless of clinical presentation A patient with MRSA prosthetic valve endocarditis should
(ie, pocket infection or endocarditis). Delaying device receive antibiotic therapy for at least 6 weeks. With bio-
explantation until a blood culture is negative is not recom- film formation by staphylococci on prosthetic valves, the
mended. Because staphylococci (Staphylococcus aureus and addition of rifampin significantly improves cure rates of
coagulase-negative staphylococci) are the most common S aureus prosthetic valve endocarditis. Gentamicin is rec-
pathogens, vancomycin is the preferred drug for empirical ommended for the first 2 weeks of therapy, not for the
therapy. Cefazolin is not appropriate for empirical treat- entire 6-week course. Daptomycin is not superior to vanco-
ment because of the high rate of methicillin resistance in mycin and is reserved for situations in which vancomycin
staphylococci (up to 90% in coagulase-negative staphylo- cannot be used. (See Baddour et al, 2005, in the Suggested
cocci). Combination therapy with rifampin is not recom- Reading list.)
mended in pacemaker infections. (See Baddour et al, 2010,
38. Answer e.
and Sohail et al in the Suggested Reading list.)
This diabetic man has pneumococcal pneumonia and men-
34. Answer b.
ingitis. The most appropriate empirical therapy includes
Prevention of central venous CRBSIs has become a major the initial use of dexamethasone in combination with the
national issue. Studies have shown that use of a bundle of 2 antimicrobials to ensure coverage for -lactamresistant
evidence-based practices is effective at reducing central pneumococci. Dexamethasone (0.15 mg/kg every 6 hours
venous CRBSIs. Several different technologies, including for 24 days) should be started before or with the first dose

56 M AYO C L I N I C I N T E R N A L M E D I C I N E B OA R D R E VI EW: Q U E S T I O N S A N D A N S W E R S
of antibiotics. The greatest benefit is achieved in the most Mandell LA, Wunderink RG, Anzueto A, Bartlett JG, Campbell GD,
severely ill patients. There is no benefit when corticoster- Dean NC, et al; Infectious Diseases Society of America; American
Thoracic Society. Infectious Diseases Society of America/American
oids are given after antibiotic therapy has already begun. Thoracic Society consensus guidelines on the management of
There is no need to use acyclovir because the clinical pre- community-acquired pneumonia in adults. Clin Infect Dis. 2007 Mar
sentation and the spinal fluid findings are not consistent 1;44 Suppl 2:S2772.
with herpes simplex encephalitis. (See Tunkel et al in the Mermel LA, Allon M, Bouza E, Craven DE, Flynn P, OGrady NP,
Suggested Reading list.) et al. Clinical practice guidelines for the diagnosis and manage-
ment of intravascular catheter-related infection: 2009 Update by
the Infectious Diseases Society of America. Clin Infect Dis. 2009 Jul
1;49(1):145. Erratum in: Clin Infect Dis. 2010 Feb 1;50(3):457.
SUGGESTED RE ADING Clin Infect Dis. 2010 Apr 1;50(7):1079. Dosage error in article
text.
American Thoracic Society; Infectious Diseases Society of America. Oehler RL, Velez AP, Mizrachi M, Lamarche J, Gompf S. Bite-
Guidelines for the management of adults with hospital-acquired, related and septic syndromes caused by cats and dogs. Lancet Infect
ventilator-associated, and healthcare-associated pneumonia. Am J Dis. 2009 Jul;9(7):43947. Erratum in: Lancet Infect Dis. 2009
Respir Crit Care Med. 2005 Feb 15;171(4):388416. Sep;9(9):536.
Baddour LM, Epstein AE, Erickson CC, Knight BP, Levison ME, Lockhart OGrady NP, Alexander M, Burns LA, Dellinger EP, Garland J, Heard SO,
PB, et al; American Heart Association Rheumatic Fever, Endocarditis, et al; Healthcare Infection Control Practices Advisory Committee
and Kawasaki Disease Committee; Council on Cardiovascular (HICPAC). Guidelines for the prevention of intravascular
Disease in Young; Council on Cardiovascular Surgery and Anesthesia; catheter-related infections. Clin Infect Dis. 2011 May;52(9):e16293.
Council on Cardiovascular Nursing; Council on Clinical Cardiology; Epub 2011 Apr 1.
Interdisciplinary Council on Quality of Care; et al. Update on cardio- Quintiliani R Jr, Quintiliani R. Inhalational anthrax and bioterrorism.
vascular implantable electronic device infections and their manage- Curr Opin Pulm Med. 2003 May;9(3):2216.
ment: a scientific statement from the American Heart Association. Sohail MR, Uslan DZ, Khan AH, Friedman PA, Hayes DL, Wilson
Circulation. 2010 Jan 26;121(3):45877. Epub 2010 Jan 4. WR, et al. Management and outcome of permanent pacemaker and
Baddour LM, Wilson WR, Bayer AS, Fowler VG Jr, Bolger AF, Levison implantable cardioverter-defibrillator infections. J Am Coll Cardiol.
ME, et al; Committee on Rheumatic Fever, Endocarditis, and Kawasaki 2007 May 8;49(18):18519. Epub 2007 Apr 23.
Disease; Council on Cardiovascular Disease in the Young; Councils Tunkel AR, Hartman BJ, Kaplan SL, Kaufman BA, Roos KL, Scheld
on Clinical Cardiology, Stroke, and Cardiovascular Surgery and WM, et al. Practice guidelines for the management of bacterial men-
Anesthesia; American Heart Association; Infectious Diseases Society of ingitis. Clin Infect Dis. 2004 Nov 1;39(9):126784. Epub 2004
America. Infective endocarditis: diagnosis, antimicrobial therapy, and Oct 6.
management of complications: a statement for healthcare professionals Wilson W, Taubert KA, Gewitz M, Lockhart PB, Baddour LM, Levison
from the Committee on Rheumatic Fever, Endocarditis, and Kawasaki M, et al; American Heart Association Rheumatic Fever, Endocarditis,
Disease, Council on Cardiovascular Disease in the Young, and the and Kawasaki Disease Committee; American Heart Association
Councils on Clinical Cardiology, Stroke, and Cardiovascular Surgery Council on Cardiovascular Disease in the Young; American Heart
and Anesthesia, American Heart Association: endorsed by the Infectious Association Council on Clinical Cardiology; American Heart
Diseases Society of America. Circulation. 2005 Jun 14;111(23): Association Council on Cardiovascular Surgery and Anesthesia;
e394434. Erratum in: Circulation. 2007 Apr 17;115(15):e408. Quality of Care and Outcomes Research Interdisciplinary Working
Circulation. 2008 Sep 16;118(12):e497. Circulation. 2007 Nov Group. Prevention of infective endocarditis: guidelines from the
20;116(21):e547. Circulation. 2005 Oct 11;112(15):2373. American Heart Association: a guideline from the American Heart
Capelastegui A, Espana PP, Quintana JM, Areitio I, Gorordo I, Association Rheumatic Fever, Endocarditis, and Kawasaki Disease
Egurrola M, et al. Validation of a predictive rule for the manage- Committee, Council on Cardiovascular Disease in the Young, and
ment of community-acquired pneumonia. Eur Respir J. 2006 the Council on Clinical Cardiology, Council on Cardiovascular
Jan;27(1):1517. Surgery and Anesthesia, and the Quality of Care and Outcomes
Carpenter J, Stapleton S, Holliman R. Retrospective analysis of 49 cases of Research Interdisciplinary Working Group. Circulation. 2007 Oct
brain abscess and review of the literature. Eur J Clin Microbiol Infect 9;116(15):173654. Epub 2007 Apr 19. Erratum in: Circulation.
Dis. 2007 Jan;26(1):111. 2007 Oct 9;116(15):e3767.

5. I N F E C T I O U S D I S E A S E S Q U E S T I O N S A N D A N S W E R S 57
This page intentionally left blank
6.
RHEUMATOLOGY QUESTIONS AND ANSWER S

QUESTIONS becomes fatigued if she tries to comb her hair. She is


a nonsmoker and denies having any respiratory com-
Multiple Choice (choose the best answer) plaints. On examination, her temperature is 37.5C.
Her right radial pulse is decreased compared with the
N O NA RT I CU L A R R H EUM AT I S M A N D
left, and the blood pressure in her right arm is decreased
VA S C U L I T I S
compared with the left. Laboratory studies show mild
normochromic anemia with a hemoglobin of 11.2 g/dL
1. A 72-year-old woman presents with complaints of (reference range >12.0 g/dL), mildly elevated erythro-
new-onset headache. She is fatigued and has lost about cyte sedimentation rate (ESR) at 36 mm/h (reference
2.3 kg over the past month. The patient describes daily range <29 mm/h), negative antineutrophil cytoplasmic
bitemporal headaches. On further questioning, she autoantibody (ANCA) test, and normal blood chemis-
describes morning stiffness lasting about 1 hour and try panel results. Urinalysis and chest radiograph find-
aching in the shoulders and hips. She reports transient ings are normal. Which of the following is the most
loss of vision in her right eye for about 30 minutes yes- likely diagnosis?
terday before her vision returned. Physical examination a. Polymyalgia rheumatica (PMR)
findings are unremarkable for her age. Initial labora- b. Giant cell arteritis (GCA)
tory studies show a mild normochromic anemia and c. Buerger disease
elevated erythrocyte sedimentation rate (ESR) (101 d. Wegener granulomatosis
mm/h). Which would be the most appropriate next e. Takayasu arteritis
step?
a. Request a temporal artery biopsy.
4. A 59-year-old man has a 1-week history of low back dis-
b. Prescribe prednisone 1 mg/kg orally daily.
comfort. He says that he has been moving furniture over
c. Prescribe prednisone 5 mg orally 3 times daily.
the past several weeks, but he does not recall a specific
d. Request computed tomography (CT) of the head.
injury. The pain is worse if he is active; it improves if he
e. Refer the patient to a neurologist.
is at rest. He denies having pain radiating to the legs.
The patient has been taking ibuprofen 400 mg twice
2. A 55-year-old man presents with a 3-week history of left daily with food, and this seems to help. He has no prior
knee pain. He does not recall an injury. He says that the history of lower back pain. Neurologic examination
front of his leg is tender. On examination, he is afebrile findings are normal, with a downgoing Babinski sign,
and overweight. There is no left knee effusion, but the equal and symmetrical knee jerks, and normal strength
patient is markedly tender over the anterior medial tibia in the lower extremities. He has somewhat diffuse ten-
just distal to the knee joint. This area seems somewhat derness over the lumbar spine. Laboratory study results
puff y but is not particularly erythematous. The patient are normal for the complete blood cell count, erythro-
has good range of motion in both hips, but the area over cyte sedimentation rate, and blood chemistry panel.
the left trochanteric bursa is tender. What would be the What would be the most appropriate recommendation
most appropriate step for pain relief ? at this point?
a. Inject the left knee joint with corticosteroid and local a. Bed rest for 2 weeks
anesthetic. b. Radiograph of the lumbar spine
b. Inject the left trochanteric bursa with corticosteroid and c. Electromyographic (EMG) study
local anesthetic. d. Neurologic consultation
c. Inject the left pes anserine bursa with corticosteroid and e. HLA-B27 testing
local anesthetic.
d. Prescribe propoxyphene 65 mg orally every 6 hours.
5. A 26-year-old woman presents to your office because
e. Prescribe prednisone 60 mg orally daily for 7 days.
she aches all over. She tells you that this condition has
been present for several years but has gotten worse over
3. A 26-year-old woman presents for evaluation of the past 6 months. She has problems getting to sleep and
low-grade fevers that have been present about does not feel rested when she wakes up. She denies hav-
6 months. She also has lost 3.6 kg and has noted some ing depression. She is stiff for 5 minutes in the morning
arthralgias and myalgias. She says that her right arm and has not noticed any joint swelling. On examination,

59
her body mass index (BMI) is 20. Her muscle strength catching, or giving way of the knees. She takes glu-
is normal, and there is no synovitis. There is no rash. cosamine chondroitin sulfate (1,200 mg daily) and
She has multiple tender points. Results of the follow- acetaminophen (up to 1,000 mg 3 times daily as needed
ing laboratory studies are normal: complete blood cell for pain). She has hypertension and mild renal insuf-
count, erythrocyte sedimentation rate, blood chemis- ficiency (serum creatinine 1.6 mg/dL, reference range
try panel, and sensitive thyrotropin. What would be the 1.2 mg/dL). She also has a history of coronary artery
most appropriate next step? disease. On examination, she has a small amount of
a. Obtain an electromyogram (EMG). effusion in her right knee, mild genu varus deformity,
b. Prescribe prednisone 15 mg orally daily. and mild tenderness along the medial joint line of the
c. Prescribe duloxetine 30 mg orally daily. right knee. The knees appear stable on examination.
d. Obtain overnight oximetry results. Radiographs show medial joint space narrowing of the
e. Prescribe oxycodone 5 mg orally every 6 hours, as needed for right knee. Which would be the best next step in her
pain control. care?
a. Obtain a magnetic resonance imaging (MRI) scan of the
6. A 50-year-old man has sinus drainage, cough, and
right knee.
hemoptysis. He also says that he has had joint pain for
b. Administer an intra-articular corticosteroid injection.
several weeks and swelling in the feet. On examination,
c. Administer a series of injections with hylan G-F 20.
he has synovitis of several proximal interphalangeal
d. Prescribe naproxen 500 mg twice daily.
joints of the hands and bilateral lower extremity edema.
e. Obtain an orthopedic consultation for possible arthroscopic
Results of laboratory studies show a normochromic
surgery.
anemia (hemoglobin 9.6 mg/dL), an elevated erythro-
cyte sedimentation rate (67 mm/h), and an elevated 9. A 65-year-old man with rheumatoid arthritis (RA) has
creatinine level (1.6 mg/dL). The urinalysis shows noticed an increase in small rheumatoid nodules on
proteinuria (2+) and red blood cell casts. Chest radi- his hands over the past 3 months. Otherwise, he feels
ography shows multiple nodular lesions in both lungs. well. His RA has been well controlled with methotrex-
What would be the most appropriate next test to help ate 17.5 mg weekly, etanercept 50 mg subcutaneously
establish a diagnosis? weekly, folic acid 1 mg daily, and nabumetone 750 mg
a. Renal biopsy twice daily. On examination, he is afebrile. There are
b. Open lung biopsy numerous small nodules on the extensor surface of the
c. Antineutrophil cytoplasmic autoantibody (ANCA) panel fingers, but the patients RA appears quiescent, and he
for vasculitis does not have active synovitis in the hands or wrists.
d. Rheumatoid factor test Which of the following would be the most appropriate
e. Cyclic citrullinated peptide (CCP) antibody test recommendation?
7. A 41-year-old woman presents with a several-week his- a. Stop taking nabumetone.
tory of severe headaches and episodes that resemble a b. Add trimethoprim-sulfamethoxazole 1 double-strength tab-
transient ischemic attack (TIA). She has no history of let twice daily.
smoking or of having asthma. On examination, she is c. Add prednisone 60 mg daily.
afebrile but dysarthric. Her laboratory test results are d. Decrease the methotrexate dosage to 7.5 mg weekly.
normal for complete blood cell count with differential e. Switch etanercept to infliximab.
count, erythrocyte sedimentation rate, creatinine, and
10. A 43-year-old woman with rheumatoid arthritis pres-
urinalysis. The antineutrophil cytoplasmic autoanti-
ents to your office for optimization of her therapy. She
body (ANCA) test is negative. Chest radiography is
is taking methotrexate 15 mg weekly, prednisone 5 mg
unrevealing, but magnetic resonance imaging of the
daily, and nabumetone 750 mg daily. She saw a televi-
head shows multiple areas suggestive of infarctions in
sion advertisement for infliximab and wants to try it.
several arterial distributions. What is the most likely
She has a past medical history significant for multiple
diagnosis?
sclerosis (MS) at age 25, which has not been active in
a. Wegener granulomatosis (WG)
recent years. On examination, she does have synovi-
b. Giant cell arteritis (GCA)
tis in several joints. Which of the following would be
c. Buerger disease
most appropriate to tell her about her request to add
d. Isolated central nervous system (CNS) vasculitis
infliximab?
e. Churg-Strauss vasculitis
a. She will need a purified protein derivative (PPD) skin test
before starting infliximab therapy.
O S T EOA RT H R IT I S , R H EUM ATO I D A RT H R IT I S ,
b. Etanercept would be a better choice for her.
A N D A N T I R H EUM AT I C D RU G S
c. Infliximab can be given either orally or subcuteneously.
8. A 56-year-old woman has bothersome, but not dis- d. Infliximab cannot be used with prednisone.
abling, osteoarthritis of her right knee. She has pain e. Infliximab is relatively contraindicated in patients with a
if she walks more than 2 blocks but denies locking, history of MS.

60 M AYO C L I N I C I N T E R N A L M E D I C I N E B OA R D R E VI EW: Q U E S T I O N S A N D A N S W E R S
11. A 57-year-old woman comes to your office for mild knee and 30 tender joints. Laboratory test results include
discomfort. She says that if she walks more than 2 miles, the following: mild normochromic anemia (hemoglo-
both knees hurt. With daily activity, she is not symp- bin 12.1 g/dL), elevated erythrocyte sedimentation
tomatic. She currently is not taking any medications for rate (39 mm/h), negative rheumatoid factor test (14
her occasional knee pain. She is most interested in what international units/mL, reference range <15 interna-
she can do to prevent progression of her condition. On tional units/mL), weakly positive antinuclear antibody
examination, you note mild varus deformity of both (ANA) test (1.2 units, reference range <1.0 units), and
knees and tenderness over the medial joint line. What an increased level of antibodies to cyclic citrullinated
would be the best advice for her? peptide (anti-CCP) (>100 units, reference range <5
a. Take naproxen 500 mg twice daily. units). What can you tell him from these results?
b. Take acetaminophen 1,000 mg 3 times daily. a. You are referring him to a hematologist because of anemia.
c. Take glucosamine chondroitin sulfate 1,200 mg daily. b. Patients with RA do not have positive ANA tests.
d. Undergo magnetic resonance imaging (MRI) scanning of c. The anti-CCP results suggest that he is at higher risk of
both knees. radiographic progression than RA patients who have nega-
e. Undergo ultrasound-guided aspiration of 1 knee. tive anti-CCP results.
d. Since his rheumatoid factor test is negative, it is unlikely that
12. A 79-year-old woman has had seropositive rheumatoid
he has RA.
arthritis for 3 years. She takes methotrexate 15 mg on
e. Anti-CCP antibodies occur only in RA.
1 day weekly, hydroxychloroquine 400 mg daily, folic
acid 1 mg daily, and diclofenac-misoprostol 75 mg up
to twice daily as needed for joint pain. She says that S P O N DY L OA RT H RO PAT H I E S
these medications have helped her. She weighs 70 kg
15. What is the goal with allopurinol therapy in the treat-
and is normotensive. On examination, you find only
ment of hyperuricemia in gout patients?
2 tender joints and no swollen joints. Radiographs of
a. Decrease the uric acid level to 8.0 mg/dL or less (reference
the hands and feet do not show any erosions. She was
range 8.0 mg/dL).
told by an ophthalmologist that she has early cataracts
b. Prevent further gout attacks without regard to uric acid
in both eyes. Of the following, what would be the best
level.
recommendation for her?
c. Decrease the uric acid level to less than 6.0 mg/dL (reference
a. Stop taking hydroxychloroquine.
range 8.0 mg/dL).
b. Increase the methotrexate dosage to 20 mg weekly, and stop
d. Decrease urinary uric acid excretion to normal.
taking hydroxychloroquine.
e. Prevent uric acidinduced renal disease.
c. Increase the hydroxychloroquine dosage to 600 mg daily.
d. Have an ophthalmologic examination at least annually while 16. In a patient with ankylosing spondylitis, which of the
taking hydroxychloroquine. following would be most helpful for treating back pain
e. Stop taking the medications since therapy for rheumatoid and preventing episodes of uveitis?
arthritis is no longer needed. a. Nonsteroidal anti-inflammatory drugs
b. Methotrexate
13. A 26-year-old woman presents to your office for treat-
c. Sulfasalazine
ment of newly diagnosed rheumatoid arthritis (RA).
d. Etanercept
She has had symptoms for about 3 months. She is not
e. Adalimumab
functionally limited. So far, she has been taking ibupro-
fen 600 mg 3 times daily and has received some relief. 17. A 25-year-old man presents with a 2-week history of
On examination, she has 2 swollen joints and 4 tender pain and swelling of the right Achilles tendon and left
joints. She does not have any rheumatoid nodules or ankle. He has a previous history of uveitis on 2 occa-
other extra-articular features of RA. The rheumatoid sions. He describes stiffness in his lower back, which is
factor test is mildly positive, but the cyclic citrullinated worse in the morning. Which disease best accounts for
peptide antibody test is negative. Her radiographs do his symptoms?
not show any erosions. She would like to start therapy a. Ankylosing spondylitis
with a tumor necrosis factor (TNF) inhibitor. What b. Rheumatoid arthritis
would be the best advice for this patient? c. Lupus
a. Start infliximab 3 mg/kg intravenously. d. Gout
b. Start etanercept 50 mg/wk subcutaneously. e. Pseudogout
c. Start methotrexate 10 mg orally on 1 day weekly.
18. A 62-year-old woman experiences her first attack of
d. Additional therapy is not required now.
gout in her great toe. She takes nifedipine for hyperten-
e. Start glucosamine chondroitin sulfate 1,200 mg daily.
sion, simvastatin for hyperlipidemia, and naproxen for
14. A 29-year-old man presents to you with a 10-week his- osteoarthritis of her hips. Laboratory study results (and
tory of joint pain. His mother has severe rheumatoid reference ranges) include the following: uric acid 9.1
arthritis (RA). On examination, he has 20 swollen joints mg/dL (<8 mg/dL), creatinine 1.9 mg/dL (<1.1 mg/dL),

6. R H E U M ATO L O GY Q U E S T I O N S A N D A N S W E R S 61
and hemogloblin 11.3 g/dL (>12 g/dL). Besides treating M I S C E L L A N E O US R H EUM ATO L O G I C
her with tapering doses of methylprednisolone, which C O N D IT I O NS
of the following should be done next?
22. A 42-year-old man from Rhode Island has bilateral
a. Stop naproxen.
intermittent painful knee eff usions. Synovial fluid anal-
b. Stop simvastatin.
ysis is negative with Gram staining and culture, with
c. Begin colchicine.
a leukocyte count of 6.510 9/L. No crystals are seen.
d. Begin allopurinol.
There is no history of skin rash or low back pain. There
e. Begin probenecid.
is no other joint involvement. After a flulike illness
19. Acute monoarthritis of the left knee developed in a 2 years previously, the patient did have Bell palsy,
62-year-old man receiving long-term hemodialysis. which resolved. His father has gout. Which of the
Joint aspiration analysis is negative with Gram staining, following diseases is most likely to account for his
and polarization microscopy shows birefringent bipy- symptoms?
ramidal crystals that stain with alizarin red, indicative a. Rheumatoid arthritis
of calcium. What is the most likely diagnosis? b. Lyme disease
a. Gout c. Systemic lupus erythematosus (SLE)
b. Infection d. Gout
c. Basic calcium phosphate disease (hydroxyapatite deposition e. Spondyloarthropathy
disease)
23. A 63-year-old woman with a 20-year history of
d. Pseudogout
CREST syndrome presents with a 6-month history
e. Calcium oxalate arthropathy
of dyspnea with walking. A technetium Tc 99m ses-
20. A 43-year-old man has had a 3-year history of pro- tamibi study with exercise does not show any evi-
gressive psoriatic arthritis unresponsive to nonsteroi- dence of ischemia. A chest radiograph is normal.
dal anti-inflammatory drugs. On examination, he has Pulmonary function test results are normal except
inflammatory arthritis, and radiographs show erosive for an isolated decrease in the diffusing capacity of
changes in his hands and feet. He currently is being lung for carbon monoxide (Dlco) to 35% of the pre-
treated for hepatitis C infection, which he acquired dicted value. Which of the following would be the
from a blood transfusion 10 years ago. Which of the most likely diagnosis?
following would be the best treatment option at this a. Deconditioning
time? b. Atypical angina
a. Methotrexate c. Recurrent pulmonary embolism
b. Prednisone 40 mg daily d. Interstitial lung disease
c. Prednisone 10 mg daily e. Pulmonary hypertension
d. Etanercept
24. A 32-year-old male intravenous drug user has arthral-
e. Rituximab
gias and biopsy-proven cutaneous leukocytoclastic
21. An 82-year-old woman had sudden onset of pain and vasculitis. Laboratory study results are shown in Table
swelling in her left knee. There was no trauma. She had 6.Q24.
no previous episodes or history of fever. She was taking
hydrochlorothiazide for hypertension. Joint aspiration
Table 6.Q24
analysis is negative with Gram staining, the synovial
fluid leukocyte count is 12.5109/L, and polarization COMPONENT RESULT
microscopy shows weakly positive rhomboid bire-
fringent crystals. Laboratory study results include the Hemoglobin, g/dL 10.2
following: hemoglobin 11.7 g/dL, leukocyte count Leukocyte count, 109/L 8.2
7.2109/L, erythrocyte sedimentation rate 45 mm/h,
creatinine 1.6 mg/dL, and uric acid 8.6 mg/dL (refer- Erythrocyte sedimentation rate, mm/h 59
ence range <7 mg/dL). Which of the following is the Rheumatoid factor 1:640
cause of her acute monoarthritis?
a. Gout C4 Low
b. Pseudogout Aspartate aminotransferase, U/L (AST) 3 times upper limit of
c. Basic calcium phosphate disease reference range
d. Calcium oxalate arthropathy
Cryoglobulins Positive
e. Septic joint

62 M AYO C L I N I C I N T E R N A L M E D I C I N E B OA R D R E VI EW: Q U E S T I O N S A N D A N S W E R S
Which of the following tests would be most likely to Table 6.Q27
establish the diagnosis?
COMPONENT RESULT
a. Anticyclic citrullinated peptide
b. Antinuclear antibody Hemoglobin, g/dL 13.1
c. Anti-dsDNA antibody
d. Human immunodeficiency virus (HIV) Leukocytes, 109/L 3.2
e. Hepatitis C serology Platelets, 109/L 162
25. A 48-year-old woman with polymyositis began taking Erythrocyte sedimentation rate, mm/h 3
prednisone 60 mg daily 6 months ago. Her initial cre-
atine kinase (CK) level was 7,028 U/L (reference range Creatinine, mg/dL 0.6
<176 U/L). Now her CK is in the reference range and her Anti-dsDNA antibody Negative
prednisone dose has been decreased to 20 mg. She still
has significant proximal muscle weakness. What would C3 Normal
be the best course of action at this time to improve her Anti-Sm antibody Negative
muscle strength?
a. Increase the dose of prednisone. Anti-RNP antibody Negative
b. Continue to decrease the prednisone dose and have her AntiSS-A antibody Positive
begin physical therapy.
c. Begin methotrexate. AntiSS-B antibody Negative
d. Begin azathioprine. Antiphospholipid antibody Negative
e. Begin intravenous immune globulin.
Antibodies IgG, IgM
26. A 53-year-old woman has a 15-year history of systemic
lupus erythematosus. Multiple flares of her disease dur-
ing the past several years have necessitated high-dose a. Deep vein thrombosis
prednisone therapy. She now presents with a 3-week b. Miscarriage secondary to placental infarction
history of pain in the left groin with ambulation. She c. Lupus renal disease
denies having fevers, chills, or other symptoms that she d. Pregnancy complications with fetal heart block
had equated with a flare in the past. Findings on routine e. Hemolytic anemia
radiography of her pelvis, including hips, are normal.
28. A 65-year-old man noted slowly progressive muscle
Which of the following would be most helpful in deter-
weakness in his arms and legs over the past 2 years. He
mining the cause of her symptoms?
considers himself healthy otherwise. He has had no
a. Electromyography
pain. He takes simvastatin and hydrochlorothiazide. He
b. Erythrocyte sedimentation rate and anti-nDNA antibody
is weak both proximally and distally. An electromyo-
determination
gram (EMG) shows both myopathic and neuropathic
c. Magnetic resonance imaging (MRI) of the hips
findings. Laboratory test results include the following:
d. Empirical trial of a corticosteroid injection into the hip
hemoglobin 14.4 g/dL, erythrocyte sedimentation rate
under fluoroscopy
3 mm/h, creatinine 1.0 mg/dL, antinuclear antibodies
e. MRI of the lumbar spine
1:40 (negative <1:40), and creatine kinase (CK) 639
27. A 31-year-old woman with systemic lupus erythemato- U/L (reference range <250 U/L). What is the most
sus is considering having a child. She is asymptomatic. likely diagnosis?
She has no history of a previous pregnancy or of renal a. Polymyositis
disease. She takes hydroxychloroquine 200 mg daily. On b. Amyotrophic lateral sclerosis (ALS)
the basis of her laboratory study results (Table 6.Q27), c. Simvastatin toxicity
which of the following would she be at increased risk d. McArdle disease
for during a pregnancy? e. Inclusion body myositis (IBM)

6. R H E U M ATO L O GY Q U E S T I O N S A N D A N S W E R S 63
ANSWER S

1. Answer b. 5. Answer c.
In a patient of this age with a new onset of headache, anemia, This patient presents with fibromyalgia-like symptoms. The
and highly elevated ESR, giant cell arteritis (GCA) would be patient does not have muscle weakness, so an EMG would
the number one consideration. Temporal artery biopsy would not be indicated. Her symptoms, physical examination
be appropriate, but with the high degree of suspicion for GCA findings, and laboratory test results are not suggestive of
(because of the loss of vision), prednisone therapy should inflammation, so prednisone is not indicated. Duloxetine
begin without waiting for the biopsy results. The appropriate has been shown to be efficacious treatment for fibromyalgia
dose of prednisone for GCA is 1 mg/kg daily. Lower doses in women. Although the patients sleep is nonrestorative,
are appropriate for polymyalgia rheumatica. A CT scan of the she is young and has a low BMI, so sleep apnea is not likely.
head would not be helpful for a patient with suspected GCA. She has a chronic pain syndrome, and it is usually best to
Likewise, referral to a neurologist would not be helpful. avoid using narcotics to treat fibromyalgia pain.
2. Answer c. 6. Answer c.
This patient has pes anserine bursitis. Injection of the bursa This patient has features of Wegener granulomatosis
is often helpful. There is not an indication to inject the left (WG) with upper and lower respiratory tract involvement
knee joint. Although the area over the left trochanteric bursa and renal involvement. An ANCA panel would likely be
is tender, the patient is not symptomatic in this area, so there positive in a c-ANCA pattern with a positive proteinase 3
is no indication for an injection there. Propoxyphene has enzyme-linked immunosorbent assay (PR3 ELISA). A renal
been taken off the market because of safety concerns, includ- biopsy would be expected to show glomerulonephritis,
ing accidental overdose, suicide, and arrhythmias. High-dose which would not be specific for WG. An open lung biopsy
prednisone is typically not used for bursitis. likely would show histologic features of WG (granuloma-
tous vasculitis), but the procedure is invasive and may not
3. Answer e.
be necessary if the ANCA tests are positive. Patients with
Of the choices listed, this patient would most likely have WG can have synovitis. The rheumatoid factor and CCP
Takayasu arteritis. This would be most common in women tests are for rheumatoid arthritis. Patients with rheumatoid
younger than 40. Weight loss, arthralgias, myalgias, and arthritis would not typically have hemoptysis, although
low-grade fevers are all common features. In addition, they could have pulmonary involvement, and they would
patients often present with upper extremity claudication. not usually have glomerulonephritis.
The decreased radial pulse in the right arm compared with
7. Answer d.
the left and the decreased blood pressure in the right arm
suggest upper aortic arch involvement. Typically, mild Patients with WG usually have upper and lower respiratory
anemia is present. Some patients have active disease but a involvement and often renal disease, which this patient
normal ESR. Many patients with Takayasu arteritis have a does not have. Also, most patients with WG have a posi-
negative ANCA test. Both PMR and GCA occur in an older tive ANCA test. GCA would not occur in this age group
age group. Buerger disease occurs in smokers. Patients with and usually does not manifest with TIAs. Buerger disease
Wegener granulomatosis usually have respiratory symptoms affects the peripheral circulation and occurs in smokers.
and a positive ANCA test; they often have upper and lower Churg-Strauss vasculitis occurs in patients with a history
respiratory involvement and kidney involvement. of asthma and is usually associated with eosinophilia.
Isolated CNS vasculitis can manifest with TIAs and head-
4. Answer b.
aches. Often patients do not have other features of systemic
New onset of lower back pain after age 50 would be an indi- vasculitis, so the term isolated CNS vasculitis is used. Most
cation for radiograph. Bed rest for more than 3 days is not patients with isolated CNS vasculitis are ANCA negative.
helpful. There is no suggestion of radiculopathy, so EMG
8. Answer b.
and neurologic consultation are not indicated. The patient
is not describing inflammatory back pain and is older than This patient has osteoarthritis. Her examination, history,
50, so HLA-B27 testing for spondyloarthropathy would and radiographs do not suggest another process, so MRI
not be helpful. is likely to yield little more information. Because she has

64
renal insufficiency, hypertension, and a history of coronary therapy. It would be appropriate to start a disease-modifying
artery disease, nonsteroidal anti-inflammatory drugs are antirheumatic drug, so methotrexate would be a reasonable
relatively contraindicated. A corticosteroid injection would choice. Glucosamine chondroitin sulfate has been shown
be a consideration, especially since she has pain in only to be potentially helpful in osteoarthritis of the knee, but
1 joint. If the patient had no response to the corticosteroid there is no support from evidence-based medicine that it
injection, viscosupplementation would be a consideration. helps RA.
The patient does not have mechanical symptoms, so there
is no clear indication for an arthroscopic procedure. 14. Answer c.
The patient likely has an anemia of chronic disease related
9. Answer d.
to inflammatory arthritis. High levels of anti-CCP anti-
Methotrexate can cause increased nodulosis in some RA bodies are specific for RA; occasionally, lower values are
patients. Depending on how problematic it is, one may need present with other conditions, including systemic lupus
to consider decreasing the dose or even stopping the medi- erythematosus. A positive ANA test, especially a weakly
cation. Trimethoprim-sulfamethoxazole can interact with positive result such as this patients, is common in RA. The
methotrexate and potentially cause toxicity, so it should be anti-CCP test is more specific than the rheumatoid factor
avoided, especially if there is not a specific reason to add it. test for RA but, depending on the assay used, may be less
The RA appears relatively quiescent, so there would not be sensitive than the rheumatoid factor test. Anti-CCP anti-
a need to add high-dose prednisone. Likewise, there would bodies sometimes occur in patients who are rheumatoid
not be a reason to switch to a different tumor necrosis fac- factornegative; occasionally, anti-CCP antibodies occur
tor inhibitor. before rheumatoid factor occurs in patients with RA. The
presence of anti-CCP antibodies has been associated with
10. Answer e. more radiographic progression in patients with RA.
All the tumor necrosis factor (TNF) inhibitors have been
15. Answer c.
reported to potentially cause demyelinating lesions in
patients. Trials of infliximab in patients with MS lead to The goal of allopurinol therapy in the treatment of gout is
worsening of the disease. Therefore, TNF inhibitors are to decrease the uric acid level to less than 6.0 mg/dL. The
relatively contraindicated for patients with a history of MS. solubility quotient of uric acid is 6.8 mg/dL. Therefore,
Infliximab can be used in patients who are taking nonsteroi- decreasing the uric acid level to less than 6.0 mg/dL pro-
dal anti-inflammatory drugs, methotrexate, and prednisone. motes shrinkage of tophi and results in better clinical
A PPD skin test should be checked before starting any of the efficacy.
TNF inhibitors because tuberculosis has reactivated in some
patients. Infliximab is given by intravenous infusion only. 16. Answer e.
Although both etanercept and adalimumab would be help-
11. Answer c.
ful in treating the back pain, only adalimumab has been
Nonsteroidal anti-inflammatory drugs and acetaminophen shown to decrease the frequency of uveitis.
are analgesics, but they do not prevent progression of osteoar-
thritis. There is some support from evidence-based medicine 17. Answer a.
for the use of glucosamine chondroitin sulfate to retard pro- Achilles tendinitis (enthesopathy), low back pain with
gression of osteoarthritis of the knee. There is no indication morning stiffness, and uveitis are characteristic of spondy-
for MRI or ultrasound-guided arthrocentesis of the knee. loarthropathies, such as ankylosing spondylitis. The other
diseases do not predispose characteristically to back pain,
12. Answer d.
uveitis, or enthesopathy.
Hydroxychloroquine is typically used at a dosage of up
18. Answer a.
to 6.5 mg/kg daily. Higher doses may be associated with
increased ocular toxicity and so are generally avoided. The Naproxen should be stopped because of the elevated crea-
toxicity is retinal toxicity, and hydroxychloroquine use is tinine. Stopping naproxen may also result in decreased
not associated with cataract formation. Patients taking creatinine and uric acid levels. Only if the patient contin-
hydroxychloroquine should have an ophthalmologic exam- ued to have recurrent attacks of gout would allopurinol be
ination at least annually. Patients with rheumatoid arthritis used to lower the uric acid. With an elevated creatinine,
usually require long-term therapy with a disease-modifying probenecid would not be effective.
antirheumatic drug. This patients disease is well controlled
with the current therapy, so there is no indication to change 19. Answer e.
her regimen. Calcium oxalate arthropathy usually occurs in patients
receiving long-term hemodialysis, and the bipyramidal
13. Answer c. crystals stain positive for calcium. Dialysis patients are also
According to present treatment guidelines, initiation of a at increased risk for basic calcium phosphate disease, gout,
TNF would not be indicated for this patient as the first and a septic joint. Negative Gram staining rules against a

6. R H E U M ATO L O GY Q U E S T I O N S A N D A N S W E R S 65
septic joint, and no urate crystals indicative of gout were in the Dlco suggests pulmonary hypertension, and the
seen. In basic calcium phosphate disease, crystals are seen patient should undergo right heart catheterization.
only with electron microscopy, not with polarization
24. Answer e.
microscopy.
Hepatitis C infection can cause cryoglobulinemia, which
20. Answer d.
is responsible for the positive rheumatoid factor and cryo-
Etanercept is approved for both psoriasis and psoriatic globulinemic vasculitis. The low C4 is also associated
arthritis and has no deleterious effect on viral titers or with the cryoglobulinemic vasculitis. The elevated AST is
treatment of hepatitis C. Methotrexate is contraindicated indicative of ongoing hepatitis liver disease. Rheumatoid
for a patient with hepatitis C. A high dose of prednisone arthritis, systemic lupus erythematosus, and HIV do not
can increase viral titers, and a low dose of prednisone characteristically cause cryoglobulinemia.
should be used only to give temporary benefit until a
25. Answer b.
disease-modifying antirheumatic agent provides benefit.
Rituximab is not approved for psoriatic arthritis. This patient has steroid myopathy. With the CK value being
normal, the polymyositis is under good control. There is no
21. Answer b.
reason to add a steroid-sparing agent at this time. A slow,
Weakly positive rhomboid birefringent crystals are char- continued corticosteroid taper in conjunction with physi-
acteristic of pseudogout. This patient had an elevated cal therapy is indicated.
uric acid level, but monosodium urate crystals, which are
26. Answer c.
needle shaped and strongly negative birefringent, were
not seen. Gram staining was negative, which rules against The most helpful test would be MRI of the hips. This
a septic joint. Calcium oxalate crystals are bipyramidal. In patient has 2 risk factors for avascular necrosis of the hip:
basic calcium phosphate disease (calcium hydroxyapatite), 1) systemic lupus erythematosus and 2) prednisone use.
no crystals are seen with polarization microscopy.
27. Answer d.
22. Answer b.
The positive results for antiSS-A antibody give this patient
Not all patients with Lyme disease recall a tick bite or a 3% risk of a complete heart block developing in the fetus
have erythema chronicum migrans. This patient had in the second or third trimester. There are no antiphospho-
flulike symptoms and Bell palsy several years before lipid antibodies, so there is no increased risk of deep vein
the onset of his Lyme arthritis, which characteristically thrombosis or miscarriage. The patient has had no previ-
involves the knees several years after the initial infec- ous renal disease, and with no anti-dsDNA there is not an
tion. Bell palsy is a characteristic neurologic manifesta- increased risk of active renal disease during pregnancy.
tion of Lyme disease. Rheumatoid arthritis would be less
likely without polyarticular involvement and would not 28. Answer e.
be associated with Bell palsy. No crystals were seen with IBM occurs in older patients and causes both proximal and
joint aspiration, which would rule against gout. There is distal muscle weakness. In contrast, polymyositis causes
no history of low back pain or uveitis, which would make predominantly proximal weakness. In IBM, the elevation of
a spondyloarthropathy unlikely. There is no clinical his- CK values is usually relatively small, and characteristically
tory to suggest SLE. the EMG has both myopathic and neuropathic findings.
23. Answer e. Polymyositis, simvastatin toxicity, and McArdle disease
would not show neuropathic findings on EMG. ALS can
Pulmonary hypertension is more likely to develop in be associated with an elevated CK, but the muscle weak-
patients with CREST syndrome than in patients with dif- ness is usually asymmetrical and EMG would show only
fuse systemic sclerosis (scleroderma). The isolated decrease neuropathic findings.

66 M AYO C L I N I C I N T E R N A L M E D I C I N E B OA R D R E VI EW: Q U E S T I O N S A N D A N S W E R S
7.
ENDOCRINOLOGY QUESTIONS AND ANSWER S

QUESTIONS 3. A 72-year-old man has new-onset atrial fibrillation and


abnormal results on thyroid tests (thyrotropin <0.01
Multiple Choice (choose the best answer) mIU/L, free thyroxine 2.3 ng/dL). He begins antiar-
rhythmic therapy and is referred to you. He reports palpi-
T H Y RO I D D I S E A S E tations and weight loss for 3 months along with redness,
1. A 62-year-old farmer is evaluated as a new patient. He swelling, and pain over his eyes. His past medical history
has a 20-year history of hypothyroidism and hypogo- is significant for hypertension, congestive heart failure,
nadism. He takes levothyroxine (88 mcg daily) and and ongoing nicotine dependence. On examination, his
testosterone gel 1% (2.5 g daily) and feels well. He heart rate is irregular at 92 beats per minute, his blood
weighs 75 kg and has a small thyroid. Physical exami- pressure is 148/86 mm Hg, and his lungs are clear. The
nation findings are unremarkable. Laboratory test thyroid is nontender, about twice the normal size, and
results include the following: thyrotropin 0.2 mIU/L increased in consistency. He has bilateral exophthalmos
(reference range, 0.35.0 mIU/L) and free thyroxine with red and significantly swollen eyelids and injected
1.3 ng/dL (reference range, 0.81.8 ng/dL). You decide conjunctiva. There is pitting edema (trace) of the lower
to decrease the levothyroxine dosage to 75 mcg daily. extremities and an area of brownish thickening of the
After 2 months, thyrotropin was 0.1 mIU/L, free thy- skin over the pretibial areas. What is the best manage-
roxine was 0.9 ng/dL, and he feels tired and cold. Which ment for his thyroid dysfunction at this point?
of the following should you do now? a. Observation, antiarrhythmic treatment, and reevaluation in
a. Decrease the levothyroxine dosage to 50 mcg daily and 6 weeks for possible hypothyroidism
recheck in 6 weeks. b. Propylthiouracil (PTU)
b. Stop levothyroxine altogether and evaluate thyroid uptake c. Methimazole treatment
for hyperthyroidism. d. Total or near-total thyroidectomy
c. Discontinue the testosterone gel since it increases the e. Radioactive iodine treatment
thyroxine-binding globulins.
d. Check his thyroglobulin to distinguish between primary D I A B ET E S M E L L IT US , H Y P O G LYC E M I A , A N D
and secondary hypothyroidism. HYPERLIPIDEMIA
e. Continue levothyroxine at 125 mcg daily, get his old records,
4. A 54-year-old woman comes to your office for her rou-
and consider cortisol testing.
tine 6-month follow-up for the management of type
2. A 29-year-old woman has a 7-year history of primary 2 diabetes mellitus. She has no immediate concerns.
hypothyroidism due to Hashimoto thyroiditis. She has Over the past year, she has consulted a dietitian and has
been treated with 125 mcg of levothyroxine with a good been trying to adhere to her diabetic diet. She has also
clinical and biochemical response so far. A few months started a walking program. Her medications include
ago, she decided to start taking oral contraceptives metformin 1,000 mg twice daily, simvastatin 20 mg
because she had persistently irregular periods related daily, lisinopril 20 mg daily, a multivitamin daily, and
to polycystic ovary syndrome. Her periods are regular a calcium supplement daily. She monitors her blood
now, but she feels tired and has cold intolerance. Her glucose levels at home in the morning 2 or 3 times per
thyrotropin is 10.8 mIU/L; free thyroxine is 1.1 ng/dL. week, and the metered glucose values (in milligrams per
Which of the following should you do next? deciliter) are in the mid 100s. She denies having hypo-
a. Check the titer of thyroperoxidase antibodies to assess the glycemia. She works outside the home but eats meals
severity of Hashimoto thyroiditis. at regular times. On physical examination, her blood
b. Advise the patient to improve her adherence to the medical pressure is 135/80 mm Hg, her heart rate is 75 beats
therapy and recheck in 2 to 3 months. per minute, her height is 160 cm, her weight is 85 kg,
c. Advise the patient to take the oral contraceptive 12 hours and her body mass index is 33. Other examination find-
after taking levothyroxine, and recheck in 2 to 3 months. ings are unremarkable. Laboratory test results include
d. Add liothyronine to the levothyroxine for a better clinical hemoglobin A1c (HbA1c) 8.5% and normal serum crea-
response. tinine and liver transaminase values. What is your best
e. Increase the levothyroxine dose to 137 mcg, and recheck management option to improve this patients glycemic
thyrotropin in 2 to 3 months. control?

67
a. Make no changes in her current glucose-lowering regimen, d. Insulin sensitizers such as metformin and rosiglitazone have
and congratulate her. been shown to slow the progression to type 2 diabetes mel-
b. Increase the metformin dose to 2,500 mg daily, with a goal litus with minimal risk.
HbA1c of less than 7%.
6. A 63-year-old woman with a history of type 2 diabe-
c. Add a sulfonylurea to her regimen, with a goal HbA1c of less
tes mellitus for the past 8 years comes to your office
than 7%, and discuss the risk of hypoglycemia.
with complaints of worsening paresthesias involving
d. Add rosiglitazone to her regimen, with a goal HbA1c of less
her hands. She has peripheral neuropathy involving
than 7%, and discuss the risk of heart failure and cardiovas-
her lower extremities; the neuropathy is unchanged
cular events.
and has been attributed to diabetes. She feels more
e. Add a single injection of insulin glargine to her regimen,
fatigued, which she attributes to poor sleep as a result
with a goal HbA1c of less than 7%, and discuss the risk of
of her neuropathic symptoms. Additional medical
hypoglycemia.
diagnoses are hypertension and hyperlipidemia. She is
5. A 48-year-old man comes to your office for a routine up-to-date for all her medical preventive services. Her
physical examination. He feels well but is concerned current medications are metformin 1,000 mg twice
about his family medical history. Both his mother and daily, lisinopril 20 mg daily, and atorvastatin 20 mg
his father have type 2 diabetes mellitus, and his father daily. Physical examination findings include the fol-
(age 73 and an ex-smoker) had coronary bypass surgery lowing: height 155 cm, weight 74.4 kg, body mass index
recently. The patients medical history is positive for 31, pulse 75 beats per minute, blood pressure 135/82
hypertension. He is a nonsmoker. Current medications mm Hg, dry skin on the lower extremities, no areas
include hydrochlorothiazide-triamterene 25 mg/12.5 of skin breakdown, decreased sensation to monofila-
mg 1 tablet daily and fish oil capsules 1,000 mg daily. On ment in the lower extremities to the ankles, and absent
physical examination, his pulse is 85 beats per minute, ankle reflexes bilaterally. The upper extremities have
his blood pressure is 140/85 mm Hg, his height is 177 normal hand grip and normal reflexes, but the patient
cm, his weight is 120 kg, and his body mass index is 38. describes paresthesias. Laboratory test results include
He has central obesity. The other cardiac and physical the following: hemoglobin 11.5 g/dL, hemoglobin A1c
examination findings are unremarkable. Diagnostic 6.5%, and normal values for serum creatinine, liver
test results are shown in Table 7.Q5. transaminases, and serum thyrotropin. What should
be your next step in the evaluation and management of
Table 7.Q5 this patient?
a. Obtain an electromyogram.
COMPONENT VALUE b. Determine the vitamin B12 level.
c. Start therapy with a tricyclic antidepressant.
Fasting blood glucose, mg/dL 116
d. Intensify her glucose-lowering therapy by adding a second
Hemoglobin A1c, % 6.2 glucose-lowering agent.
Serum creatinine, mg/dL 1.0 7. A 59-year-old man with type 2 diabetes mellitus
comes to your office for a routine visit. He has a his-
Total cholesterol, mg/dL 210
tory of coronary artery disease and had a myocardial
Triglycerides, mg/dL 225 infarction 3 years ago. He is currently free of any car-
diac symptoms. He is an ex-smoker. He monitors his
High-density lipoprotein cholesterol, mg/dL 34
blood glucose levels daily before breakfast, and the
Calculated low-density lipoprotein cholesterol, mg/dL 131 majority of values are less than 150 mg/dL. He denies
Nonhigh-density lipoprotein cholesterol, mg/dL 176
having hypoglycemia. Current medications are met-
formin 1,000 mg twice daily, glimepiride 4 mg twice
daily, simvastatin 40 mg daily, metoprolol 50 mg
daily, quinapril 40 mg daily, and aspirin 81 mg daily.
Which of the following statements about this patient is
Physical examination findings include the following:
false?
height 175 cm, weight 113 kg, pulse 65 beats per min-
a. He has prediabetes, and his 5-year risk for diabetes mellitus
ute, blood pressure 145/85 mm Hg, body mass index
can be as high as 50%.
37, overweight with central obesity, systolic ejection
b. Lifestyle changes that lower his intake of saturated fats,
murmur 2/6 at the left sternal border, and decreased
increase his exercise, and cause modest weight loss can lower
sensation to pinprick in the feet up to the ankles with
his risk for type 2 diabetes mellitus by 58%.
absent ankle reflexes bilaterally. Laboratory test results
c. His lipid profile puts him at increased risk for a premature
are shown in Table 7.Q7.
cardiac event.

68 M AYO C L I N I C I N T E R N A L M E D I C I N E B OA R D R E VI EW: Q U E S T I O N S A N D A N S W E R S
Table 7.Q7 c. Glucose 54 mg/dL, C peptide 0.6 ng/mL, -hydroxybutyrate
0.2 mmol/L with negative results for the sulfonylurea screen-
COMPONENT VALUE
ing test
Hemoglobin A1c, % 7.5 d. Glucose 54 mg/dL, C peptide 3.0 ng/mL, -hydroxybutyrate
0.3 mmol/L
Serum creatinine, mg/dL 1.2 e. Glucose 54 mg/dL, C peptide 3.0 ng/mL, -hydroxybutyrate
Total cholesterol, mg/dL 150 0.3 mmol/L with negative results for the sulfonylurea screen-
ing test
Triglycerides, mg/dL 195
9. A 65-year-old man comes to your office for follow-up of
High-density lipoprotein cholesterol, mg/dL 42 his diabetes mellitus. The diagnosis was made a year ago
Calculated low-density lipoprotein cholesterol, mg/dL 69 with a random blood glucose value above 300 mg/dL.
The patient has been struggling to adhere to the dietary
Nonhigh-density lipoprotein cholesterol, mg/dL 108 recommendations and has been unable to establish an
activity program. He monitors his blood glucose lev-
Which intervention is associated with a lower risk for a els a few times a week, generally in the morning before
cardiovascular event? breakfast. All blood glucose values are over 160 mg/dL.
a. Increasing his aspirin dosage to 325 mg daily He is a nonsmoker. He has no history of cardiovascu-
b. Adding fenofibrate to his lipid-lowering regimen lar disease. His current medications are metformin
c. Adding insulin to improve his glycemic control until the 1,000 mg twice daily, glimepiride 4 mg twice daily,
hemoglobin A1c is less than 6.5% atorvastatin 40 mg daily, lisinopril 20 mg twice daily,
d. Increasing his simvastatin dosage to 80 mg daily hydrochlorothiazide-triamterene 50 mg/25 mg 1 tablet
e. Continuing metformin therapy daily, and fish oil capsules 1,000 mg twice daily. Physical
examination findings include the following: height
8. A 55-year-old woman is seen in your office accompa-
170 cm, weight 95 kg, body mass index 33, pulse 75
nied by her husband for follow-up after dismissal from
beats per minute, and blood pressure 138/82 mm Hg.
her local emergency department (ED) for further evalu-
Laboratory test results are shown in Table 7.Q9.
ation of reactive hypoglycemia. For years, she has had
intermittent symptoms of diaphoresis and excessive
Table 7.Q9
hunger that resolve after eating. She remembers having
a glucose tolerance test, during which her blood glucose COMPONENT VALUE
level decreased to about 60 mg/dL. She was advised to
see a dietitian, eat regularly, and always have hard candy Hemoglobin A1c, % 9.2
available to abort her symptoms. She has been experi- Serum creatinine, mg/dL 1.3
encing more frequent symptoms and has gained weight
over the past year. She is healthy otherwise. She has no Total cholesterol, mg/dL 198
history of diabetes. She is currently taking no medi- Triglycerides, mg/dL 335
cations. The day of her evaluation in the ED, she was
involved in her usual activities at home when she expe- High-density lipoprotein cholesterol, mg/dL 35
rienced her usual symptoms related to hypoglycemia, Calculated low-density lipoprotein cholesterol, mg/dL 96
diaphoresis, and hunger. She recalled missing breakfast
but does not recall any additional events until she was Nonhigh-density lipoprotein cholesterol, mg/dL 162
in the ED. Her husband recognized that she was not
well and was worried that she was having a stroke, so he Which of the following statements about the manage-
called the paramedics. She was sweaty and would not ment of this patient is false?
respond to his questions. At the ED, several laboratory a. Use of pioglitazone (a thiazolidinedione [TZD]) should be
tests were performed. She was told that her blood glu- avoided because of the higher risk of congestive heart failure
cose was low. She was given fluids and dextrose intra- and cardiovascular events.
venously. She was dismissed and advised to follow up b. Use of glucagon-like peptide 1 analogues (exenatide, lira-
as an outpatient. Which test results support abnormal glutide) will improve glycemic control with the potential
endogenous insulin secretion as a cause for hypogly- for weight loss.
cemia (reference ranges: C peptide, 0.94.3 ng/mL; c. Use of dipeptidyl-peptidase-4 inhibitors (sitagliptin, saxa-
-hydroxybutyrate, <0.4 mmol/L)? gliptin) will improve glycemic control without weight loss
a. Glucose 54 mg/dL, C peptide 0.6 ng/mL, -hydroxybutyrate or gain.
4.9 mmol/L d. Insulin therapy will improve glycemic control but with a
b. Glucose 54 mg/dL, C peptide 0.6 ng/mL, -hydroxybutyrate higher risk of weight gain and hypoglycemia.
0.2 mmol/L

7. E N D O C R I N O L O GY Q U E S T I O N S A N D A N S W E R S 69
O B E S IT Y A N D NU T R IT I O N a. She meets the National Institutes of Health consensus crite-
ria for considering bariatric surgery.
10. A 45-year-old woman comes to your office for her
b. Laparoscopic adjustable gastric banding and Roux-en-Y gas-
routine physical examination. She has no immedi-
tric bypass are associated with similar outcomes for inducing
ate concerns except that her weight has been increas-
remission of type 2 diabetes mellitus.
ing in the past few years. She has a clerical job and a
c. Untreated obstructive sleep apnea is a risk factor for higher
busy home life. Her eating habits have not changed
perioperative morbidity and mortality with bariatric
dramatically, although she admits that she eats at fast
surgery.
food restaurants several times a week on her way to
d. The use of nonsteroidal anti-inflammatory drugs is contrain-
extracurricular activities. She currently has no exercise
dicated after Roux-en-Y gastric bypass surgery.
program, although she owns a treadmill. She is other-
wise healthy. She has regular menses. She has had tubal 12. You are called to the emergency department to see one
ligation. She has a family history of type 2 diabetes of your patients. He is a 48-year-old man who under-
mellitus; her mother received the diagnosis in her 50s. went a Roux-en-Y gastric bypass for the management
The patients medications are vitamin D supplement of his obesity 2 months ago. He has severe nausea and
400 international units daily and calcium supplement vomiting and has not been able to keep any food or flu-
600 mg daily. Physical examination findings include ids down for the past 48 hours. He has abdominal pain,
the following: height 162 cm, weight 83 kg, body mass particularly in the epigastric region. He feels dizzy and
index 33, pulse 72 beats per minute, and blood pressure complains of tingling and numbness of his hands. He
135/80 mm Hg. She is overweight with central obesity. has lost 16 kg since his surgery. He has a diagnosis of
Examination findings are otherwise unremarkable. type 2 diabetes mellitus and takes metformin, which
Which is not an appropriate step in the management of was discontinued at his hospital dismissal after surgery.
this patient? Other diagnoses are hypertension, hyperlipidemia, and
a. Screen for hypothyroidism with a sensitive thyrotropin obstructive sleep apnea, for which he uses continuous
(sTSH) test. positive airway pressure. Current medications (which
b. Screen for type 2 diabetes mellitus by measuring the fasting he has not been able to take regularly) are simvastatin
blood glucose level. 20 mg daily, losartan 100 mg daily, chewable multivi-
c. Advise that she start a walking program of 10- to 30-minute tamin 1 tablet twice daily, calcium carbonate 300 mg
sessions, working toward a goal of 150 minutes of walking 3 times daily, and vitamin B12 1,000 mcg subcutane-
per week. ously monthly. Physical examination findings include
d. Recommend that she eat a low-carbohydrate diet, which is the following: height 177.5 cm, weight 124 kg, pulse
superior for promoting and maintaining a weight loss of 5% 112 beats per minute, blood pressure 105/68 mm Hg,
to 10%. and temperature 37.8C. The patient appears uncom-
fortable and has frequent dry heaves. Cardiac auscul-
11. A 52-year-old woman comes to your office to discuss
tation is unremarkable except for tachycardia. Lung
bariatric surgery. She has struggled to lose weight dur-
sounds are normal. The abdomen has hyperactive bowel
ing most of her adult life. She has participated in indi-
sounds and is tender to palpation in the epigastric
vidual and commercial weight loss programs with no
region. There are no acute peritoneal signs. Laboratory
long-term success at managing her weight, and she is
test results are shown in Table 7.Q12.
concerned about her health. She has type 2 diabetes
mellitus, hypertension, hyperlipidemia, and joint pain.
Her joint pain and fatigue interfere with her ability to Table 7.Q12
be physically active. She does not smoke or drink alco-
COMPONENT VALUE
holic beverages. She has no prior psychiatric history.
After she recently spoke to a friend who had bariatric Sodium, mg/dL 146
surgery and is doing very well, she became motivated to
consider this option. Her current medications are met- Potassium, mg/dL 3.2
formin 1,000 mg twice daily, glipizide 20 mg twice daily, Glucose, mg/dL 118
atorvastatin 20 mg daily, fenofibrate 145 mg daily, fos-
inopril 20 mg daily, hydrochlorothiazide-triamterene Creatinine, mg/dL 1.5
25 mg/12.5 mg 1 tablet daily, aspirin 81 mg daily, and Aspartate aminotransferase, U/L 52
naproxen 500 mg twice daily as needed. Physical exami-
nation findings include the following: height 157 cm, Alkaline phosphatase, U/L 120
weight 110 kg, body mass index 45, pulse 72 beats per Lipase, U/L 68
minute, and blood pressure 138/75 mm Hg. She is
overweight with central obesity. Examination findings Hemoglobin, g/dL 14.1
are otherwise unremarkable. Which statement about Leukocyte count, 10 /L9
10.1
bariatric surgery in the management of this patient is
false? Platelet count, 109/L 158

70 M AYO C L I N I C I N T E R N A L M E D I C I N E B OA R D R E VI EW: Q U E S T I O N S A N D A N S W E R S
You started intravenous fluid hydration with 0.9% P IT U ITA RY, G O NA DA L , A N D A D R E NA L
sodium chloride solution and 20 mEq of potassium DISORDERS
per liter at 100 mL/h. You have consulted the general
14. A 31-year-old married white man presents with breast
surgeon on call. The patient is scheduled for computed
enlargement and tenderness of 6 months duration.
tomography of the abdomen within the next hour.
He reports normal sexual functions and has 2 biologic
What is the most appropriate next step in the manage-
children. He is not taking any medications, he does not
ment of this patient?
smoke or drink, and he has not used any recreational
a. Give the patient 100 mg of thiamine intravenously.
drugs. Physical examination findings are unremark-
b. Call a gastroenterologist to perform an emergent esophago-
able except for bilateral, tender, symmetrical gyne-
gastroduodenoscopy for the possibility of an anastomotic
comastia. His secondary sex characteristics, external
stricture.
genitalia, and testicular size and consistency are all
c. Give the patient 1,000 mcg of cyanocobalamin (vitamin B12)
normal. Laboratory test results are normal for serum
intramuscularly.
total and free testosterone, prolactin, thyrotropin, and
d. Give the patient a multivitamin injection intravenously.
dehydroepiandrosterone-sulfate (DHEA-S). Other
13. A 62-year-old man comes to your office for a routine results (and reference ranges) include serum estradiol
physical examination. He tries to live a healthy life- 78 pg/mL (1040 pg/mL) and -human chorionic gonad-
style. He follows a low-fat diet, rich in fruits and vege- otropin (hCG) 50,000 IU/L (<3.0 IU/L). Which of the
tables, and limits his intake of red meats. He exercises following tests is the most appropriate next step?
regularly, 3 to 4 times per week for 30 to 40 minutes a. Ultrasonography of the testicles
each time. He does not smoke. His medical history b. Computed tomography imaging of the adrenals
is positive for hypertension, hyperlipidemia, benign c. Magnetic resonance imaging of the pituitary
prostatic hypertrophy, and erectile dysfunction. He d. Mammography
does not have diabetes mellitus. He is interested in e. Liver biopsy
disease prevention and wonders about his nutritional 15. A 26-year-old woman presents at 4 weeks post partum
supplements. His family history is significant for with headaches, profound weakness, nausea, and vomit-
coronary artery disease in his father, who had a myo- ing. She had been breast-feeding. Physical examination
cardial infarction at age 65 years, and for type 2 dia- findings are unremarkable. Magnetic resonance imaging
betes mellitus in his mother. His current medications (MRI) of the head shows a sellar mass with suprasellar
are atorvastatin 20 mg daily, metoprolol 50 mg daily, extension but without chiasmal compression. Laboratory
tamsulosin 0.4 mg daily, tadalafil 10 mg as directed test results (and reference ranges) include the following:
before intercourse, folic acid 2.5 mg daily, fish oil serum sodium 125 mEq/L (136142 mEq/L), serum pro-
1,000 mg daily, vitamin E 400 international units lactin 72 g/L (430 g/L), 8 am serum cortisol 3 g/dL
daily, and a multivitamin daily. Physical examination (525 g/dL), serum adrenocorticotropic hormone
findings include the following: height 177 cm, weight (ACTH) 10 pg/mL (1060 pg/mL), and normal values
85 kg, body mass index 27, pulse 60 beats per minute, for serum free thyroxine and thyrotropin. Which of the
and blood pressure 135/70 mm Hg. He appears well following is the most likely diagnosis?
and physically fit. There are no abnormal findings on a. Primary adrenal insufficiency (Addison disease)
the cardiac examination. His prostate has mild dif- b. Prolactin-producing pituitary tumor
fuse enlargement without discrete nodularities. The c. Nonfunctioning pituitary tumor
remainder of the examination findings are unremark- d. Pituitary apoplexy (Sheehan syndrome)
able. Which of the following statements is false about e. Lymphocytic hypophysitis
vitamin and nutritional supplementation to prevent
disease? 16. A 32-year-old woman presented with amenorrhea of
a. Routine multivitamin supplementation has not been associ- 9 months duration. Physical examination findings were
ated with health benefits in population-based studies. unremarkable except for bilateral expressible galactor-
b. Antioxidant vitamin supplementation (beta carotene, vita- rhea. Laboratory test results included a negative preg-
min E) has not been associated with a lower risk of cardiovas- nancy test, evidence of a hypogonadotropic state, and a
cular events or mortality in randomized, placebo-controlled prolactin level of 250 g/L. Findings on magnetic reso-
clinical trials. nance imaging of the pituitary were consistent with a pitu-
c. Supplementation with fish oil (1,000 mg daily) is associated itary microadenoma. The patient was unable to tolerate
with a greater decrease in cardiovascular mortality compared dopaminergic-agonist drugs, and a decision was made to
with consumption of 2 or 3 meals of fatty fish (salmon) per treat her surgically. On the second day after transsphenoi-
week. dal microadenomectomy, postoperative diabetes insipi-
d. Folic acid supplementation with normalization of homo- dus developed but resolved by the third postoperative day.
cysteine levels has not been associated with a lower risk of Her medications at dismissal included an analgesic and
cardiovascular events in randomized, placebo-controlled replacement doses of hydrocortisone. She was instructed
clinical trials. in hydrocortisone dosage modifications during an acute

7. E N D O C R I N O L O GY Q U E S T I O N S A N D A N S W E R S 71
illness. A week after her hospital dismissal, she was evalu- a. Check the parathyroid hormone (PTH) level.
ated in an emergency department for headache, fatigue, b. Check the 25-hydroxyvitamin D level.
lethargy, confusion, and nausea. She had been taking c. Perform a parathyroid scan.
her medications regularly. Physical examination findings d. Emphasize the use of acetaminophen if flulike symptoms
were unremarkable. Which of the following is the most occur.
important laboratory test to perform? e. Use bisphosphonates only if her creatinine clearance is
a. Pregnancy test greater than 35 mL/min.
b. Serum free thyroxine
19. A 70-year-old man was hospitalized because of
c. Serum cortisol
anterior chest pain. His vital signs were normal. An
d. Serum sodium
urgent coronary angiogram led to angioplasty, which
e. Serum prolactin
was successfully carried out, and the patient was dis-
charged home 2 days later. During the next month,
he had some nervousness, anxiety, and a 10-kg
B O N E A N D PA R AT H Y RO I D D I S E A S E
weight loss while eating his usual diet. His cardiolo-
17. A 34-year-old woman is referred to you for a preopera- gist recently determined that the patient had atrial
tive evaluation from an otolaryngology colleague who fibrillation and prescribed a calcium channel blocker.
diagnosed a medullary thyroid carcinoma. That evalu- When you see him a few days later in the emergency
ation was prompted by a call from 1 of the patients department, he has a ventricular rate of 130 beats per
sisters who recently received the same diagnosis. The minute and shortness of breath. On examination, he
patient has no complaints, she has no past medical is in no acute distress at rest, but he is in atrial fibril-
problems, and she does not take any medications. Her lation with a rate of 120 to 130 beats per minute. He
vital signs are normal. You can feel a 1.5-cm right thy- has no tremors and no signs of Graves ophthalmopa-
roid lobe nodule without cervical adenopathy. The rest thy or dermopathy. His thyroid is low lying, multi-
of the examination findings are unremarkable. Which nodular, nontender, and slightly enlarged. You learn
of the following should you do preoperatively? of a long-standing history of nontoxic multinodular
a. Measure levels of serum calcium, gastrin, and insulin. goiter. Because of the atrial fibrillation, thyroid tests
b. Request magnetic resonance imaging of the abdomen. were performed, and the results were as follows: thy-
c. Obtain genetic testing for RET proto-oncogene mutation. rotropin (TSH) less than 0.01 mIU/L, free thyrox-
d. Recommend proceeding with surgery if the patients blood ine 3.7 ng/dL, and negative for thyroid peroxidase
pressure is normal. (TPO) antibodies. What is the most likely cause of
e. Measure levels of serum calcium and plasma fractionated this patients thyrotoxicosis?
metanephrines. a. Lymphocytic thyroiditis
b. Autonomous thyroid nodule
18. A 79-year-old woman broke her hip 3 months ago by
c. Iodine-induced hyperthyroidism
tripping on a stair. Subsequent evaluation (her first)
d. Graves disease
with dual energy x-ray absorptiometry (DEXA) identi-
e. TSH-producing pituitary adenoma
fied low bone mineral density with T scores less than
2.5 at the contralateral hip and spine. She has come 20. A 41-year-old woman has an elevated ionized calcium
to you with her daughter to discuss osteoporosis treat- level of 5.36 mg/dL during an infertility evaluation.
ment. You are told that she had normal periods until Her only complaints are fatigue and occasional head-
menopause at age 51, she never took estrogens but has aches. She denies having a history of constipation, nau-
begun taking 1,200 mg of elemental calcium daily since sea, vomiting, kidney stones, hematuria, or fractures.
she returned home from the hospital, and she takes 400 Her appetite was normal without significant weight
international units of vitamin D daily. Her calcium level changes. She denies having any known family history
is normal. She has had difficulty swallowing tablets after of hypercalcemia, metabolic bone disease, or kidney
she had a stroke 5 years ago, and she wants to discuss the stones. Her medications include fexofenadine and
use of intravenous (IV) bisphosphonates. You discuss multivitamins. Examination findings are unremark-
their side effects, including the risk of hypocalcemia. able: heart rate 70 beats per minute and regular, weight
You plan to identify and minimize that risk. Which of 72 kg, and blood pressure 132/78 mm Hg. Laboratory
the following should you do next? test results are shown in Table 7.Q20.

72 M AYO C L I N I C I N T E R N A L M E D I C I N E B OA R D R E VI EW: Q U E S T I O N S A N D A N S W E R S
Table 7.Q20 21. A 65-year-old woman with recurrent kidney stones
receives a diagnosis of hyperparathyroidism (total cal-
COMPONENT FINDING
cium 13.3 mg/dL, parathyroid hormone [PTH] 380
Ionized calcium, mg/dL 5.48 pg/mL). An experienced parathyroid surgeon performs
a minimally invasive parathyroidectomy and removes a
Total calcium, mg/dL 10.4 large parathyroid adenoma (6 g). Postoperatively, the
Albumin, g/dL 4.0 patient feels well and begins eating a normal diet. At
24 hours postoperatively, she begins experiencing
Phosphorus, mg/dL 3.1 perioral tingling and the Chvostek sign is positive. The
Creatinine, mg/dL 1.0 calcium level decreases to 8.1 mg/dL, and calcium car-
bonate (600 mg elemental calcium 3 tablets daily) is
Parathyroid hormone, pg/mL 59 given orally. On postoperative day 2, the tingling contin-
Serum protein electrophoresis Normal ues and the results of her laboratory tests are as follows:
total calcium 7.4 mg/dL, phosphorus 1.5 mg/dL, albu-
25-hydroxyvitamin D, ng/mL 22 min 3.9 g/dL, and creatinine 1.0 mg/dL. You administer
Blood cell counts Normal intravenous calcium, increase her oral calcium dosage,
and begin calcitriol therapy, and her symptoms improve.
Urinary creatinine, mg/d 1,500 What is the best explanation for these findings?
Urinary calcium, mg/d 129 a. Hungry bone syndrome
b. Severe vitamin D deficiency
Calcium clearance to creatinine clearance ratio 0.0075 c. Transient hypoparathyroidism from surgical manipulation
d. Transient hypoparathyroidism from long-term suppression
What is the best test for diagnosing familial hypocal- of normal parathyroids
ciuric hypercalcemia (FHH)? e. Lack of absorption of oral calcium
a. Another 24-hour urine collection
b. Parathyroid scan
c. Neck ultrasonography
d. Screening for FHH among family members
e. Calcium-sensing receptor mutational analysis

7. E N D O C R I N O L O GY Q U E S T I O N S A N D A N S W E R S 73
ANSWER S

1. Answer e. eye condition, lead to a transient increase in thyroid hor-


mone values, and not achieve euthyroidism for another 2
The replacement dose of levothyroxine is relatively low for
to 3 months. Surgery would be associated with additional
this patients weight (the usual dose is 1.6 mcg/kg body
risks because this patient has several cardiac comorbidities.
weight), and it would be unlikely to cause iatrogenic thyro-
The safest and most effective treatment for him would be
toxicosis. The free thyroxine level decreased with a decrease
antithyroid drug treatment. Methimazole would be the
in the levothyroxine dosage without a significant change in
first choice. PTU is a second-line drug because of its asso-
the thyrotropin level. The patient has symptoms of hypo-
ciation with severe liver failure and death, but PTU is still
thyroidism, and he has hypogonadism. His old records
preferred in the first trimester of pregnancy and during
might explain why he has central hypothyroidism. If those
thyroid storm. (See Bahn et al and Bartalena et al in the
records are not available, testing his adrenal axis and imag-
Suggested Reading list.)
ing his pituitary would be the next step. For the same
reason, decreasing the dosage to 50 mcg daily for a 75-kg 4. Answer c.
person or evaluating him for hyperthyroidism would not
address his symptoms or biochemical response to the first Metformin and sulfonylurea medications have been exten-
change in dosage. Androgens actually decrease the level of sively studied as monotherapy and combination therapy
thyroxine-binding globulins. (See Daly et al and Singer et for type 2 diabetes mellitus. For the overweight or obese
al in the Suggested Reading list.) patient, metformin is the recommended first-line therapy
because of its insulin-sensitizing effects, low risk of hypo-
2. Answer e. glycemia, and neutral impact on weight. Sulfonylurea
Increased estrogen levels, which were likely present in medications provide additional glucose lowering to the
this patient during oral contraceptive therapy, lead to patient who has inadequate glycemic control with optimal
an increase in thyroxine-binding globulins, and that doses of metformin and a recognized risk of hypoglycemia
requires an increase in the levothyroxine replacement and weight gain. No additional glucose-lowering effect
dose in hypothyroid patients to maintain the free thy- is observed with metformin doses higher than 2,000 mg
roxine levels unchanged. Thyroperoxidase antibodies are daily. The use of rosiglitazone is restricted because of recog-
not pathogenic, and their titer does not correlate with nized risks of heart failure and cardiovascular events, and it
the replacement dose. The patients hypothyroidism had is not appropriate for this patient. (See Nathan et al in the
been well controlled with a stable dosage of levothyroxine Suggested Reading list.)
for several years, so that nonadherence is probably not a
5. Answer d.
problem. The estrogen action is sustained, is not related
to the timing of the administration of levothyroxine This patient meets the body mass index (BMI) criterion
and oral contraceptive, and does not interfere with the for obesity (BMI >30) and is at risk for type 2 diabetes
absorption of levothyroxine. The combination of levothy- mellitus over the next 5 years. Many patients are unaware
roxine and liothyronine has not been proved to be more of their risk for progression toward type 2 diabetes mel-
effective clinically than levothyroxine monotherapy, but litus. Interventions to delay progression to type 2 diabe-
it has been associated with significantly more iatrogenic tes mellitus should be discussed. Lifestyle changes with
thyrotoxicosis in clinical trials; therefore, it is not recom- dietary modification, regular physical activity, and mod-
mended. (See Ain et al and Sawka et al in the Suggested est weight loss can substantially lower his risks for pro-
Reading list.) gressing to type 2 diabetes mellitus. Several medications
have been studied in type 2 diabetes mellitus prevention,
3. Answer c.
including metformin, rosiglitazone, acarbose, and orlistat.
This patient has thyrotoxicosis with Graves ophthalmopa- However, rosiglitazone was associated with significant
thy and dermopathy, which are pathognomonic for Graves risk for weight gain and heart failure despite the younger
disease. Therefore, the possibility of thyroiditis is effec- population studied (mean age, 55 years) in the Diabetes
tively excluded. The presence of moderate to severe oph- Reduction Assessment With Ramipril and Rosiglitazone
thalmopathy in a smoker is a relative contraindication for Medication (DREAM) trial. Since the DREAM trial, the
the use of radioactive iodine, which might exacerbate his use of rosiglitazone has been significantly restricted because

74
it is associated with a higher risk of cardiovascular events. patient currently meets criteria for obesity, and, with her
(See American Diabetes Association, Gerstein et al, and family history, screening by measuring the fasting blood
Knowler et al in the Suggested Reading list.) glucose level is indicated. Lifestyle changes with dietary
modification and regular physical activity are the corner-
6. Answer b. stone of weight management. Randomized studies have
In patients with diabetes mellitus, neuropathic symptoms shown that calorie restriction and not macronutrient com-
are often attributed to the underlying disease. However, in position (proportions of carbohydrate, protein, and fat)
a large percentage of patients (30%), long-term metformin determine success in weight loss and weight maintenance.
therapy can lead to vitamin B12 malabsorption and subse- (See Gardner et al in the Suggested Reading list.)
quent deficiency. If not recognized, a treatable cause for 11. Answer b.
irreversible neurologic symptoms would be overlooked.
(See Bell in the Suggested Reading list.) Bariatric surgery has become well accepted in the manage-
ment of medically complicated obesity mainly because of
7. Answer e. the benefits to the patient with type 2 diabetes mellitus.
Among the interventions listed, metformin therapy is the Remission rates for diabetes vary according to the operation
only one that has been clearly associated with a lower- performed: Remission rates are highest with biliopancreatic
ing of cardiovascular events and mortality in the United diversion and duodenal switch (92%) and Roux-en-Y gas-
Kingdom Prospective Diabetes Study. The Action to tric bypass (85%) and lowest with laparoscopic adjustable
Control Cardiovascular Risk in Diabetes (ACCORD) gastric banding (47%). A patient considering bariatric sur-
trial showed a trend toward lower cardiovascular events gery should be well informed about the currently accepted
only when fenofibrate was used with a statin and triglycer- criteria, risk factors that affect perioperative morbidity and
ide values were greater than 200 mg/dL. For a patient with mortality, and long-term management. (See Greenway
a low-density lipoprotein cholesterol level less than 70 mg/ et al in the Suggested Reading list.)
dL, there would be no additional benefit to increasing the 12. Answer a.
statin dosage. (See Stratton et al and Toth in the Suggested
Reading list.) Acute thiamine deficiency can occur in a patient with severe
persistent vomiting. It has been reported in patients who
8. Answer e. have had bariatric surgery that involved a restrictive mecha-
The normal physiologic response to hypoglycemia is low nism for weight loss, such as vertical banded gastroplasty,
insulin secretion with gradual development of ketosis. laparoscopic adjustable gastric banding, and Roux-en-Y
Endogenous insulin secretion is best determined not only gastric bypass. Early recognition and treatment are key to
by measuring insulin levels but by measuring C peptide, avoid neurologic complications and Wernicke encephal-
the peptide cleaved from the insulin molecule after secre- opathy. (See Serra et al in the Suggested Reading list.)
tion renders it active. To assure that true excessive endog- 13. Answer c.
enous insulin secretion is involved, the presence of insulin
secretagogues must be ruled out as a potential explanation Health benefits of fish oil supplementation vary according
for hypoglycemia with elevated or inappropriately nor- to the dosage of active docosahexaenoic acid (DHA) and
mal insulin and C peptide levels. (See Cryer et al in the eicosapentaenoic acid (EPA) consumed. At lower dosages
Suggested Reading list.) (1,000 mg daily), omega-3 fatty acids are associated with a
lower risk of cardiovascular mortality. The same benefit is
9. Answer a. observed when individuals consume 2 servings weekly of a
This patient is receiving maximal dosages of the recom- fatty fish such as salmon. For individuals already consum-
mended first- and second-line therapies for the manage- ing this amount of omega-3 fatty acids in their diet, there
ment of type 2 diabetes mellitus. The choice of a third agent is no additional benefit to additional omega-3 fatty acid
should take several factors into consideration. The use of supplementation. Higher doses of DHA and EPA (24 g
rosiglitazone, a TZD, is significantly restricted because daily) are needed to lower triglyceride levels. (See Jacobson
of the recognized risk of cardiovascular events. The same in the Suggested Reading list.)
risks may not apply to pioglitazone. (See Loke et al in the
14. Answer a.
Suggested Reading list.)
Gynecomastia is the most common disorder of the male
10. Answer d.
breast and accounts for more than 80% of all male breast
For a patient with recent weight gain, it is reasonable to masses. It may result from a trivial cause, or it may be an early
screen for thyroid function abnormalities with an sTSH sign of a sinister disorder. The basic mechanism of gyne-
test. Other endocrinopathies such as Cushing syndrome comastia is relative estradiol excess, which can result from
are rare, and screening should likely not be pursued in the decreased androgen production or effect, or an increase in
absence of other suspicious physical findings such as facial estrogen production or effect. In adults, the 2 most com-
plethora, purple striae, hirsutism, and hypertension. The mon causes of gynecomastia are drugs and alcohol-related

7. E N D O C R I N O L O GY Q U E S T I O N S A N D A N S W E R S 75
liver disease. Less common causes include recovery from and lasts a few days. Permanent diabetes insipidus is rare.
malnutrition or other serious chronic illness. Rarely Transient excess AVP leading to syndrome of inappropriate
encountered are hCG-producing tumors, adrenal tumors, secretion of antidiuretic hormone (SIADH) and hypona-
and testicular tumors. In this patient, drugs and alcohol, tremia occurs in 10% of patients within 5 to 14 days postop-
gonadal failure, hyperthyroidism, and prolactin-producing eratively. Pregnancy in the first week after transsphenoidal
pituitary tumor have been excluded as causes. This patient surgery is unrealistic and a pregnancy test is not warranted.
has an hCG-producing tumor and increased estrogen If symptomatic postoperative hypothyroidism occurs, it
production. Although the most common manifestation usually occurs several weeks postoperatively. The patient
of testicular tumors is a testicular enlargement or mass, has been taking her hydrocortisone regularly; therefore,
small testicular tumors may often be occult. Of primary she would not have cortisol deficiency, so measuring serum
testicular tumors, 97% arise from seminiferous tubules cortisol would be unnecessary. Although the patient may
(germ cell tumors) and about 3% arise from interstitial tis- have persistent hyperprolactinemia after surgery for micro-
sue. Germ cell tumors are the most common solid tumors prolactinoma, she would not be symptomatic in the first
in men between the ages of 15 and 34 years; these tumors week postoperatively. The most important consideration
occur primarily in white men and are the main tumors that in this patient is SIADH; therefore, measuring the serum
secrete hCG. Stimulation of Leydig cells by hCG results sodium level is the most important laboratory test. (See
in an increased secretion of both estrogen and andro- Lien and Shapiro in the Suggested Reading list.)
gen. Testicular ultrasonography is the most appropriate
17. Answer e.
step. A normal DHEA-S value excludes a diagnosis of an
estrogen-producing adrenal tumor, a normal prolactin Multiple endocrine neoplasia (MEN) type 2 must be con-
value excludes the diagnosis of a prolactin-producing pitu- sidered because of the familial association described. The
itary tumor, and imaging studies of these glands are not conditions that may also be present in this patient, there-
warranted. Mammograms are not required for evaluation fore, are pheochromocytoma (in 40%-50% of patients with
of symmetrical concentric gynecomastia; they are indi- MEN type 2) and hyperparathyroidism (in 10%-20%).
cated in the evaluation of unilateral breast masses or when Both are important from an operative perspective:
breast cancer is considered. Nothing in the presentation Hemodynamic fluctuations can occur with pheochromocy-
of this patient suggests an occult liver disease that would toma, and hyperparathyroidism can be addressed with the
necessitate a liver biopsy. (See Braunstein in the Suggested thyroid surgery. Therefore, screening for both with highly
Reading List.) sensitive biomarkers is important. The testing for calcium,
gastrin, and insulin is a starting point for abnormalities
15. Answer e.
in MEN type 1. Imaging the abdomen for diagnosing
Headaches, pituitary insufficiency, and MRI findings con- pheochromocytoma has a very poor specificity (because
sistent with a pituitary tumor in relation to pregnancy of adrenal incidentalomas) without any significant gain
should always lead to consideration of lymphocytic hypo- in sensitivity compared with measuring levels of plasma
physitis. Absence of a high ACTH level excludes primary metanephrines. Genetic testing takes time (26 weeks),
adrenal insufficiency. Pituitary tumors, whether prolactin and a positive test does not indicate the presence of 1 or
producing or nonfunctioning, are predominantly benign both of these associated endocrinopathies. Normal blood
and slow growing. A pituitary tumor would have been pres- pressure is present in about 10% of patients with pheochro-
ent for a while and would probably have made the patient mocytoma (the percentage is higher among patients with
hypogonadotropic and infertile, so that she would not familial syndromes). (See Kloos et al and Raue et al in the
have presented initially in the postpartum state. Patients Suggested Reading list.)
with Sheehan syndrome have a history of postpartum
18. Answer b.
hemorrhage-induced hypotension or shock that requires
blood transfusion; the signs and symptoms of hypopitu- Hypocalcemia is more likely to occur in patients who have
itarism are usually recognized early after delivery, and the vitamin D deficiency; therefore, the risk of hypocalcemia
patient loses the ability to breast-feed. Lymphocytic hypo- can be minimized by vitamin D and calcium supplemen-
physitis is therefore the most likely diagnosis. (See Rivera tation. The serum calcidiol (25-hydroxyvitamin D) con-
in the Suggested Reading list.) centration should be assessed before the patient receives
IV bisphosphonate infusions. Patients with vitamin D
16. Answer d.
deficiency (25-hydroxyvitamin D <15 ng/mL) should be
Major complications of pituitary surgery include postop- treated before the infusion. Results from checking the PTH
erative hemorrhage, cerebrospinal fluid leak, meningitis, level and performing the parathyroid scan would be inad-
and visual disturbances; these occur in less than 5% of equate for diagnosing vitamin D deficiency. In addition,
patients and occur most frequently in patients who have having a normal serum calcium level excludes the diagnosis
large tumors and not microadenomas. Diabetes insipidus, of hypoparathyroidism. Acetaminophen will not mini-
the most common disorder of arginine vasopressin (AVP) mize the risk of hypocalcemia since it only affects the sys-
secretion in the postoperative state, occurs in less than 15% temic inflammation that occurs with IV bisphosphonates.
of patients and usually occurs early after pituitary surgery The risk of hypocalcemia is independent of the creatinine

76 M AYO C L I N I C I N T E R N A L M E D I C I N E B OA R D R E VI EW: Q U E S T I O N S A N D A N S W E R S
clearance even though IV bisphosphonates should not be disease (reflected by the very large parathyroid gland
used in patients with a creatinine clearance less than 30 to and the significant calcium and PTH elevations), and it
35 mL/min because of the risk of renal osteodystrophy. occurs early after surgery. Continuing calcium therapy
(See Black et al and Rosen and Brown in the Suggested beyond symptom control will likely improve her overall
Reading list.) bone mass. With the high calcium value preoperatively, it
is quite unlikely that she had significant vitamin D defi-
19. Answer c.
ciency to explain these findings. The minimal access of
Iodine-induced hyperthyroidism can be easily overlooked parathyroidectomy avoids bilateral neck exploration, thus
and is likely more common than is realized. It usually mani- eliminating the possibility of surgical manipulation of all
fests as apathetic hyperthyroidism, which can be masked the parathyroid glands and its consequences. The relative
even further by the use of -blockers. The time frame for insufficiency of the remaining parathyroids from pro-
exposure to iodinated contrast agents a month earlier is typ- longed suppression is not consistent with a low phospho-
ical for the development of thyrotoxicosis in patients with rus level since hypoparathyroidism is associated with low
preexisting multinodular goiter. Treatment involves anti- renal phosphate clearance. Oral calcium supplementation
thyroid drugs and supportive measures. The other choices might be decreased initially, but it should improve in the
can be excluded on clinical grounds. Lymphocytic thy- second day, and the phosphorus level should be normal.
roiditis is relatively rare in the elderly and is almost always (See Brasier and Nussbaum, and Tohme and Bilezikian in
associated with anti-TPO antibodies, which were absent the Suggested Reading list.)
in this patient. This disorder can occur in patients who
have Graves disease and preexisting iodine insufficiency.
However, iodine insufficiency is rare in North America, SUGGESTED RE ADING
and the multinodular goiter also argues against Graves
disease. An autonomous toxic nodule evolves slowly over Ain KB, Mori Y, Refetoff S. Reduced clearance rate of thyroxine-binding
many years rather than arising abruptly. A TSH-producing globulin (TBG) with increased sialylation: a mechanism for
adenoma can be excluded by the suppressed TSH value. estrogen-induced elevation of serum TBG concentration. J Clin
Endocrinol Metab. 1987 Oct;65(4):68996.
(See Bahn Chair et al and Martin et al in the Suggested American Diabetes Association. Standards of medical care in diabetes:
Reading list.) 2011. Diabetes Care. 2011 Jan;34(Suppl 1):S1161.
Bahn RS, Burch HS, Cooper DS, Garber JR, Greenlee CM, Klein IL,
20. Answer d. et al. The role of propylthiouracil in the management of Graves dis-
FHH is an autosomal dominant disorder characterized ease in adults: report of a meeting jointly sponsored by the American
Thyroid Association and the Food and Drug Administration. Thyroid.
by asymptomatic hypercalcemia, normal parathyroid 2009 Jul;19(7):6734.
hormone levels, and low urinary calcium levels. The cause Bahn Chair RS, Burch HB, Cooper DS, Garber JR, Greenlee MC, Klein I,
is usually an inactivating mutation in the calcium-sensing et al; American Thyroid Association; American Association of Clinical
receptor, which makes the parathyroid glands less sensi- Endocrinologists. Hyperthyroidism and other causes of thyrotoxico-
tive to calcium and increases the tubular reabsorption of sis: management guidelines of the American Thyroid Association and
American Association of Clinical Endocrinologists. Thyroid. 2011
calcium. Screening for FHH among family members is Jun;21(6):593646. Epub 2011 Apr 21. Erratum in: Thyroid. 2011
the most reliable method of making the diagnosis, and Oct;21(10):1169.
genetic testing should be used only in atypical cases if Bartalena L, Marcocci C, Bogazzi F, Manetti L, Tanda ML, DellUnto E,
family members are not available. This patients sister had et al. Relation between therapy for hyperthyroidism and the course of
asymptomatic hypercalcemia with a calcium clearance Graves ophthalmopathy. N Engl J Med. 1998 Jan 8;338(2):738.
Bell DS. Metformin-induced vitamin B12 deficiency presenting as a
to creatinine clearance ratio of 0.005. For approximately peripheral neuropathy. South Med J. 2010 Mar;103(3):2657.
80% of patients, the ratio is less than 0.01; in contrast, for Black DM, Delmas PD, Eastell R, Reid IR, Boonen S, Cauley JA, et al;
primary hyperparathyroidism patients, the ratio is usually HORIZON Pivotal Fracture Trial. Once-yearly zoledronic acid for
greater than 0.02. Parathyroid scanning and ultrasonog- treatment of postmenopausal osteoporosis. N Engl J Med. 2007 May
raphy will not reliably distinguish these 2 entities and 3;356(18):180922.
Brasier AR, Nussbaum SR. Hungry bone syndrome: clinical and bio-
should be used only for localization purposes if the diag- chemical predictors of its occurrence after parathyroid surgery. Am J
nosis of primary hyperparathyroidism has been estab- Med. 1988 Apr;84(4):65460.
lished. Given that the urinary creatinine value from the Braunstein GD. Clinical practice: gynecomastia. N Engl J Med. 2007 Sep
24-hour collection appears complete, there is no need to 20;357(12):122937.
repeat it. (See Heath, and Tfelt-Hansen and Brown in the Cryer PE, Axelrod L, Grossman AB, Heller SR, Montori VM, Seaquist
ER, et al; Endocrine Society. Evaluation and management of adult
Suggested Reading list.) hypoglycemic disorders: an Endocrine Society Clinical Practice
21. Answer a. Guideline. J Clin Endocrinol Metab. 2009 Mar;94(3):70928.
Daly RC, Su TP, Schmidt PJ, Pagliaro M, Pickar D, Rubinow DR.
When PTH values are not available, the most useful evi- Neuroendocrine and behavioral effects of high-dose anabolic steroid
administration in male normal volunteers. Psychoneuroendocrinology.
dence of hungry bone syndrome is low calcium and phos-
2003 Apr;28(3):31731.
phorus levels. Both calcium and phosphorus are moving Gardner CD, Kiazand A, Alhassan S, Kim S, Stafford RS, Balise RR,
rapidly into the unmineralized osteoid of this patient. et al. Comparison of the Atkins, Zone, Ornish, and LEARN diets
Hungry bone syndrome occurs in patients with severe for change in weight and related risk factors among overweight

7. E N D O C R I N O L O GY Q U E S T I O N S A N D A N S W E R S 77
premenopausal women: the A TO Z Weight Loss Study: a random- Association for Study of Diabetes. Medical management of hyper-
ized trial. JAMA. 2007 Mar 7;297(9):96977. Erratum in: JAMA. glycemia in type 2 diabetes: a consensus algorithm for the initiation
2007 Jul 11;298(2):178. and adjustment of therapy: a consensus statement of the American
Gerstein HC, Yusuf S, Bosch J, Pogue J, Sheridan P, Dinccag N, et al; Diabetes Association and the European Association for the Study
DREAM (Diabetes REduction Assessment with ramipril and rosigli- of Diabetes. Diabetes Care. 2009 Jan;32(1):193203. Epub 2008
tazone Medication) Trial Investigators. Effect of rosiglitazone on the Oct 22.
frequency of diabetes in patients with impaired glucose tolerance Raue F, Frank-Raue K, Grauer A. Multiple endocrine neoplasia type 2:
or impaired fasting glucose: a randomised controlled trial. Lancet. clinical features and screening. Endocrinol Metab Clin North Am.
2006 Sep 23;368(9541):1096105. Erratum in: Lancet. 2006 Nov 1994 Mar;23(1):13756.
18;368(9549):1770. Rivera JA. Lymphocytic hypophysitis: disease spectrum and approach to
Greenway SE, Greenway FL 3rd, Klein S. Effects of obesity surgery diagnosis and therapy. Pituitary. 2006;9(1):3545.
on non-insulin-dependent diabetes mellitus. Arch Surg. 2002 Rosen CJ, Brown S. Severe hypocalcemia after intravenous bisphospho-
Oct;137(10):110917. nate therapy in occult vitamin D deficiency. N Engl J Med. 2003 Apr
Heath H 3rd. Familial benign (hypocalciuric) hypercalcemia: a trouble- 10;348(15):15034.
some mimic of mild primary hyperparathyroidism. Endocrinol Metab Sawka AM, Gerstein HC, Marriott MJ, MacQueen GM, Joffe RT. Does
Clin North Am. 1989 Sep;18(3):72340. a combination regimen of thyroxine (T4) and 3,5,3-triiodothyronine
Jacobson TA. Beyond lipids: the role of omega-3 fatty acids from fish oil in improve depressive symptoms better than T4 alone in patients with
the prevention of coronary heart disease. Curr Atheroscler Rep. 2007 hypothyroidism? Results of a double-blind, randomized, controlled
Aug;9(2):14553. trial. J Clin Endocrinol Metab. 2003 Oct;88(10):45515.
Kloos RT, Eng C, Evans DB, Francis GL, Gagel RF, Gharib H, et al; Serra A, Sechi G, Singh S, Kumar A. Wernicke encephalopathy
American Thyroid Association Guidelines Task Force. Medullary after obesity surgery: a systematic review. Neurology. 2007 Aug
thyroid cancer: management guidelines of the American Thyroid 7;69(6):615.
Association. Thyroid. 2009 Jun;19(6):565612. Erratum in: Thyroid. Singer PA, Cooper DS, Levy EG, Ladenson PW, Braverman LE, Daniels
2009 Nov;19(11):1295. G, et al; Standards of Care Committee, American Thyroid Association.
Knowler WC, Barrett-Connor E, Fowler SE, Hamman RF, Lachin JM, Treatment guidelines for patients with hyperthyroidism and hypothy-
Walker EA, et al; Diabetes Prevention Program Research Group. roidism. JAMA. 1995 Mar 8;273(10):80812.
Reduction in the incidence of type 2 diabetes with lifestyle interven- Stratton IM, Adler AI, Neil HA, Matthews DR, Manley SE, Cull CA,
tion or metformin. N Engl J Med. 2002 Feb 7;346(6):393403. et al. Association of glycaemia with macrovascular and microvascular
Lien YH, Shapiro JI. Hyponatremia: clinical diagnosis and management. complications of type 2 diabetes (UKPDS 35): prospective observa-
Am J Med. 2007 Aug;120(8):6538. tional study. BMJ. 2000 Aug 12;321(7258):40512.
Loke YK, Kwok CS, Singh S. Comparative cardiovascular effects of thi- Tfelt-Hansen J, Brown EM. The calcium-sensing receptor in normal
azolidinediones: systematic review and meta-analysis of observational physiology and pathophysiology: a review. Crit Rev Clin Lab Sci.
studies. BMJ. 2011 Mar 17;342:d1309. 2005;42(1):3570.
Martin FI, Tress BW, Colman PG, Deam DR. Iodine-induced hyper- Tohme JF, Bilezikian JP. Diagnosis and treatment of hypocalcemic emer-
thyroidism due to nonionic contrast radiography in the elderly. Am J gencies. Endocrinologist. 1996 Jan;6(1):1018.
Med. 1993 Jul;95(1):7882. Toth PP. Fibrate therapy in the management of diabetic dyslipidemia:
Nathan DM, Buse JB, Davidson MB, Ferrannini E, Holman RR, there is no ACCORD to be found. Curr Atheroscler Rep. 2010
Sherwin R, et al; American Diabetes Association; European Sep;12(5):3315.

78 M AYO C L I N I C I N T E R N A L M E D I C I N E B OA R D R E VI EW: Q U E S T I O N S A N D A N S W E R S
8.
ONCOLOGY QUESTIONS AND ANSWER S

QUESTIONS not tender. Findings from mammography, breast ultra-


sonography, and computed tomography of the chest are
Multiple Choice (choose the best answer) negative except for the presence of right axillary ade-
1. An otherwise healthy 32-year-old man asks you about nopathy. Biopsy of a lymph node shows a moderately
screening tests for colon cancer. He reports that his differentiated adenocarcinoma of unknown primary
2 older sisters had colon cancer at age 40 and 42, his origin. What should be the next step in evaluation of
mother had colon cancer at age 40 and endometrial this patient?
a. Perform a mediastinoscopy.
cancer at age 45, and a maternal aunt had breast cancer
b. Assume that nonsmall cell carcinoma is present, and treat
at a young age. He notes that there was not a history of
with cisplatin-based chemotherapy.
colon polyps in any family member. What should you
c. Clarify the histogenetic origin of the tumor by testing for
recommend for this patient?
tumor markers: carcinoembryonic antigen, cancer antigen
a. Annual fecal occult blood testing
153, and neuron-specific enolase.
b. Colonoscopy now and every 1 to 2 years thereafter
d. Perform breast magnetic resonance imaging (MRI).
c. Colonoscopy with random biopsies to look for inflamma-
e. Recommend bilateral mastectomies.
tory bowel diseaseif not present, follow routine screening
recommendations for average-risk Americans 4. A 38-year-old woman presents for intermittent abdom-
d. Prophylactic colectomy inal pain and bloating that has been getting worse for
e. Reassurance only, since no polyps were found in family the past several months. She has been reading medical
members information on the Internet and is very concerned about
2. A 72-year-old male smoker with a 42-pack-year history ovarian cancer. She has no family history of malig-
presents with anorexia, cough, and altered mental sta- nancy and has been otherwise healthy. She is not tak-
tus. A chest radiograph shows a right-sided mass that, on ing any medications. Physical examination findings are
bronchoscopy, is identified as squamous cell carcinoma. remarkable only for some tenderness to movement of
On physical examination, the patient is thin, cachec- the uterus. No pelvic masses are detected. She requests
tic, dehydrated, and disoriented with no focal neuro- a serum cancer antigen (CA) 125 test; the result is 86
logic deficits. The calcium level is elevated (15 mg/dL), U/mL (reference range <35 U/mL). How should you
creatinine is 2.5 mg/dL, and albumin is 2.2 g/dL. advise her at this time?
A bone scan shows only some degenerative changes. a. Recommend combination chemotherapy with cisplatin and
What is the most appropriate next step in the manage- paclitaxel.
ment of this patient? b. Tell her that this degree of CA 125 elevation occurs only in
a. Cisplatin-based chemotherapy ovarian cancer.
b. Radiotherapy to the brain c. Tell her that although the CA 125 elevation is concerning,
c. Intravenous fluids and bisphosphonates multiple conditions can cause such an elevation, and further
d. Dexamethasone 100 mg given as an intravenous push investigation is warranted.
e. Emergent magnetic resonance imaging of the head d. Recommend exploratory laparotomy with total abdomi-
nal hysterectomy, bilateral salpingo-oophorectomy, pelvic
3. A 55-year-old woman with a history of hypertension, a lymphadenectomy, omentectomy, and aggressive surgical
30-pack-year history of smoking, mild chronic obstruc- debulking of all disease.
tive pulmonary disease, and moderate obesity presents e. Recommend only observation now, and recheck the CA 125
with a right axillary mass. She has a family history of level in 3 months.
coronary artery disease and strokes. Current medica-
tions include a statin, a diuretic, and a -blocker. On 5. A 38-year-old woman with recently diagnosed
examination, she is moderately obese and in no dis- node-positive breast cancer presents to the emergency
tress. Her lungs have increased sound in the expiratory department with a temperature of 38.3C. She reports
phase diffusely but no frank wheezing or other sounds. having mild chills and fever but denies having nausea,
Findings from examination of the heart, abdomen, and vomiting, diarrhea, cough, or dysuria. Seven days ago
breasts are unremarkable. On lymph node examina- she received her third cycle of doxorubicin and cyclo-
tion, a palpable right axillary mass is firm, mobile, and phosphamide chemotherapy; thus far, she has tolerated

79
the cycles well. On physical examination, she is pleasant a. He is at increased risk for secondary cancers of the bladder,
and appears fatigued but in no distress, with the follow- prostate, and rectum, but the risk is only about 1 in 70 at 10
ing findings: blood pressure 122/78 mm Hg, pulse 82 years.
beats per minute and regular, respiration rate 14 breaths b. There is no need to worry since he has no increased risk for
per minute, and temperature 38.2C. The remainder of secondary malignancies.
the examination is remarkable only for alopecia. A chest c. He is at high risk for lung cancer in the future, so he should
radiograph is clear of abnormalities. Urinalysis shows have routine screening chest radiographs.
no leukocytes. Laboratory data include the following: d. He is right to worry since secondary cancers are very com-
hemoglobin 11.4 g/dL, leukocyte count 0.8109/L, mon, and screening should be done at regular intervals.
absolute neutrophil count 0.25109/L, and platelet e. There is no longer a reason to screen for prostate cancer since
count 90109/L. She lives in town with her husband he has had radiotherapy to this area.
and 2 children (aged 10 and 14). No one else is ill at
7. An 82-year-old man comes to your office for rou-
home. At this time, what should you do?
tine follow-up care. He has a prior history of chronic
a. Admit her to the hospital to receive broad-spectrum
obstructive pulmonary disease with a forced expiratory
antibiotics.
volume in 1 second of 25% of the predicted value, coro-
b. Administer granulocyte colony-stimulating factor now.
nary artery disease with mild congestive heart failure,
c. Send her home, and ask her to follow up with her oncologist
hypertension, and type 2 diabetes mellitus. He denies
in the morning.
having any urinary symptoms. He takes the following
d. Collect blood and urine samples for cultures, begin therapy
medication: enalapril 5 mg twice daily, hydrochloro-
with amoxicillinclavulanate potassium and ciprofloxacin
thiazide 25 mg twice daily, lovastatin 20 mg once daily,
orally, discharge to home, and ask her to follow up with her
albuterol inhaler as needed, fluticasone propionate 250
oncologist by telephone within 24 hours.
mcg and salmeterol 50 mcg inhalation powder daily,
e. Obtain a throat swab for influenza virus.
aspirin 325 mg daily, glipizide 10 mg twice daily, and a
6. A 67-year-old man with a history of stage II rectal can- multivitamin daily. On examination, he has poor breath
cer was treated with resection and combined chemo- sounds in all areas, distant heart tones, a normal abdo-
therapy and radiotherapy 6 years ago. He has recovered men, and edema (2+) of the lower extremities bilater-
well from the operations and treatments; he still has ally. On rectal examination, he has an enlarged prostate
some rectal and bladder urgency but no incontinence. with a firm nodule. The prostate-specific antigen level is
He recently retired from his job as an office manager 8.5 ng/mL, and transrectal needle biopsy shows adeno-
and is physically active. His hypertension is well con- carcinoma (Gleason grade 6). A bone scan shows only
trolled with a -blocker, and his cholesterol levels are some changes consistent with degenerative disease.
controlled with diet. There is no significant family his- What can you tell him at this time?
tory. His most recent follow-up colonoscopy was done a. Radical prostatectomy is likely to improve his overall sur-
10 months ago and showed no evidence of recurrence vival and decrease his chance of death from prostate cancer.
or other disease. He is a lifetime nonsmoker. On physi- b. Given his lack of symptoms from prostate cancer, combined
cal examination, he is thin and pleasant, and he appears with his age and comorbid conditions, a watchful waiting
fit. General examination findings are unremarkable. approach is reasonable.
On rectal examination, his prostate feels normal and c. External beam radiotherapy is not effective against prostate
smooth without palpable masses. A stool sample is neg- cancer.
ative for heme. Liver function test results are normal, d. Chemotherapy can be used to decrease his risk of recurrence
the level of carcinoembryonic antigen (CEA) is within of prostate cancer.
the reference range, and the level of prostate-specific e. Orchiectomy is the standard of care.
antigen is 1.4 ng/mL. He is concerned about late side
effects of his prior therapy, specifically about the devel-
opment of new cancers. What should you tell him?

80 M AYO C L I N I C I N T E R N A L M E D I C I N E B OA R D R E VI EW: Q U E S T I O N S A N D A N S W E R S
ANSWER S

1. Answer b. MRI for routine screening should be limited to high-risk


women since it has not been shown to be beneficial for
With his family history, this patient is at very high risk for
average-risk patients. The detection of hormone receptors
colon cancer. It is unlikely that he would have a hereditary
in the pathologic specimen has diagnostic and therapeu-
polyposis syndrome since no family member had polyps.
tic implications. Serum tumor markers are rarely useful
His family history is highly concerning for hereditary
diagnostic tools (with few exceptions). (See Chen et al and
nonpolyposis colorectal cancer (Lynch syndrome) because
Saslow et al in the Suggested Reading list.)
multiple first-degree relatives were affected at an early age
and because there is a family history of breast and endome- 4. Answer c.
trial cancer. This syndrome is associated with a defect in
mismatch repair enzymes and leads to microsatellite insta- Further investigation is warranted to determine the cause
bility. Screening with fecal occult blood testing is not ade- of her discomfort and the reason for her elevated CA
quately sensitive for patients at high riskor even normal 125 level. This nonspecific serum marker can be elevated
risk. Inflammatory bowel disease does significantly increase in many benign conditions, such as endometriosis, preg-
the risk of colon cancer, but nothing in the patients history nancy, menstruation, and peritonitis. The positive pre-
suggests that it is present. Prophylactic colectomy would dictive value of CA 125 for screening is only about 2% to
be a consideration only if testing is positive for the defec- 3%. Although very high levels (several hundred to several
tive gene. (See Rex et al and Umar et al in the Suggested thousand units per milliliter) typically occur only in ovar-
Reading list.) ian cancer, patients with endometriosis can have levels
around 200 U/mL with stage IV disease. Chemotherapy is
2. Answer c. never indicated until there is tissue confirmation of disease.
The patient needs intravenous fluids and restoration of Extensive resection would be indicated if ovarian carci-
intravascular volume along with bisphosphonates to cor- noma were diagnosed, and this would typically be followed
rect the hypercalcemia. In the absence of any focal neuro- by systemic chemotherapy. Since the patient is having sig-
logic deficits, it is unlikely that his disorientation is due to nificant symptoms, observation only is not warranted. (See
metastatic disease; altered mental status is very common Partridge et al in the Suggested Reading list.)
with hypercalcemia. If correction of the hypercalcemia 5. Answer d.
reverses the altered mental status, central nervous system
imaging is not required. Cranial radiotherapy should be Febrile neutropenia is common with many forms of che-
given only after metastatic disease is identified in a patient motherapy. The vast majority of patients have negative
with nonsmall cell carcinoma. Dexamethasone is used to cultures. Patients who are medically stable, are able to
decrease peritumoral edema from intracranial metastases maintain oral intake, are reliable for close follow up, and
and is not indicated for this patient. Dexamethasone can live near a medical facility can be safely treated with an
help to significantly correct hypercalcemia due to multiple outpatient regimen. After neutropenia develops, there is
myeloma or lymphoma, but it is unlikely to correct hyper- no role for administration of growth factors. Observation
calcemia due to squamous cell carcinoma. (See Behl et al alone is insufficient since the low absolute neutrophil count
and Halfdanarson et al in the Suggested Reading list.) puts her at significant risk for sepsis. (See Behl et al and
Halfdanarson et al in the Suggested Reading list.)
3. Answer d.
6. Answer a.
Women presenting with axillary lymph node metastases
of adenocarcinoma of unknown primary origin should Patients treated with pelvic radiotherapy for rectal or pros-
undergo thorough evaluation for breast cancer. Breast tate cancer are at increased risk for secondary malignancies
MRI has greater sensitivity than mammography or ultra- in the area, but this risk is low (estimated to be 1 in 125 at 5
sonography, and among women who have occult adeno- years and 1 in 70 at 10 years). Patients should still undergo
carcinoma in the axillary lymph nodes, MRI can detect a screening for cancers that they are at risk for, as long as
primary lesion in up to 75% of patients. Although breast their general health and other medical conditions warrant
MRI is helpful for evaluation of women with adenocar- screening. This patient does not have an increased risk for
cinoma metastatic to axillary lymph nodes, use of breast lung cancer per se, but lung metastases are a common site

81
for recurrence of rectal cancer. The pattern of recurrence are typically reserved for patients with metastatic or symp-
is different from that of colon cancer, which much more tomatic disease. (See National Comprehensive Cancer
commonly metastasizes to the liver before traveling to the Network [NCCN] in the Suggested Reading list.)
lung. The venous drainage of the rectum is into the inferior
vena cava, while most of the colons venous drainage is to
the portal system. Routine follow-up for otherwise healthy SUGGESTED RE ADING
patients who have colorectal cancer includes the follow-
ing: 1) Evaluate with a history and physical examination Behl D, Hendrickson AW, Moynihan TJ. Oncologic emergencies. Crit
every 3 months for 2 years and then every 6 months for Care Clin. 2010 Jan;26(1):181205.
a total of 5 years. 2) Perform a colonoscopy in 1 year. If Chen C, Orel SG, Harris E, Schnall MD, Czerniecki BJ, Solin LJ.
Outcome after treatment of patients with mammographically
results are abnormal, perform another colonoscopy in 1 occult, magnetic resonance imaging-detected breast cancer pre-
year; if results are normal, perform a colonoscopy as clini- senting with axillary lymphadenopathy. Clin Breast Cancer. 2004
cally indicated. 3) Determine CEA levels every 3 months Apr;5(1):727.
for 2 years and then every 6 months for years 3 to 5. 4) Halfdanarson TR, Hogan WJ, Moynihan TJ. Oncologic emergencies:
Consider computed tomography of the chest, abdomen, diagnosis and treatment. Mayo Clin Proc. 2006 Jun;81(6):83548.
Erratum in: Mayo Clin Proc. 2006 Nov;81(11):1509.
and pelvis annually for 3 years if the patient is at high risk. National Comprehensive Cancer Network (NCCN) guidelines [Internet].
(See National Comprehensive Cancer Network [NCCN] Fort Washington (PA): National Comprehensive Cancer Network;
in the Suggested Reading list.) c2012 [cited 2011 Mar 20]. Available from: www.nccn.org.
Partridge E, Kreimer AR, Greenlee RT, Williams C, Xu JL, Church
7. Answer b. TR, et al; PLCO Project Team. Results from four rounds of ovar-
ian cancer screening in a randomized trial. Obstet Gynecol. 2009
For patients older than 65, especially those with signifi- Apr;113(4):77582.
cant comorbid conditions, it is unclear whether radical Rex DK, Johnson DA, Anderson JC, Schoenfeld PS, Burke CA, Inadomi
prostatectomy improves the patients overall survival and JM; American College of Gastroenterology. American College of
it certainly has adverse effects on quality of life. Patients Gastroenterology guidelines for colorectal cancer screening 2009 [cor-
rected]. Am J Gastroenterol. 2009 Mar;104(3):73950. Epub 2009
in good condition who are younger than 65 do seem to
Feb 24. Erratum in: Am J Gastroenterol. 2009 Jun;104(6):1613.
have a survival advantage if treated with radical prosta- Saslow D, Boetes C, Burke W, Harms S, Leach MO, Lehman CD, et al;
tectomy instead of watchful waiting; however, this elderly American Cancer Society Breast Cancer Advisory Group. American
patient with multiple other illnesses is unlikely to benefit Cancer Society guidelines for breast screening with MRI as an adjunct
from aggressive treatment. External beam radiotherapy or to mammography. CA Cancer J Clin. 2007 Mar-Apr;57(2):7589.
Erratum in: CA Cancer J Clin. 2007 May-Jun;57(3):185.
brachytherapy would be a reasonable treatment option, but
Umar A, Boland CR, Terdiman JP, Syngal S, de la Chapelle A, Ruschoff
each carries a risk of impotence, rectal injury, and inconti- J, et al. Revised Bethesda Guidelines for hereditary nonpolyposis col-
nence. Chemotherapy has no role except for patients with orectal cancer (Lynch syndrome) and microsatellite instability. J Natl
metastatic disease. Orchiectomy or hormonal therapies Cancer Inst. 2004 Feb 18;96(4):2618.

82 M AYO C L I N I C I N T E R N A L M E D I C I N E B OA R D R E VI EW: Q U E S T I O N S A N D A N S W E R S
9.
HEMATOLOGY QUESTIONS AND ANSWER S

QUESTIONS 2. A 45-year-old woman is admitted to the surgical service


with severe arterial insufficiency of the right second toe.
Multiple Choice (choose the best answer) She has no prior medical history and takes no medica-
tions. Physical examination findings are normal except
A N E M I A S A N D MY E L O I D M A L I G NA N C I E S
for mild splenomegaly and signs of early gangrene in the
right second toe. All pulses are full and equal through-
1. A 67-year-old man is evaluated for exertional dyspnea. out. Diagnostic testing results are shown in Table 9.Q2.
He recalls that 3 years ago he was told that he had ane-
mia. In reviewing his records, you note that at that time Table 9.Q2
his hemoglobin level was 9.5 g/dL and his hematocrit
was 33% with an increased mean corpuscular volume COMPONENT FINDING
(MCV); the remainder of his complete blood cell count Hemoglobin, g/dL 13.2
was normal. On physical examination, he had conjunc-
tival pallor, normal heart and lung findings, no lymph- Hematocrit, % 39
adenopathy, no hepatomegaly or splenomegaly, and no Leukocyte count, 109/L 15.5
petechiae or ecchymoses. Diagnostic testing results are Segmented neutrophils, % 78
shown in Table 9.Q1. Band cells, % 4
Lymphocytes, % 20
Table 9.Q1 Monocytes, % 5
Basophils, % 2
COMPONENT FINDING Eosinophils, % 1

Hemoglobin, g/dL 7.5 Platelet count, 109/L 1,300

Hematocrit, % 23 Mean corpuscular volume, f L 88

Mean corpuscular volume, f L 110 (reference range, 8698) Erythrocyte sedimentation 28


rate, mm/h
Leukocyte count, 109/L 2.1
Neutrophils, % 20 Leukocyte alkaline 110 (reference range, 13130)
Lymphocytes, % 70 phosphatase score
Monocytes, % 6 Serum ferritin Within reference range
Basophils, % 3
Eosinophils, % 1 Serum iron Within reference range
9
Platelet count, 10 /L 64 Serum total iron-binding Within reference range
capacity
Reticulocyte count, % of 0.3 (reference range, 0.51.5)
erythrocytes Peripheral blood film Increased large platelets with
some clustering; leukocytes and
Absolute reticulocyte 10.0 (reference range, 29.587.3) erythrocytes are unremarkable
count, 109/L
Bone marrow aspiration and Increased cellularity with increased
Peripheral blood film Dimorphic erythrocyte population biopsy and atypical megakaryocytes in
with pronounced macrocytes clusters; reticulin staining is normal
Lactate dehydrogenase, U/L 150 (reference range, 140280) Chromosomal analysis Normal female karyotype (46XY)

Which of the following is the most likely explanation


for these findings? Which of the following is the most likely diagnosis?
a. Acute myeloid leukemia (AML) a. Essential thrombocythemia
b. Vitamin B12 deficiency b. Vasculitis
c. Hemolytic anemia c. Philadelphia chromosomenegative chronic myeloid leuke-
d. Myelodysplastic syndrome (MDS) mia (CML)
e. Primary myelofibrosis d. Primary myelofibrosis (PMF)

83
3. A 22-year-old man is admitted to the hospital for an Table 9.Q4
elective cholecystectomy. You are asked to see him
COMPONENT FINDING
because he had anemia on preoperative testing. He tells
you that he has always been told by his physicians that Hemoglobin, g/dL 7.9
he has mild anemia; his medical history is otherwise
unremarkable. His vital signs are normal. His conjunc- Hematocrit, % 25
tivae are mildly icteric, and the spleen is palpable in the Mean corpuscular volume, f L 80
left upper quadrant. Findings on the remainder of the
physical examination are normal. Diagnostic testing Leukocyte count, 109/L 16.0
Segmented neutrophils, % 70
results are shown in Table 9.Q3. Band cells, % 7
Lymphocytes, % 15
Monocytes, % 5
Table 9.Q3 Basophils, % 2
Eosinophils, % 1
COMPONENT FINDING
Platelet count, 109/L 490
Hemoglobin, g/dL 11.2
Creatinine, mg/dL 1.4
Hematocrit, % 34
Peripheral blood film Anisopoikilocytosis with multiple
Leukocyte count, 109/L 9.0 sickle cells
Differential count Within reference ranges

Platelet count, 109/L 295 A chest radiograph shows a right middle and upper
Mean corpuscular volume, f L 89 lobe air space infiltrate. The patient is given supplemen-
tal oxygen, adequate pain control, and intravenous anti-
Reticulocyte count, % 4 biotics. Which of the following should you now order?
Absolute reticulocyte count, 109/L 200 (reference range, a. Hydroxyurea
29.587.3) b. Erythrocyte exchange transfusion
c. Plasma exchange
Peripheral blood film Polychromasia with numerous
microspherocytes d. Anticoagulation with unfractionated heparin
e. Aggressive intravenous fluid hydration
5. A 70-year-old man presents with weakness of his right
Which of the following tests would most likely help arm and leg. His symptoms began yesterday and are
confirm the diagnosis? now resolved. He also reports a 6-month history of
a. Hemoglobin electrophoresis recurrent headaches and fatigue. He is a nonsmoker.
b. Osmotic fragility test His medical history is significant for high blood pres-
c. Direct and indirect antiglobulin (Coombs) tests sure. His blood pressure is 167/88 mm Hg, his oxygen
d. Bone marrow aspiration and biopsy saturation is 93% on room air, his face is plethoric, and
a right carotid bruit is heard. Other findings on physi-
4. A 28-year-old black man with sickle cell disease pres- cal examination are normal. Diagnostic testing results
ents to the emergency department with abdominal are shown in Table 9.Q5.
pain, chest pain, and shortness of breath. His dyspnea
evolved over 36 hours after a visit with his niece and
Table 9.Q5
nephew. His history is significant for approximately
2 emergency department visits or hospital admissions COMPONENT FINDING
per year for painful crises. Three years ago, he spent 4
weeks in the hospital after an episode of acute chest Hemoglobin, g/dL 20.5
syndrome. He has been taking hydroxyurea but only Hematocrit, % 58
intermittently because of financial concerns. His pulse
is 116 beats per minute and regular, his blood pressure Mean corpuscular volume, f L 88
is 138/76 mm Hg, his respiratory rate is 18 breaths per Leukocyte count, 109/L 12.5
minute, and his temperature is 38.3C. Pulse oximetry Neutrophils, % 83
shows 91% oxygen saturation with room air and 93% Lymphocytes, % 12
with 4 L of oxygen by nasal cannula. His lungs have Monocytes, % 3
Basophils, % 2
scattered inspiratory crackles in the right midlung field.
His spleen is not palpable. The remainder of the physi- Platelet count, 109/L 600
cal examination findings are normal. Diagnostic testing
Erythropoietin, mIU/mL <2 (reference range, 019)
results are shown in Table 9.Q4.

84 M AYO C L I N I C I N T E R N A L M E D I C I N E B OA R D R E VI EW: Q U E S T I O N S A N D A N S W E R S
Carotid ultrasonography shows a 30% stenotic lesion in Which of the following is the best interpretation of
the right carotid. The patient is hospitalized and begins these data?
antiplatelet therapy. Which of the following should you a. The hemolysis is predominantly intravascular.
order next? b. The bone marrow is not responding to the anemia.
a. JAK2 V617F mutation testing c. Direct Coombs testing results should be positive.
b. Fluorescence in situ hybridization (FISH) for BCR-ABL d. Urine hemoglobin testing results should be positive.
testing
7. A 58-year-old woman with active rheumatoid arthritis
c. Arterial blood gas analysis
presents with fatigue and joint pain. She received the
d. Bone marrow aspiration and biopsy
diagnosis of rheumatoid arthritis 5 years earlier and has
6. A 42-year-old woman with a history of systemic lupus been taking prednisone 10 mg daily and methotrexate
erythematosus (SLE) presents with fatigue. She has been with folate weekly. She has had chronic fatigue and ane-
receiving antitumor necrosis factor therapy and has mia. Her vital signs are normal. Her conjunctivae are
been managing the SLE well. However, she has recently pale, and she has active synovitis affecting both knees,
experienced worsening fatigue. Her vital signs are nor- her wrists, and elbows, with rheumatoid nodules on
mal. Her face and conjunctivae are jaundiced, and she the extensor surface of her right forearm. The remain-
has a fading butterfly rash on her face. The spleen is pal- der of the physical examination findings are normal.
pable on deep inspiration. Diagnostic testing results are Diagnostic testing results are shown in Table 9.Q7.
shown in Table 9.Q6, and the peripheral blood film is
shown in Figure 9.Q6. Table 9.Q7
Table 9.Q6 COMPONENT FINDING

COMPONENT FINDING Hemoglobin, g/dL 9.0


Hemoglobin, g/dL 7.8 Hematocrit, % 30
Hematocrit, % 27 Mean corpuscular volume, f L 80
Mean corpuscular volume, f L 95 Leukocyte count, 109/L 11.5
Neutrophils, % 90
Leukocyte count, 109/L 4.5 Lymphocytes, % 8
Differential count Within reference ranges Monocytes, % 2
Platelet count, 109/L 450 Platelet count, 109/L 500
Reticulocyte count, % 4 Erythrocyte sedimentation rate, mm/h 50
Absolute reticulocyte count, 109/L 170.0 (reference range, Erythropoietin, mIU/mL 15 (reference range, 019)
29.587.3)

Erythrocyte sedimentation rate, mm/h 25


Which of the following laboratory findings are consis-
Lactate dehydrogenase, U/L 400 (reference range, 140280) tent with this condition?
Total bilirubin, mg/dL 3.5 (reference range, 0.11.0)
a. Elevated hepcidin, elevated ferritin, elevated total
iron-binding capacity (TIBC), elevated serum iron
Indirect bilirubin, mg/dL 3.0 b. Elevated hepcidin, elevated ferritin, decreased TIBC, ele-
vated serum iron
c. Decreased hepcidin, elevated ferritin, decreased TIBC, ele-
vated serum iron
d. Elevated hepcidin, elevated ferritin, decreased TIBC, nor-
mal serum iron
e. Decreased hepcidin, elevated ferritin, elevated TIBC, nor-
mal serum iron

C OAGU L AT I O N
8. A 62-year-old man underwent right total knee replace-
ment 8 days ago. Swelling has developed in his right
lower extremity, and Doppler ultrasonography confirms
the presence of a right superficial femoral vein throm-
bosis. His current medications include oxycodone and
subcutaneous unfractionated heparin. Results of pre-
Figure 9.Q6 operative tests, including a complete blood cell count

9. H E M ATO L O GY Q U E S T I O N S A N D A N S W E R S 85
and liver and kidney function, were normal. Other lab- Food and Drug Administration (FDA) approval of dab-
oratory data include the following: hemoglobin 12.2 igatran, which requires no monitoring, and he would
g/dL, leukocyte count 8.5109/L, and platelet count like a prescription for this new drug. Which of the fol-
60109/L. In addition to stopping the use of subcuta- lowing statements is true about the use of dabigatran in
neous heparin, what is the next most appropriate step atrial fibrillation compared with the well-managed use
in management of this patient? of warfarin?
a. Start low-molecular-weight heparin therapy. a. Switching to dabigatran would result in superior outcomes.
b. Start intravenous therapeutic doses of heparin. b. Switching to dabigatran would result in inferior outcomes.
c. Start direct thrombin inhibitor therapy. c. Switching to dabigatran would provide no significant
d. Start aspirin therapy. benefit.
d. Dabigatran is FDA approved for postoperative thrombo-
9. A 45-year-old man presents with deep vein thrombosis
prophylaxis for knee and hip replacement surgery.
of the right femoral vein. Three months ago, he received
e. Dabigatran is FDA approved as an anticoagulant for patients
a diagnosis of systemic lupus erythematosus (SLE). In
who have received a mechanical heart valve.
addition to confirming SLE, laboratory testing also
documented the presence of a lupus anticoagulant 12. A 22-year-old woman is brought to the emergency
(LAC). There is no family history of venous thrombo- department after having 1 witnessed tonic-clonic sei-
sis. Current medications include hydroxychloroquine. zure. She had appeared confused for the preceding
Laboratory testing shows normal results for a complete few hours. On examination, she is febrile and appears
blood cell count and for tests of liver and kidney func- slightly confused; otherwise, neurologic and physical
tion. Special coagulation testing confirms the persis- examination findings are normal. Laboratory testing
tence of an LAC. What is the most reasonable duration results are shown in Table 9.Q12, and the peripheral
of warfarin anticoagulation for this patient? blood smear is shown in Figure 9.Q12.
a. 3 months
b. 1 year
Table 9.Q12
c. 6 months
d. Long-term COMPONENT FINDING REFERENCE RANGE
e. 6 weeks
Hemoglobin, g/dL 8 1215
10. A 20-year-old white woman has been admitted to the
hospital with pulmonary embolism. She has no chronic Platelet count, 109/L 50 150450
illnesses and is receiving no medications except for com- Leukocyte count, 109/L 8 3.510.0
bination estrogen-progesterone birth control pills that
she started using approximately 1 year earlier. Results Creatinine, mg/dL 2.5 0.81.3
were normal for a complete blood cell count, baseline
prothrombin time, activated partial thromboplastin
time (aPTT), and tests of kidney and liver function.
The patient is currently receiving therapeutic doses of
intravenous unfractionated heparin, and her aPTT is
therapeutic at 72 seconds. A panel of thrombophilia
tests has been performed. Which of the following state-
ments about her thrombophilia test results is correct?
a. DNA-based testing for factor V Leiden and prothrombin
G20210A mutations are reliable.
b. Low antithrombin confirms a hereditary deficiency state.
c. A positive result on lupus anticoagulant (LAC) testing con-
firms antiphospholipid antibody syndrome.
d. Low protein S confirms the presence of a hereditary defi-
ciency state.
11. A 62-year-old man with chronic atrial fibrillation has
been treated with warfarin. He has no other chronic ill- Figure 9.Q12
nesses and is receiving no other medications long-term
except for lipid-lowering agents. Results of his complete What is the most appropriate next step in
blood cell count and tests of renal and kidney function management?
are normal. He checks his prothrombin time monthly a. Red blood cell transfusion
and has kept the international normalized ratio (INR) b. Platelet transfusion
within the therapeutic range (23) for the duration of c. Gamma globulin administration
his therapy with warfarin. He has heard about recent US d. Plasma exchange

86 M AYO C L I N I C I N T E R N A L M E D I C I N E B OA R D R E VI EW: Q U E S T I O N S A N D A N S W E R S
13. A 72-year-old man with chronic atrial fibrillation has 16. An 80-year-old man is admitted to the hospital after
been receiving dabigatran 75 mg twice daily for the past falling on an icy sidewalk and fracturing his hip.
6 months. He has not had any thrombotic or hemor- He undergoes open reduction and internal fixation
rhagic complications. He has a history of colon pol- of the fracture. At surgery, there does not appear to
yps, for which he needs to undergo a colonoscopy with be any bone disease at the fracture site. The patient
possible polypectomy. Apart from an irregular pulse, was previously asymptomatic. Physical examination
his physical examination findings are normal. Results findings are otherwise unremarkable. Serum protein
were normal for a complete blood cell count and tests electrophoresis and immunofixation show an IgM
of renal and liver function. The calculated creatinine monoclonal protein (0.3 g/dL). The complete blood
clearance is 28 mL/min. For how long should dabiga- cell count and serum creatinine levels are normal.
tran use be discontinued before the colonoscopy? Skeletal survey shows no additional bone defects.
a. No need to discontinue Which of the following statements is true for this
b. 24 hours patient?
c. 48 hours a. He has multiple myeloma and requires treatment.
d. 3 days b. He has a lower risk of a clinically significant lymphocytic or
e. 7 days plasma cell malignancy than patients with an IgG monoclo-
nal protein.
c. He requires a radioisotope bone scan to evaluate his bone
LY M P H O I D M A L I G NA N C I E S
integrity.
14. At her annual physical examination, an asymptomatic d. He requires regular follow-up and serial measurements of
68-year-old woman has lymphocytosis (32109/L) his monoclonal protein level.
with a normal hemoglobin level and platelet count. e. He has a 10% annual risk of multiple myeloma.
On examination, she has 1-cm lymphadenopathy in the
17. A 75-year-old African American man was seen last week
cervical region and no palpable liver or spleen enlarge-
by his primary care physician for mild dyspnea. He has
ment. A peripheral blood smear shows identically
also noted intermittent peripheral edema. During the
appearing mature lymphocytes with smudge cells. Flow
evaluation, an electrocardiogram showed low-voltage
cytometry of the peripheral blood lymphocytes shows
QRS complexes in the limb leads. The troponin T level
a monoclonal B population with dim expression of
was elevated (0.07 ng/mL). This finding suggested the
light chain and CD20 that is positive for expression of
need for a coronary angiogram, which showed no sig-
CD5, CD19, and CD23. Which of the following is the
nificant coronary artery disease. An echocardiogram
best next step in her management?
showed diffuse left ventricular thickening with a granu-
a. Combination chemoimmunotherapy
lar texture to the myocardium and a septal thickness
b. Chlorambucil therapy
of 2.5 cm (normal <1.1 cm). The complete blood cell
c. Allogeneic peripheral blood stem cell transplant
count results were normal. Serum and urine protein
d. Combination monoclonal antibody therapy
electrophoresis and immunofixation were unremark-
e. Active monitoring for disease progression and complications
able. Serum free light chain levels were not increased.
15. Ten years ago, a previously healthy 20-year-old woman What is the most likely diagnosis?
presented to her physician with a 2-month history of a. AA amyloidosis
pruritis, drenching night sweats, unintentional weight b. Light chainrelated amyloidosis
loss, and nonproductive cough. On examination, she c. Hypertrophic obstructive cardiomyopathy
had 2-cm cervical lymphadenopathy. A computed d. Amyloidosis due to transthyretin deposition
tomographic scan showed a 12-cm-diameter anterior e. Amyloidosis due to 2-microglobulin deposition
mediastinal mass. An excisional biopsy of a cervical
18. A 55-year-old man presented to his primary care phy-
lymph node showed nodular sclerosing Hodgkin lym-
sician for evaluation of fatigue. He was previously
phoma. After she was treated with ABVD (doxorubicin
healthy with the exception of chronic musculoskeletal
[Adriamycin], bleomycin, vinblastine, and dacarbazine)
low back pain, for which he occasionally takes non-
combination chemotherapy followed by involved field
steroidal anti-inflammatory drugs. On examination,
radiotherapy, the disease was in complete remission.
he is pale. Complete blood cell count results are as
Now you see her for the first time for an annual physical
follows: hemoglobin 8.3 g/dL, mean corpuscular vol-
examination. The disease remains in complete remis-
ume 73 f L, leukocyte count 6.9109/L, and platelet
sion. Compared to her peers, this patient is at increased
count 398109/L. Results of the fecal occult blood
risk of which of the following conditions?
test are positive. During upper and lower endoscopy, a
a. Breast cancer
1.22.5-cm ulcerative lesion is noted in the lesser cur-
b. Coronary artery disease
vature of the stomach. The lesion is biopsied and identi-
c. Hypothyroidism
fied as a MALT lymphoma. Which of the following is
d. Skin cancer
characteristic of MALT lymphoma?
e. All of the above

9. H E M ATO L O GY Q U E S T I O N S A N D A N S W E R S 87
a. Most cases are treated with anthracycline-based 20. A 48-year-old man presents to the emergency depart-
chemotherapy. ment with a 6-week history of progressively worsening
b. It is caused by chronic stimulation with Chlamydophila abdominal pain and night sweats. Physical examination
psittaci. findings were significant for palpable bilateral 2-cm
c. Radiotherapy is necessary in most cases. axillary lymph nodes and diffuse abdominal tenderness
d. It frequently undergoes transformation to a large-cell with no rebound or guarding. Computed tomography
lymphoma. of the abdomen and pelvis showed retroperitoneal and
e. The combination of amoxicillin, omeprazole, and clarithro- mesenteric lymphadenopathy. Excisional biopsy of
mycin is the most appropriate first-line treatment. an axillary node was positive for diffuse, large B-cell
lymphoma. Positron emission tomography showed
19. A 73-year-old woman presented to the emergency
fluorodeoxyglucose-avidity in the axillary, mesenteric,
department with new-onset back pain, confusion, and
and retroperitoneal lymph nodes. Results of the bone
constipation over the past week. Her past medical his-
marrow examination were normal. Which of the fol-
tory is significant only for hypertension. On examina-
lowing is the best next step?
tion, she is slightly pale with slow cognition and point
a. Combination therapy with rituximab, cyclophosphamide,
tenderness over the lumbar spine. Plain films of the
doxorubicin, vincristine, and prednisone (R-CHOP)
lumbar spine show osteolytic lesions in L2, L3, and L5.
b. Observation
Laboratory values are as follows: hemoglobin 9.3 g/dL,
c. Combination therapy with cyclophosphamide, doxorubi-
leukocyte count 4.6109/L with a normal differential
cin, vincristine, and prednisone (CHOP)
count, platelet count 230109/L, creatinine 1.6 mg/dL,
d. Autologous stem cell transplant
total calcium 13.1 mg/dL, albumin 3.6 g/dL, and total
e. Involved field radiotherapy
protein 9.1 g/dL. What is the most likely diagnosis?
a. Metastatic breast cancer
b. Hydrochlorothiazide use
c. Multiple myeloma
d. Primary hyperparathyroidism
e. Milk alkali syndrome

88 M AYO C L I N I C I N T E R N A L M E D I C I N E B OA R D R E VI EW: Q U E S T I O N S A N D A N S W E R S
ANSWER S

1. Answer d. hemolytic anemia, which produces positive Coombs test


results, can cause spherocytes as well; however, the history of
MDS most commonly manifests as isolated macrocytic
lifelong anemia makes this diagnosis unlikely. A hemoglobin
anemia. MDS can evolve to include pancytopenia over
electrophoresis would help in diagnosing thalassemia or a
several years; the typical peripheral smear findings include
hemoglobinopathy; however, these conditions do not mani-
a dimorphic erythrocyte population (microcytes and oval
fest with microspherocytes on the peripheral blood film.
macrocytes) with an overall prominent macrocytosis and
There is no indication for a bone marrow biopsy since the
an MCV around 110 fL. The chronicity of MDSin par-
reticulocyte response is appropriate and no other cytopenias
ticular, anemia preceding the diagnosis of pancytopenia by
are apparent. (See Gallagher in the Suggested Reading list.)
several yearsis in contrast to the typically acute manifes-
tation of AML, which is therefore an unlikely possibility 4. Answer b.
in this patient. Primary myelofibrosis, a myeloproliferative
neoplasm, causes fibrosis in the bone marrow, resulting The patient has acute chest syndrome, a sickle cell anemia
in extramedullary hematopoiesis and significant spleno- complication that is an indication for urgent red cell (not
megaly, and typically does not cause a macrocytic anemia. plasma) exchange transfusion to decrease the hemoglobin
Vitamin B12 deficiency can cause a megaloblastic anemia S level to less than 30% to 35%. Gentle fluid resuscitation
and manifest with slowly evolving macrocytic anemia and is appropriate (along with oxygen support and antibiot-
eventually pancytopenia, but the peripheral smear would ics, since about one-third of acute chest syndrome events
not show a dimorphic erythrocyte population. (See Tefferi are initiated by or associated with bacterial pneumonia).
and Vardiman in the Suggested Reading list.) Aggressive fluid resuscitation, leading to overhydration,
might cause pulmonary edema and worsen the oxygen-
2. Answer a. ation. Pulmonary embolism is possible, but full anticoagu-
Extreme thrombocytosis may be reactive and occur with lation is not warranted until embolism is documented. Use
severe iron deficiency or inflammatory states (with elevated of hydroxyurea might have prevented this crisis, but it is of
erythrocyte sedimentation rates) or after splenectomy; no value for the acute condition. (See Vij and Machado in
patients are typically asymptomatic. Clonal thrombocy- the Suggested Reading list.)
tosis is related to a myeloproliferative neoplasm, which 5. Answer a.
usually causes splenomegaly. Typical bone marrow find-
ings include a hypercellular bone marrow with increased Polycythemia may be secondary, as with erythropoietin-
atypical megakaryocytes in clusters. Essential throm- mediated causes such as chronic hypoxemia, living at high
bocythemia may cause extreme thrombocytosis (plate- altitude, and high oxygen affinity hemoglobinopathies.
let count >1,000109/L); however, it can also occur less Polycythemia vera is a myeloproliferative neoplasm that
commonly with polycythemia rubra vera (typically with can manifest with arterial thrombosis secondary to hyper-
erythrocytosis), the cellular phase of PMF, or rarely CML. viscosity from the increased concentration of erythrocytes.
The normal karyotype makes CML much less likely since The low erythropoietin rules out erythropoietin-mediated
it typically manifests with the Philadelphia chromosome causes, leaving the presumptive diagnosis of polycythemia
t(9;22). Increased reticulin fibrosis would have been seen vera. With JAK2 V617F mutation testing of peripheral
on the bone marrow biopsy if the patient had PMF. (See blood, results are positive for approximately 90% of patients
Tefferi in the Suggested Reading list.) who have polycythemia vera. FISH for BCR-ABL testing
would screen for chronic myeloid leukemia, which does not
3. Answer b. manifest with polycythemia. Although bone marrow aspira-
When a patient presents with premature gallstones, one tion and biopsy would be helpful, it is not immediately nec-
should consider whether they may be due to pigment gall- essary and could be considered later. (See Patnaik and Tefferi
stones from chronic hemolysis causing indirect hyperbili- in the Suggested Reading list.)
rubinemia. The presence of microspherocytes is consistent
6. Answer c.
with hereditary spherocytosis, and the diagnostic test is
an osmotic fragility test, which identifies a congenital Hematologic complications of SLE include anemia of
membrane defect. Typically, acquired warm autoimmune chronic disease, pure red cell aplasia, and warm autoimmune

89
hemolytic anemia (WAIHA). The presentation and labo- systemic embolism in patients with nonvalvular atrial
ratory data suggest hemolysis, and the blood smear shows fibrillation.
spherocytes. These findings are consistent with WAIHA,
12. Answer d.
which causes extravascular hemolysis. The reticulocytosis
suggests that the bone marrow response is adequate. In Plasma exchange is the treatment of choice for thrombotic
intravascular hemolysis, the urine is positive for hemoglo- thrombocytopenic purpura (TTP). Although red blood
bin. (See Packman in the Suggested Reading list.) cell transfusion may be indicated, it does not address the
underlying pathogenesis of TTP. Platelets are thought to
7. Answer d.
be contraindicated in TTP because of the theoretical pos-
Rheumatoid arthritis is a chronic inflammatory disorder sibility of worsening the TTP. Gamma globulin is ineffec-
that may lead to anemia of chronic disease. Anemia of tive in increasing the platelet count in TTP.
chronic disease results from the effect of elevated cyto-
13. Answer d.
kines on hematopoiesis, including upregulation of hepci-
din, leading to increased ferritin from iron malutilization Dabigatran is cleared through the kidneys. It has a pro-
and downregulation of ferroportin, the main iron export- longed half-life in patients who have a creatinine clearance
ing system. Transferrin is also downregulated, leading to less than 30 mL/min compared with patients who have a
decreased TIBC and normal to decreased serum iron lev- creatinine clearance greater than 30 mL/min.
els. (See Weiss and Goodnough in the Suggested Reading
list.) 14. Answer e.

8. Answer c. Chronic lymphocytic leukemia (CLL) is a clonal lym-


phoproliferative disorder of mature lymphocytes. The clin-
The timing and degree of thrombocytopenia are consistent ical diagnosis requires a B-lymphocyte count of more than
with immune-mediated heparin-induced thrombocytope- 5109/L. Peripheral blood smears typically show smudge
nia type II. Unfractioned heparin and low-molecular-weight cells, which are lymphocytes that have broken during pro-
heparin are contraindicated. Aspirin would not be the sole cessing of the slide. The clinical course of CLL is chronic in
management agent for established thrombosis. The most most patients. For those with early-stage disease, standard
appropriate step is to start a direct thrombin inhibitor. practice is to withhold treatment until the disease is active
or progressive. However, patients need to be monitored
9. Answer d.
for disease progression, autoimmune complications, infec-
Presentation with a vascular thrombosis and persistence tions, and second cancers.
of a LAC for 12 weeks or more satisfies the criteria for an
15. Answer e.
antiphospholipid syndrome. This patient has a high risk
for recurrent venous thrombosis on discontinuing antico- Hodgkin lymphoma therapy is curative in about 80% of
agulation; thus, long-term warfarin is recommended with cases. However, there are late complications of therapy,
periodic reassessment for safety. particularly in those treated before modern chemotherapy
and radiotherapy. At 15 years, the risk of death from other
10. Answer a.
causes surpasses that of risk of death from Hodgkin lym-
DNA-based testing is reliable for patients receiving hepa- phoma. Patients are at higher risk of secondary malignan-
rin or warfarin and for patients who have acute thrombosis. cies, cardiovascular disease, thyroid disorders, and infertility
However, acute thrombosis and heparin can cause lower than the general population. Many of these conditions can
antithrombin activity results, which should be verified be attributed to chemotherapy and radiotherapy.
at another time, when heparin and acute thrombosis are
16. Answer d.
not factors. A single positive test result for LAC does not
confirm antiphospholipid syndrome; follow-up testing at This patient has monoclonal gammopathy of undetermined
12-week intervals is required to demonstrate persistence of significance (MGUS), the most common dysproteinemia.
LAC. Acute thrombosis and estrogen use can lower pro- In MGUS, the M protein level is typically less than 3 g/dL,
tein S levels; thus, abnormally low results require follow-up the bone marrow has less than 10% plasma cells, and the
confirmation. hemoglobin, creatinine, calcium, and bone radiographs are
normal. The risk of progression to a lymphocytic or plasma
11. Answer c.
cell malignancy is about 1% per year. Patients with an IgM
Among patients randomly assigned to receive dabigatran, or IgA monoclonal protein are at higher risk of progression
overall outcomes were noninferior when compared with than those with an IgG protein. Patients with MGUS need
the well-managed use of warfarin, thus providing no sig- to be observed.
nificant advantages. The group of patients that derived the
17. Answer d.
most benefit from dabigatran was the group with INRs
outside the recommended therapeutic range. Dabigatran The patient has senile cardiac amyloidosis. This syndrome
is FDA approved only to reduce the risk of stroke and is usually isolated to the heart with few clinically significant

90 M AYO C L I N I C I N T E R N A L M E D I C I N E B OA R D R E VI EW: Q U E S T I O N S A N D A N S W E R S
deposits elsewhere, and the echocardiographic findings care. Rituximab is an anti-CD20 monoclonal antibody
are often out of proportion to the degree of symptoms. that improves overall survival when added to CHOP che-
Transthyretin is the protein causing the amyloid deposits; motherapy for aggressive B-cell lymphomas. For patients
most patients have wild-type transthyretin. whose disease relapses or is refractory, autologous stem
cell transplant is the standard therapy. Radiotherapy can
18. Answer e.
be used in combination with chemotherapy in early-stage
With combination antibiotic therapy, 70% of gastric (I-IIA) nonbulky disease but is not standard therapy for
MALT lymphomas are cured. In cases refractory to anti- advanced disease.
biotics, tumors may carry the t(11;18) translocation, and
involved field radiotherapy is effective. Combination
chemotherapy is reserved for advanced disease. The SUGGESTED RE ADING
majority of cases are associated with Helicobacter pylori
infection. Gallagher PG. Red cell membrane disorders. Hematology Am Soc
Hematol Educ Program. 2005:138.
19. Answer c. Packman CH. Hemolytic anemia due to warm autoantibodies. Blood Rev.
2008 Jan;22(1):1731. Epub 2007 Sep 27.
This patient has multiple myeloma with evidence of Patnaik MM, Tefferi A. The complete evaluation of erythrocytosis: con-
end-organ damage from the plasma cell proliferative dis- genital and acquired. Leukemia. 2009 May;23(5):83444. Epub 2009
order (hypercalcemia, renal failure, anemia, and osteolytic Mar 19.
Tefferi A. Annual clinical updates in hematological malignancies: a con-
bone lesions). The other answer choices are possible causes tinuing medical education series: polycythemia vera and essential
of hypercalcemia, but only multiple myeloma accounts for thrombocythemia: 2011 update on diagnosis, risk-stratification, and
all the presenting symptoms, including the elevated level of management. Am J Hematol. 2011 Mar;86(3):292301.
total protein. Tefferi A, Vardiman JW. Myelodysplastic syndromes. N Engl J Med. 2009
Nov 5;361(19):187285.
20. Answer a. Vij R, Machado RF. Pulmonary complications of hemoglobinopathies.
Chest. 2010 Oct;138(4):97383.
This patient has advanced-stage, diffuse, large B-cell lym- Weiss G, Goodnough LT. Anemia of chronic disease. N Engl J Med. 2005
phoma, and R-CHOP chemotherapy is the standard of Mar 10;352(10):101123.

9. H E M ATO L O GY Q U E S T I O N S A N D A N S W E R S 91
This page intentionally left blank
10.
NEPHROLOGY QUESTIONS AND ANSWER S

QUESTIONS Which of the following contributed to his


hyponatremia?
Multiple Choice (choose the best answer) a. HCTZ
b. Decreased intravascular volume
c. Beer drinking
E L E C T RO LY T E D I S O R D E R S
d. Chronic kidney disease
1. A 68-year-old man, a formerly heavy smoker, presents e. Syndrome of inappropriate secretion of antidiuretic
with a blood-tinged cough and weakness. He uses an hormone
ipratropium bromide inhaler. On physical examination,
his vital signs are normal, his jugular venous pressure is 3. An 80-year-old woman who is a nursing home resident
normal, and he has no edema. His laboratory values are with hypertension, multi-infarct dementia, dysphasia,
as follows: sodium 125 mEq/L, potassium 3.6 mEq/L, and atrial fibrillation was admitted because her men-
bicarbonate 25 mEq/L, serum urea nitrogen 7 mg/dL, tal status changed. Physical examination findings and
and serum creatinine 0.9 mg/dL. What should you laboratory test results are shown in Table 10.Q3.
obtain next?
a. Serum osmolality value Table 10.Q3
b. Urine osmolality value
COMPONENT FINDING
c. Thyrotropin and morning cortisol values
d. Computed tomographic (CT) scan of the chest Blood pressure, mm Hg 106/58
e. Magnetic resonance imaging of the brain
Pulse, beats per minute 90 (irregular)
2. A 56-year-old man with a history of hypertension and
drinking alcoholic beverages (mostly beer) presents Respiratory rate, breaths per minute 24
with episodic confusion, weakness, and imbalance. For Oxygen saturation with room air, % 97
the past 3 weeks, he has been taking hydrocholorothi-
azide (HCTZ) 25 mg once daily. On physical examina- Weight, kg 60
tion, his blood pressure is 103/50 mm Hg, his pulse is Mucosal surfaces Dry
110 beats per minute, his respiratory rate is 20 breaths
per minute, his jugular venous pressure is normal, the Mental status Sleepy, oriented to name only
liver edge is palpable, and he has no edema. Laboratory Lung fields Clear, no edema
test results are shown in Table 10.Q2.
Sodium, mEq/L 176

Potassium, mEq/L 3.5


Table 10.Q2
Chloride, mEq/L 129
COMPONENT FINDING
Bicarbonate, mEq/L 35
Sodium, mEq/L 105
Serum creatinine, mg/dL 1.2
Potassium, mEq/L 3.2
Urine osmolality, mOsm/kg 655
Chloride, mEq/L 70

Bicarbonate, mEq/L 30
What is her estimated water deficit?
Serum creatinine, mg/dL 1.3 a. 6.5 L
b. 7.3 L
Serum osmolality, mOsm/kg 224
c. 9.3 L
Urine osmolality, mOsm/kg 410 d. 10.5 L
e. 2.4 L
Thyrotropin Within reference range
Morning cortisol Within reference range

93
4. For the patient in the preceding question, what is the only medication she uses is artificial tears. Physical
maximum rate of correction for her hypernatremia in examination findings and laboratory test results are
the next 24 hours? shown in Table 10.Q7. A computed tomographic scan
a. 10 mEq/L daily of the abdomen is shown in Figure 10.Q7.
b. 12 mEq/L daily
c. 14 mEq/L daily Table 10.Q7
d. 16 mEq/L daily
COMPONENT FINDING
e. 18 mEq/L daily
Body mass index 19
5. A 66-year-old man presents with a history of general-
ized weakness for 1 week. Otherwise, he has hyperten- Blood pressure, mm Hg 98/60
sion, diabetes mellitus, and obstructive sleep apnea (he
Pulse, beats per minute 98
uses nocturnal continuous positive airway pressure).
He has been taking furosemide 20 mg twice daily for Respiratory rate, breaths per minute 20
3 years and metolazone 2.5 mg twice daily for 2 weeks.
Edema Absent
Physical examination findings and laboratory test
results are shown in Table 10.Q5. Sodium, mEq/L 134

Potassium, mEq/L 2.3


Table 10.Q5
Chloride, mEq/L 114
COMPONENT FINDING
Bicarbonate, mEq/L 12
Body mass index 36.5
Serum creatinine, mg/dL 0.8
Blood pressure, mm Hg 108/78
Anion gap, mEq/L 8
Pulse, beats per minute 84
Arterial blood gas
Edema Absent pH 7.27
Pco2, mm Hg 27
Sodium, mEq/L 134
Urinalysis
Potassium, mEq/L 2.4 pH 6.7
Protein Trace
Bicarbonate, mEq/L 32 Glucose Negative
Red blood cells per high-power field 310
Chloride, mEq/L 92

Serum creatinine, mg/dL 1.5

Calcium, mg/dL 8.1


Glucose, mg/dL 600

Which of the following did not contribute to this


patients hypokalemia?
a. Metolazone
b. Furosemide
c. Transcellular shift
d. High aldosterone caused by a reduced intravascular volume
e. Chronic kidney disease
6. To the patient in the preceding question, what should
be given next?
a. Insulin intravenously
b. Insulin subcutaneously
c. Potassium intravenously
d. Calcium intravenously
e. Potassium orally

AC I D BA S E D I S O R D E R S
7. A 38-year-old woman who has had Sjgren syndrome
for 2 years presents with diffuse muscle weakness. The Figure 10.Q7

94 M AYO C L I N I C I N T E R N A L M E D I C I N E B OA R D R E VI EW: Q U E S T I O N S A N D A N S W E R S
What is the diagnosis? Table 10.Q9
a. Proximal renal tubular acidosis (RTA)
COMPONENT FINDING
b. Distal RTA
c. Idiopathic nephrolithiasis Mental status Incoherent
d. Hyporeninemic hypoaldosteronism
Blood pressure, mm Hg 106/70
e. Gout
Heart rate, beats per minute 110
8. A 68-year-old woman presents with new-onset back
pain, weakness, episodic light-headedness, and a recent Respiratory rate, breaths per minute 18
spontaneous left rib fracture. Physical examination
Edema Absent
findings and laboratory test results are shown in Table
10.Q8. Sodium, mEq/L 146

Potassium, mEq/L 5.0


Table 10.Q8
Chloride, mEq/L 100
COMPONENT FINDING
Bicarbonate, mEq/L 8
Blood pressure, mm Hg 98/50
Glucose, mg/dL 110
Pulse, beats per minute 106
Serum urea nitrogen, mg/dL 28
Respiratory rate, breaths per minute 20
Serum creatinine, mg/dL 1.2
Heart examination Unremarkable
Arterial blood gas
Lung examination Unremarkable pH 7.20
Pco2, mm Hg 20
Edema Extremities: pitting edema
(trace) Serum osmolality, mOsm/kg 378

Hemoglobin, g/dL 10.5 Toxicity screen Pending

Sodium, mEq/L 134


What is the most appropriate next step?
Potassium, mEq/L 3.4 a. Calculate the osmolal gap.
b. Examine a urine specimen microscopically.
Chloride, mEq/L 116
c. Initiate 4-methylpyrazole (fomepizole) therapy.
Bicarbonate, mEq/L 17 d. Initiate hemodialysis.
e. Do all of the above.
Serum creatinine, mg/dL 1.6
10. A 47-year-old woman with a history of substance abuse
Phosphorus, mg/dL 2.0
was found unresponsive. Physical examination findings
Uric acid, mg/dL 1.9 and laboratory test results are shown in Table 10.Q10.
Glucose, mg/dL 99
Table 10.Q10
Arterial blood gas
pH 7.32 COMPONENT FINDING
Pco2, mm Hg 32
Blood pressure, mm Hg 136/92
Urinalysis
pH 5.1 Heart rate, beats per minute 106
Protein 1+
Glucose 2+ Respiratory rate, breaths per minute 18
Red blood cells per high-power field 13
Edema Absent

Sodium, mEq/L 144


What is the diagnosis?
a. Proximal renal tubular acidosis (RTA) Potassium, mEq/L 5.1
b. Distal RTA
Chloride, mEq/L 110
c. Drug-induced diarrhea
d. Hypoaldosteronism Bicarbonate, mEq/L 6
e. Multiple myeloma
Glucose, mg/dL 89
9. A 39-year-old man with a history of polysubstance abuse Serum urea nitrogen, mg/dL 17
was found confused. Physical examination findings and
laboratory test results are shown in Table 10.Q9. (continued)

10. N E P H R O L O GY Q U E S T I O N S A N D A N S W E R S 95
Table 10.Q10 (CONTINUED) 12. A 70-year-old man who receives nocturnal oxygen
for severe chronic obstructive pulmonary disease was
COMPONENT FINDING
admitted for acute pneumonia. He has some nausea but
Serum creatinine, mg/dL 1.2
denied vomiting or having diarrhea. Physical examina-
tion findings and laboratory test results are shown in
Arterial blood gas Table 10.Q12.
pH 6.80
Pco2, mm Hg 32 Table 10.Q12
Serum osmolality, mOsm/kg 461 COMPONENT FINDING

Anion gap, mEq/L 28 Mental status Somnolent

Blood pressure, mm Hg 110/70


What is the most appropriate next step?
a. Calculate the osmolal gap. Pulse, beats per minute 90
b. Examine a urine specimen microscopically. Respiratory rate, breaths per minute 14
c. Initiate 4-methylpyrazole (fomepizole) therapy.
d. Initiate hemodialysis. Edema Dependent (1+)
e. Do all of the above. Bicarbonate, mEq/L 32
11. A 47-year-old man who has a history of polysubstance Arterial blood gas
abuse presents with delirium. Reportedly, he ingested pH 7.29
an unknown substance. Physical examination findings Pco2, mm Hg 70
and laboratory test results are shown in Table 10.Q11. Po2, mm Hg 40

Table 10.Q11
What is the patients acid-base status?
a. Acute respiratory acidosis
COMPONENT FINDING b. Chronic respiratory acidosis
c. Acute and chronic respiratory acidosis
Mental status Disoriented
d. Acute respiratory acidosis and metabolic alkalosis
Blood pressure, mm Hg 110/70 e. Chronic respiratory acidosis and metabolic acidosis
Pulse, beats per minute 110 (normal heart rhythm) 13. An 18-year-old woman with no past medical history
reports having nausea and occasional vomiting in the
Respiratory rate, breaths per minute 34
morning for about 2 months. She denies having diar-
Lung fields Clear rhea. Physical examination findings and laboratory test
results are shown in Table 10.Q13.
Edema Absent

Sodium, mEq/L 138 Table 10.Q13

Potassium, mEq/L 3.8 COMPONENT FINDING

Bicarbonate, mEq/L 14 Blood pressure, mm Hg 98/60

Chloride, mEq/L 100 Heart rate, beats per minute 92

Serum urea nitrogen, mg/dL 20 Respiratory rate, breaths per minute 16

Serum creatinine, mg/dL 1.5 Sodium, mEq/L 134

Arterial blood gas Potassium, mEq/L 4.0


pH 7.43
Pco2, mm Hg 20 Chloride, mEq/L 108
Po2, mm Hg 85
Bicarbonate, mEq/L 20

What is the patients acid-base status? Serum creatinine, mg/dL 0.4


a. Metabolic acidosis with respiratory compensation Glucose, mg/dL 88
b. Metabolic acidosis and respiratory acidosis
c. Metabolic acidosis and respiratory alkalosis Arterial blood gas
pH 7.45
d. Acute respiratory alkalosis with appropriate metabolic Pco2, mm Hg 30
compensation Po2, mm Hg 99
e. Chronic respiratory alkalosis and metabolic compensation

96 M AYO C L I N I C I N T E R N A L M E D I C I N E B OA R D R E VI EW: Q U E S T I O N S A N D A N S W E R S
What is the patients acid-base status? 15. A 54-year-old woman who had been previously healthy
a. Acute respiratory alkalosis is evaluated for a rash on her lower extremities that has
b. Chronic respiratory alkalosis been present for 1 week. She has noticed tea-colored
c. Acute and chronic respiratory alkalosis urine for several weeks. She takes no medications. Her
d. Metabolic acidosis and acute respiratory alkalosis temperature is 37.3C, her pulse is 88 beats per minute,
e. Metabolic acidosis and chronic respiratory alkalosis and her blood pressure is 154/90 mm Hg. Palpable pur-
pura is present on both legs and feet. The remainder of
the examination findings are unremarkable. Results of
ACU T E R E NA L FA I LU R E
laboratory studies are notable for creatinine 1.8 mg/
14. A 73-year-old man who has a history of hypertension dL. The erythrocyte sedimentation rate is 80 mm/h.
and metastatic prostate cancer presents with dizziness Antinuclear antibody, antibodies to double-stranded
and weakness. He has no known history of kidney dis- DNA, myeloperoxidase, and proteinase 3 assays are
ease; his creatinine was 1.2 mg/dL 2 months ago. His negative. The C4 complement level is low, and the
medications include lisinopril-hydrochlorothiazide (20 results of cryoglobulin testing are positive. Urinalysis
mg/12.5 mg daily), multivitamin daily, tramadol (50 mg shows proteinuria (2+) and hematuria (3+). Urine
23 times daily as needed for pain), and leuprolide (30 microscopy shows 31 to 40 erythrocytes per high-power
mg intramuscularly every 4 months). His blood pressure field (HPF) and 3 to 10 leukocytes per HPF. Which of
is 130/86 mm Hg, his pulse is 90 beats per minute, and the following viruses is most likely to be associated with
his temperature is 36.9C. On examination, he appears this disorder?
fatigued, heart sounds are normal, lungs are clear, and a. Epstein-Barr virus
there is pretibial edema (trace). Laboratory test results b. Cytomegalovirus
are shown in Table 10.Q14. Renal ultrasonography c. Human immunodeficiency virus
shows no evidence of hydronephrosis. d. Parvovirus B19
e. Hepatitis C virus
Table 10.Q14

COMPONENT FINDING

Hemoglobin, g/dL 11.1 G L O M E RU L A R D I S E A S E

Leukocyte count, 109/L 7.5 16. A 51-year-old truck driver is referred for evaluation of
persistent asymptomatic microhematuria. He has not
Platelet count, 109/L 187 seen a physician since he was first told about blood
Sodium, mEq/L 142 in his urine during a Department of Transportation
physical examination 9 years ago. At that time, he had
Potassium, mEq/L 5.2 a computed tomographic scan of the abdomen and pel-
Bicarbonate, mEq/L 22 vis, cystoscopy with retrograde pyelograms, and urine
cytology. He was told last year that his blood glucose
Chloride, mEq/L 105 level was elevated. He has never smoked and takes no
Serum creatinine, mg/dL 3.2 medications other than ibuprofen 400 mg approxi-
mately twice monthly for headaches. Physical examina-
Serum urea nitrogen, mg/dL 50 tion findings and laboratory test results are shown in
Calcium, mg/dL 13.1 Table 10.Q16. Results of erythrocyte sedimentation
rate, antinuclear antibody testing, testing for antibod-
Albumin, g/dL 3.9 ies to myeloperoxidase and proteinase 3, serum pro-
Phosphorus, mg/dL 3.3 tein electrophoresis, and hepatitis B and C and human
immunodeficiency serologies all are negative or normal.
Urinalysis Renal biopsy is performed.
Protein 1+
Microscopic examination Occasional granular casts
Sodium, mEq/L 68 Table 10.Q16
Creatinine, mg/dL 31
COMPONENT FINDING

What is the most likely cause of this patients acute renal Blood pressure, mm Hg 164/94
failure? Pulse, beats per minute 76
a. Membranous nephropathy
b. Acute tubular necrosis Weight, kg 92
c. Dehydration
Height, cm 185
d. Obstruction
e. Tumor lysis syndrome (continued)

10. N E P H R O L O GY Q U E S T I O N S A N D A N S W E R S 97
Table 10.Q16 (CONTINUED) Table 10.Q17 (CONTINUED)

COMPONENT FINDING COMPONENT FINDING

Heart, lungs, and abdomen Normal Chloride, mEq/L 108

Jugular venous distention Absent Serum creatinine, mg/dL 2.8

Pitting edema Both lower extremities (trace) Serum urea nitrogen, mg/dL 39

Rashes Absent Creatine kinase, U/L 12,870

Serum creatinine, mg/dL 2.1 Urinalysis


Color Brown
Fasting blood glucose, mg/dL 130 Blood 3+
Protein 2+
Spot urine microalbumin, mg/g 1,586 Leukocytes 13
Urinalysis Microscopic examination Granular casts, epithelial casts,
Blood 3+ renal epithelial cells (1120)
Protein 3+
24-h total protein, g 2.1
What is the most likely cause of her renal failure?
What is the most likely diagnosis? a. Rhabdomyolysis
a. Minimal change nephropathy b. Acute interstitial nephritis
b. Membranous nephropathy c. Thrombotic thrombocytopenic purpura (TTP)
c. Chronic interstitial nephritis d. Membranoproliferative glomerulonephritis (MPGN)
d. Diabetic nephropathy e. Antiglomerular basement membrane nephritis
e. IgA nephropathy
18. Oliguric acute renal failure developed in a 72-year-old
17. A 34-year-old woman presents to the emergency depart- man after left femoral-popliteal bypass surgery. He
ment with diffuse myalgias. Her past medical history is had a computed tomographic angiogram of the lower
significant for arthroscopic knee surgery 3 years ago. extremity arteries 5 days ago and underwent left
She has no history of recent trauma. She admits to using femoral-popliteal bypass surgery for critical ischemia
cocaine regularly. She takes an oral contraceptive tablet of the left foot 3 days ago. During the procedure, his
daily and ibuprofen 400 mg 2 to 3 times daily as needed systolic blood pressure decreased to 80 to 89 mm Hg
for pain, most recently this morning. On auscultation, several times. Over the past 24 hours, his urine output
the heart rhythm is regular with no murmur, rub, or through the indwelling bladder catheter was 200 mL
gallop, and the lungs are clear. Findings on abdominal despite intravenous fluid administration; he has a posi-
examination are normal. There is diffuse tenderness in tive fluid balance of 4 L postoperatively. His current
both upper and lower extremities without any ecchy- medications include insulin, metoprolol, aspirin, sim-
moses or rashes. Ankle edema (trace) is present bilat- vastatin, and fentanyl patient-controlled analgesia. In
erally. Additional physical examination findings and addition to peripheral vascular disease, his past medi-
laboratory test results are shown in Table 10.Q17. cal history is significant for type 2 diabetes mellitus for
8 years, hypertension, hyperlipidemia, and coronary
Table 10.Q17 artery disease. Baseline creatinine was 1.1 mg/dL last
month, and he had a urine microalbumin to creatinine
COMPONENT FINDING ratio of 28 mg/g (reference range <17 mg/g) at that
time. His blood pressure is 107/60 mm Hg, his pulse
Blood pressure, mm Hg 158/90
is 68 beats per minute, and his temperature is 37.1C.
Pulse, beats per minute 100 On physical examination, he has a regular cardiac rate
and rhythm and a grade 3/6 systolic ejection murmur
Temperature, C 37.5
at the right upper sternal border without a rub or gal-
Hemoglobin, g/dL 12.1 lop, there are diffuse pulmonary crackles bilaterally,
and the abdomen is soft and nontender. Both lower
Leukocyte count, 109/L 8.3
extremities have pitting edema (2+). Pedal pulses are
Platelet count, 109/L 189 diminished bilaterally. His creatinine was 1.2 mg/dL
on postoperative day 1 and 3.2 mg/dL on postoperative
Sodium, mEq/L 137
day 3. Urine Gram staining is negative, and the results
Potassium, mEq/L 5.8 of the urinalysis with microscopy are pending. Which
of the following sets of findings on urinalysis and urine
Bicarbonate, mEq/L 17
microscopy would be most characteristic of this acute
(continued) presentation?

98 M AYO C L I N I C I N T E R N A L M E D I C I N E B OA R D R E VI EW: Q U E S T I O N S A N D A N S W E R S
a. Normal urinalysis; occasional hyaline casts is 90 beats per minute, his respiratory rate is 16 breaths
b. Protein (4+); occasional free fat per minute, his temperature is 37.4C, and he weighs 87
c. Protein (2+); blood (3+); 41 to 50 red blood cells; occa- kg (body mass index 25.4). He appears fatigued and is
sional red blood cell casts in no acute distress. His skin is pale with scattered red-
d. Protein (trace); blood (trace); positive nitrite and positive dish lesions on the lower legs. The heart rate is regular
leukocyte esterase tests; 1 to 3 red blood cells; 51 to 100 leu- with a grade 2/6 systolic ejection murmur. Scattered
kocytes; occasional leukocyte casts pulmonary crackles are present. There is no hepatosple-
e. Protein (1+); less than 3 red blood cells; 4 to 10 renal epithe- nomegaly, and the abdomen is nontender with normal
lial cells bowel sounds. Bilateral pretibial edema (1+) is present.
Laboratory test results are shown in Table 10.Q20. Chest
19. A 70-year-old man is found to have a serum creatinine
radiography shows multiple small bilateral pulmonary
level of 3.2 mg/dL while undergoing an evaluation for
nodules that appear slightly smaller than 3 months ago.
bilateral lower extremity edema that had been present
for approximately 2 weeks. He has a history of hyperten-
sion, hyperlipidemia, mitral regurgitation, and Barrett
esophagus. Neither the patient nor any family members Table 10.Q20
are known to have kidney disease. Other than the edema,
COMPONENT FINDING
he has had no symptoms and denies having hematuria,
decreased urine output, increased urinary frequency, Hemoglobin, g/dL 6.8
hesitancy, or urgency. His medications are metoprolol,
Leukocyte count, 109/L 8.7
simvastatin, omeprazole, and aspirin. His blood pressure
is 136/80 mm Hg, his pulse is 62 beats per minute, his Platelet count, 109/L 37
temperature is 37.2C, his respiratory rate is 14 breaths
Sodium, mEq/L 141
per minute, and his weight is 78.9 kg. He does not appear
ill. His skin turgor is normal. His heart rate and rhythm Potassium, mEq/L 5.6
are regular. A 2/6 holosystolic murmur is present at
Chloride, mEq/L 108
the apex, and there is no rub or gallop. His lungs are
clear, and abdominal examination findings are normal. Bicarbonate, mEq/L 21
There is pitting pretibial edema (1+ to 2+) bilaterally.
Laboratory test results are as follows: serum creatinine Serum creatinine, mg/dL 4.7
3.2 mg/dL, serum urea nitrogen 54 mg/dL, hemoglo- Serum urea nitrogen, mg/dL 62
bin 12.8 g/dL, leukocyte count 9.8109/L, and platelet
count 204109/L. Urinalysis shows protein (1+), posi- Calcium, mg/dL 9.7
tive results for leukocyte esterase, 31 to 40 leukocytes Phosphorus, mg/dL 4.9
per high-power field, and negative Gram staining. Renal
ultrasonography shows increased echogenicity bilater- Albumin, g/dL 3.2
ally and no evidence of hydronephrosis. Venous Doppler Lactate dehydrogenase, U/L 540
ultrasonography of the lower extremities is negative for
deep vein thrombosis. Which of the following interven-
tions is most likely to improve this patients acute renal Which of the following is the most likely cause of this
failure by addressing the underlying cause? patients acute renal failure?
a. Initiate subcutaneous low-molecular-weight heparin therapy. a. Thrombotic microangiopathy
b. Discontinue the use of omeprazole. b. Acute tubular necrosis
c. Insert an indwelling urinary catheter. c. Atheroemboli
d. Decrease the metoprolol dose. d. Microscopic polyangiitis
e. Start ciprofloxacin therapy with 250 mg twice daily. e. Goodpasture syndrome

20. A 74-year-old man presents with a 1-week history of


C H RO N I C R E NA L FA I LU R E
weakness and fatigue. Two years ago, he underwent right
nephrectomy for renal cell carcinoma. Three months 21. A 56-year-old woman who is doing well is seeing you for
ago, pulmonary metastases were diagnosed, and therapy her annual checkup. Her medical history is significant
with sunitinib was initiated. Other past medical his- for type 2 diabetes mellitus for 20 years, hypertension,
tory includes hypertension and stage 3 chronic kidney and diabetic neuropathy. On physical examination, she
disease with baseline creatinine 1.6 mg/dL (estimated is obese and has decreased sensation in her feet. Other
glomerular filtration rate 42 mL/min per 1.73 m2). His physical examination findings and laboratory test
medications are amlodipine 10 mg daily, furosemide 20 results are shown in Table 10.Q21. Her current medi-
mg daily, and sunitinib 50 mg daily for the past 3 weeks cations are metformin 1,000 mg twice daily, simvasta-
(following 2 weeks without sunitinib after the previous tin 40 mg daily, gabapentin 300 mg 3 times daily, and
cycle). His blood pressure is 178/96 mm Hg, his pulse atenolol 50 mg twice daily.

10. N E P H R O L O GY Q U E S T I O N S A N D A N S W E R S 99
Table 10.Q21 What should you recommend?
a. Exchange atenolol with metoprolol.
COMPONENT FINDING
b. Increase the dosage of lisinopril to 80 mg daily.
Heart rate, beats per minute 65 c. Add losartan 100 mg daily.
d. Increase the dosage of allopurinol to 600 mg daily.
Blood pressure, mm Hg 125/65 e. Add metformin 1,000 mg twice daily.
Height, cm 165 23. A 58-year-old man has lower extremity swelling,
Weight, kg 91 decreased stamina, and fatigue. He is obese and has a
15-year history of type 2 diabetes mellitus. His other
Hemoglobin A1c, % 6.5 medical problems include hyperlipidemia and recur-
Serum creatinine, mg/dL 1.2 rent gout attacks. He weighs 147 kg, he is 176 cm tall,
his blood pressure is 156/78 mm Hg, and his heart
Total cholesterol, mg/dL 190 rate is 78 beats per minute. Laboratory test values are
Triglycerides, mg/dL 123 as follows: serum creatinine 2.3 mg/dL, hemoglobin
A1c 8.6%, and urine albumin to creatinine ratio 2,600
High-density lipoprotein cholesterol, mg/dL 34 mg/g. He takes losartan 25 mg daily, metformin 1,000
Low-density lipoprotein cholesterol, mg/dL 89 mg twice daily, allopurinol 100 mg daily, simvastatin
20 mg daily, and aspirin 81 mg daily. What therapeutic
What additional tests should you order for this patient? intervention should you recommend that is effective in
a. Renal ultrasonography treating patients with diabetic kidney disease and can
b. A 24-hour urine protein collection prevent its progression?
c. A 24-hour urine albumin collection a. Increase the losartan dosage to 50 mg daily and monitor
d. Random urine albumin to creatinine ratio serum potassium and creatinine.
e. Creatinine clearance b. Add hydrochlorothiazide 12.5 mg daily.
c. Stop the use of metformin and start insulin therapy.
22. A 72-year-old woman presents with fatigue, decreased d. Initiate dietary modification with protein and salt
stamina, and loss of appetite. She has a 25-year history of restriction.
type 2 diabetes mellitus; she has coronary artery disease e. All of the above would be effective.
and has had coronary artery bypass graft surgery; she
received a diagnosis of breast cancer 10 years ago and had 24. A 72-year-old man presents with acute chest pain. He
a mastectomy and radiotherapy; she has hypertension, has stage 3 chronic kidney disease (CKD) and a 20-year
hyperlipidemia, and a history of gout. Her heart rate is 48 history of type 2 diabetes mellitus. He is a smoker and
beats per minute, her blood pressure is 132/78 mm Hg, has hyperlipidemia and degenerative joint disease. His
her height is 174 cm, and her weight is 88 kg. On physical blood pressure is 165/85 mm Hg, and his heart rate
examination, she is pale, she is in no acute distress, and is 110 beats per minute. His troponin level is elevated.
she has lower extremity edema; otherwise, examination His electrocardiogram shows ST-segment elevation.
findings are normal. Laboratory test results are shown in The patient is being transported to undergo coronary
Table 10.Q22. Results of liver function studies are nor- angiography. What should you recommend be done
mal. The patient takes atenolol 50 mg daily, lisinopril 40 next?
mg daily, NPH insulin 30 units twice daily, simvastatin a. No treatment is indicated; proceed with coronary
20 mg daily, and allopurinol 150 mg daily. angiography.
b. Start therapy with N-acetylcysteine 1,200 mg twice daily
Table 10.Q22 with 1 L of oral hydration.
c. Start therapy with N-acetylcysteine 1,200 mg twice daily
COMPONENT FINDING with 1 L of 0.45% saline.
Hemoglobin, g/dL 9.8
d. Start therapy with N-acetylcysteine 1,200 mg twice daily
with 1 L of 0.9% saline.
Serum creatinine, mg/dL 2.2 e. Start therapy with N-acetylcysteine 1,200 mg twice daily
Sodium, mEq/L 140
with 1 L of sodium bicarbonate.

Bicarbonate, mEq/L 17
25. A 52-year-old woman presents with anasarca, decreased
stamina, and fatigue. She has had type 2 diabetes mel-
Chloride, mEq/L 113 litus for 8 years, hypertension, and hyperlipidemia. She
Potassium, mEq/L 5.4
quit smoking 2 years ago. She has a history of migraine
headaches. She has pale skin and pitting edema (4+) of
Uric acid, mg/dL 11 the thigh. On auscultation, breath sounds are mildly
Hemoglobin A1c, % 7.6
decreased in the bases bilaterally. Other physical exam-
ination findings and laboratory test results are shown
Urine albumin to creatinine ratio, mg/g 1,800

100 M AYO C L I N I C I N T E R N A L M E D I C I N E B OA R D R E VI EW: Q U E S T I O N S A N D A N S W E R S


in Table 10.Q25. She is taking NPH insulin, simvas- per minute. He is mildly obese; examination find-
tatin 40 mg daily, aspirin 81 mg daily, metoprolol 100 ings are otherwise unremarkable. Laboratory test
mg twice daily, and ibuprofen 200 mg twice daily as results include the following: serum creatinine 1.8
needed. A renal biopsy was performed. mg/dL, urine albumin to creatinine ratio 10 mg/g,
low-density lipoprotein cholesterol 89 mg/dL, and
hemoglobin A1c 6.5%. He takes metformin 1,000
Table 10.Q25
mg twice daily, allopurinol 300 mg daily, losartan
COMPONENT FINDING 100 mg daily, aspirin 81 mg daily, simvastatin 80 mg
daily, a multivitamin daily, vitamin E daily, fish oil
Height, cm 170 capsules 2,000 mg twice daily, and sildenafil 50 mg
Weight, kg 82 as needed. Which of the following is true about the
patients current condition?
Blood pressure, mm Hg 98/62 a. His blood pressure and diabetes are controlled well, so his
Heart rate, beats per minute 102 risk of death from cardiovascular disease is not increased.
b. His blood pressure and diabetes are controlled well, so he
Serum creatinine, mg/dL 1.2 will not progress to end-stage renal disease (ESRD).
Serum albumin, g/dL 2.0 c. He should begin renin-inhibitor therapy to further reduce
his risk of death from cardiovascular disease and his risk of
Total cholesterol, mg/dL 260 progression of chronic kidney disease (CKD).
Low-density lipoprotein cholesterol, mg/dL 180 d. He should initiate or intensify lifestyle modifications,
including diet, exercise, and weight loss.
High-density lipoprotein cholesterol, mg/dL 30 e. All of the above are true.
Triglycerides, mg/dL 198 27. A 68-year-old man presents with fatigue, bone pain,
Hemoglobin, g/dL 9.6 and arthralgias. His history is significant for chronic
kidney disease (CKD), left nephrectomy for renal cell
Hemoglobin A1c, % 9.2 carcinoma, hypertension, and degenerative joint dis-
ease. His height is 173 cm, he weighs 65 kg, his blood
pressure is 133/77 mm Hg, and his heart rate is 62 beats
What renal biopsy finding would be most likely?
per minute. He has diffuse arthralgias and mild pitting
a. Fusion of podocyte foot processes on electron microscopy
edema of the lower extremities. Laboratory test results
b. Mesangial matrix expansion and cell proliferation
include the following: serum creatinine 2.3 mg/dL,
c. Thickening of the glomerular basement membrane
serum calcium 8.2 mg/dL, phosphorus 5.8 mg/dL, para-
d. Mesangial nodular sclerosis
thyroid hormone (PTH) 298 pg/mL, and hemoglobin
e. All of the above
10.2 g/dL. His only medication is the extended-release
26. A 46-year-old man presents for an annual checkup formulation of diltiazem 360 mg. What is your next
with no medical concerns. He has had type 2 dia- recommendation?
betes mellitus for 9 years, and he has a history of a. Refer the patient for parathyroidectomy.
hypertension, degenerative joint disease with use of b. Schedule a parathyroid sestamibi scan.
nonsteroidal anti-inflammatory drugs, and gout. His c. Schedule an ultrasonographic study of the neck.
height is 181 cm, he weighs 101 kg, his blood pres- d. Start vitamin D therapy.
sure is 122/72 mm Hg, and his heart rate is 68 beats e. Determine the 1,25-dihydroxyvitamin D level.

10. N E P H R O L O GY Q U E S T I O N S A N D A N S W E R S 101
ANSWER S

1. Answer a. aldosterone secretion, which creates an abnormal coupling


of high distal sodium delivery and high aldosterone level,
This patient presented with a history suspicious for lung
leading to urinary potassium loss. Transcellular shift would
cancer, which is known to be associated with syndrome
lead to a shift of potassium from the intracellular compart-
of inappropriate secretion of antidiuretic hormone.
ment to extracellular compartments; this shift would not
Serum osmolality is necessary to confirm the presence
contribute to the patients hypokalemia.
of hypo-osmolar hyponatremia. Urine osmolality is
necessary to confirm the presence of arginine vasopres- 6. Answer c.
sin (AVP); concentrated urine (absence of maximally
diluted urine) would indicate the presence of AVP. The best choice is potassium given intravenously at an
Hypothyroidism and adrenal insufficiency could cause appropriate rate. Insulin given intravenously or subcu-
hyponatremia and should be ruled out. The presence of taneously promotes intracellular potassium shifting and
AVP in a patient with normal intravascular volume, low would be expected to further exacerbate hypokalemia.
serum osmolality, and lack of other known AVP stimu- The use of insulin should be avoided in patients who have
lating factors (pain, nausea, and medications) would hypokalemia. Administration of calcium intravenously
indicate ectopic AVP production. Given the patients would not solve the problem of hypokalemia.
history, high-resolution CT of the chest is indicated to 7. Answer b.
determine the presence of lung cancer, which could be
the source of AVP. This is a classic scenario for distal RTA, which is associated
with calcium phosphate nephrolithiasis and nephrocal-
2. Answer d. cinosis. Without treatment, the hypokalemia and non
HCTZ blocks absorption of sodium and chloride in the anion gap metabolic acidosis can become severe. Patients
distal convoluted tubule; the result is a mild degree of with RTA tend to have slightly low intravascular volume.
intravascular volume depletion, which stimulates secretion Therefore, hyporeninemic hypoaldosteronism would
of arginine vasopressin (AVP). Beer is hypotonic and con- not be a possibility. Proximal RTA would not be associ-
tains a large amount of free water. When passing through ated with a severe reduction in serum bicarbonate or with
the distal convoluted tubule, free water is absorbed nephrolithiasis.
under the influence of AVP; the result is hyponatremia.
8. Answer a.
Therefore, HCTZ, decreased intravascular volume (which
stimulates AVP), and beer drinking all contributed to the This patient likely has plasma cell dyscrasia (multiple
hyponatremia. myeloma or AL amyloidosis, or both). The unusually low
anion gap indicates the presence of positively charged
3. Answer c.
paraproteins. The presence of glucosuria with euglycemia
Water deficit is calculated as 0.6 body weight (kg) indicates a proximal tubular dysfunction. Typically, renin
[(sodium concentration/140) 1]. For this patient, the and aldosterone levels are high because the blood pressure
water deficit would be 36 [(176/140) 1] = 9.3 L. is low.
4. Answer b. 9. Answer a.
The correction of hypernatremia should be limited to no The history indicates some type of intoxication. The patient
more than 0.5 mEq/L per hour or 12 mEq/L daily. The has a large anion gap, and the most appropriate next step
daily rate is a limit, not a target. would be to calculate the osmolal gap to rule out alcohol
(methanol and ethylene glycol) intoxication. Urine micros-
5. Answer c.
copy would be helpful. The presence of calcium oxalate
The combination of a loop diuretic (furosemide) and a dis- crystals would indicate ethylene glycol intoxication. The
tal tubular diuretic (metolazone) is a powerful kaliuretic severe acidosis and large osmolal gap would require the
regimen. In patients who have reasonably good renal clear- initiation of hemodialysis. If ethylene glycol or methanol
ance, the combination inevitably results in hypokalemia. intoxication is suspected, 4-methylpyrazole therapy should
Volume depletion from the diuretic effect increases be initiated.

102
10. Answer a. would manifest with pyuria; patients with TTP have ane-
mia and thrombocytopenia; and MPGN and antiglo-
The anion gap acidosis and massively elevated osmolal gap
merular basement membrane nephritis would manifest
suggest small-molecule (ie, methanol) intoxication. The
with hematuria with or without red blood cell casts.
severe acidosis and large osmolal gap require combina-
tion treatment with 4-methylpyrazole (fomepizole) and 18. Answer e.
hemodialysis.
The clinical manifestation of oliguric acute renal failure
11. Answer c. after hypotension during surgery (along with exposure
to intravenous contrast material 2 days preoperatively) is
This patient has anion gap acidosis and respiratory alkalo-
typical of acute tubular necrosis. Urinalysis would show
sis, which is typical for salicylate intoxication.
low-grade proteinuria, and the renal epithelial cells seen on
12. Answer c. microscopy would indicate renal tubular injury.
A change in Pco2 of 30 mm Hg with a change in bicar- 19. Answer b.
bonate of 7 mEq/L would be consistent with a mixture of
chronic and acute respiratory acidosis. Sterile pyuria with acute renal failure suggests acute
interstitial nephritis. Discontinuing the use of omepra-
13. Answer b. zole would be the most appropriate treatment since pro-
ton pump inhibitors are associated with this condition.
This patients arterial blood gas and serum electrolyte val-
Inserting a urinary catheter in the absence of hydronephro-
ues are consistent with chronic respiratory alkalosis. In an
sis or a history of voiding dysfunction would not be help-
18-year-old woman, chronic respiratory alkalosis is consis-
ful. Low-molecular-weight heparin should be avoided in
tent with pregnancy. Progesterone stimulates the respira-
patients with this degree of renal functional impairment
tory center, causing chronic respiratory alkalosis.
and is not indicated in the absence of deep vein thrombo-
14. Answer b. sis. Decreasing the metoprolol dose in the absence of sig-
nificant bradycardia would not be beneficial. (See Myers et
Granular casts on urine microscopy and a fractional excre- al in the Suggested Reading list.)
tion of sodium that is greater than 3% (4.9% in this case)
are consistent with a diagnosis of acute tubular necrosis. 20. Answer a.
Proteinuria would be higher in membranous nephropa-
thy, which is associated with malignancies. Tumor lysis Renal thrombotic microangiopathy is an important side
syndrome is associated with hyperphosphatemia and, typi- effect of vascular endothelial growth factor inhibitors such
cally, hypocalcemia. Absence of hydronephrosis on ultra- as sunitinib and would manifest with hemolytic anemia,
sonography is not consistent with obstruction. thrombocytopenia, and renal failure. Microscopic poly-
angiitis can also manifest with anemia and acute renal
15. Answer e. failure, but severe thrombocytopenia is not a feature.
Cryoglobulinemia can develop in patients with asymp- Hemolytic anemia and severe thrombocytopenia are not
tomatic hepatitis C infection, and this can cause a vasculitis features of Goodpasture syndrome, atheroembolic renal
involving small vessels (skin) and membranoproliferative disease, or acute tubular necrosis. (See Eremina et al in the
glomerulonephritis. Suggested Reading list.)

16. Answer e. 21. Answer d.

A common manifestation of IgA nephropathy is persistent In the United States, only 20% to 30% of patients with
asymptomatic microhematuria with various degrees of pro- type 2 diabetes mellitus are evaluated for diabetic kidney
teinuria. Membranous nephropathy and minimal change disease (DKD) with testing for proteinuria. Patients with
nephropathy typically manifest with nephrotic range pro- DKD and proteinuria have a higher risk of end-stage renal
teinuria without significant hematuria. Patients who have disease and have a high associated cardiovascular mortal-
chronic interstitial nephritis and diabetic nephropathy do ity. Initiation of timely screening and appropriate therapy
not present with persistent microhematuria. can decrease the rate of progression of DKD. (See The
National Kidney Foundation Kidney Disease Outcomes
17. Answer a. Quality Initiative [NKF KDOQI] in the Suggested
Rhabdomyolysis precipitated by cocaine use can cause Reading list.)
acute renal failure from acute tubular necrosis that results 22. Answer a.
from injury to renal tubular epithelial cells after myoglobin
is released by myocytes. The urine sediment findings are Exchange atenolol with metoprolol because atenolol is
characteristic of acute tubular necrosis. Urinalysis typically cleared by the kidneys and may accumulate to adverse
is positive for blood by dipstick since the assay detects myo- plasma levels in patients with stage 3 or 4 chronic kidney
globin in addition to hemoglobin, but red blood cells are disease (CKD). The patient has a Modification of Diet
not seen on urine microscopy. Acute interstitial nephritis in Renal Disease (MDRD) glomerular filtration rate

10. N E P H R O L O GY Q U E S T I O N S A N D A N S W E R S 103
(GFR) of 22 mL/min per 1.73 m2. Metoprolol is metab- including intensification of lifestyle modifications for diet,
olized by the liver and therefore has fewer side effects. exercise, and weight loss. (See Keith et al in the Suggested
Patients with progressive CKD (stages 3 and 4) have Reading list.)
decreased elimination of medications that are excreted
27. Answer d.
by the kidneys. These medications can accumulate and
lead to adverse effects, such as bradycardia from renally Deficiency of 1,25-dihydroxyvitamin D is common among
excreted -blockers (eg, atenolol). Switching to medica- CKD patients, and initiation of vitamin D therapy to
tions metabolized in the liver is more advantageous in treat secondary hyperparathyroidism in CKD patients is
these patients. Furthermore, although inhibitors of the indicated along with achieving the following target PTH
renin-angiotensin system decrease progression of CKD levels:
and diabetic kidney disease, their benefit is limited in
patients with progressive CKD. The adverse effects of PTH 3570 pg/mL for patients with an estimated
these agents from decreasing the GFR and causing hyper- glomerular filtration rate (GFR) of 3059 mL/min per
kalemia might be harmful. (See Micromedix 2.0, and The 1.73 m2 (stage 3 CKD)
National Kidney Foundation Kidney Disease Outcomes
Quality Initiative [NKF KDOQI] in the Suggested PTH 70110 pg/mL for patients with an estimated
Reading list.) GFR of 1529 mL/min per 1.73 m2 (stage 4 CKD)

23. Answer e. PTH 150300 pg/mL for patients who receive dialysis
or who have an estimated GFR <15 mL/min per 1.73
The following therapies are effective for treating chronic m2 (stage 5 CKD)
kidney disease or diabetic kidney disease: titration of
angiotensin-converting enzyme inhibitor or angiotensin (See The National Kidney Foundation Kidney Disease
receptor blocker therapy, blood pressure control, glycemic Outcomes Quality Initiative [NKF KDOQI] in the
control, and weight loss. (See Saiki et al in the Suggested Suggested Reading list.)
Reading list.)
24. Answer d.
SUGGESTED RE ADING
Patients with CKD have an increased risk of
contrast-induced acute kidney injury (AKI) and high Calvin AD, Misra S, Pflueger A. Contrast-induced acute kidney injury
cardiovascular mortality. Recurrent episodes of AKI can and diabetic nephropathy. Nat Rev Nephrol. 2010 Nov;6(11):67988.
lead to progression of CKD. Appropriate therapies for Epub 2010 Sep 28.
Eremina V, Jefferson JA, Kowalewska J, Hochster H, Haas M, Weisstuch J,
contrast-induced AKI prevention should be initiated. (See et al. VEGF inhibition and renal thrombotic microangiopathy. N Engl
Calvin et al in the Suggested Reading list.) J Med. 2008 Mar 13;358(11):112936.
Keith DS, Nichols GA, Gullion CM, Brown JB, Smith DH. Longitudinal
25. Answer e. follow-up and outcomes among a population with chronic kidney dis-
Diabetic nephropathy is characterized by glomerulopa- ease in a large managed care organization. Arch Intern Med. 2004 Mar
22;164(6):65963.
thy involving all segments, including the mesangium, glo- Micromedex 2.0 [Internet]. Truven Health Analytics. c2012. Available
merular basement membrane, and podocytes. Some of the from: http://www.micromedex.com/.
phenotypes of diabetic nephropathy are characterized by Myers RP, McLaughlin K, Hollomby DJ. Acute interstitial nephritis due
rapid progression. Even so, patients with diabetes can have to omeprazole. Am J Gastroenterol. 2001 Dec;96(12):342831.
non-diabetic kidney disease (IgA, membranous nephropa- The National Kidney Foundation Kidney Disease Outcomes Quality
Initiative (NKF KDOQI) [Internet]. National Kidney Foundation.
thy, etc). Nearly 80% of renal biopsies in this patient popu- New York (NY). c2012. Available from: http://www.kidney.org/pro-
lation show diabetic nephropathy. (See Pflueger et al in the fessionals/kdoqi/index.cfm.
Suggested Reading list.) Pflueger A, Abramowitz D, Calvin AD. Role of oxidative stress in
contrast-induced acute kidney injury in diabetes mellitus. Med Sci
26. Answer d. Monit. 2009 Jun;15(6):RA12536.
Saiki A, Nagayama D, Ohhira M, Endoh K, Ohtsuka M, Koide N,
Patients with CKD have a high cardiovascular mortality et al. Effect of weight loss using formula diet on renal function in
and risk for progression to ESRD. Patient management obese patients with diabetic nephropathy. Int J Obes (Lond). 2005
should focus on cardiovascular risk reduction therapy, Sep;29(9):111520.

104 M AYO C L I N I C I N T E R N A L M E D I C I N E B OA R D R E VI EW: Q U E S T I O N S A N D A N S W E R S


11.
ALLERGY QUESTIONS AND ANSWER S

QUESTIONS Within minutes, the area becomes red, painful, pru-


ritic, and swollen. During the next several hours, the
Multiple Choice (choose the best answer) redness and swelling spread to the elbow. He denies
1. A 28-year-old woman presents with year-round symp- having other symptoms; specifically, he denies having
toms of nasal congestion, rhinorrhea, and sneezing. dyspnea, light-headedness, nausea, vomiting, or diar-
Although the symptoms are present year-round, they rhea. There are no skin manifestations elsewhere. What
worsen in the spring and fall. Her symptoms are bilat- is the most appropriate management?
a. Apply ice to the arm, provide symptomatic relief, and review
eral and have responded partially to treatment with
strategies to avoid stinging insects.
over-the-counter antihistamines. In an effort to control
b. Perform skin testing to bee only; if results are positive,
her symptoms, which medication should not be used as
administer immunotherapy.
part of her treatment plan?
c. Perform skin testing to apids (honeybee) and vespids (yel-
a. Nonsedating antihistamine
low jacket, wasp, and hornet); if any of the results are posi-
b. Leukotriene antagonist
tive, administer immunotherapy.
c. Topical decongestant
d. Perform an in vitro test for IgE specific for apids (honeybee)
d. Intranasal corticosteroid
and vespids (yellow jacket, wasp, and hornet); if results are
e. Saline nasal spray
positive, administer immunotherapy.
2. A 26-year-old woman is treated for Neisseria gonor- e. Administer 0.3 mL epinephrine 1:1,000 intramuscularly
rhoeae infection. She has had 2 episodes of Neisseria and observe.
meningitidis infection. As a child, she had 1 episode
of pneumonia. She has no history of skin infections or 5. A 31-year-old woman presents in her fifth month of
abscesses. What type of defect is the most likely under- pregnancy with increasing asthma symptoms. Her
lying immunologic abnormality? asthma had been well controlled throughout pregnancy
a. C2
but worsened over the past 4 days with the onset of an
b. C59
upper respiratory tract infection. She has daily symp-
c. IgG
toms and nighttime awakenings due to dyspnea. She
d. Neutrophil chemotaxis
uses her albuterol inhaler every 3 hours and has adhered
e. Lymphocyte count
to her usual inhaler regimen of budesonide 2 puffs daily.
She denies having fever or purulent mucus. On exami-
3. A 22-year-old man presents with a 10-year history of nation, she has scattered expiratory wheezes through-
recurrent angioedema, particularly of the lips and eyes. out all lung fields, a respiratory rate of 18 breaths per
Throat swelling occurred on 3 occasions. The episodes minute, oxygen saturation of 97%, and forced expira-
seem to occur randomly, approximately monthly, and tory volume in 1 second (FEV1) of 64% of the predicted
there is no association with food, medications (includ- value, which improves to 72% after albuterol. How
ing over-the-counter products), or environmental should you change her therapy?
exposures. His father reports having similar symptoms a. Add montelukast 10 mg daily.
for years. The patient is otherwise healthy. Current b. Increase the dosage of budesonide to 2 puffs twice daily.
physical examination findings are normal, although he c. Start the use of amoxicillin 500 mg 3 times daily.
brings in photographs that show marked angioedema d. Discontinue the use of the budesonide inhaler.
of his orbits and lips. What type of abnormality does e. Add prednisone 40 mg daily for 5 days.
this patient have?
a. C4
6. A 43-year-old woman reports that she had pruritic
b. C1q
hives and wheezing after her second dose of intravenous
c. Tryptase
penicillin approximately 10 years ago. She has avoided
d. Lymphocyte count
penicillin since then and has not had any further drug
e. 24-hour urine N-methylhistamine
reactions. Currently, she is being treated for endocardi-
tis, and high-dose penicillin is considered the drug of
4. A 32-year-old man is stung on the distal right forearm choice. What recommendation can be given for penicil-
by what he describes as a bee. No stinger is visible. lin use?

105
a. Penicillin cannot be given; use an alternate medication. recurrent infections, allergic rhinitis, or asthma before
b. Skin test to the major determinant of penicillin; if results are the past 3 years. Sputum cultures have shown growth of
negative, penicillin can be given. Streptococcus pneumoniae and Haemophilus influenzae.
c. Skin test to the major and minor determinants of penicillin; Currently, he has a chronic productive cough. With
if results are negative, penicillin can be given. which of the following laboratory tests is this patient
d. Patch test to penicillin; if results are negative, penicillin can most likely to have abnormal results?
be given. a. Total leukocyte count
e. Avoid penicillin and use a -lactam antibiotic. b. IgG level
c. Neutrophil chemotaxis assay
7. A 62-year-old man presents with a 3-year his-
d. Total complement level
tory of recurrent sinopulmonary infections docu-
e. IgE level
mented by computed tomographic scan of the sinuses
and chest radiographs. He does not recall having

106 M AYO C L I N I C I N T E R N A L M E D I C I N E B OA R D R E VI EW: Q U E S T I O N S A N D A N S W E R S


ANSWER S

1. Answer c. the absence of other symptoms, does not require the use
The patient has perennial and seasonal allergic rhinocon- of epinephrine.
junctivitis. Various medications can help control the symp- 5. Answer e.
toms and are often used in combination. These include
nonsedating antihistamines, intranasal corticosteroids, and This pregnant patient is having a significant flare of her
leukotriene antagonists. Saline nasal spray has also been asthma, and aggressive treatment is warranted by the
shown to improve symptoms. Topical decongestants, such decreased FEV1, increased use of albuterol, and wheezing.
as oxymetazoline, should be avoided. Use of this medica- In this situation, systemic corticosteroids are required. The
tion for more than 3 consecutive days can result in rhinitis main risk to the mother and child is hypoxia. Systemic cor-
medicamentosa. ticosteroids, inhaled corticosteroids, long- and short-acting
inhaled -agonists, and leukotriene receptor blockers are
2. Answer b. acceptable for use in pregnant patients who have asthma.
For ongoing management, adding montelukast or increas-
The patient has had recurrent Neisseria infections, which ing the budesonide dosage may be helpful, but these
are most commonly associated with defects in the ter- options are not the most effective in treating an acute
minal complement components (C59). Defects in the exacerbation.
other components of the immune system are associated
with other types of infections. Hypogammaglobulinemia 6. Answer c.
is associated primarily with upper and lower respiratory The clinical history is consistent with an IgE-mediated
infections caused by encapsulated bacteria. Neutrophil reaction to penicillin. Over time, the majority of patients
defects are associated with skin and pulmonary abscesses. lose their sensitivity to penicillin. Penicillin allergy can be
3. Answer a. assessed by skin testing to the major and minor determi-
nants of penicillin. These are allergenic metabolites of pen-
The patient has hereditary angioedema resulting from C1 icillin. If the test results are negative, the patient can receive
esterase inhibitor deficiency. The usual screening test for penicillin with the same risk of a significant reaction as for
this is C4 testing; C4 levels are decreased in these patients. persons in the general population who have never had a
The C1q level is decreased in the acquired form of this reaction. Skin testing to only the major determinant will
disease but not in the hereditary form. The acquired form miss approximately 25% of patients who are penicillin
occurs in older patients who do not have a family history of allergic. If the results of skin testing are positive, the patient
the disease, and in the acquired form there is often an asso- is at high risk for an IgE-mediated reaction and penicillin
ciated underlying malignancy, particularly a hematologic should be given only under a desensitization protocol.
malignancy. Tryptase and 24-hour N-methylhistamine are
used to evaluate for mast cell disease, which does not mani- 7. Answer b.
fest solely with intermittent angioedema. The lymphocyte Recurrent sinopulmonary infections with common respi-
count is normal in hereditary and acquired angioedema ratory microorganisms are the primary manifestations
and in the majority of patients with mast cell disease. of common variable immunodeficiency disease (CVID).
The main laboratory abnormality in CVID is a depressed
4. Answer a.
level of IgG. Hypogammaglobulinemia predisposes to
The patient has a large local reaction to a stinging insect. recurrent sinus and pulmonary infections. Other mani-
A large local reaction to a stinging insect is not considered festations include autoimmune processes and infectious
a risk factor for a more serious reaction with a subsequent diarrhea. The neutrophil chemotaxis assay is decreased in
sting. Therefore, only symptomatic treatment is required chronic granulomatous disease, which is characterized by
for treatment. Skin testing to the apids and vespids would recurrent skin and pulmonary abscesses. Deficiencies of
be required only if immunotherapy were being contem- the late-acting complement components (C59) typically
plated. This should be considered for all adults who have manifest as recurrent infections with meningococci and
systemic reactions to the sting. A large local reaction, in gonococci.

107
This page intentionally left blank
12.
PSYCHIATRY QUESTIONS AND ANSWER S

QUESTIONS her hands are now raw from all the scrubbing. With fur-
ther queries, she says that she feels compelled to count
Multiple Choice (choose the best answer) ceiling tiles in your examination room and she worries
1. A 72-year-old man has a history of poorly controlled irrationally about her childrens health. She has sleep
type 2 diabetes mellitus, coronary heart disease with QT difficulties, and her anxiety is so high throughout the
prolongation (corrected QT interval 482 ms), and major day that she has difficulty working. Which of the fol-
depression. He presents to your office with a 2-month lowing would not be an appropriate recommendation
history of worsening depressive symptoms, including for this patient?
a. Treat with bupropion titrated to 150 mg twice daily.
severely depressed mood, tearfulness, decreased con-
b. Treat with clonazepam 0.5 mg twice daily as a bridge until
centration, hopelessness, middle insomnia, and passive
other treatment is effective.
suicidal ideation without a specific intent or plan. He
c. Treat with fluoxetine 20 mg daily.
has been taking sertraline 150 mg daily (the dosage was
d. Taper and eliminate her intake of caffeine.
recently increased from 100 mg daily) without much
e. Recommend cognitive behavioral psychotherapy (CBT).
benefit. He has been experiencing life stresses including
financial strain and his wifes declining health. Which 4. A 42-year-old woman presents to your office again
of the following would not be a reasonable next step in after 16 years of intermittent severe left lower quadrant
managing his depression? abdominal pain. She denies having weight loss, fever, or
a. Cross-taper his medication from sertraline to amitriptyline. chills. The cause of her symptoms is not apparent from
b. Cross-taper his medication from sertraline to bupropion. previous workups, which included a complete blood
c. Perform a preanesthetic medical examination and refer the cell count, electrolyte evaluations, urinalysis, computed
patient to a psychiatrist for consideration of electroconvul- tomographic scan of the abdomen and pelvis, colonos-
sive therapy (ECT). copy, and gynecologic examination. She has previ-
d. Refer the patient to a psychologist for psychotherapy. ously been thoroughly evaluated for episodic dizziness,
e. Ask the patient about his access to firearms and enlist family headaches, flulike syndromes, back pain, and pain with
members to remove them from the home. intercourse. The results of all these workups were nega-
2. A 33-year-old woman has recently received a diagnosis tive. What is the most likely diagnosis?
a. Conversion disorder
of bipolar disorder and is now taking lithium carbonate
b. Somatization disorder
600 mg orally twice daily. She wants to know some of
c. Hypochondriasis
the long-term risks of taking this medication. Which of
d. Body dysmorphic disorder
the following is false about long-term management of
e. Factitial disorder
bipolar disorder with lithium?
a. Nonsteroidal anti-inflammatory drugs (NSAIDs) should 5. A 56-year-old man with a history of schizophrenia has
be avoided because of nephrotoxicity associated with taking a history of mild congestive heart failure and osteoar-
the combination of an NSAID and lithium. thritis. He harbors the paranoid belief that his wife is
b. There is an increased risk of birth defects in children of poisoning him, and he gets special messages from the
women taking lithium during pregnancy. television. These symptoms have been partially con-
c. Thyroid function needs to be monitored because of the trolled with a second-generation antipsychotic, olan-
potential for thyrotoxicity. zapine 5 mg orally daily. You recommend increasing the
d. Acetaminophen should be avoided because of hepatotoxic- olanzapine dosage to 10 mg daily. Which of the follow-
ity associated with taking the combination of acetamino- ing is false about the use of olanzapine in this patient?
phen and lithium. a. He is at increased risk of neuroleptic malignant syndrome.
e. Lithium doses may need to be decreased temporarily if an b. He is at increased risk of parkinsonism.
acute illness leads to dehydration. c. He is at increased risk of tardive dyskinesia.
d. He is at increased risk of diabetes mellitus.
3. A 32-year-old woman presents with anxiety about
e. He is at increased risk of anorexia.
germs. She washes her hands 30 to 40 times a day, and

109
6. A 19-year-old woman is brought to the emergency 7. You contact the family of the patient in the previous
department by police after she was discovered alone on question and learn that she has a strong family history
a rural bike trail dancing nude. I am the lizard queen! of bipolar disorder. The family asks you questions about
she joyfully proclaims. You learn that she has been acting management of this patient. Which of the following
quite erratically and sleeping poorly for the past week. would not be appropriate?
Which of the following is the least likely diagnosis? a. Hospitalize on a psychiatric unit.
a. Major depression with psychotic features b. Begin use of amitriptyline 50 mg orally.
b. Bipolar disorder c. Begin use of ziprasidone 20 mg orally.
c. Substance dependence d. Perform a urine drug screen.
d. Schizophrenia e. Begin use of valproate sodium 1,000 mg orally.
e. Methamphetamine intoxication

110 M AYO C L I N I C I N T E R N A L M E D I C I N E B OA R D R E VI EW: Q U E S T I O N S A N D A N S W E R S


ANSWER S

1. Answer a. Somatization disorder entails a history of multiple somatic


Amitriptyline would be a poor antidepressant choice complaints over many years, including 4 pain symptoms,
because the patient has QT prolongation (tricyclic anti- 2 gastrointestinal tract symptoms, 1 sexual symptom, and
depressants can prolong the QT interval) and diabetes 1 neurologic symptom. Conversion disorder is marked by
(tricyclic antidepressants can cause increased appetite and a neurologic symptom, such as a motor or sensory deficit,
weight gain). Bupropion would be a reasonable medication and is unconscious in origin (unlike factitial disorder, in
alternative. ECT could also be considered because of the which the symptoms are consciously produced). Body
severity of his symptoms and the failure of a medication dysmorphic disorder is characterized by a perception that
trial. Psychotherapy may help treat the depression and help a normally appearing body part is misshapen or otherwise
the patient deal with life stresses. Most completed suicides has an abnormal appearance. Hypochondriasis is charac-
are from firearms. terized by an irrational fear that one has an illness or serious
disease.
2. Answer d.
5. Answer e.
Lithium is not hepatotoxic and may be used safely with
acetaminophen. Lithium can cause nephrotoxicity, an Second-generation antipsychotics are associated with
effect that can be increased when taken concomitantly with hyperglycemia and an increased risk of diabetes mellitus.
NSAIDs. Lithium is a pregnancy category D medication Similar to traditional neuroleptics, second-generation
(associated with cardiac and facial structural abnormali- antipsychotics may cause extrapyramidal symptoms, tar-
ties) and should be avoided if possible during pregnancy, dive dyskinesia, and neuroleptic malignant syndrome.
especially in the first trimester. Lithium can cause thyro- Olanzapine is not usually associated with anorexia.
toxicity. Because it has a positive ion with a valence similar 6. Answer a.
to that of sodium, lithium will be retained by the kidneys
during periods of dehydration, rapidly reaching toxic lev- Although any of the disorders mentioned can cause psycho-
els in some patients. Renal function, thyroid function, and sis and odd behavior, major depression is unlikely. Patients
electrolytes should be checked periodically (at least annu- who have major depression with psychotic features would
ally) for all patients taking lithium. be unlikely to appear animated and joyful; more often,
they have ego-dystonic (very unpleasant) delusions, such
3. Answer a. as a belief that their family hates them or that people are
This patient has obsessive-compulsive disorder, which is conspiring against them.
responsive to selective serotonin reuptake inhibitors and
7. Answer b.
CBT. Because she is very anxious, taking a scheduled low
dose of a benzodiazepine and eliminating consumption With this patients age, behavior, and family history,
of caffeine may improve sleep and provide some relief bipolar disorder is high in the differential diagnosis.
while waiting for other treatments to become effective. Antidepressants, especially tricyclics, would be contrain-
Bupropion is not active at the serotonin receptor and will dicated because they can induce or prolong mania. With
not provide any benefit. the severity of her symptoms and her poor judgment,
she poses a danger to herself, so hospitalization is neces-
4. Answer b.
sary. Ziprasidone or valproate sodium is a reasonable ini-
Somatization disorder is characterized by physical symp- tial option to manage her symptom of mood elevation.
toms without an identifiable organic cause. Patients believe Substance-induced psychosis or mania is high in the dif-
that they have a physical problem and are not consciously ferential diagnosis; thus, a drug screen would be important
generating the symptoms or pretending to have them. in the evaluation of this patient.

111
This page intentionally left blank
13.
NEUROLOGY QUESTIONS AND ANSWER S

QUESTIONS except for mild symmetrical facial weakness. On motor


testing, distal muscle weakness is greater than proximal
Multiple Choice (choose the best answer) muscle weakness. Vibratory sensation is decreased dis-
1. A 52-year-old woman is evaluated for dizziness. She tally. Reflexes are absent. The Babinski test is negative.
has a history of well-controlled diabetes mellitus and Coordination is difficult to test because of weakness.
treated hypertension. On neurologic examination, she Which of the following would be expected on further
has a moderate loss of all sensory modalities distally evaluation?
a. Magnetic resonance imaging (MRI) of the head showing
and to the knees and wrists symmetrically. Her blood
multiple areas of increased T2 signal in the subcortical white
pressure is 146/84 mm Hg in the supine position (with
matter
a heart rate of 85 beats per minute). Upon standing, her
b. Cerebrospinal fluid (CSF) examination showing an
blood pressure is 100/70 mm Hg (with a heart rate of
increased protein level and a normal cell count
88 beats per minute), and she complains of a dizzy sen-
c. Ophthalmoscopic examination showing optic disc pallor
sation. Which of the following interventions should be
bilaterally
instituted first?
d. Elevated blood glucose concentration
a. Administer fludrocortisone.
e. Electroencephalography showing left temporal sharp waves
b. Perform magnetic resonance angiography.
c. Discontinue use of antihypertensive drugs. 4. An 83-year-old man has an abrupt, painless onset of
d. Perform a vestibular evaluation. right upper limb weakness and garbled speech. On
e. Perform magnetic resonance imaging of the cervical spine. examination in the emergency department 2.5 hours
2. A 37-year-old man has a 1-month history of fluctuating after the onset of symptoms, he has moderate weakness
difficulties with his speech and swallowing. He notes of the right deltoid, triceps, and intrinsic hand muscles
that with long conversations he has increasing difficulty and a mild right foot drop. His speech is halting, and
speaking; this is apparent during the history when his he appears frustrated when trying to speak, but he fol-
speech becomes nearly unintelligible. He describes sev- lows commands without difficulty. His blood pressure
eral choking episodes and nasal regurgitation of liquids is 163/87 mm Hg, with a heart rate of 85 beats per min-
when swallowing. On examination, he has normal eye ute. His electrocardiogram indicates atrial fibrillation.
movements at baseline, but after 2 minutes of sustained What is the most appropriate next step in the evalua-
upgaze, asymmetrical ptosis (worse on the left) and tion and management of this patient?
right hypertropia occur. Strength is initially normal, but a. Infusion of intravenous (IV) tissue plasminogen activator
with sequential strong contractions of the deltoid and (tPA)
iliopsoas muscles, mild weakness develops. Sensation, b. Initiation of warfarin therapy
muscle stretch reflexes, coordination, and alternating c. Computed tomography (CT) of the head
motion rates are normal. Which of the following would d. Infusion of IV labetalol
be the most appropriate next step in his evaluation? e. Electroencephalography (EEG)
a. Autonomic testing, including tilt-table testing
5. A 53-year-old woman has a 6-week history of increasing
b. Urgent magnetic resonance imaging of the cervical spine
clumsiness with both hands and imbalance that has led
c. Cerebrospinal fluid examination
to several falls. Her family has also noted that her speech
d. Muscle biopsy
sounds drunk. On neurologic examination, she has a
e. Serum acetylcholine receptor antibody testing
wide-based, cautious, unstable gait. Alternating motion
3. A 42-year-old woman presents with a 3-day history of rates of the limbs are of normal frequency and ampli-
difficulty walking. She has also noted a tingling sen- tude but are irregular and imprecise. Finger-to-nose
sation in her fingers and toes. On neurologic exami- testing is inaccurate, and the patient becomes tearful
nation, she has a normal mental status and normal during the examination. Her strength, sensation, and
neurovascular examination findings. She cannot rise muscle stretch reflexes are normal, and magnetic reso-
from the couch without assistance and cannot ambu- nance imaging (MRI) of her brain is normal. Which of
late without assistance. She appears somewhat short of the following is the most appropriate next step in her
breath. Cranial nerve examination findings are normal evaluation?

113
a. Serum paraneoplastic antibodies, including Purkinje cell c. Substitute phenytoin for carbamazepine.
cytoplasmic autoantibody type 1 (PCA-1) (also known as d. Perform prolonged video-EEG monitoring for possible
anti-Yo) anterior temporal lobectomy.
b. Consultation with a psychiatrist e. Discontinue use of valproate and consider vagus nerve
c. Cervical spine MRI with contrast medium stimulation.
d. Nerve conduction studies, including needle electromyog-
7. A 38-year-old obese woman has a history of depression,
raphy
chronic obstructive pulmonary disease, constipation,
e. Sural nerve biopsy
and recurrent migraine headaches. Up to 3 years ago,
6. A 20-year-old man is evaluated for spells during which the headaches averaged 1 attack every 2 to 4 months
he appears alert but stares ahead, is unresponsive to and responded to sumatriptan. Over the past 3 months,
the environment, and exhibits automatic lip-smacking they have increased in frequency to 1 to 3 attacks
movements. Results of his neurologic examination are weekly. Results of neurologic and ophthalmoscopic
normal. Magnetic resonance imaging of the head shows examinations are normal. Which of the following is
atrophy of the left hippocampus. Results of awake elec- the most appropriate prophylactic treatment for these
troencephalography (EEG) are normal. The spells, headaches?
which occur up to 3 times weekly despite treatment a. Daily long-acting triptan
with maximally tolerated doses of carbamazepine and b. Valproate
valproate over the past 10 months, have prevented him c. Topiramate
from driving and working. Which of the following is d. Propranolol
the most appropriate approach? e. Amitriptyline
a. Substitute topiramate for valproate.
b. Add gabapentin or levetiracetam.

114 M AYO C L I N I C I N T E R N A L M E D I C I N E B OA R D R E VI EW: Q U E S T I O N S A N D A N S W E R S


ANSWER S

1. Answer c. dysfunction), and systemic symptoms, such as nausea and


vomiting and diarrhea, may precede these symptoms.
Orthostatic hypotension is an important cause of dizzi-
ness, particularly in patients at risk of autonomic failure,
such as those with diabetic neuropathy or synucleinopa- 3. Answer b.
thies (eg, parkinsonism). The hallmark of orthostatic Weakness may be caused by abnormalities in many areas
hypotension due to autonomic failure is inability to of the central nervous system and peripheral nervous sys-
increase the heart rate when the blood pressure decreases tem. Central nervous system abnormalities causing weak-
profoundly upon standing. The first step in managing ness include supratentorial, infratentorial, and spinal cord
orthostatic hypotension is to correct potentially reversible lesions, each of which would cause an upper motor neuron
causes, particularly the use of vasodilators, diuretics, and pattern of weakness with increased reflexes and extensor
anticholinergics. Simple maneuvers, such as increasing salt plantar responses. The weakness is usually associated with
and water intake, elevating the head of the bed, and per- other findings that localize the lesion somewhere along the
forming postural maneuvers, should be tried before phar- corticospinal tract. Disorders of the nerve roots (ie, radicu-
macologic management. lopathies) causing weakness are often associated with pain,
2. Answer e. they may be associated with decreased reflexes, and they are
commonly associated with sensory complaints. Disorders
Myasthenia gravis is an autoimmune disorder of the neu- of the muscle are associated with normal reflexes early in
romuscular junction. It causes fatigable weakness of the the disease but may be diminished later. Weakness is typi-
axial musculature or bulbar muscles (or both). Diplopia, cally greater proximally than distally. Plantar responses are
ptosis, speech slurring, and difficulty with chewing or swal- flexor, and there is no sensation loss. Neuromuscular dis-
lowing are typical symptoms of the fatigable weakness of eases such as myasthenia gravis may cause fatigable weak-
the bulbar muscles. Additional diagnostic studies include ness of both the axial and the bulbar muscles. Reflexes are
electromyography, which shows a decrement in amplitude typically normal, and sensation is intact.
of compound muscle action potentials with repetitive Peripheral nerve disorders typically cause more diffuse
stimulation. Computed tomography of the chest should weakness that is greater distally than proximally. They are
be performed because thymoma is occasionally associated often associated with some degree of sensory loss (there
with myasthenia gravis. Thymectomy has a role in the treat- are more pure motor neuropathies that are associated with
ment of myasthenia gravis, particularly in a person with minimal sensory symptoms). Reflexes are often decreased
generalized myasthenia gravis and in younger patients who early in the course and the plantar responses are flexor.
are not at significant surgical risk. Acetylcholine receptor Guillain-Barr syndrome (GBS) is an acute inflamma-
antibody testing is also useful for diagnosis. The Tensilon tory demyelinating disorder of the peripheral nerves and
(edrophonium chloride) test involves the administra- nerve roots. Patients present with subacute onset of weak-
tion of edrophonium, a short-acting acetylcholinesterase ness and sensory symptoms (eg, paresthesias) that slowly
inhibitor, which temporarily increases the concentration progess proximally from the hands and feet. The weak-
of acetylcholine in the synaptic cleft, so that the signs of ness usually begins in the legs but may occur simultane-
myasthenia gravis are temporarily improved. Atropine ously in the upper and lower extremities. Facial weakness
should be available because of the potential for bradycardia occurs later in 50% of cases. The reflexes are diminished
or other cholinergic symptoms. Autoimmune thyroid dis- early with large fiber joint position and vibratory sensa-
ease can also occur in myasthenia gravis, and appropriate tion loss. Antecedent infection with viral or bacterial infec-
blood tests should be performed. Fatigable weakness argues tions occur from 1 to several weeks before symptom onset.
against muscle disease, and muscle biopsy is not useful in Further findings include abnormal CSF with an elevated
myasthenia gravis. Another neuromuscular junction dis- protein level and a normal cell count (ie, albuminocyto-
ease, botulism, occurs typically several hours after ingesting logic dissociation). In most GBS patients, these abnormali-
food containing Clostridium botulinum. The patients pres- ties can be found within 1 week after onset. MRI is not
ent with diplopia, dysphagia, dysarthria, ptosis, and weak- needed because the lesion is localized to the peripheral
ness of the jaw muscles. Autonomic symptoms can also nerve. Electromyographic findings may be normal early
occur (eg, constipation, dilation of the pupils, and urinary but later show conduction block and nerve conduction

115
velocity slowing, with findings more apparent on motor paraneoplastic mechanism. In this patient, the most appro-
testing than on sensory testing; fibrillation potentials indi- priate next step among the choices listed is assessment for
cating denervation also occur within 2 weeks of symptom paraneoplastic antibodies, such as PCA-1, that may mediate
onset. a cerebellar syndrome. These antibodies and this syndrome
Patients with GBS are usually hospitalized and closely are predictive of gynecologic malignancies such as ovarian
monitored for respiratory compromise. Treatment is usu- carcinoma or breast carcinoma. Other diagnostic consider-
ally initiated with plasmapheresis or intravenous immu- ations for this patient might include structural cerebellar
noglobulin and is effective particularly if it is started early. lesions such as a tumor (effectively excluded by the normal
Close attention is also paid to deep vein thrombosis pro- imaging), intoxication with anticonvulsant medications or
phylaxis, risk of aspiration pneumonia, constipation, back alcohol, or infectious cerebellitis.
pain, and autonomic instability (variable heart rate and
blood pressure). 6. Answer d.

4. Answer c. This young man has temporal lobe epilepsy with complex
partial seizures that is intractable since it is not responding
The patients presentation suggests an acute ischemic cere- to 2 medications. Typically, if a patient does not achieve
bral infarction in the distribution of the anterior division seizure control with 2 medications, additional medical
of the left middle cerebral artery, but CT of the head is therapy will not help. In this case, the next consideration
required to exclude the management-changing possibility is for a surgical procedure to remove the epileptogenic por-
of cerebral hemorrhage. The commonly accepted window tion of his brain. Continuous EEG monitoring is done in
for IV thrombolysis is 3 hours, but imaging, laboratory a hospitalized setting, often with withdrawal of the seizure
tests, and a careful history cannot be sacrificed to beat the medications, in order to map the seizure onset. If it local-
deadline. Selected patients may be considered for IV tPA izes to the temporal lobe, there is about a 90% chance that
after 3 hours, and intra-arterial intervention can be con- the patient will be free of seizures after temporal lobec-
sidered up to 6 or more hours after the onset of symptoms. tomy. Vagus nerve stimulation is done in patients who are
Anticoagulation does not typically have a role in the man- not surgical candidates.
agement of acute ischemic cerebral infarction, particularly
before imaging has excluded hemorrhage, although it will 7. Answer c.
likely be considered for secondary prevention of stroke in The frequency of the migraine headaches is such that
this patient who has atrial fibrillation. The treatment of preventive medication is appropriate. Answer choices b
mild, asymptomatic hypertension in this patient could lead through e would be reasonable for prevention of migraine,
to neurologic deterioration. An EEG might be helpful if a but triptans are abortive agents and would not be used
postictal paresis is suspected (Todd paralysis), but there is daily. A patients comorbidities must be reviewed before an
no history of seizure. appropriate preventive agent is chosen. Since this patient
5. Answer a. has lung disease, depression, and obesity, topiramate would
be the best choice. Potential adverse effects of topiramate
Gait ataxia, limb dysmetria, and ataxic dysarthria all sug- include paresthesias, cognitive clouding, taste perturba-
gest a cerebellar disorder, and the subacute onset of the tion (particularly with carbonated beverages), anorexia,
symptoms raises the likelihood of an autoimmune or and nephrolithiasis.

116 M AYO C L I N I C I N T E R N A L M E D I C I N E B OA R D R E VI EW: Q U E S T I O N S A N D A N S W E R S


14.
DERMATOLOGY QUESTIONS AND ANSWER S

QUESTIONS 2. An 86-year-old man has an itchy, new rash. At the


nursing home where he lives, he has been treated
Multiple Choice (choose the best answer) with different antihistamines and topical corticoster-
oids without response. A clinic visit is arranged. On
1. A patient presents with recurrent episodes of the skin
examination, he has red papules and excoriations on
findings shown in Figure 14.Q1. What is the most likely
the hands, groin, and axillae (Figure 14.Q2). He has
infectious association?
nodular areas on his scrotum. What diagnosis should
a. Herpes simplex virus
you consider?
b. Pseudomonas
a. Chronic idiopathic urticaria
c. Dermatophyte
b. Urticarial vasculitis
d. Borrelia burgdorferi
c. Bullous pemphigoid
e. Mycobacterium marinum
d. Pemphigus vulgaris
e. Scabies

Figure 14.Q2

3. A 26-year-old white woman presents with a facial


rash that first appeared last summer and returned
during her recent trip to Arizona. Initially, she says
that she feels well, but then she describes fatigue
and malaise. She has a family history of rheumatoid
arthritis. On examination, she is afebrile and she has
no active synovitis. Both cheeks are erythematous
and have small papules (Figure 14.Q3). There are no
other concerning signs on examination. What is the
diagnosis?
a. Malar rash of lupus
b. Seborrheic dermatitis
c. Allergic contact dermatitis
Figure 14.Q1. (Adapted from Drage LA, Bundrick JB, Litin SC. Clinical
pearls in dermatology. Mayo Clin Proc. 2012 Jul;87[7]:6959. Used with d. Rosacea
permission of Mayo Foundation for Medical Education and Research.) e. Acne vulgaris

117
Figure 14.Q3

4. A 49-year-old woman presents with a persistent, pru-


ritic, red rash on her face, chest, elbows, knees, and
hands. The rash began last summer and is unrespon-
sive to topical corticosteroids. Her past medical his-
tory is significant for asthma and migraines. For the
past month, she has been having more problems with
shortness of breath and headaches. Some of her skin
findings are shown in Figure 14.Q4. She has mild
wheezing. Neurologic examination findings are nor- Figure 14.Q4
mal. Laboratory test results are positive for antinuclear
antibody and normal for the complete blood cell count,
liver function tests, and renal function tests. What other
laboratory testing would be most important?
a. Chest radiography and mammography
b. Patch testing
c. Lupus serologies
d. Tissue transglutaminase testing
e. Light testing

5. A 52-year-old man with chronic obstructive pulmonary


disease has a significant flare. You prescribe tapered
dosages of corticosteroid and a course of azithromycin.
A new rash develops (Figure 14.Q5). What is the most
likely cause of the rash?
a. Tapered dosages of corticosteroid
b. Drug rash
c. Pityriasis rosea
d. Tinea corporis
e. Mycoplasma pneumonia

Figure 14.Q5

118 M AYO C L I N I C I N T E R N A L M E D I C I N E B OA R D R E VI EW: Q U E S T I O N S A N D A N S W E R S


6. A 52-year-old woman presents with a nonhealing ulcer a. Tissue transglutaminase antibody serology
of the lower extremity. Three weeks earlier, she noted b. Creatine kinase measurement
a purple papule on her shin. It quickly enlarged, ulcer- c. Perinuclear antineutrophil cytoplasmic antibody testing
ated, and became extremely painful. Dbridement was d. Human immunodeficiency virus serology
performed at a local wound care center; afterward, the e. A 24-hour urine sample for measuring the level of
ulcer expanded to 3 times its original size. The woman 5-hydroxyindoleacetic acid
is now admitted for intravenous antibiotics and pain
control. On her left shin, she has a large, tender ulcer
with a purple border (Figure 14.Q6). Her biopsy from
the wound care center was interpreted as abscess, but
the culture results are negative so far. What is the most
likely diagnosis?
a. Brown recluse spider bite
b. Venous insufficiency ulcer
c. Necrotizing fasciitis
d. Atypical mycobacteria infection
e. Pyoderma gangrenosum

Figure 14.Q6

7. A 43-year-old man presents with weight loss and mild


anemia. He is also concerned about a new rash that is
intensely pruritic and burning. He has received a
diagnosis of irritable bowel syndrome from his refer-
ring physician and is being treated with fiber. He takes
no other medication. He still has intermittent diar-
rhea and bloating but no other symptoms. On physical Figure 14.Q7
examination, you note the skin changes shown in Figure
14.Q7. What testing will help confirm your diagnosis?

14 . D E R M ATO L O GY Q U E S T I O N S A N D A N S W E R S 119
ANSWER S

1. Answer a. 4. Answer a.
Patients who have erythema multiforme present with Dermatomyositis is an inflammatory myositis with distinc-
target lesions that are often located on the palms and tive skin findings. Skin signs seen with dermatomyositis
soles but may occur as generalized eruptions. Drugs such may include heliotrope rash, Gottron papules, photosen-
as sulfonamides, barbiturates, and anticonvulsants are sitivity, Gottron sign (erythema over the extensor surfaces
commonly associated with development of erythema of the finger joints, knees, and elbows), periorbital edema,
multiforme, but recurrent lesions are most often linked scalp erythema and pruritus, violaceous erythema of
to a herpes simplex infection. Although the skin signs of sun-exposed or extensor surfaces, periungual erythema and
an active herpes simplex infection may be apparent, the telangiectasia, cuticle hypertrophy, generalized pruritus,
outbreak may be subclinical. Discontinuing use of any shawl sign, and holster sign. Patients may present with skin
culprit drugs and a trial of acyclovir or other appropri- findings but without clinical weakness or muscle enzyme
ate antiviral would be appropriate in cases of recurrent abnormalities (amyopathic dermatomyositis or dermato-
erythema multiforme. myositis sine myositis). Dermatomyositis may be a skin
sign of internal malignancy with a significantly higher risk
2. Answer e.
of lung, breast, or ovarian cancer. A workup for underlying
Scabies is caused by infestation with the mite Sarcoptes malignancy should be pursued.
scabiei var hominis. Infection occurs as a result of direct
5. Answer a.
skin-to-skin contact; fomite transmission is uncommon.
It causes epidemics in schools, hospitals, and nursing The man has psoriasis vulgaris. Psoriasis may be precipitated
homes. The rash results from a hypersensitivity reaction by many factors, including infection (streptococcal phar-
to the mite protein. The main complaint is pruritus, espe- yngitis, human immunodeficiency virus disease), stress,
cially at night. Clinical features include inflammatory, smoking, alcoholism, physical trauma (koebnerization),
excoriated papules in the web spaces of the hands and and medications. Common medications that cause flares
feet, axillae, groin, and wrists and the areolae and sub- or unmask psoriasis include lithium, -blockers, calcium
mammary sites of women. Nodules or thickened areas in channel blockers, angiotensin-converting enzyme inhibi-
the scrotum are also helpful clues. The pathognomonic tors, some nonsteroidal anti-inflammatory drugs, anti-
finding is a burrow, commonly located on finger webs. malarials, and tapered dosages of corticosteroid. Psoriasis
Identification of mites on a scabies preparation is diag- manifests with sharply demarcated plaques with silver scale
nostic. In immunocompromised patients, a highly conta- involving the elbows, knees, scalp, lumbar region of the
gious form of scabies may appear as a generalized scaling back, and the gluteal cleft. Nail signs (nail pits, oil spots,
eruption (ie, crusted scabies, formerly called Norwegian and onycholysis), scalp scale, gluteal cleft pinking, and
scabies). Treatment of classic scabies includes the topical koebnerization (development of psoriatic skin disease at
application of permethrin; oral ivermectin may be used to sites of trauma) are subtle signs that can help with the diag-
treat crusted scabies. nosis. Psoriatic arthritis occurs in 5% to 8% of patients who
have psoriasis of the skin and commonly manifests as asym-
3. Answer d.
metric oligoarthritis or enthesitis.
Not all malar rashes result from lupus. Rosacea is a com-
6. Answer e.
mon facial rash with red papules and small pustules on a
base of erythema and telangiectasia. It commonly affects Pyoderma gangrenosum is primarily a clinical diagnosis
the central facial area. Sun, vasodilators, caffeine, and hot based on the patients history and physical examination
or spicy foods may trigger flares. It may be associated with findings. Patients typically present with an expanding
eye findings, but otherwise it has no systemic associa- nonhealing ulcer on the lower extremities and may relate
tions. Mild rosacea is treated with topical metronidazole, a history of trauma that initiated or worsened the lesion
use of sunscreens, and avoidance of triggering factors. (ie, pathergy). The ulcer is extremely painful and unre-
Anti-inflammatory antibiotics, such as tetracycline, are sponsive to conservative wound care measures. On physi-
used in more severe cases. Rhinophyma can be treated with cal examination, the rapidly enlarging, necrotic ulcer has
laser and other surgical methods. a purple undermined margin and a surrounding border of

120
erythema. The histopathologic features are nonspecific, the patient. Thus, on physical examination, excoriations,
and on skin biopsy the neutrophilic infiltrate may be inter- erosions, and crusts are more commonly noted than intact
preted as cellulitis or abscess. Pyoderma gangrenosum vesicles. Most patients with dermatitis herpetiformis have
may occur in otherwise healthy people or may be a skin subclinical celiac disease (gluten-sensitive enteropathy).
sign of internal disease. Important associations include Adults may present with episodic or nocturnal diarrhea,
inflammatory bowel disease, IgA monoclonal gammo- flatulence, or weight loss. Abdominal discomfort and bloat-
pathy, rheumatoid arthritis, chronic active hepatitis, ing may lead to a mistaken diagnosis of irritable bowel syn-
and hematologic malignancies. Treatment of pyoderma drome. Biopsies show classic jejunal villous atrophy in 75%
gangrenosum includes intralesional corticosteroids, oral of the patients; the remainder have minor mucosal changes.
corticosteroids, potassium iodide, dapsone, minocycline, The standard for diagnosis of dermatitis herpetiformis is a
azathioprine, and cyclosporine. The response to cortico- skin biopsy that shows granular immunoglobulin A depos-
steroids is dramatic. its in the dermal papillae on direct immunofluorescence
examination. Serologic tests for tissue transglutaminase
7. Answer a.
antibody or endomysial antibody are highly sensitive and
Dermatitis herpetiformis manifests with intensely pru- specific markers for an associated gluten-sensitive enterop-
ritic vesicles and papules occurring symmetrically over the athy. Treatment of dermatitis herpetiformis includes a
knees, elbows, lower back, neck, and scalp. The classically gluten-free diet and dapsone. Dapsone provides rapid
grouped (herpetiform) vesicles are rarely intact because of relief from pruritus and the rash but has no effect on the
the intensity of the pruritus and subsequent scratching by enteropathy.

14 . D E R M ATO L O GY Q U E S T I O N S A N D A N S W E R S 121
This page intentionally left blank
15.
CROSS-CONTENT AREA QUESTIONS AND ANSWER S

QUESTIONS 3. A 76-year-old woman undergoes a right L5 forami-


notomy and an L5-S1 fusion. Past history is signifi-
Multiple Choice (choose the best answer) cant for mixed connective tissue disease. Preoperative
medications are methylprednisolone, hydroxychloro-
G E R I AT R I C S
quine, and warfarin for symptoms of connective tis-
sue disease (arterial occlusive disease was considered
1. An 80-year-old man is brought in by his son. The a component of the connective tissue disease). She is
patient has no complaints, but the son is concerned that also taking nortriptyline and tramadol for back pain.
his fathers memory is failing. The patient resides in an Immediately postoperatively, she has mild hypoxemia
assisted living facility. He receives 2 meals daily and an (thought to be from narcotics). On the evening after
aide helps him with housework weekly. His family moved surgery, she is agitated and awake all night. She is
him there several months ago when it was clear that he drowsy and difficult to arouse on postoperative day 1
could no longer live alone. Last week, a staff member and refuses to eat or participate in therapy. She does
found him wandering in the street, dressed in only a bath- not sleep. On postoperative day 2, she is not oriented
robe and pajamas (the outside temperature was 20C). to time or place and is complaining of chest pain.
The patient told the nurses that he was following a man What is the most appropriate way to manage her
who had entered his room. He has a history of hyperten- symptoms?
sion treated with lisinopril 20 mg daily. His son states a. Vest restraints, bed rails, and benzodiazepines titrated to a
that he found an empty pill vial at home, and it does not dose that causes sedation
appear that the prescription has been refilled recently. b. Vest restraints, haloperidol for agitation, and 24-hour
On examination, the patients blood pressure is 160/95 supervision
mm Hg. He is unkempt. He has a resting tremor and a c. Sleep enhancement protocol, family support, and haloperi-
slow, shuffling gait. His Mini-Mental State Examination dol for agitation
(MMSE) score is 19 points (out of 30). Six months ago, d. Benzodiazepines for sleep and removal of urinary catheter
his MMSE score was 22 points (out of 30). What is the e. Psychiatric consultation
most appropriate next step in his management?
a. Perform magnetic resonance imaging (MRI) of his brain.
4. An 87-year-old woman presents with a 5-kg weight loss
b. Start donepezil therapy.
over the past 9 months. She had a myocardial infarc-
c. Advise the son to move him into a nursing home.
tion 10 years ago but has been asymptomatic ever since.
d. Start low-dose haloperidol therapy for his psychotic
She has been treated with atenolol and aspirin, and her
symptoms.
mild hypertension is well controlled with the atenolol.
e. Start citalopram therapy.
She fractured her hip when she fell on ice about 7
years ago, but she does not have a history of recurrent
2. A 66-year-old female nursing home resident complains of falls. She states that her mood is good. She does her
urinary incontinence. She describes the sensation of hav- own shopping, but finds it hard to carry the grocer-
ing an urgent need to void. However, with her arthritic ies into the house because of generalized fatigue. She
knees, she often cannot reach the toilet before she loses lives alone and her only daughter lives 5 hours away.
urine. These urinary symptoms have been present for sev- She does not have any gastrointestinal tract (GI) com-
eral years but have gradually worsened. She is now afraid plaints. Her only real complaint is fatigue. Her body
to go out in public. She has no dysuria or hematuria. She mass index is 20 (it was 23 at her previous examination
takes metoprolol, hydrochlorothiazide, and a stool soft- 9 months ago). Vital signs are normal. The remainder
ener. Which of the following is the most appropriate first of the examination findings are normal. Her health
step in the management of her incontinence? screening is up-to-date and unremarkable. Laboratory
a. Begin tolterodine therapy. test results are normal for erythrocyte sedimentation
b. Try a trial of topical estrogen. rate, complete blood cell count, and creatinine. What
c. Refer to a urologist for further evaluation. should be done next?
d. Try a trial of timed voiding.
e. Discontinue the use of hydrochlorothiazide.

123
a. Perform a complete GI workup, including computed tomog- P R EV E N T I V E M E D I C I N E
raphy of the abdomen and upper and lower GI endoscopy.
8. A 54-year-old man presents to an urgent care clinic
b. Discontinue the use of atenolol.
after stepping on a piece of broken glass in his front
c. Start antidepressant therapy.
yard. The glass is lodged in the sole of his left foot. He
d. Help her arrange assistance with shopping and cooking.
says that he has received several tetanus vaccinations
e. Recommend that she move in with her daughter.
during his life, but he does not remember all the dates.
5. A 68-year-old woman presents for her annual examina- Immunization records show that he received his lat-
tion. She has no complaints. She has a history of hyperten- est tetanus-diphtheria (Td) vaccination 6 years ago.
sion, which is well controlled with hydrochlorothiazide. What further therapy is needed for optimal tetanus
She is physically active. She received an influenza vacci- prophylaxis?
nation last year and plans to receive one again this year. a. Tetanus immune globulin
She received her pneumococcal vaccination at age 66. She b. Tetanus toxoid and tetanus immune globulin now
received her most recent tetanus toxoid 10 years ago. She c. Tetanus toxoid
helps her daughter run a family day care for 12 children d. Tetanus toxoid followed by tetanus immune globulin in
aged 3 months to 6 years. In addition to the influenza vac- 7 days
cine, what other vaccines should you recommend? e. No further therapy
a. Tetanus and diphtheria toxoids vaccine booster
b. Tetanus and diphtheria toxoids and acellular pertussis 9. An asymptomatic, active 77-year-old man presents for
(Tdap) vaccine a general medical examination. He does not have any
c. Polyvalent pneumococcal vaccine medical problems, but he has a 10-pack-year smoking
d. Measles-mumps-rubella vaccine history. He has no family history of colon cancer. He
e. Hepatitis A vaccine had his latest colonoscopy 5 years ago; results were nor-
mal without polyps. He received tetanus-diphtheria
6. An 83-year-old woman is seen in the office for her (Td) vaccine and pneumococcal vaccine polyvalent
annual examination. She has a history of osteoporosis 9 years ago and zoster vaccine live 3 years ago. Which
and is taking alendronate. Her body mass index is 27. preventive service should you recommend?
She is the primary caretaker for her 85-year-old hus- a. Abdominal aortic aneurysm (AAA) screening
band, who has Parkinson disease. She is a nonsmoker. ultrasonography
She had a normal colonoscopy at age 77. Which of the b. Pneumococcal vaccine polyvalent
following screening tests is appropriate? c. Td vaccine
a. Spirometry d. Td vaccine and AAA screening ultrasonography
b. Colonoscopy e. Tetanus and diphtheria toxoids and acellular pertussis
c. Carotid ultrasonography (Tdap) vaccine
d. Hypertension screening
e. Exercise stress test 10. A 35-year-old woman presents for a general medical
examination. She is healthy but is concerned that her
7. A 78-year-old man is brought to your office by his wife.
father had colon cancer at age 57. She wants to know
He has a known history of Alzheimer disease and has
at what age she should undergo her first colonoscopy.
had a rapid loss of memory. Therapy with donepezil
What age should you recommend?
was stopped because he had gastrointestinal tract side
a. 37
effects. He did not want to try memantine. He can no
b. 40
longer care for himself. His wife, who has diabetes mel-
c. 42
litus, osteoarthritis, and hypertension, provides all his
d. 47
care. She is feeling increasingly stressed. He has periods
e. 50
of agitation several times each day. On 1 or 2 occasions,
he has struck his wife. He often resists getting dressed 11. A 28-year-old woman presents for a routine general
or eating. His wife fears that she can no longer continue medical examination. She has had long-standing,
to care for him at home. However, she is struggling well-controlled asthma. She is otherwise healthy. She
with this decision because he has told her that he would does not smoke or drink alcoholic beverages, and she
rather die than live in a nursing home. What is the most works as an office secretary. Which of the following
appropriate way to manage his symptoms? should you recommend?
a. He can no longer be cared for safely at home, so he should a. Influenza and pneumococcal vaccination
live in a nursing home. b. Human papillomavirus (HPV) vaccination
b. He should be enrolled in an adult day care that incorporates c. Influenza vaccination only
music therapy and aromatherapy. d. Pneumococcal vaccination only
c. He should take risperidone. e. Influenza and HPV vaccination
d. Low-dose lorazepam should be given as needed for agitation.
e. He should take citalopram.

124 M AYO C L I N I C I N T E R N A L M E D I C I N E B OA R D R E VI EW: Q U E S T I O N S A N D A N S W E R S


12. A 42-year-old woman presents to your office for a menopause 2 years ago without the use of hormone
routine general medical examination. She is healthy therapy. She has been free of vasomotor symptoms
and does not have a family history of breast cancer. for 6 months but comments that intercourse with
She had menarche at age 14 and her first child at age her partner of 3 years is uncomfortable, so she began
30, and she breast-fed her 3 children. She has never using a vaginal lubricant with some improvement in
had a mammogram and asks about the need for mam- symptoms. Otherwise, she is doing very well. Her pre-
mography. What should you recommend for this ventive health screening tests are all up-to-date with
patient? the exception of her Papanicolaou (Pap) test; results
a. Perform screening now and then annually. of her latest test, done 3 years ago, were normal. On
b. Perform screening now and then at age 50. examination, she appears healthy and comfortable.
c. Begin screening at age 50. Her pelvic examination shows vaginal atrophy with no
d. Perform screening now and then biennially. other findings. You recommend vaginal estradiol. A
e. Weigh the individual risks and values with the benefits and Pap test is performed. Two days later, the results show
harms of screening. atypical squamous cells of undetermined significance
(ASC-US). You schedule an appointment for her to
13. A 32-year-old man presents for a general medical exam-
return to the office to discuss the results and your rec-
ination. He is healthy and does not routinely take any
ommendations. Which of the following should you
medications. He has had 3 lifetime partners and has
recommend?
been in a monogamous relationship for 5 years. He is
a. Human papillomavirus (HPV) DNA testing and, if the
a nonsmoker, he drinks 1 or 2 beers each week, and he
results are positive, loop electrosurgical excision
does not use illicit drugs. His father had a myocardial
b. Loop electrosurgical excision
infarction at age 69. His blood pressure in the office
c. Pap tests in 6 and 12 months and, if the results still show
is 118/68 mm Hg. Which of the following screening
ASC-US, colposcopy
services would be most appropriate to offer to this
d. Vaginal estradiol for 2 months and then another Pap test
patient?
e. Colposcopy
a. Blood chemistry panel
b. Fasting blood glucose level 16. A healthy 47-year-old woman wants your recommenda-
c. Lipid panel tions for breast cancer prevention because her mother
d. Chlamydia screening had breast cancer at age 59. The patient performs
e. Voluntary human immunodeficiency virus (HIV) testing regular breast self-examinations and practices breast
14. A 51-year-old healthy bus driver is planning to join self-awareness, and she has noticed no changes. She
a local gym with his wife. He does not exercise on a began having mammograms at age 40 and has had no
regular basis. He does not take daily medications or abnormalities; her latest mammogram 13 months ago
use tobacco. His father died of a myocardial infarc- was unremarkable, showing only dense tissue. Her past
tion at age 63. His blood pressure in the office is medical history is unremarkable. Her latest menstrual
122/78 mm Hg, his pulse is 76 beats per minute, period was 1 month ago, but the period before that one
and his body mass index is 30. His total cholesterol was 9 months earlier; before that, her menses had been
level is 180 mg/dL, and his high-density lipoprotein irregular. Family history is otherwise notable for ovar-
cholesterol level is 42 mg/dL. His Framingham risk ian cancer in her maternal aunt at age 61, breast cancer
score is 5%. He asks for heart testing before start- in her maternal grandmother at age 63, prostate cancer
ing his new exercise program. What further testing in her maternal uncle at age 52, and pancreatic cancer in
should you recommend? her maternal great grandfather at age 67. Findings on her
a. Electrocardiogram (ECG) physical examination, including breast examination, are
b. Computed tomographic (CT) coronary calcium scan normal. The mammogram done the same day is normal
c. No further testing and unchanged compared with prior mammograms.
d. Exercise treadmill testing Dense tissue is still observed. You recommend a medi-
e. High-sensitivity C-reactive protein (hs-CRP) testing cal genetics consultation to further evaluate her family
history because you are concerned about the possibil-
WO M E N S H E A LT H
ity of hereditary breast and ovarian cancer syndrome.
Three weeks later, the patient returns and reports that
15. A 51-year-old woman who you know well comes for her mother accompanied her to the genetics consulta-
her annual preventive gynecologic examination. She tion. The geneticist estimated that the patients mother
feels well. She has a history of type 2 diabetes melli- had a 24% likelihood of a mutation; thus, the patient
tus, which she manages with her lifestyle, and rheu- has a 12% likelihood. The patients mother was tested
matoid arthritis. She tells you that her rheumatologist and the patient has learned that her mother carries a
prescribed adalimumab shortly after her visit with you truncation mutation in BRCA2. Which of the follow-
last year and that her symptoms are well controlled; ing should you recommend that the patient consider as
she is pleased with the results. She went through her next step?

15. C R O S S - C O N T E N T A R E A Q U E S T I O N S A N D A N S W E R S 125
a. Annual digital mammogram and clinical breast examination and left subareolar ultrasonography are unremark-
b. Bilateral mastectomy and oophorectomy able, except for mild duct ectasia, with no abnormali-
c. Tamoxifen 20 mg daily for 5 years ties identified. Which of the following should you
d. Raloxifene 60 mg daily for 5 years recommend?
e. Genetic testing a. Reassurance and recommendation to discontinue checking
so frequently
17. A healthy 21-year-old premedical student presents to b. Ductoscopy
your office for further evaluation of a change in her c. Breast magnetic resonance imaging (MRI)
menstrual periods. She reports that her periods began d. Subareolar duct excision
at age 13. Until 7 months ago, her menses were regular e. Pituitary MRI
for 5 years and occurred every 27 to 28 days. She feels
well but comments that she has been stressed about tak- 20. A healthy 37-year-old woman presents to your office
ing the Medical College Admission Test and has been for further evaluation of vaginal symptoms. She reports
staying up late studying for several weeks. She attri- that for the past 3 months she has had persistent vagi-
butes her headaches and blurry vision to stress and lack nal irritation, with itching and burning and a somewhat
of sleep. She also tells you that her weight has increased thick discharge. She has normal menses. She has been
since freshman year, probably because she snacks while in a monogamous relationship with the same partner
studying and eats fast food too often. She says that she for the past 15 years. She uses oral contraceptives. She
started exercising and dieting last year, but she has not appears mildly uncomfortable but otherwise well. Her
lost weight. Records she brought from a student health examination is significant for vulvar erythema without
clinic visit for an ankle sprain freshman year note body any lesions; she has a mildly thick discharge. The spec-
mass index of 25 and normal vital signs. Her blood ulum examination shows normal findings, but she is
pressure is 127/73 mm Hg, her heart rate is 70 beats per uncomfortable during it and during the bimanual exam-
minute and regular, and her body mass index is 28. She ination. A Papanicolaou test is performed. The pH of
appears to be a healthy, somewhat overweight young the vagina is 4.3. The whiff test is negative. Microscopy
woman who is comfortable. Findings from the physical shows only a moderate number of large rods, and Gram
examination, including a pelvic examination, are unre- staining shows gram-positive rods. Culture results are
markable. Which of the following tests would be most negative for Trichomonas vaginalis. What is the most
informative? likely diagnosis?
a. Serum -human chorionic gonadotropin a. Vaginal candidiasis
b. Thyrotropin and free thyroxine b. Trichomoniasis
c. Serum estradiol c. Contact dermatitis
d. Serum prolactin d. Bacterial vaginosis
e. Serum follicle-stimulating hormone e. Lichen planus

18. Which of the following vaccinations should not be 21. A 54-year-old healthy woman asks for a recommenda-
administered during pregnancy? tion for hot flushes and vaginal symptoms. She com-
a. Human papillomavirus (HPV) pleted natural menopause about 18 months ago. She
b. Meningococcus began having hot flushes and sweats during perimeno-
c. Pneumococcus pause, and the symptoms continued after menopause.
d. Hepatitis A She wakes up multiple times at night because of hot
e. Tetanus-diphtheria (Td) flushes and has to change her nightclothes at least once
nightly. She is fatigued and irritable. She is embarrassed
19. A 35-year-old woman presents to your office concerned at work and in social situations because she sweats
about milky discharge from the left breast. She feels through her clothes. She has followed all recommenda-
well and has no other concerns. She reports normal tions about clothing, diet, and her immediate environ-
menstrual periods and no prior breast problems. Her ment, but her symptoms have continued. Soy and red
sister had breast cancer at age 43. The patient has been clover have not helped, either. She has also had worsen-
very worried and checks regularly to see if the discharge ing vaginal symptoms with dryness and burning caus-
is still present. Her self-examination findings are other- ing significant discomfort during intercourse despite
wise unchanged. She has 2 children and finished nurs- use of vaginal lubricants. This has led to avoidance of
ing the second child 18 months ago. On examination, intimacy with her partner. She reports exercising regu-
she appears comfortable, she has no adenopathy, and larly, and she has never smoked. She has no significant
her breasts are normal bilaterally, with no skin abnor- medical or family history. On examination, she is a
malities and no palpable abnormalities. Several drops healthy-appearing woman in no distress. Her breasts
of milky fluid are easily expressible from several ducts of are normal on examination. Pelvic examination shows
the left nipple. The fluid tests negative for blood. Serum significant vaginal atrophy. Her mammogram from 3
thyrotropin (TSH) and prolactin (PRL) levels are nor- months ago was unremarkable. She has never had an
mal. Results from bilateral diagnostic mammography abnormal result from a Papanicolaou test, including

126 M AYO C L I N I C I N T E R N A L M E D I C I N E B OA R D R E VI EW: Q U E S T I O N S A N D A N S W E R S


the latest one, done 2 years ago. Which of the following e. Bilateral diagnostic mammography, ultrasonography, and
is the most appropriate recommendation? magnetic resonance imaging
a. Start use of vaginal estrogen tablet 25 mcg twice weekly, and
24. Which of the following statements is true?
reassess within 12 months.
a. Rubella is the most common cause of congenital viral
b. Add black cohosh 40 mg daily, and reassess within 12
infection.
months.
b. A pregnant woman presenting with pulmonary edema and
c. Start use of venlafaxine 37.5 mg daily, titrating up to 75 mg
atrial fibrillation has a high likelihood of having mitral
daily as needed, and reassess within 12 months.
stenosis.
d. Encourage her to wait it out and use a different vaginal
c. The lifetime risk of ovarian cancer is greater than 75% in a
lubricant.
carrier of the BRCA1 mutation.
e. Begin combination hormone therapy, and reassess within 12
d. Dysfunctional uterine bleeding is benign and requires no
months.
evaluation.
22. A healthy 47-year-old woman presents to your office e. Annual clinical breast examinations decrease breast cancer
with a 2-week history of a grape-sized breast lump that mortality.
she felt while showering. She reports that the lump is
nontender and she has not noticed any skin changes G E N E R A L I N T E R NA L M E D I C I N E
or nipple discharge. The appearance of the breast is
unchanged. Her latest menstrual period occurred last 25. Your clinic has access to magnetic resonance imaging
week and was normal. She has no prior medical prob- (MRI) technology. You are evaluating a patient who has
lems, and her family history is negative for malignan- a diabetic foot infection. There is no exposed bone. You
cies. She has no notable risk factors for malignancies. are concerned that he has not responded well to anti-
On examination, she appears healthy. Breast examina- biotic treatment, and you determine that his current
tion is unremarkable with the exception of a 2.5-cm pretest likelihood of having underlying osteomyelitis
firm mass in the lateral portion of the right breast. A is 50%. Recently, you read an article that evaluated the
mammogram shows dense breast tissue that appears role of MRI in diagnosing foot osteomyelitis. The sensi-
normal, symmetrical, and, when compared with a prior tivity of this test is 90%, and the specificity is 85% (com-
screening mammogram, stable. What should you rec- pared with bone biopsy as the referenced standard). If
ommend as the next step? you use MRI for this patient, and it is positive for osteo-
a. Breast ultrasonography
myelitis, what is the likelihood that he indeed will have
b. Reassurance
osteomyelitis on bone biopsy?
a. 94%
c. Surgical consultation
b. 89%
d. Follow-up mammogram midcycle in 1 to 2 months
c. 86%
e. Breast magnetic resonance imaging
d. 50%
23. A 31-year-old nulliparous woman in good health pres- e. 11%
ents for evaluation of bilateral breast pain. She reports
aching, burning pain (pain level 5 on a scale from 0 26. Refer to the information in question 25. If the mag-
to 10) in both breasts; the discomfort has been inter- netic resonance imaging (MRI) findings were negative,
fering with sleep and intimacy. Her breasts feel heavy what is the likelihood that this patient would not have
and uncomfortable, especially with activity that moves osteomyelitis?
a. 94%
them. She has not noted any other changes in either
b. 89%
breast, but she does not do self-examinations. Her
c. 86%
latest menstrual period was 3 weeks ago, and her peri-
d. 50%
ods have been normal. She has not paid attention to
e. 11%
whether there is any pattern to the pain, but her qual-
ity of life has been affected since she avoids intimacy 27. You have recently read about a randomized controlled
and exercise when her breasts hurt. Her mother had trial that compared the efficacy of warfarin (interna-
breast cancer at age 62. On examination, the patient is tional normalized ratio 2.03.0) with that of aspirin
overweight and has large, symmetrical breasts that are (325 mg) in stroke prevention in a subset of elderly
normal in appearance. She has nodular, lumpy breasts patients (75 years or older) who had atrial fibrillation.
bilaterally but no focal lumps or discrete masses. The The patients were followed for 3 years. The final results
nodularity is thought to be relatively symmetrical when showed that the chance of major stroke, intracranial
the breasts are compared. What is the most appropri- hemorrhage, or systemic embolism was 5% among
ate next step? patients treated with warfarin and 10% among patients
a. Reassurance and breast support with a well-fitting bra treated with aspirin. From these results, what is the rela-
b. Bilateral diagnostic mammography and ultrasonography tive risk reduction (RRR) of a major event in the group
c. Danazol 100 mg daily in divided doses treated with warfarin compared with the group treated
d. Fine-needle aspiration biopsy from the area of greatest pain with aspirin?

15. C R O S S - C O N T E N T A R E A Q U E S T I O N S A N D A N S W E R S 127
a. 5% He is otherwise healthy. His creatinine value is 1.1 mg/
b. 10% dL, and his platelet count is 220109/L. Which of the
c. 20% following statements is correct?
d. 50% a. Because the clot is confined to his calf veins, anticoagulation
e. 100% is not required, but ultrasonography should be repeated in 5
to 7 days to rule out proximal extension.
28. Refer to the information in question 27. For the group
b. A temporary vena cava filter should be placed to decrease
treated with warfarin (international normalized ratio
the risk of pulmonary embolism because anticoagulation
2.03.0) compared with the group treated with aspirin,
would be too risky in this early postoperative situation.
what is the absolute risk reduction (ARR) of a major
c. For convenience, oral dabigatran could be used to treat the
event at 3 years?
clot because no laboratory monitoring would be needed.
a. 5%
d. He should receive anticoagulants for 3 months; an appro-
b. 10%
priate low-molecular-weight heparin (LMWH) (eg, enox-
c. 20%
aparin) dosage would be 120 mg subcutaneously once daily
d. 50%
while administering warfarin.
e. 100%
e. He should receive anticoagulants for 6 months; an appro-
29. Refer to the information in question 27. What is the priate LMWH (eg, enoxaparin) dosage would be 120 mg
number needed to treat (NNT) for 3 years with warfa- subcutaneously once daily while administering warfarin.
rin anticoagulation to prevent 1 major event?
a. 5
G E N E R A L I N T E R NA L M E D I C I N E A N D Q UA L IT Y
b. 10
I M P ROVE M E N T
c. 20
d. 25 32. You serve on the sentinel event review committee at your
e. 50 hospital. An event occurred in which a patient received
30. A 70-year-old woman presents with acute pain in an overdose of heparin. Your committee completes a
her left leg. Ultrasonography has confirmed a new root cause analysis and finds that the error resulted from
thrombus in the left superficial femoral vein. She a gap in physician knowledge about heparin dosing, the
is otherwise healthy and states that she has not had lack of an institutional consensus on heparin dosing,
recent surgery or trauma. Her only medication is con- and a cumbersome order entry system. From the root
jugated estrogen 0.625 mg daily for hot flushes. She cause analysis, which of the following interventions is
does not have a personal or family history of venous most likely to have a sustained effect?
a. An online education module on heparin dosing
thromboembolism, and she has no major bleeding
b. Distribution of a heparin dosing pocket card
risks. Her creatinine value is 0.9 mg/dL. She has a
c. A heparin order set
supportive husband at home who can help her with
d. A new institutional policy on heparin dosing
activities of daily living. She weighs 60 kg. What is
e. A physician education conference on heparin dosing
the most appropriate next step in the management of
this patient? 33. An 83-year-old woman comes to your office with a
a. Elevate the leg, apply an elastic bandage wrap to the leg, and 2-day history of a red left eye associated with dull,
administer anti-inflammatory drugs until she has recovered. achy pain and tears. She denies having any trauma
b. Discontinue use of conjugated estrogen, elevate the leg, or visual problems. On examination, she has clear
apply an elastic bandage wrap to the leg, and administer tears and a focal area of redness in the medial sclera.
anti-inflammatory drugs until she has recovered. What would you recommend for control of her
c. Hospitalize and treat with either low-molecular-weight hep- symptoms?
arin (LMWH) or unfractionated heparin while administer- a. Acetaminophen and avoiding contact with others until the
ing warfarin. symptoms resolve
d. Provide outpatient treatment with LMWH while adminis- b. Aspirin and reassurance
tering warfarin. c. Urgent referral to ophthalmology
e. Request computed tomography (CT) of the chest to look d. Gentamicin eyedrops for 7 to 10 days
for pulmonary emboli before deciding on hospitalization or e. Corticosteroid eyedrops for 1 week
outpatient treatment.
34. A disheveled 20-year-old college student comes into
31. A 62-year-old man weighing 80 kg presents for evalua- the university student health center. It is Monday and
tion of an acute increase in swelling and pain of his right he has just returned from a spring break trip to Florida.
lower extremity. One week earlier, he had a right total He complains of a bright red right eye associated with
knee arthroplasty and is currently walking with a walker. copious purulent discharge, which began abruptly last
Ultrasonography of his right lower extremity confirms night. On examination, you note a brightly erythema-
that a thrombus in the right anterior tibial vein extends tous sclera associated with purulent discharge and a
into the popliteal vein. The femoral vein is free of clot. prominent right preauricular lymph node. In addition

128 M AYO C L I N I C I N T E R N A L M E D I C I N E B OA R D R E VI EW: Q U E S T I O N S A N D A N S W E R S


to immediate referral to the local ophthalmologist for 38. Recently, several complaints have been filed by patients
oral and topical antibiotics, what other testing should in your clinic about excessive wait times in the lobby.
be considered next? As a member of the quality team at your clinic, you are
a. Allergy testing for pollen and ragweed charged to study and fix this problem. Which of the
b. Screening for human immunodeficiency virus/AIDS following improvement methodologies would be most
c. Urinary polymerase chain reaction for Chlamydia trachoma- successful at reducing wait times for patients in the
tis and Neisseria gonorrhoeae clinic lobby?
d. Rapid streptococcal antigen test for group A streptococci a. LEAN
e. Complete blood cell count with erythrocyte sedimentation b. Six Sigma
rate c. Cause-and-effect diagramming
d. Swim lane diagramming
35. A 42-year-old kindergarten teacher presents to your
e. Failure mode and effects analysis
office with a 2-week history of an upper respiratory
illness. She initially noted a low-grade fever, achiness, 39. A 72-year-old woman with long-standing diabetes mel-
sinus congestion with clear discharge, and cough, which litus is admitted to your hospital service for pain control
improved over 10 days but then seemed to recur with a for a pelvic insufficiency fracture. She receives hemodi-
vengeance. Now she has a severe sore throat, enlarged alysis. An electrocardiogram shows that her corrected
glands, and fever. She does not have a cough or short- QT interval is 480 ms. What pain medication should
ness of breath. On examination, she has a fever (39C), be ordered for this patient?
tonsillar exudate, and cervical adenopathy, and she a. Fentanyl
appears ill and uncomfortable. Abdominal examina- b. Methadone
tion findings are unremarkable. Which of the following c. Propoxyphene napsylate
is most appropriate? d. Codeine
a. Mononucleosis spot test e. Meperidine
b. Throat culture and Gram stain
c. Rapid streptococcal antigen test
M E D I C A L ET H I C S A N D PA L L I AT I VE C A R E
d. Empirical penicillin V for 10 days
e. Chest radiograph 40. For 6 years, you have been caring for a 54-year-old
unmarried man. He has bipolar disease but has been
36. Which features could potentially distinguish between
faithful in taking his medication. He is intelligent and
acute angle-closure glaucoma and acute anterior
works as a computer programmer. He has not had an
uveitis?
episode of significant mania or depression for the past 4
a. Circumcorneal injection
years. He is a former smoker, but he quit smoking when
b. Blurry vision
his bipolar disease was adequately treated. Recently,
c. Hypopyon
he presented with the complaint of a new cough with-
d. Pain
out other symptoms of upper respiratory tract infec-
e. Unilateral presentation of symptoms
tion, and he reported that on a couple of occasions he
37. A 60-year-old male patient presents with gradual-on- coughed up blood in his sputum. Chest radiography,
set erectile dysfunction (ED). He is in good health, followed by computed tomography, showed that he had
and he takes no medications. On physical examina- a solitary lung mass on the right side. Sputum analysis
tion, there is no evidence of penile abnormalities or confirmed the presence of adenocarcinoma. His evalu-
testicular atrophy. Visual field testing and cardiopul- ation suggested that the lesion was stage I, which would
monary examination findings are normal. Serum tes- indicate a high probability of cure by resection (5-year
tosterone levels are normal. He reports that he has a survival about 85%). When you share the diagnosis and
healthy libido and is involved in a stable and otherwise recommendations for operation with him, he declines
satisfactory marital relationship. What should you the procedure. In fact, he wants no treatment at all. He
recommend? acknowledges that he will probably die of this cancer if
a. He should undergo coronary angiography to rule out he does not pursue treatment, but he says that he accepts
vascular disease as a cause of the ED before initiating any this as his fate. He is oriented and does not appear to be
treatment. manic or depressed. What should you do next?
b. He should undergo further evaluation for hypogonadism as a. Comply with his refusal and schedule regular appointments
a likely cause of the ED. to pursue palliative care.
c. He should undergo magnetic resonance imaging (MRI) of b. Recommend psychiatric consultation to assess capacity
the brain to rule out pituitary disease as a cause of his ED. because of his history of bipolar disease.
d. He should undergo psychotherapeutic intervention for per- c. Declare the patient incompetent because of his irrational
formance anxiety. choice and seek a court-appointed guardian.
e. He should take a phosphodiesterase-5 (PDE-5) inhibitor 1 d. Contact his parents and coworkers and enlist them to con-
hour before sexual intercourse. vince the patient to pursue treatment.

15. C R O S S - C O N T E N T A R E A Q U E S T I O N S A N D A N S W E R S 129
e. Schedule another appointment in 1 week to discuss the a. Negotiate a time-limited period of continued aggressive
issues again after he has had a chance to think about his intervention with the family, and encourage them to reach a
situation. consensus.
b. Stop the use of the ventilator as requested by the patients
41. A 63-year-old man had a permanent pacemaker placed a
rightful surrogate, his wife.
year ago for symptomatic complete heart block. He has
c. Refuse to withdraw any treatment at any point because there
now had a massive myocardial infarction and is respon-
is no advance directive.
sive to only deep pain. His ejection fraction is estimated
d. Request an ethics consultation.
to be 12%, and recovery of additional myocardial func-
e. Tell the family members that they must obtain a court order
tion is not expected. The patient has a living will stating
before use of the ventilator can be stopped.
that he would not want life-sustaining treatments initi-
ated or continued if his chances of recovery from a seri- 43. A 16-year-old adolescent boy is brought to the emer-
ous illness were small. Aggressive treatments are being gency department after a motorcycle accident. Severe,
withheld according to the familys request in com- continuous hemorrhage has decreased the patients
pliance with the living will. The family now comes to hemoglobin value to 4.5 g/dL. His systolic blood pres-
you requesting that the pacemaker be deprogrammed. sure is 70 mm Hg. He is unconscious. His parents arrive
What should you do? soon after the ambulance and refuse for their son to
a. Comply with the request and deprogram the pacemaker. receive any blood products, in keeping with their reli-
b. Give the family the deprogrammer and ask them to depro- gious beliefs. They want to take him home, where they
gram the pacemaker. and their neighbors can pray for the boy. What should
c. Request an ethics consultation. you do?
d. Refuse to comply with the request because it would con- a. Contact the hospitals legal department to get a court order
stitute actively taking the patients life, and euthanasia and for transfusions and operation.
assisted suicide are illegal in your state. b. Avoid transfusion and manage with crystalloid.
e. Refuse to comply with the request because the living will is c. Dismiss the patient to the family against medical advice.
invalid in this circumstance. d. Start transfusions immediately and prepare the patient for
the operation to stop the bleeding.
42. A 72-year-old man is admitted to the hospital with
e. Ask the hospital chaplain to reason with the parents.
an acute onset of severe bilateral pneumonia after his
third round of chemotherapy for diffuse large cell
non-Hodgkin lymphoma. He is in respiratory fail- P E R I O P E R AT I VE M E D I C I N E
ure and consents to intubation. After intubation, he
44. A 72-year-old woman will be undergoing a left total
requires chemical paralysis to tolerate mechanical ven-
knee arthroplasty because of degenerative joint dis-
tilation. Cultures and bronchoalveolar lavage do not
ease. You are asked by her orthopedist to evaluate
identify an infectious cause, and corticosteroids are
her preoperatively. She has a history of hypertension,
added to broad-spectrum antibiotic therapy in case the
which is currently well controlled with medication.
patient has cyclophosphamide-induced pneumonitis.
She also has chronic kidney disease, with a baseline
After 1 week of treatment, a little progress is observed,
creatinine value of 2.3 mg/dL, which is thought to be
although attempts to wean the patient from chemical
the cause of the hypertension. She does not have dia-
paralysis have failed. At this point, the patients wife
betes mellitus. She had a stroke 3 years ago, with no
and 3 children demand to meet with the treating phy-
residual impairment. She does not have known coro-
sicians. The wife and oldest son insist that use of the
nary artery disease. Her functional capacity is limited
ventilator be discontinued and the patient be allowed
by her knee pain. She cannot walk up a flight of stairs
to die. They claim that he would not want this therapy
without stopping. Her current medications are lisino-
and that it is torture for him. When queried, they indi-
pril, metoprolol, simvastatin, metformin, and aspirin.
cate that there is no known advance directive and that
Physical examination findings are remarkable for a
neither of them has ever overtly discussed these sorts of
blood pressure of 146/75 mm Hg and a heart rate of
issues with the patient. At this point, the youngest son
55 beats per minute. Her electrocardiogram shows a
dissents and states that his father is a fighter and would
first-degree atrioventricular block and normal sinus
want the ventilator therapy to continue in hopes that he
rhythm. The only laboratory test abnormality is the
could improve. He states that his father was always an
creatinine value. What should you recommend for
aggressive businessman and overcame 2 previous can-
further preoperative testing?
cers, lung cancer and prostate cancer. The middle child,
a. No further testing
a daughter, is very confused and does not know what
b. Dobutamine stress echocardiogram
to think or what her family would want. All again reit-
c. Dipyridamole thallium testing
erate that none of them have ever discussed issues such
d. Exercise stress test
as life-sustaining treatment or goals of therapy with the
e. Transthoracic echocardiogram
patient. At this point, what should the physician do?

130 M AYO C L I N I C I N T E R N A L M E D I C I N E B OA R D R E VI EW: Q U E S T I O N S A N D A N S W E R S


45. A 72-year-old woman is admitted to the hospital with 48. A 78-year-old woman has peripheral vascular disease
a hip fracture, which she sustained while shoveling her and is scheduled for an aortofemoral bypass. Her past
driveway. She has been in good health. She denies having history includes hypertension, diabetes mellitus, and
syncope, shortness of breath, chest pain, or palpitations hyperlipidemia. She has no symptoms of coronary
before the fall. She has no history of diabetes mellitus, artery disease, but her activity is limited by claudica-
hypertension, coronary artery disease, or hyperlipidemia. tion and arthritis in her knee. Current medications
She takes no medications. She does all her own house- include metformin 1,000 mg twice daily, metoprolol
work, yard work, and snow shoveling without symptoms. 100 mg twice daily, lovastatin 20 mg daily, aspirin 81
On physical examination, her blood pressure is 135/88 mg daily, and lisinopril 10 mg daily. On examination,
mm Hg, and her heart rate is 76 beats per minute and her heart rate is 60 beats per minute, and her blood
regular. Her lungs are clear. On cardiac examination, she pressure is 135/80 mm Hg. Heart and lung examina-
has a coarse, ejection-type murmur at the second right tion findings are normal. She has no peripheral edema,
intercostal border, which radiates to the carotids, and but she does have markedly diminished pulses in her
her carotid upstrokes are mildly diminished. There is feet. Dipyridamole thallium testing shows a moder-
no peripheral edema. What is the most appropriate next ately sized, fixed inferior wall defect with peri-infarct
step in the management of this patient? ischemia and an ejection fraction of 50%. Cardiac
a. Delay surgery until an echocardiogram can be obtained. catheterization showed a 30% to 50% stenosis in the
b. Recommend that the patient begin taking atenolol 25 mg left anterior descending coronary artery and in the
daily before surgery. right coronary artery. Which of the following strate-
c. Cancel the surgery until a dobutamine stress echocardio- gies should you recommend?
gram can be obtained. a. Proceed with surgery, continuing her current medications.
d. Advise the anesthesiologist of the patients condition. b. Refer her for coronary artery bypass graft (CABG) surgery
e. Send the patient to the telemetry unit for further before vascular surgery.
monitoring. c. Refer her for angioplasty and stent placement before vascu-
lar surgery.
46. A 75-year-old man is seen preoperatively before a neph-
d. Cancel the surgery and try to manage her claudication
rectomy for renal cell carcinoma. He has a history of
medically.
chronic obstructive pulmonary disease (COPD) due to
e. Prescribe perioperative nitrate therapy.
smoking, and he has had multiple unsuccessful attempts
at smoking cessation. He currently smokes about 15 49. A 68-year-old man is being evaluated for a transurethral
cigarettes daily. He has a chronic cough but denies hav- resection of the prostate for prostate cancer. He has a
ing dyspnea, and he walks 2 to 3 miles daily without Starr-Edwards mitral valve and is taking long-term
symptoms. He has no other medical problems. He has warfarin for anticoagulation. His international nor-
had 2 exacerbations of COPD in the past 2 years, dur- malized ratio (INR) has been in the therapeutic range.
ing which he was treated as an outpatient and did not The urologist asks for recommendations about the
require systemic corticosteroids. What is the best way perioperative management of his anticoagulation.
to manage him preoperatively? Other medical history includes hypertension, type 2
a. Perform spirometry and check arterial blood gases. diabetes mellitus, obesity, and peripheral arterial dis-
b. Perform chest radiography, spirometry, and a 6-minute walk ease. Physical examination findings are unremarkable.
test. His blood pressure is 130/85 mm Hg, and his heart
c. Perform spirometry and check lung volumes. rate is 65 beats per minute. His medications include
d. No further testing is needed. warfarin, lisinopril, atenolol, metformin, hydrochlo-
e. Delay his surgery for 8 weeks until he has stopped smoking. rothiazide, and atorvastatin. Preoperative laboratory
studies show normal values for the complete blood cell
47. A 70-year-old man is seen for preoperative medical
count and electrolyte levels, creatinine 1.1 mg/dL, and
evaluation before elective cataract removal. He received
INR 2.5. What is the most effective way to manage his
2 drug-eluting stents for symptomatic coronary artery
anticoagulation?
disease 4 months ago. He is taking both aspirin (81
a. Stop use of warfarin 4 days before surgery.
mg daily) and clopidogrel (75 mg daily). What is your
b. Stop use of warfarin 4 days before surgery, and initiate
advice to the patient and his eye surgeon about medica-
low-molecular-weight heparin (LMWH) therapy.
tion management in the perioperative period?
c. Continue warfarin therapy through the entire perioperative
a. Stop use of clopidogrel 7 days before surgery, but continue
period.
use of aspirin.
d. Stop use of warfarin, and admit 4 days before surgery for
b. Stop use of clopidogrel and aspirin 7 days before surgery.
infusion of unfractionated heparin.
c. Stop use of aspirin 7 days before surgery, but continue use of
e. Stop use of warfarin and begin aspirin therapy 325 mg
clopidogrel.
daily.
d. Defer surgery for an additional 6 months.
e. Continue use of both clopidogrel and aspirin.

15. C R O S S - C O N T E N T A R E A Q U E S T I O N S A N D A N S W E R S 131
50. A 66-year-old woman has degenerative joint disease a. Proceed with surgery without any intervention.
in her left knee and wants to have a total knee replace- b. She should receive nonsurgical therapy since her surgical
ment. She has a history of type 2 diabetes mellitus, risk is prohibitive.
which is controlled with insulin. Her body mass index c. She should be treated with a transjugular intrahepatic por-
(BMI) is 41, and she has cirrhosis, which is thought to tosystemic shunt (TIPS) before surgery.
be from nonalcoholic steatohepatitis. Her aspartate d. She should receive a transplant evaluation before surgery.
aminotransferase and alanine aminotransferase levels e. Postpone surgery until her BMI is less than 35.
are twice the upper limit of the reference ranges. Her
Model for End-Stage Liver Disease (MELD) score is 7.
Her Child-Turcotte-Pugh score is 6. What should you
recommend?

132 M AYO C L I N I C I N T E R N A L M E D I C I N E B OA R D R E VI EW: Q U E S T I O N S A N D A N S W E R S


ANSWER S

1. Answer b. GI workup is not indicated. She shows no signs of depres-


sion. Her blood pressure is well controlled, and there does
This patient shows signs of dementia with Lewy bod-
not appear to be a good reason to discontinue the use of
ies. Patients who have dementia with Lewy bodies have
atenolol. Moving in with her daughter may be an option,
signs of parkinsonism, visual hallucinations, and rapid eye
but this would involve moving away from a familiar envi-
movement sleep disorder. They are extremely sensitive to
ronment. There are many agencies that could assist this
neuroleptic agents, which may severely worsen the clinical
woman with her shopping and cooking needs.
course. Donepezil and therapy for Parkinson disease may
be effective. MRI would not contribute anything to his 5. Answer b.
management.
This patient has close contact with infants younger than
2. Answer d. 6 months. She should receive 1 dose of Tdap vaccine. She
This patient has classic symptoms of overactive bladder was born before 1957, so she is presumed to have immunity
with urinary urgency incontinence. Although tolterodine to measles, mumps, and rubella. She has already had 1 dose
is likely to give the patient benefit from her symptoms, of pneumococcal vaccine and will not need another.
there are other, nonpharmacologic treatments that should 6. Answer d.
be tried before medications. Topical estrogen may benefit
some patients with overactive bladder, but the evidence The US Preventive Services Task Force recommends hyper-
supporting benefit is weak. Generally, the best approach tension screening for all adults. Screening for carotid artery
starts with the least aggressive measures and then leads to stenosis and chronic obstructive pulmonary disease is not
more invasive, more expensive, and higher-risk procedures. recommended for asymptomatic patients who have no risk
Since this patient has overactive bladder associated with factors. This patient has no signs or symptoms of coronary
urinary urgency incontinence, bladder suspension surgery artery disease, and an exercise stress test is not indicated.
is unlikely to give her any benefit; that procedure is for Colonoscopy in patients older than 75 is controversial and
stress urinary incontinence. Although a referral to an urol- probably not indicated for patients who have had negative
ogist is not inappropriate, there are certainly several treat- colonoscopy findings in the past.
ment measures that the primary caregiver could attempt
7. Answer b.
first. This patient may be a candidate for timed voiding,
which is often successful in nursing homes. The patient is This patient may ultimately need to enter a nursing home,
toileted every 2 hours or at a frequency determined by the but that situation would be distressing for both the patient
patients voiding diary. This is a simple, noninvasive treat- and his wife. Nonpharmacologic therapies such as music
ment option that effectively decreases the number of daily therapy and aromatherapy have been shown to be success-
incontinence episodes. ful for patients who have moderate to severe dementia. It
is unclear how long the effect will last, but those therapies
3. Answer c.
should probably be tried before nursing home placement.
Vest restraints often cause or worsen delirium and should not Medications have only limited success in treating behav-
be used except in extreme circumstances. Benzodiazepines ioral symptoms of dementia, and nonpharmacologic ther-
may also cause or worsen delirium. Haloperidol may be apy should be tried first.
appropriate for short-term management of behavioral
8. Answer c.
symptoms. Orienting strategies, familiar objects and peo-
ple, and normalizing the sleep-wake cycle help to decrease For a clean minor wound, revaccination with Td is required
the severity and shorten the duration of delirium. if the latest vaccination was more than 10 years ago.
However, for a dirty deep wound, revaccination with Td
4. Answer d.
is required if the latest vaccination was more than 5 years
Many elderly patients exhibit signs of frailty for socioeco- ago (Table 15.A8). (See Centers for Disease Control and
nomic reasons. This patient has no GI symptoms, and a Prevention, 2012 in the Suggested Reading list.)

133
Table 15.A8
TETANUS PROPHYLAXIS FOR CLEAN MINOR
WOUNDS TETANUS PROPHYLAXIS FOR ALL OTHER WOUNDS

VACCINATION HISTORY Tetanus-Diphtheriaa Tetanus Immune Globulin Tetanus-Diphtheriaa Tetanus Immune Globulin

Unknown or <3 doses Yes No Yes Yes


b c
3 doses No No No No
a
Tetanus-diphtheria-acellular pertussis (Tdap) may be substituted for tetanus-diphtheria if the person has not previously received Tdap.
b
Yes, if the latest dose was given more than 10 years ago.
c
Yes, if the latest dose was given more than 5 years ago.
Adapted from Centers for Disease Control and Prevention. Epidemiology and Prevention of Vaccine-Preventable Diseases. 12th ed. Washington DC: Public Health
Foundation; c2012. Chapter 20, Tetanus; p. 291300.

9. Answer e. Screening for Breast Cancer, topic page in the Suggested


The Advisory Committee on Immunization Practices Reading list.)
recently released an updated recommendation for Tdap 13. Answer e.
vaccination: for adults 65 years or older who have not pre-
viously received Tdap, a single dose of Tdap may be given The Centers for Disease Control and Prevention recom-
instead of Td vaccine. Tdap can be administered regard- mends that all persons between 13 and 64 years of age be
less of the interval since the latest tetanus vaccine. The US tested for HIV regardless of recognized risk factors. The
Preventive Services Task Force recommends screening for US Preventive Services Task Force makes no recommenda-
AAA in men aged 65 to 75 who have smoked more than tion for or against routinely screening for HIV in adults
100 cigarettes. Pneumococcal vaccination is not routinely who are not at increased risk of HIV infection (grade C
repeated for otherwise healthy patients. Zoster vaccine live recommendation). Given that this patient is otherwise
is administered once to adults who are 60 or older; they do healthy and has few cardiac risk factors, he does not need
not require another vaccination. (See Centers for Disease fasting blood glucose or lipid screening. Lipid screen-
Control and Prevention [CDC], 2011 in the Suggested ing is recommended at age 35 for otherwise healthy men.
Reading list.) Blood glucose screening is recommended for individuals
with hypertension. A blood chemistry panel is not recom-
10. Answer b. mended as a routine screening test. (See Branson et al in
The American Cancer Society, the US Multisociety Task the Suggested Reading list.)
Force on Colorectal Cancer, and the American College of 14. Answer c.
Radiology developed consensus guidelines for colon cancer
screening. Accordingly, persons who have a first-degree rel- The US Preventive Services Task Force (USPSTF) rec-
ative who had colon cancer or adenomatous polyps before ommends against routine screening with resting ECG,
age 60 should start colonoscopy at age 40, or 10 years CT scanning for coronary calcium, and exercise treadmill
before the youngest case in the immediate family, which- testing. The USPSTF also concludes that there is insuf-
ever is earlier. Therefore, this patient should start screening ficient evidence to balance the risks and benefits of using
at age 40. (See Levin et al in the Suggested Reading list.) nontraditional risk factors such as hs-CRP testing. CT
coronary calcium scanning and hs-CRP testing can be
11. Answer a. considered for asymptomatic individuals if their 10-year
Pneumococcal vaccination is recommended for adults risk of a cardiovascular event is between 10% and 20%.
aged 19 to 64 who have asthma or who are smokers. HPV The patient in this question has a Framingham risk score of
vaccination is recommended for women until age 26. only 5%. (See Screening for Coronary Heart Disease With
Influenza vaccination is now recommended for everyone Electrocardiography, topic page and Using Nontraditional
aged 6 months or older. (See Centers for Disease Control Risk Factors in Coronary Heart Disease Risk Assessment,
and Prevention; Advisory Committee on Immunization topic page in the Suggested Reading list.)
Practices and Fiore et al in the Suggested Reading list.)
15. Answer c.
12. Answer e.
The 2001 guidelines for a postmenopausal woman with
The US Preventive Services Task Force updated its recom- vaginal atrophy recommended short-term vaginal estro-
mendations on mammography in December 2009: The gen treatment and a follow-up Pap test, which generally
decision to start regular, biennial screening mammog- would yield normal findings. Those guidelines, however,
raphy before the age of 50 years should be an individual recommended that immunosuppressed women undergo
decision that accounts for patient context, including the colposcopy immediately because they had a higher risk of
patients values related to specific benefits and harms. (See neoplasia. More recent guidelines (2006) offer the same

134 M AYO C L I N I C I N T E R N A L M E D I C I N E B OA R D R E VI EW: Q U E S T I O N S A N D A N S W E R S


recommendations for postmenopausal women, immu- 18. Answer a.
nosuppressed women, and women in the general popu-
The HPV vaccine is an inactivated viral vaccine that is likely
lation because additional data have shown that women
safe (pregnancy category B), but it has not been studied
with ASC-US have a low prevalence of invasive cancer.
well in this population, and current guidelines recommend
Loop electrosurgical excision is not appropriate for any
against its use during pregnancy. Women inadvertently
population with ASC-US. HPV DNA testing and colpos-
vaccinated during pregnancy should have their exposure to
copy are also reasonable choices for this patient (and the
the vaccine reported to the Vaccine in Pregnancy Registry.
general public) but choice c is the least invasive and most
The other vaccines listed as answer choices are known to
cost-effective. Additional management guidelines for dif-
be safe during pregnancy. Hepatitis A, pneumococcal, and
ferent populations and higher-grade cytologic findings
meningococcal vaccines should be administered to preg-
appear in the article by Wright et al (see the Suggested
nant women who are considered to be at high risk rather
Reading list).
than to all pregnant women. Td should be administered to
16. Answer e. pregnant women whose latest booster was 10 or more years
earlier or who have a deep or dirty wound. (See Pickering
Although bilateral mastectomy and oophorectomy may
et al in the Suggested Reading list.)
be a reasonable choice, this patient should be offered
genetic testing because her mother is a known mutation 19. Answer a.
carrier. If the patient declines, prophylactic surgery or
more intense surveillance would be a reasonable option. The patients nonbloody nipple discharge is consistent
Intense surveillance for a BRCA mutation carrier or for with galactorrhea. With otherwise normal results for the
a first-degree relative of a carrier should include annual examination, for the TSH and PRL levels, and for the
breast magnetic resonance imaging in addition to mam- breast imaging and with no objective evidence of blood in
mography, biannual clinical breast examination, annual the fluid, there is no need for further breast imaging or any
gynecologic examination, biannual or annual testing other testing. There is also no need for a surgical evaluation.
for cancer antigen 125, and pelvic ultrasonography with Given the normal PRL level, the galactorrhea is not indica-
appropriate counseling on the benefits, limitations, and tive of a prolactinoma and pituitary MRI is unnecessary. If
potential harms of each test. Raloxifene would not be there is uncertainty as to whether the secretion is milk, a fat
appropriate for 2 reasons: 1) There are no safety data stain could be done, but this is rarely necessary. In addition
from premenopausal or perimenopausal women, so it to reassurance, the patient should be instructed to discon-
is indicated only for postmenopausal women, and this tinue checking for discharge so frequently to avoid stimu-
patient is perimenopausal. 2) There are no data assess- lating the breast tissue to continue fluid production. (See
ing whether raloxifene decreases breast cancer incidence Hussain et al in the Suggested Reading list.)
among BRCA mutation carriers or women suspected of 20. Answer c.
being BRCA mutation carriers. Tamoxifen may be an
option, but the data on tamoxifen in BRCA2 mutation Although the sensitivity of the whiff test is relatively low,
carriers are not extensive and are inconclusive; however, this patients vaginal pH is normal, effectively excluding
the data that are available suggest a benefit. (See Nelson bacterial vaginosis. Large rod-shaped bacilli in the vaginal
et al in the Suggested Reading list.) secretions represent normal vaginal flora (lactobacilli) and
are not indicative of infection. No motile species are noted
17. Answer d. on microscopy, so Trichomonas is unlikely and the nega-
Although pregnancy is the most common cause of sec- tive culture results confirm the absence of Trichomonas.
ondary amenorrhea and must always be considered and The patient should eliminate potential irritants (laundry
excluded as a cause of amenorrhea (secondary or primary), detergent, douching, vulvar sprays, fragranced cleansers,
prolactinoma is the cause of secondary amenorrhea in 1 of etc) and be reassessed if her symptoms do not resolve. (See
7 women. This patient does not complain specifically of Laine et al in the Suggested Reading list.)
diplopia, but she reports visual changes that she attributes
21. Answer e.
to stress and lack of sleep and may not realize that she is
experiencing diplopia. The possibility of a prolactinoma Hot flushes and night sweats (vasomotor symptoms)
should not be overlooked. Further, the peak incidence of are the most frequent menopause-related complaints.
prolactinomas is in the third to fifth decades of life and Although symptoms generally decrease over time, they
is most common in women in that age range. Testing for persist for up to 10 years in up to 50% of women, with 10%
hypothyroidism should be considered if the clinical presen- reporting symptoms for even longer. Vaginal atrophy and
tation suggests that hypothyroidism is possible. Although dryness due to menopause are also common symptoms.
this patient has gained weight and is having difficulty losing This patients quality of life is significantly affected by her
it, there are other reasons for her weight gain; weight gain vasomotor symptoms and vaginal symptoms. Since she
due to hypothyroidism usually occurs with other symp- has no significant risk factors for the use of combination
toms and at an older age. (See Mindermann and Wilson in hormone therapy (breast cancer, smoking, or coronary
the Suggested Reading list.) artery disease), it is the best choice; combination hormone

15. C R O S S - C O N T E N T A R E A Q U E S T I O N S A N D A N S W E R S 135
therapy will address vaginal and vasomotor symptoms. arm. When women keep a monthly chart, they note that
Since she has an intact uterus, unopposed estrogen can- the pain develops during the 2 weeks before menstrual
not be used because of the risk of uterine malignancy. If bleeding; after menstruation starts, the pain decreases and
this patients symptoms had been primarily vaginal, vagi- typically resolves completely between cycles. For a woman
nal estrogen alone would have been an appropriate choice. who provides this history and has unremarkable clinical
Some data have shown benefit from antidepressants (eg, examination findings, no diagnostic testing is necessary.
venlafaxine) for vasomotor symptoms, but her symptoms If a woman is eligible for a screening mammogram and
are significant and venlafaxine would not help the vaginal if she is overdue for screening, an up-to-date screening
dryness. There is insufficient evidence for the effectiveness mammogram should be obtained. Otherwise, mammog-
of herbal remedies. Given the severity of her symptoms and raphy is not necessary for evaluation of this type of breast
the negative effect on her quality of life, waiting longer to pain. Typically, reassurance and education are sufficient.
see if her symptoms improve is not a reasonable choice. The A well-fitting support bra (eg, a sports bra) is very help-
lowest effective dose of hormone therapy for the shortest ful; for some women, decreasing the intake of caffeine and
duration in low-risk women is recommended, and its use salt can be helpful, although the evidence is inconsistent.
should be reassessed at least annually. Initiation of hor- In addition, there is some inconsistent evidence for the use
mone therapy should be avoided for women who smoke, of supplements. Despite multiple studies, the underlying
have vascular disease, have a history of breast cancer, or are causes are undefined.
well past menopause. (See Utian et al in the Suggested Breast pain can be characterized as cyclic, noncyclic,
Reading list.) and extramammary. Noncyclic mastalgia is not associated
with the menstrual cycles and is usually constant or inter-
22. Answer a.
mittent. There are multiple causes of noncyclic mastalgia,
Most palpable breast masses are benign, but evaluation is and, although it is uncommon for breast cancer to manifest
necessary to exclude malignancy, even with a normal mam- with only pain, focal and persistent pain must be evaluated
mogram and the absence of any breast cancer risk factors. to exclude an underlying malignancy. Noncyclic breast
Of the palpable malignancies, 10% to 15% are mammo- pain usually occurs at a postmenopausal age, but it may
graphically occult. In a woman older than 30, the likeli- occur earlier, and it is usually unilateral with an anatomical
est cause is a fibroadenoma, a cyst, fibrocystic change, or cause that can be identified. In addition, multiple medica-
hyperplasia. Evaluation should include mammography and tions have been associated with noncyclic breast pain.
ultrasonography with a strong consideration for biopsy of The cause of extramammary pain commonly involves
a solid mass (fine-needle aspiration, core needle biopsy, the underlying chest wall, but other conditions may cause
or excision). Ultrasonography reliably distinguishes solid extramammary pain (eg, ischemia, pulmonary disease, tho-
masses from cystic lesions. A benign biopsy in a woman racic nerve root irritation) and should be considered.
with a negative mammogram does not preclude care- Evaluation must include a careful history (including
ful follow-up to assess for long-term stability. In women quality, location, severity, aggravating and alleviating fac-
younger than 30, evaluation should begin with ultrasonog- tors, and the effect on quality of life). Medications should
raphy; if the results are abnormal, mammography should be carefully reviewed (including any supplements and
be performed. In this situation, it is also acceptable to allow herbs). A thorough clinical examination should include the
1 or 2 menstrual cycles to pass before reassessing clinically chest wall. Focal, persistent pain should be evaluated with
and proceeding with imaging if the palpable mass persists. ultrasonography, and mammography should be included
Diffuse lumpiness or nodularity is usually from fibrocystic for women 30 or older. Any solid mass should be evaluated
change. Discrete cysts occur most often in women younger with a biopsy.
than 40. Both diffuse lumpiness and discrete cysts may be After exclusion of malignancy and other causes that
tender and can fluctuate with the menstrual cycle. Cystic require specific treatment, multiple strategies for man-
masses should be aspirated; if the fluid is bloody or the cyst agement may be considered. Severe pain may warrant
is recurrent after prior aspiration, the fluid should be evalu- treatment, but often other measures are helpful, and
ated cytologically. Any palpable mass in a postmenopausal medications can be avoided in most cases. Reassurance, a
woman requires immediate evaluation to exclude malig- supportive bra, and discontinuation of an offending medi-
nancy without a period of observation. One of the most cation or supplement (if possible) are often sufficient.
common causes of litigation is failure to diagnose breast Taking essential fatty acids, vitamin E, and evening prim-
cancer. (See Health Care Guideline: Diagnosis of Breast rose oil or decreasing or eliminating the intake of salt and
Disease in the Suggested Reading list.) methylxanthines (eg, caffeine) may be helpful for some
women, although the evidence is conflicting, and in mul-
23. Answer a. tiple studies there was no benefit compared with placebo.
This is a classic history of cyclic breast pain associated with Multiple medications have been studied and are effective
the luteal phase of the menstrual cycle. Typically, women for the treatment of severe mastalgia, including danazol
describe burning, aching pain and a heavy sensation that (US Food and Drug Administration approved), bro-
is most often located in the upper, outer quadrants and mocriptine, nonsteroidal anti-inflammatory drugs, and
often radiates into the axillae and sometimes the upper tamoxifen. However, the side effects often cause patients

136 M AYO C L I N I C I N T E R N A L M E D I C I N E B OA R D R E VI EW: Q U E S T I O N S A N D A N S W E R S


to discontinue use, and medications are rarely necessary 29. Answer c.
with other measures. (See Smith et al in the Suggested
As discussed in the answer to question 28, the absolute risk
Reading list.)
reduction (ARR) is 5%. Now you are asked to determine
24. Answer b. the NNT in this randomized controlled trial. The formula
1 1
Cytomegalovirus is the most common cause of con- for NNT is . Therefore, NNT = = 20. That is,
genital viral infection. The lifetime risk of ovarian can- ARR 0.05
cer for a BRCA1 mutation carrier is approximately 40%. 20 elderly people with atrial fibrillation would need to be
Dysfunctional uterine bleeding is a diagnosis of exclusion; treated with anticoagulant therapy for 3 years to prevent
therefore, abnormal uterine bleeding must be evaluated 1 event.
before this benign diagnosis is made. Annual screening
mammograms have been shown to decrease mortality 30. Answer d.
related to breast cancer, but there are no such data for clini- The superficial femoral vein is in the deep venous system;
cal breast examinations. therefore, this patient needs to be treated with heparin
25. Answer c. and warfarin, overlapped for 5 days, until the international
normalized ratio is greater than 2.0. Outpatient treatment
A 22 table must be constructed to determine the posi- with LMWH has been approved by the US Food and Drug
tive predictive value for MRI among patients who have a Administration for the treatment of deep vein thrombosis.
pretest probability of 50% in an evaluation for foot osteo- The patient has a good support system at home; therefore,
myelitis. As determined from Table 15.A25, the correct it would be appropriate for her to be treated as an outpa-
a tient. Since she has no symptoms to suggest pulmonary
answer is or 90/105 = 86%.
a b embolism, CT of the chest is not indicated.

Table 15.A25
31. Answer d.
The patient has a clot in the popliteal system. This is con-
DISEASE DISEASE
PRESENT ABSENT
sidered a deep vein thrombosis (DVT) and does need anti-
coagulation. The patient is not at high risk of bleeding 1
Positive 90 15 105 week postoperatively; therefore, he does not need a vena
Diagnostic a b a+b caval filter. Dabigatran is not approved by the US Food and
Test Result c d c+d Drug Administration for DVT treatment. Three months
Negative
10 85 95 of treatment is adequate. The dosage of enoxaparin in the
outpatient setting is 1 mg/kg twice daily or 1.5 mg/kg once
a+c b+d a+b+c+d
daily; for this 80-kg patient, 120 mg subcutaneously once
100 100 200 daily would be appropriate.
26. Answer b. 32. Answer c.
d Quality improvement interventions have a hierarchy for
The negative predictive value for MRI is
or 85/95 = 89%. c d sustainability. Education tools, such as online modules,
conferences, pocket cards, and signs, tend to have a posi-
27. Answer d. tive initial effect, but this effect is not lasting. Although an
You are asked to determine the RRR in a random- institutional consensus or policy may be a good idea, it is
ized controlled trial that showed that the anticoagu- difficult to implement and develop consistent adherence.
lant therapy group had an experimental event rate In this question, the order set is most likely to provide a sus-
(EER) of 5%. The aspirin group had a control event rate tained improvement. If it is integrated into a computerized
CER EER order entry system, it will prompt physicians about correct
(CER) of 10%. The formula for RRR is . dosing and thus decrease the risk of human error.
CER
10% 5% 33. Answer b.
In this trial, RRR = = 50%.
10% This patient has episcleritis, which is a self-limited, possi-
bly autoimmune-mediated inflammation of the episcleral
28. Answer a.
vessels. Although vision is unaffected and tearing may be
You are asked to determine the ARR in a randomized con- minimal, rapid-onset redness associated with achy pain
trolled trial that showed that the anticoagulant therapy and tenderness is evident. In this scenario, one would
group had an experimental event rate (EER) of 5%. The also want to rule out the possibility of temporal arteritis,
aspirin group had a control event rate (CER) of 10%. The although it does not manifest with eye symptoms. If symp-
formula for ARR is CER EER. In this trial, ARR = 10% toms persist or recur, the patient should be referred to an
5% = 5% over 3 years. ophthalmologist.

15. C R O S S - C O N T E N T A R E A Q U E S T I O N S A N D A N S W E R S 137
34. Answer c. is a tool for classifying errors by severity and likelihood of
recurrence for use in prioritizing quality initiatives.
This patient likely has gonorrhea and gonococcal conjunc-
tivitis. Immediate ophthalmologic referral is advised for 39. Answer a.
treatment with topical antibiotics (bacitracin or ciproflox-
acin) and systemic antibiotics (ceftriaxone 1 g intramuscu- This patient has renal failure and a prolonged QT inter-
larly). Sexual symptoms and contacts should be reviewed val. Fentanyl does not affect the QT and can be used with
and treated appropriately. caution and at lower doses in renal failure. Methadone
prolongs the QT interval but can be used in dialysis.
35. Answer d. Propoxyphene, codeine, and meperidine should be
On the basis of the Centor criteria (fever, tonsillar exu- avoided because they have metabolites that can accumu-
dates, tender anterior cervical lymphadenopathy, and lack late in dialysis patients.
of cough), this patient has a high probability of Group 40. Answer e.
A -hemolytic streptococcal (GABHS) pharyngitis.
Guidelines from the Infectious Diseases Society of America From all appearances, the patient is decisionally capable.
recommend rapid streptococcal antigen testing for adults Answer choice a is not completely inappropriate. Given
with suspected GABHS infection (ie, 2 Centor criteria) that this is the first time the patient is confronting a horri-
and no testing or treatment for those at low risk (0 or 1 ble diagnosis, he may need to take some time to process the
Centor criterion). Patients at high risk (3 or 4 Centor cri- information and could change his mind. Answer choice b
teria) should receive empirical antibiotic therapy. points to a very important issuethe patients capacity in
light of his significant psychiatric history. Ultimately, psy-
36. Answer c. chiatric consultation might be an important step, but it is
Both acute angle-closure glaucoma and acute anterior probably not the best first response. Since the patient has
uveitis are emergent processes associated with rapid onset, just received horrible news, it might not be the best time
circumcorneal injection, pain, and unilateral manifesta- to remind him of his psychiatric issues. Rather, it is best to
tion. Acute angle-closure glaucoma is associated with a give him time to reflect and continue the conversation in
tense affected eye on palpation in an older patient; acute the near future. Answer choice d is not appropriate because
anterior uveitis usually manifests in a young or middle-aged such action would violate patient confidentiality unless
adult and is associated with the presence of inflammatory you received specific consent from the patient to make
cells and protein in the anterior chamber of the eye. This such contacts.
collection of pus (hypopyon) may occasionally be observed
41. Answer a.
in severe cases of acute anterior uveitis.
Granting a request to discontinue a form of life-sustaining
37. Answer e.
therapy is not the same as euthanasia or assisted suicide. In
You have already taken a good history and examined the both euthanasia and assisted suicide, the cause of death is
areas most likely to physically contribute to ED. The onset the lethal agent administered by the physician. With deac-
of symptoms was gradual in a patient who has excellent tivation of a pacemaker, the cause of death is the underlying
health and who likely has ED due to age-related changes. conduction defect or the pathologic disorder. Respect for
There is no need for MRI of the brain to rule out pituitary patient autonomyin this case, the right to be free from
disease since his testosterone level and testicular examina- unwanted medical interventionunderpins the right to
tion are normal. PDE-5 inhibitors are accepted as first-line refuse or to request the withdrawal of treatments no lon-
therapy for patients in this category. If his symptoms per- ger serving the patients goals of care. There is no ethical
sist, consultation with a sex psychotherapist would be or legal distinction between withholding a treatment and
reasonable to evaluate the possibility of anxiety related to withdrawing it after it has been initiated. Both are expres-
sexual performance or loss of sensate focus (mental distrac- sions of the patients desire to be left alone, free from
tion). (See Beckman et al in the Suggested Reading list.) unwanted touching.
38. Answer a. 42. Answer a.
LEAN is a quality improvement methodology developed It is not at all clear what the patient would want at this
by Toyota Motor Corporation to provide value by reducing point. The last directive from the patient was that he wanted
waste. In medicine, wait times are the most common form treatment. At the same time, the patient may indeed be in
of waste. Six Sigma is an improvement methodology devel- an irreversible situation in which eventual withdrawal of
oped by Motorola to reduce defects. A cause-and-effect use of the ventilator would be appropriate. The goals are to
diagram (also known as a fish bone diagram because of try to do what is best for the patient and to demonstrate a
its shape) is a tool to complete and organize a root cause good-faith commitment to work with the family. A reason-
analysis. Swim lane diagramming is a quality improvement able time limit on the aggressive treatment could accom-
tool used to visually show who is responsible for various plish both and allow the family members to resolve their
processes within a system. Failure mode and effects analysis differences.

138 M AYO C L I N I C I N T E R N A L M E D I C I N E B OA R D R E VI EW: Q U E S T I O N S A N D A N S W E R S


43. Answer d. 48. Answer a.
The patient is a minor; therefore, under presumed consent, This patient has a reduced functional status, and noninva-
life-sustaining interventions are to be pursued even in the sive stress testing preoperatively is appropriate, given that
face of parental refusal. Although surrogates can refuse she will undergo a high-risk surgical procedure. Her car-
life-sustaining therapy for adults for whatever reason, par- diac catheterization shows noncritical disease. Her stress
ents have not been allowed to refuse appropriate medical test results would be considered low-risk positive because
treatment, especially life-sustaining treatment, of minors she has only peri-infarct ischemia. Perioperative -blocker
for religious convictions. In such cases, physicians are obli- therapy is appropriate for her, and her goal heart rate
gated to obtain a court order appointing a guardian and should be 55 to 65 beats per minute. There is no role for
permitting the life-sustaining treatment. In this case, there preoperative CABG surgery since she has noncritical dis-
is not time to receive a court order before intervening, so ease. Perioperative nitrates are not indicated for a patient
it is most appropriate to pursue the lifesaving treatment who is not having angina.
while the legal issues are sorted out.
49. Answer b.
44. Answer a.
This patients Starr-Edwards valve presents a high risk
No further testing is required. Although this patient is at of thromboembolism. In addition, his surgical proce-
increased risk for coronary artery disease because of her dure confers a high risk of bleeding. Simply stopping the
history of hypertension and chronic kidney disease, she use of warfarin is inappropriate because of the throm-
does not have known coronary artery disease. Her hyper- boembolic risk. However, because of the bleeding risk,
tension is well controlled, and her heart rate is 55 beats per he should receive an agent that can be easily reversed
minute. American College of Cardiology and American either unfractionated heparin, which would require hos-
Heart Association guidelines suggest that patients who pitalization and would increase his risk of bleeding, or
take -blockers and have good heart rate control do not LMWH. The most appropriate choice in this situation,
need further noninvasive testing. because of cost and ease of administration, is LMWH.
His creatinine level is in the reference range, so there
45. Answer d.
is no contraindication to the use of LMWH. Aspirin
This patient has signs of mild to moderate aortic stenosis would not be sufficient to prevent thromboembolism in
and no symptoms. She has an excellent functional status this situation.
and shows no symptoms of aortic stenosis. Perioperative
50. Answer a.
-blocker therapy is not indicated for patients with aortic
stenosis. There is no reason to delay surgery to obtain an This patient has relatively well-compensated liver disease
echocardiogram or to do a functional stress test, since she and a low MELD score. A MELD score less than 10 is asso-
easily achieves 4 metabolic equivalent tasks of work and is ciated with a low risk of perioperative death. There is no
asymptomatic. Spinal anesthesia is relatively contraindi- role for TIPS in this situation.
cated in patients with aortic stenosis because of the risk of
induced hypotension, so the anesthesiologist must be told
of her condition. SUGGESTED RE ADING
46. Answer d. Beckman TJ, Abu-Lebdeh HS, Mynderse LA. Evaluation and medi-
cal management of erectile dysfunction. Mayo Clin Proc. 2006
This patient clearly has COPD, but his functional status is Mar;81(3):38590.
good. Pulmonary function tests are not indicated for pre- Branson BM, Handsfield HH, Lampe MA, Janssen RS, Taylor AW, Lyss
operative pulmonary assessment unless the patient is hav- SB, et al; Centers for Disease Control and Prevention. Revised rec-
ing new symptoms. The patients condition appears to be ommendations for HIV testing of adults, adolescents, and pregnant
women in health-care settings. MMWR Recomm Rep. 2006 Sep
stable, and he does not need any further testing. Although
22;55(RR-14):117.
he would certainly benefit from smoking cessation, there is Centers for Disease Control and Prevention. Epidemiology and Prevention
no reason to delay his surgery to accomplish this. of Vaccine-Preventable Diseases. 12th ed. Washington DC: Public
Health Foundation; c2012. Chapter 20, Tetanus; p. 291300.
47. Answer e. Centers for Disease Control and Prevention (CDC). Updated recom-
mendations for use of tetanus toxoid, reduced diphtheria toxoid and
Cataract surgery can be done safely if a patient is taking anti- acellular pertussis (Tdap) vaccine from the Advisory Committee on
platelet drugs, although there are few data for clopidogrel. Immunization Practices, 2010. MMWR Morb Mortal Wkly Rep.
Stopping use of either aspirin or clopidogrel would put the 2011 Jan 14;60(1):135.
patient at high risk of stent thrombosis in the time frame Centers for Disease Control and Prevention; Advisory Committee on
outlined. Deferring surgery would be an option, but it would Immunization Practices. Updated recommendations for prevention of
invasive pneumococcal disease among adults using the 23-valent pneu-
require 8 months of additional time to allow the patient to mococcal polysaccharide vaccine (PPSV23). MMWR Morb Mortal
receive both antiplatelet drugs for a full 12 months. At that Wkly Rep. 2010 Sep 3;59(34):11026.
time, clopidogrel therapy could be safely stopped for sur- Fiore AE, Uyeki TM, Broder K, Finelli L, Euler GL, Singleton JA,
gery, but aspirin therapy would be continued. et al; Centers for Disease Control and Prevention. Prevention and

15. C R O S S - C O N T E N T A R E A Q U E S T I O N S A N D A N S W E R S 139
control of influenza with vaccines: recommendations of the Advisory Pickering LK, Baker CJ, Freed GL, Gall SA, Grogg SE, Poland GA, et al;
Committee on Immunization Practices (ACIP), 2010. MMWR Infectious Diseases Society of America. Immunization programs for
Recomm Rep. 2010 Aug 6;59(RR-8):162. Errata in: MMWR infants, children, adolescents, and adults: clinical practice guide-
Recomm Rep. 2010 Aug 13;59(31):993. MMWR Recomm Rep. 2010 lines by the Infectious Diseases Society of America. Clin Infect Dis.
Sep 10;59(35):1147. 2009 Sep 15;49(6):81740. Erratum in: Clin Infect Dis. 2009 Nov
Health Care Guideline: Diagnosis of Breast Disease [Internet]. 1;49(9):1465.
Bloomington (MN): Institute for Clinical Systems Improvement; Screening for Breast Cancer, Topic Page [Internet]. Rockville (MD): U.S.
c2012. Available from: http://www.icsi.org/breast_disease_diagno- Preventive Services Task Force; c2010 [cited 2011 Mar 23]. Available
sis/diagnosis_of_breast_disease_2.html. from: http://www.uspreventiveservicestaskforce.org/uspstf/uspsbrca.
Hussain AN, Policarpio C, Vincent MT. Evaluating nipple discharge. htm.
Obstet Gynecol Surv. 2006 Apr;61(4):27883. Screening for Coronary Heart Disease With Electrocardiography, Topic
Laine C, Williams D, Wilson JF. Vaginitis and cervicitis. Ann Intern Med. Page [Internet]. Rockville (MD): U.S. Preventive Services Task Force;
2009 Sep;151(5):ITC31. c2012 [cited 2011 Mar 23]. Available from: http://www.uspreventi-
Levin B, Lieberman DA, McFarland B, Smith RA, Brooks D, Andrews veservicestaskforce.org/uspstf/uspsacad.htm.
KS, et al; American Cancer Society Colorectal Cancer Advisory Smith RL, Pruthi S, Fitzpatrick LA. Evaluation and management of breast
Group; US Multi-Society Task Force; American College of Radiology pain. Mayo Clin Proc. 2004 Mar;79(3):35372.
Colon Cancer Committee. Screening and surveillance for the early Using Nontraditional Risk Factors in Coronary Heart Disease Risk
detection of colorectal cancer and adenomatous polyps, 2008: a joint Assessment, Topic Page [Internet]. U.S. Preventive Services Task
guideline from the American Cancer Society, the US Multi-Society Force; c2009 [cited 2011 Mar 21]. Available from: http://www.uspre-
Task Force on Colorectal Cancer, and the American College of ventiveservicestaskforce.org/uspstf/uspscoronaryhd.htm.
Radiology. CA Cancer J Clin. 2008 May-Jun;58(3):13060. Epub Utian WH, Archer DF, Bachmann GA, Gallagher C, Grodstein F,
2008 Mar 5. Heiman JR, et al; North American Menopause Society. Estrogen and
Mindermann T, Wilson CB. Age-related and gender-related occurrence of progestogen use in postmenopausal women: July 2008 position state-
pituitary adenomas. Clin Endocrinol (Oxf ). 1994 Sep;41(3):35964. ment of The North American Menopause Society. Menopause. 2008
Erratum in: Clin Endocrinol (Oxf ). 1994 Nov;41(5):700. Jul-Aug;15(4 Pt 1):584602.
Nelson HD, Huffman LH, Fu R, Harris EL; U.S. Preventive Services Wright TC Jr, Massad LS, Dunton CJ, Spitzer M, Wilkinson
Task Force. Genetic risk assessment and BRCA mutation testing EJ, Solomon D; 2006 American Society for Colposcopy and
for breast and ovarian cancer susceptibility: systematic evidence Cervical Pathology-sponsored Consensus Conference. 2006
review for the U.S. Preventive Services Task Force. Ann Intern Med. Consensus guidelines for the management of women with abnor-
2005 Sep 6;143(5):36279. Erratum in: Ann Intern Med. 2005 Oct mal cervical cancer screening tests. Am J Obstet Gynecol. 2007
4;143(7):547. Oct;197(4):34655.

140 M AYO C L I N I C I N T E R N A L M E D I C I N E B OA R D R E VI EW: Q U E S T I O N S A N D A N S W E R S


INDE X

Page numbers followed by f indicate a figure; page numbers followed by t indicate a table

A allergies psoriatic arthritis, 62 body mass index (BMI), 15, 17, 40, 60,
abdomen answers, 107108 asbestosis, 42 6771, 68, 74
active bowel sounds, 45 questions, 105106 ascites, 30 bone and parathyroid disease
CT scan, 94, 97 allopurinol therapy, 61, 65, 100, 101 Aspergillus, 55 answers, 7677
distention, 30 Alzheimer disease, 124 aspirin, 14, 15, 27, 37, 68, 80, 98, 99, 100, questions, 7273
hyperactive bowel sounds, 70 amenorrhea, 71 101, 130 bradycardia, 18
tenderness (pain), 27, 29, 45, 70, 84, 109 amenorrhea, secondary, 135 assisted suicide, 138 BRCA1 mutation, 137
abdominal aortic aneurysm, 14, 16, 23 amitriptyline, 111 asthma, 118 BRCA2 mutation, 125, 135
abdominal hernia repair, 46 amlodipine, 99 during pregnancy, 105, 107 breast cancer, 50, 79, 81
abdominal radiotherapy, 29 amoxicillin, 55 ataxic dysarthria, 116 node-positive, 79
abdominal wall cellulitis, 50 amoxicillin-clavulanic acid, 53 atenolol, 14, 15, 99, 103 breast enlargement, male, 71, 7576
ABIM Maintenance of Certification amyopathic dermatomyositis, 120 atherosclerosis, 16 breast-feeding, 71
(MOC) Examination, 18, 4t anasarca, 100 atorvastatin, 15, 68, 69, 70, 71 breast masses, 127, 136
ABVD combination chemotherapy, 87 anemia of chronic disease, 8990 atrial fibrillation, 12, 18, 29, 72, 93, 113 breast pain, cyclic, 127, 136
acarbose, 74 anemias chronic, 86, 87 breath sounds, diminished, 37
acetaminophen, 17, 30, 34, 60, 65 answers, 8990 new-onset, 67 Bruce protocol stress test, 15
achalasia, 32 autoimmune hemolytic anemia, 89 atrial septal defect (ASD), 11, 19 budesonide, 31, 105, 107
Achilles tendonitis (enthesopathy), 61, 65 hemolytic anemia, 83, 103 atrial septum shunts, 11, 19 Buerger disease, 59, 60, 64
Achilles tendon swelling, 61 iron deficiency anemia, 28, 31, 32 atrioventricular block, first-degree, 130 bupropion, 111
acid-based disorders macrocytic anemia, 89 atypical squamous cells of undetermined
answers, 102 megaloblastic anemia, 89 significance (ASC-US), 125 C
questions, 9497 normochromic anemia, 59 autoimmune hemolytic anemia, 89 CA 125 elevation, 81
acute angle-closure glaucoma, 129, 138 questions, 8385 autoimmune hepatitis, 34 calcitriol therapy, 73
acute anterior uveitis, 129 sickle cell anemia, 47, 89 axillary adenopathy, 49 calcium oxalate arthropathy, 6566
acute chest syndrome, 89 warm autoimmune hemolytic anemia, 90 axillary lymph node metastases of adenoma calcium oxalate nephrolithiasis, 17
acute eosinophilic pneumonia, 42 angina, progressive, 15 of unknown origin, 81 calcium supplement, 70, 73, 76
acute interstitial nephritis, 103 angioedema, recurrent, 105 axillary mass, right, 79 Candida albicans, 52
acute ischemic cerebral infarction, 116 angioplasty, 72 azithromycin, 52, 118 cardiac murmur, 18f
acute ischemic stroke, 41 anion gap acidosis, 103 azoospermia, 44 cardiac physical examination
acute myelogenous leukemia (AML), 49, ankle edema, bilateral, 98 answers, 1820, 19f
55, 83, 89 ankle reflex, absence, 68 B questions, 1112
acute pancreatitis, 31 ankle swelling, 61 back pain cardiogenic syncope, 11, 18
acute renal failure ankylosing spondylitis, 61, 65 chest/neck radiation, 15 cardiology
answers, 102103 anorexia nervosa, 79 chronic musculoskeletal, low back, 87 answers, 1824
questions, 97 anterior myocardial infarction, 1415 lumbar spine point tenderness, 88 arrhythmia, clinical syndromes, 18
acute respiratory acidosis, 96 anthrax, inhalation, 53 new-onset, 95 cardiac physical exam, 1820, 19f
acute respiratory alkalosis, 96 antiarrhythmic therapy, 67 bariatric surgery, 75 congestive heart failure, 2021
acute respiratory distress syndrome, 42 anti-CD20 monoclonal antibody, 91 B-cell lymphoma, 88, 91 coronary artery disease, 2122
acute thrombosis, 90 anticoagulation therapy recommendation, 9 bee sting symptoms, 105 hypertension, 24
acute tubular necrosis, 103 antihistamines, 105 Bell palsy, 66 myocardial infarction, 2122
acyclovir, 49 nonsedating, 107 benign prostatic hypertrophy, 37, 71 vascular medicine, 2224
adalimumab, 65, 125 antimicrobial therapy, 45, 46 benzodiazepines, 111, 133 questions
adenocarcinoma antipsychotics, second-generation, 109, 111 -agonists, 107 arrhythmia, clinical syndromes, 911
rectal, 80 antirheumatic drugs -blockers, 21, 35, 77, 79, 139 cardiac physical exam, 1112
stage I lesion, 129 answers, 6465 -lactam antibiotics, 52 congestive heart failure, 1213
adenocarcinoma of unknown primary questions, 6061 bibasilar coarse rales, 36 coronary artery disease, 1415
origin, 79 apathetic hyperthyroidism, 77 bibasilar crackles, 35, 37 hypertension, 17
adult pulmonary Langerhans cell apical systolic murmur, 11 bilateral carotid bruits, 10 myocardial infarction, 1415
histiocytosis, 42 apnea. See obstructive sleep apnea bilateral expressible galactorrhea, 71 vascular medicine, 1517
Advisory Committee on Immunization arginine vasopressin (AVP) secretion, 76, bilateral hilar lymphadenopathy, 16, 41 cardiomegaly, 30
Practices, 134 102 bilateral pitting edema, 13 carotid stenting, 2324
AL amyloidosis, 102 arm/leg weakness, 84 bilateral pulmonary emboli, 16 carotid ultrasonography, 85
albuterol inhaler, 39, 80, 105, 107 arrhythmias and clinical syndromes biliopancreatic diversion, 75 carvedilol, 11, 12
alcoholic cirrhosis, 38 answers, 18 bioterrorism, 5253 caspofungin, 55
alendronate, 37 questions, 911 bipolar disorder, 109, 110, 129 cataracts, 61, 65
aliskiren, 17 arterial insufficiency, right second toe, 83 bismuth subsalicylate, 31 surgery, 131, 139
alkaline phosphatase elevation, 29 arthralgias, 32, 59, 62, 64 bisphosphonates, IV, 72, 77 ceftriaxone, 33, 46, 55
allergic bronchopulmonary aspergillosis, 42 arthritis, 17 biventricular pacing, 21 celiac disease, 32
allergic rhinitis, 106 acute monoarthritis, 62 bloating, postprandial, 29 Centers for Disease Control and Prevention,
allergic rhinoconjunctivitis, 107 inflammatory arthritis, 65 blood glucose monitoring, 67 134

141
central hypothyroidism, 74 common variable immunodeficiency disease D lower extremity
central nervous system (CVID), 107 dabigatran, 86, 87, 90, 137 bilateral, 60, 67
cryptococcosis, 54 community-acquired pneumonia, 7, 37, 52 daptomycin, 56 bilateral ankle, 98
central venous catheter-related bloodstream computed tomography (CT), 12 daunorubicin, 49 bilateral pitting, 12
infections (CRBSIs), 50 colorectal cancer, 82 death, patient acceptance, 129, 138 leg edema, 30
cephalexin, 53 diverticulosis without abscess, 27 decongestants, topical, 107 pitting, thigh, 100
cephalosporin (third-generation), 33 familial pheochromocytoma, 24 deep vein thrombosis (DVT), 16, 23, 137 pitting, pretibial, 99
cerebellar disorder, 116 myasthenia gravis, 115 delirium, 35, 41, 96, 133 pretibial, 37
cervical adenopathy, 49 retroperitoneal/mesenteric dementia, multi-infarct, 93 periorbital edema, 120
cervical lymphadenopathy, 48, 87 lymphadenopathy, 88 dementia with Lewy bodies, 133 peritumoral edema, 81
chemical gastritis, 28 right axillary adenopathy, 79 depression, 109, 114 pulmonary edema, 13
chemotherapy sinuses, 106 dermatitis herpetiformis, 119f, 121 trace, peripheral, 14
ABVD combination, 87 confusion, 93, 95 dermatology ejection murmur, harsh, 10
for breast cancer, 50 congestive heart failure (CHF), 18, 67, answers, 120121 electrocardiogram (ECG), 9f, 10f, 11, 14
febrile neutropenia from, 81 80, 109 questions, 117119 electrolyte disorders
R-CHOP chemotherapy, 91 answers, 2021 dermatomyositis, 118f, 120 answers, 102
for rectal cancer, stage II, 80 questions, 1213 dermatomyositis sine myositis, 120 questions, 9394
chest pain constrictive pericarditis, 12, 20 dermopathy, 74 embolism
anterior, 72 continuous positive airway pressure (CPAP) descending aortic dissection, 23 answers, 43
atypical, 19 therapy, 70, 94 deviated nasal septum, 46 questions, 3839
exertional, 11, 14, 15, 36 contrast-induced acute kidney injury (AKI), dexamethasone, 5657, 81 enalapril, 12, 50, 80
at rest, 11 104 diabetes insipidus, postoperative, 71, 76 endocarditis, 105
retrosternal, 14 contrast nephropathy, 22 diabetes mellitus, 13, 15, 16, 22, 24, 47 endocrine tumors, 32
right-sided, 36 coronary angiogram, 12, 14, 15, 72, 87 answers, 7475 endocrinology
substernal, 35 coronary artery bypass graft surgery, 12, 100 questions, 6769 answers
chest radiography coronary artery disease, 50, 68, 79, 80, remission rates, 75 bone, parathyroid disease, 7677
dense upper left consolidation, 45 98, 100 type 2, 9, 45, 50, 80, 98, 109, 125 diabetes mellitus, 7475
diffuse pulmonary infiltrates, widened answers, 2122 diabetic kidney disease (DKD), 100, 103 hyperlipidemia, 7475
mediastinum, 46 questions, 1415 diabetic nephropathy, 98 hypoglycemia, 7475
questions, 3940 coronary bypass surgery, 68 diabetic neuropathy, 99 obesity, nutrition, 75
choking episodes, 113 coronary heart disease, with QT diaphoresis, 69 pituitary, gonadal, adrenal disorders,
cholecystectomy, elective, 84 prolongation, 109, 111 diaphragmatic calcification, 37 7576
chronic hepatitis C, 30 corticosteroids, 65, 107, 118 diarrhea, 27, 28, 29, 32, 45 thyroid disease, 74
chronic interstitial nephritis, 98 cough, 79 diclofenac-misoprostol, 61 questions
chronic kidney disease (CKD), 93, 94, dry, 36 dicloxacillin, 53 bone, parathyroid disease, 7273
101, 130 mild, 16 diffuse large cell non-Hodgkin lymphoma, diabetes mellitus, 6769
stage 3, 99, 100, 103 nonproductive, 37, 87 130 hypoglycemia, 6769
stage 4, 103 productive, 45, 106 diffuse lung disease obesity, nutrition, 7071
treatment recommendations, 104 recurrent, 129 answers, 4143 pituitary, gonadal, adrenal disorders,
chronic lymphocytic leukemia (CLL), 90 severe, 16 questions, 3638 7172
chronic obstructive pulmonary disease CPAP (continuous positive airway pressure) digital clubbing, 36 thyroid disease, 67
(COPD), 39, 40, 43, 79, 80, 96, therapy, 70, 94 digoxin, 9, 11 endometrial cancer, 79
114, 118, 131 CRB-65 score, 52 dilated cardiomyopathy, 11, 2021 endotracheal intubation, 37, 46
chronic renal failure, 47 C-reactive protein, 47 dilated inferior vena cava, 38 end-stage renal disease (ESRD), 101
answers, 103104 CREST syndrome, 62, 66 diltiazem, extended-release, 101 Entamoeba histolytica, 55
questions, 99101 critical care medicine distal renal tubular acidosis (RTA), 95, 102 enzyme-linked immunosorbent assay
chronic respiratory acidosis, 96 answers, 41 distal tubular diuretics, 102. See also (ELISA), 48
chronic respiratory alkalosis, 96, 103 questions, 3536 metolazone epilepsy, temporal lobe, 116
Churg-Strauss syndrome, 42, 60, 64 Crohn disease, 29 diuretics, 79 episcleritis, 31, 137
Chvostek sign, 73 cross-content areas diverticulitis, 27, 31 Epstein-Barr virus, 48, 97
ciprofloxacin, 33, 50, 52, 55 answers dizziness, 109, 113 erectile dysfunction, 71
circumcision, 55 general internal medicine, 137138 DNA-based testing, 90 gradual-onset, 129, 138
cirrhosis, 30, 33, 38 geriatrics, 133 donepezil, 133 erythema chronicum migrans, 66
claudication medical ethics, palliative care, 138139 Doppler ultrasonography, 30, 99 erythema multiforme, 117f, 120
intermittent, 15, 16, 22 perioperative medicine, 139 doxazosin, 17 erythematous nodosum, 41, 46
pseudoclaudication, 16, 22 preventive medicine, 133134 doxorubicin, 79 erythematous sclera, 128129
clindamycin, 53 womens health, 134137 doxycycline, 55 erythromycin, 53
clonal thrombocytosis, 89 questions dronedarone, 11, 18 Escherichia coli, 32, 55
clopidogrel, 14, 15 general internal medicine, 127129 drug-eluting stents, 131 esophagogastroduodenoscopy (EGD), 28,
Clostridium difficile infection, 4950, 5556 geriatrics, 123124 drug-induced lung disease, 42 30, 31
clumsiness, hands, 113 medical ethics, palliative care, 129130 D-shaped left ventricle, 38 esophagus and stomach
coagulase-negative staphylococci, 56 perioperative medicine, 130132 dual energy x-ray absorptiometry (DEXA), answers, 3132
coagulation preventive medicine, 124125 72 questions, 28
answers, 90 womens health, 125127 duodenal switch bypass, 75 essential thrombocytopenia, 83
questions, 8587 cryoglobulinemia, 103 Duplex ultrasonography, 16 etanercept, 60, 65, 66
cocaine use, 98, 103 cryoglobulinemic vasculitis, 66 dysphasia, 93 ethambutol, 47, 53
codeine, 129, 138 cryptogenic organizing pneumonia, 42 dysplasia, low-grade, 27 euthanasia, 138
colitis, microscopic, 31 crystalloid, IV, 35 dyspnea, 12, 13, 37, 40, 45, 59, 62, 72, 87 exertional dyspnea (NYHA functional class
colon and pancreas CURB-65 score, 52 acute onset, 16, 35, 36 III), 11
answers, 31 Cushing syndrome, 75 exertional, 11, 83 exophthalmos, bilateral, 67
questions, 27 cutaneous leukocytoclastic vasculitis, 62 progressive, 39, 84
colon cancer cyclophosphamide chemotherapy, 7980 dysproteinemia, 90 F
family history, 134 cystic fibrosis, 44 familial amyloidosis, 20
recurrence pattern, 82 cystic fundic gland polyps, 28 E familial hypocalciuric hypercalcemia
screening, 27, 79 cystitis, 56 early-stage (I-IIA) nonbulky disease, 91 (FHH), 73, 77
colonoscopy, 27, 28, 82 cytarabine, 49 edema fatigue, 30
colon polyps, 27, 87 cytomegalovirus, 48, 97, 137 intermittent peripheral, 87 febrile neutropenia, 81

142 INDEX
fecal leucocytes, 29 gynecologic examination, 125 hunger, excessive, 69 questions
femoral-popliteal bypass surgery, 98 gynecomastia, 71, 7576 hungry bone syndrome, 77 bone, joint infections, 4647
femoral-popliteal deep vein thrombosis hydralazine, 21 HIV infection, 4849
(DVT), 16 H hydration, IV fluid, 71 joint infections, 4647
femoral, popliteal, posterior pulses, 15, 16 H2 receptor blocker, 32 hydrochloroquine, 123 mycobacterial infections, 4647
fenofibrate, 70 Haemophilus influenzae, 52, 56, 106 hydrochlorothiazide (HCTZ), 15, 17, 63, pneumonia, zoonoses, travel,
fentanyl, 138 haloperidol, 133 80, 93, 102 bioterrorism, 4546
fentanyl patient-controlled analgesia, 98 hands hydrochlorothiazide-triamterene, 68, 69, 70 skin, soft tissue infections, 4647
fevers, low-grade, 59 hand clumsiness, 35, 113 hydrocortisone, 7172 syndromes
fexofenadine, 72 rheumatoid nodules, 60 hydroxychloroquine, 61, 63, 65 cardiovascular, bloodstream, CNS,
fibromyalgia-like symptoms, 64 synovitis, proximal interphalangeal hyperbilirubinemia, indirect, 33 5051
fibrotic lung disease, 442 joints, 60 hypergammaglobulinemia, 33 gastrointestinal tract infection,
finger telangiectases, 37, 43 Hashimoto thyroiditis, 67 hyperlipidemia, 12, 14, 15, 16, 27, 29, 35, 4950
fish oil capsules, 48, 68, 69, 71, 75, 101 hCG-producing tumor, 7576 61, 70, 100 sexually transmitted diseases, 4950
fluconazole, 55 headache answers, 7475 urinary tract infection, 4950
fluoroquinolones, 55, 56 migraine, 114, 116, 118 fish oil treatment, 48 infective endocarditis, 50
fluticasone propionate, 44, 80 new-onset, 59 questions, 6769 inferior vena cava (IVC) dilation, 38
folic acid supplement, 60, 61, 71, 85 post-partum, 71 hypernatremia, 94, 102 inferior vena cava (IVC) filter replacement,
foraminotomy, 123 heart hyperparathyroidism, 73, 76 16, 23
fosinopril, 70 bibasilar crackles, 35, 37 hyperplastic polyps, 31 inflammatory arthritis, 65
4-methylpyrazole therapy, 102, 103 cardiac murmur, 18f hypertension, 12, 13, 14, 15, 16, 27, 35, 37, inflammatory bowel disease, 31
furosemide, 11, 12, 35, 94, 99, 102 first sound, normal, 11 40, 60, 61, 67, 70, 80, 100 inflammatory synovial fluid, 54
fourth sound, 12, 13 answers, 24, 43 infliximab, 60, 65
G holosystolic murmur, 99 questions, 17, 3839 influenza vaccination, 124
gabapentin, 99 second sound, reverse splitting, 10 with type 2 diabetes, 47 infrapopliteal arterial occlusive disease, 24
gait ataxia, 113, 116 systolic ejection murmur, 68, 99 hyperthyroidism inguinal adenopathy, 49
galactorrhea, 71, 135 systolic murmur, 35 apathetic, 77 inguinal hernia, 29
gallstones, 31, 89 third sound, intermittent, 12 iodine-induced, 77 inhalation anthrax, 53
gastric banding, laparoscopic adjustable, 75 heart block, third-degree, 50 hypertrophic cardiomyopathy (HCM), insomnia, 109
gastric cardia malignancy, 32 heartburn, intermittent, 28 21, 41 insulin, 15, 98
gastric MALT lymphoma, 91 heart failure hyperuricemia, 61 intercourse pain, 109, 126
gastroenterology and hepatology due to biventricular dysfunction, 12 hypocalcemia, 72, 7677 intermittent claudication, 16
answers with preserved ejection fraction, 20 hypochondriasis, 111 invasive dental procedures, 56
colon and pancreas, 31 heart sounds hypogammaglobulinemia, 107 iodine-induced hyperthyroidism, 77
esophagus and stomach, 3132 Helicobacter pylori, 28, 31, 91 hypoglycemia ionized calcium elevation, 72
liver, 3334 hematology answers, 7475 ipratropium, 39
small bowel and intestine, 3233 answers questions, 6769 ipratropium bromide inhaler, 93
questions anemias, myeloid malignancies, 8990 hypogonadism, 67, 74 iron deficiency anemia, 28, 31, 32
colon and pancreas, 27 coagulation, 90 hypokalemia, 94 irritable bowel syndrome (IBS), 29, 3233
esophagus and stomach, 28 lymphoid malignancies, 9091 hyponatremia, 45, 93, 102 isolated central nervous system (CNS)
liver, 2930 questions hypo-osmolar hyponatremia, 102 vasculitis, 60, 64
small bowel and intestine, 2829 anemias, myeloid malignancies, 8385 hypotension, 35, 37 isoniazid, 47, 53
gastroesophageal reflux disease (GERD), coagulation, 8587 orthostatic, 115 isosorbide dinitrate, 21
3132, 37 lymphoid malignancies, 8788 postpartum hemorrhage-induced, 76
gastrointestinal tract (upper) bleeding, 30 hemoglobinopathies, high-oxygen affinity, 89 hypothyroidism, 30, 67, 74, 102 J
general internal medicine hemolytic anemia, 83, 103 hypoxemia, 37 jaundice, 29
answers, 137138 heparin, 90. See also low-molecular-weight jugular venous distention, 13
questions, 127129 heparin I jugular venous pressure ( JVP), 12, 20, 38
gentamicin, 56 dosing consensus, 128 iatrogenic thyrotoxicosis, 74
geriatrics intravenous, 14 ibuprofen, 30, 101 K
answers, 133 subcutaneous, 86 idiopathic bronchiolitis obliterans with kidney stones, recurrent, 73
questions, 123124 hepatic cytochrome P450 enzymes, 53 organizing pneumonia (BOOP), knees
giant cell arteritis (GCA), 59, 60, 64 hepatitis, autoimmune, 34 42 acute monoarthritis, 62
Gilbert syndrome, 33 hepatitis A infection, 135 idiopathic dilated cardiomyopathy, 21 arthroscopic surgery, 98
glaucoma, acute angle-closure, 129, 138 hepatitis B infection, 30, 34 idiopathic pulmonary fibrosis (IPF), 42, 44 bilateral intermittent effusions, 62
glimepiride, 68, 69 hepatitis C infection, 30, 48, 62, 66, 97 IgA nephropathy, 98, 103 joint swelling, 47
glipizide, 70, 80 asymptomatic, 103 IgE-mediated reaction to penicillin, 107 left knee pain, 59
glomerular disease hepatocellular carcinoma (HCC), 34 ileal resection, 29, 33 left, total arthroplasty, 130
answers, 103 hepatology. See gastroenterology and immune-mediated heparin-induced mild discomfort, 61
questions, 9799 hepatology thrombocytopenia type II, 90 osteoarthritis, 60
glucosamine chondroitin sulfate, 60, 65 hepatopulmonary syndrome, 44 immunosuppressive therapy, 34 sudden onset pain, swelling, 62
gluten-sensitive enteropathy, 121 hereditary hemorrhagic telangiectasia, 44 implantable cardioverter-defibrillator, 11, 12 synovitis, 85
glyburide, 37 hereditary nonpolyposis colorectal cancer infectious diseases total knee replacement, 85
glycoprotein IIb/IIIa inhibitors, 21 (Lynch syndrome), 81 answers
goiter, multinodular, 77 herpes simplex encephalitis, 57 bone, joint infections, 5354 L
gonococcal arthritis, 54 Hickman catheter tunnel, 50 HIV infection, 5455 lamivudine, 34, 54
gonococcal conjunctivitis, 138 hip arthroplasty (total), 17 joint infections, 5354 large joint monoarticular arthritis, 54
gonorrhea, 56, 138 hip fracture, 87, 131 pneumonia, zoonoses, travel, LEAN quality improvement methodology,
Gottron papules, 120 histoplasmosis, 53 bioterrorism, 5253 138
gout, 6162, 65, 101 HIV infection, 47, 4849, 5455, 97 skin, soft tissue infections, 5354 left bundle branch block (LBBB), 11, 18
pseudogout, 66 hoarseness, 28 syndromes left ventricular assist device (LVAD), 12
granulomatous disease, 41 Hodgkin lymphoma therapy, 90 cardiovascular, bloodstream, CNS, left ventricular ejection fraction (LVEF), 11,
Graves disease, 74, 77 honeycombing (lungs), 37 5657 12, 18, 20
Graves ophthalmopathy, 74 hopelessness, 109 gastrointestinal tract infection, left ventricular failure, 41
group A -hemolytic streptococcal hospital-acquired pneumonia, 46 5556 left ventricular hypertrophy, 13
(GABHS) pharyngitis, 138 hot flushes, 126, 135 sexually transmitted diseases, 5556 leg/arm weakness, 84
Guillain-Barr syndrome (GBS), 115116 HPV vaccine, 135 urinary tract infection, 5556 leg edema, 30

INDEX 143
Legionella pneumophila, 37, 52 metformin, 15, 29, 33, 67, 68, 69, 70, 74, 99, acute renal failure, 97 answers, 6465
leukocytosis, 45 100, 101, 130 chronic renal failure, 99101 questions, 6061
leukotriene receptor blockers, 107 methicillin-resistant Staphylococcus aureus electrolyte disorders, 9394 outpatient parenteral antibiotic therapy
leuprolide, 97 (MRSA), 47, 51, 53, 56 glomerular disease, 9799 (OPAT), 4647, 53
levofloxacin, 46 methimazole, 74 neuroleptic agents, 133 ovarian cancer, 79
levothyroxine, 67, 74 methotrexate, 60, 61, 85 neuroleptic malignant syndrome, 109, 111 oxygen therapy, 44
light-headedness, 95 methylprednisolone, 62, 123 neurology oxymetazoline, 107
limb dysmetria, 116 metolazone, 94, 102 answers, 115116
lip telangiectases, 37, 43 metoprolol, 9, 14, 15, 71, 98, 99, 101, 103, questions, 113114 P
lisinopril, 11, 14, 15, 17, 27, 37, 67, 68, 69, 130 new-onset headache, 59 pancreatitis, acute, 31
123, 130 microhematuria, asymptomatic, 97 niacin, 29 pansystolic murmur, 13
lisinopril-hydrochlorothiazide, 97 microscopic colitis, 31 nicotine dependence, 67 paradoxical embolism, 44
lithium carbonate, 109, 111 midsystolic click, 11, 19 nifedipine, 61 paranoid beliefs, 109
liver migrant headache, 114, 116, 118 night sweats, 135 parathyroid adenoma, 73
answers, 3334 minimal change nephropathy, 98 nitrofurantoin, 37, 55, 56 parathyroidectomy, 73
questions, 2930 Mini-Mental State Examination (MMSE), 123 nitrofurantoin lung toxicity, 42 parathyroid hormone (PTH), 73, 77
liver Doppler ultrasonography, 30 mitral regurgitant murmur, 19, 19f nitroglycerin, 14, 35 parenteral antihypertensive drug choice, 17
liver lobe abscess, 55 mitral valve disease, 29 nocturnal continuous positive airway paresthesias of the hands, 68
lobar pneumonia, 48 mitral valve prolapse, 11, 19 pressure, 94 Parkinson disease, 124, 133
Lfgren syndrome, 4142 mitral valve replacement, 51 node-positive breast cancer, 79 paroxysmal nocturnal dyspnea, 12
loop diuretics, 102. See also furosemide Model for End-Stage Liver Disease nodular sclerosing Hodgkin lymphoma, 87 parvovirus B 19, 97
loose stools, 27 (MELD), 132 nonalcoholic steatohepatitis, 38 Pasteurella multocida, 53
loperamide hydrochloride, 31 monoamine (histamine-tyramine) nonarticular rheumatism, vasculitis patent foramen ovale (PFO), 11, 19
losartan, 17, 70, 100, 101 poisoning, 53 answers, 64 PCP prophylaxis, 55
lovastatin, 80 monoarthritis, acute, 62 questions, 5960 PDE-5 inhibitors, 138
low-molecular-weight heparin (LMWH), monoclonal gammopathy of undetermined non-Hodgkin lymphoma, 30, 130 pelvic radiotherapy, 81
128, 131, 137, 139 significance (MGUS), 90 nonishemic dilated cardiomyopathy, 11 penicillin, 105
low-voltage QRS complexes, in the limb motor vehicle collision, 36 norfloxacin (oral), 33 IgE-mediated reaction to, 107
leads, 87 MRSA prosthetic valve endocarditis, 56 normochromic anemia, 59 percutaneous coronary intervention (PCI),
lumbar spine tenderness, 59 mucormycosis, 55 North American Symptomatic Carotid 1415, 22
lung cancer, 102 multi-infarct dementia, 93 Endarterectomy Trial pericardium myopathy, 11
lung contusions, bilateral, 35 multinodular goiter, 77 (NASCET), 23 perioperative medicine
lungs multiple endocrine neoplasia (MEN), type nortriptyline, 123 answers, 139
dense upper left consolidation, 45 2, 76 NPH insulin, 101 questions, 130132
diffuse bilateral crackles, 45, 46, 98 multiple myeloma, 91, 102 nucleoside, oral/nucleoside analogue, 34 periorbital edema, 120
inspiratory crackles, 45 multiple sclerosis (MS), 60 nucleoside reverse transcriptase inhibitors peripheral artery disease (PAD), 22
right midlung field, 84 multivitamin supplement, 67, 70, 71, 72, (NRTIs), 54 peripheral neuropathy, 20, 68, 98
lupus anticoagulant (LAC), 86, 90 80, 97, 101 nutrition peripheral neuropathy Lupus-like syndrome,
Lyme disease, 66 muscle weakness, 115 answers, 75 53
lymph node biopsy, 79 diffuse, 9495 questions, 7071 peripheral vascular disease, 47, 131
lymphocytic hypophysitis, 76 slowly progressive, 63 nutritional supplementation peritumoral edema, 81
lymphocytic thyroiditis, 77 myalgias, 59, 64 calcium, 70, 73, 76 periumbilical discomfort, 29
lymphocytosis, 87 diffuse, 98 fish oil capsules, 48, 68, 69, 71, 75, 101 pes anserine bursitis, 64
lymphoid malignancies myasthenia gravis, 115 folic acid, 60, 61, 71, 85 pheochromocytoma, 17, 76
answers, 9091 Mycobacterium tuberculosis, 47 multivitamin, 67, 70, 71, 72, 80, 97, 101 Philadelphia chromosome-negative chronic
questions, 8788 myelodysplastic syndrome (MDS), 83, 89 vitamin B12, 70, 75 myeloid leukemia (CML), 83, 89
Lynch syndrome (hereditary nonpolyposis myeloid malignancies vitamin D, 70, 72, 76 physiologic sleep, 36
colorectal cancer), 81 answers, 8990 vitamin E, 71, 101 pigment gallstones, 89
questions, 8385 pituitary, gonadal, adrenal disorders
M myeloproliferative neoplasm, 89 O answers, 7576
macrocytic anemia, 89 myocardial contusion, post-trauma, 41 obesity, 29, 68, 70, 100, 101. See also body questions, 7172
magnetic resonance imaging (MRI) myocardial infarction, 123, 125, 130 mass index pituitary tumors, 71, 76
arterial distribution infarctions, 60 answers, 2122 obesity and nutrition plantar ulcer, 47
breast cancer evaluation, 81 questions, 1415 answers, 75 plasma cell dyscrasia, 102. See also AL
diabetic foot infection, 127 questions, 7071 amyloidosis; multiple myeloma
head: sellar mass with suprasellar N obsessive-compulsive disorder, 111 plasma cell proliferative disorder, 91
extension, 71 nabumetone, 60 obstructive sleep apnea (OSA), 44, 70, 94 plethoric face, 84
left hippocampus atrophy, 114 N-acetylcysteine, 44 occupational lung disease pleural effusion
pituitary microadenoma, 71, 76 naproxen, 61, 70 answers, 4143 left-sided, 37
MALT lymphoma, 8788, 91 nasal passages questions, 3638 right-sided, 37
mammography recommendations, 134 congestion, 105 oculomasticatory myorhythmia, 28 pleural thickening, 37
medical ethics, palliative care hyperemic, 46 olanzapine, 109 pneumococcal pneumonia, 54, 56
answers, 138139 nasal regurgitation of liquids, 113 oliguric acute renal failure, 98, 103 pneumococcal vaccination, 124, 134
questions, 129130 nasal spray, saline, 107 omega-3 fatty acids, 75 pneumocystis pneumonia (PCP), 49
medullary thyroid carcinoma, 72 necrotizing fascitis, type 1, 53 omeprazole, 37, 99 pneumonia
megaloblastic anemia, 89 Neisseria gonorrhoeae, 107 oncology acute, 96
membranoproliferative glomerulonephritis, Neisseria gonorrhoeae infection, 54, 105 answers, 8182 acute eosinophilic, 42
103 Neisseria meningitidis infection, 105 questions, 7980 answers, 5253
membranous nephropathy, 98 nephrology onycholysis, 37 community-acquired, 7, 37, 52
memory problems, 123 answers open wound/foul-smelling pus, 47 cryptogenic organizing, 42
meperidine, 138 acid-based disorders, 102 oral contraceptives, 74, 98 hospital-acquired, 46
meropenem, 49 acute renal failure, 102103 orchiectomy, 82 lobar pneumonia, 48
mesalamine, 27 chronic renal failure, 103104 orlistat, 74 organizing, with idiopathic bronchiolitis
mesenteric ischemia, 33 electrolyte disorders, 102 orthopedic injury repairs, 35 obliterans, 42
mesenteric lymphadenopathy, 88 glomerular disease, 103 orthopnea, 12 pneumococcal, 54, 56
metabolic acidosis, 96 questions orthostatic hypotension, 115 pneumocystis pneumonia, 49
metabolic alkalosis, 96 acid-based disorders, 9497 osteoarthritis, rheumatoid arthritis questions, 4546

144 INDEX
recurrent, 37 answers, 43 selective serotonin reuptake inhibitors syphilis, IgM and IgG, 50
severe, bilateral, 130 questions, 3839 (SSRIs), 111 syphilis, secondary, 56
ventilator-associated, 45 pulmonary hypertension, 66 senile cardiac amyloidosis, 9091 systemic lupus erythematosus (SLE), 63, 85,
polyarthritis, 41 pulmonary tuberculosis, 47 septoplasty, 46 86, 8990
polycystic ovary syndrome, 67 purified protein derivative (PPD) skin test, seropositive rheumatoid arthritis, 61 systolic click, 11, 12, 19
polycythemia, 89 47, 53, 55 serum brain natriuretic peptide (BNP) systolic click murmur, 20
polymyalgia rheumatica (PMR), 59 purpura, legs and feet, 97 levels, 12, 20 systolic ejection murmur, 13
polymyositis, 63 pyelonephritis, 56 Sheehan syndrome, 76
polysomnography, overnight, 44 pyoderma gangrenosum, 119f, 120121 shock, severe, 37 T
polysubstance abuse, 95, 96 pyrazinamide, 47, 53 shunts in the atrial septum, 11 tachycardia, 70
popliteal system clot, 128, 137 pyridoxine, 47 sickle cell anemia, 47, 89 tadalafil, 71
portal hypertension, 33 sickle cell crisis, 54 Takayasu arteritis, 59, 64
post-exertion syncope, 12 Q sickle cell disease, 84 tamsulosin, 71
postpartum hemorrhage-induced quality improvement interventions, 137, 138 sigmoid polyp, 27 tardive dyskinesia, 111
hypotension, 76 QuantiFERON-TB Gold test, 47, 53 sildenafil, 101 tea-colored urine, 97
postprandial bloating, 29 quinapril, 68 simvastatin, 12, 14, 15, 27, 33, 61, 63, 67, 68, temporal arteritis, 137
potassium, IV, 102 98, 99, 100, 101, 130 temporal lobe epilepsy, 116
prasugrel, 2122 R sinopulmonary infection, 106, 107 tenofovir, 54
pravastatin, 37, 48 radial pulse diminishment, 15 sinus bradycardia, 11 testosterone gel, 67
prazosin, 37 rash sinusitis, recurrent, 40, 46 tetanus-diphtheria (Td) vaccination, 46,
prednisone, 60, 63, 64, 85 heliotrope, 120 sinus tachycardia, 35 124, 133, 134
premature coronary artery disease, 22 lower extremities, 97 Six Sigma quality improvement method, 138 thiamine deficiency, acute, 75
premature gallstones, 89 pruritic hives, 105 Sjgren syndrome, 9495 3-vessel disease, 22
premature ventricular complexes (PVCs), 13 pruritic rash, 14, 87, 119 skin dryness, lower extremities, 68 thrombocytopenia, 90
presumed consent, for minors, 139 R-CHOP chemotherapy, 91 skin hyperpigmentation, 28 thrombocytosis, extreme, 89
pretibial edema, 37 rectal cancer, stage II, 80, 81 sleep/sleepiness thrombotic thrombocytopenic purpura
preventive medicine renal biopsy, 97, 101 disrupted, fragmented, 41 (TTP), 90
answers, 133134 renal cell carcinoma, 99, 101, 131 excessive, 36 thrombus
questions, 124125 renal thrombotic microangiopathy, 103 insomnia, 109 left superficial femoral vein, 128
primary biliary cirrhosis (PBC), 33 renal ultrasonography, 99 obstructive sleep apnea, 44, 70, 94 right anterior tibial vein, 128
primary hypothyroidism, 67 respiratory illness, upper, 129 small bowel and intestine thrush, 49, 55
primary myelofibrosis (PMF), 83 retrobulbar neuritis, 53 answers, 3233 thyroid disease
procainamide, 9 retroperitoneal lymphadenopathy, 88 questions, 2829 answers, 74
prolonged QT interval, 138 rhabdomyolysis, 103 small-molecule intoxication, 103 questions, 67
propoxyphene, 138 rheumatoid arthritis, 85 smoking/smoking cessation, 15, 79 thyroid lobe nodule, 72
propylthiouracil (PTU), 67, 74 answers, 6465 snoring, 40 thyrotoxicosis, 74, 77
prostate cancer, 81 questions, 6061 socioeconomic issues, 133 tiotropium, 39, 44
metastatic, 97 rheumatology somatization disorder, 111 tissue plasminogen activator (tPA), 35
prostatectomy, radical, 35, 82 answers, 6466 sotalol, 11, 18 TMP-SMX prophylaxis, 55, 56
prostate enlargement, 80 antirheumatic drugs, 6465 speech difficulties, 113 tobacco use, 16
prosthetic hip joint, 47 miscellaneous conditions, 66 spironolactone, 11, 12, 21 Todd paralysis, 116
proteinuria, 60, 103 nonarticular rheumatism, vasculitis, 64 splenomegaly, borderline, 30 toe (right second), arterial insufficiency, 83
proton pump inhibitor (PPI) therapy, 28, osteoarthritis, rheumatoid arthritis, spondyloarthropathies tonic-clonic seizure, 86
3132, 37 6465 answers, 65 topiramate, 116
proximal renal tubular acidosis (RTA), 102 spondyloarthropathies, 65 questions, 6162 tramadol, 97, 123
pruritic hives, 105 questions, 5963 sputum, progressively purulent, 45 transfusion-related acute lung injury
pruritic rash, 14, 87 antirheumatic drugs, 6061 squamous cell carcinoma, 79 (TRALI), 36, 41
pseudoachalasia, 32 miscellaneous conditions, 6263 staphylococcal toxic shock syndrome, 53 transient ischemic attack (TIA), 16
pseudogout, 66 nonarticular rheumatism, vasculitis, Staphylococcus aureus, 52, 55, 56 transient ischemic attack (TIA)-like attacks,
Pseudomonas aeruginosa, 52 5960 Starr-Edwards mitral valve, 131, 139 60
psoriasis vulgaris, 118f, 120 osteoarthritis, rheumatoid arthritis, statins. See individual statins transsphenoidal microadenomectomy, 71
psoriatic arthritis, 62 6061 stavudine-tenofovir-lamivudine-darunavir transthoracic echocardiogram, 9
psychiatry spondyloarthropathies, 6162 with boosted ritonavir with travel, 5253
answers, 111 rhinoconjunctivitis, allergic, 107 raltegravir, 48 treadmill ECG, 11
questions, 109110 rhinorrhea, 105 steatorrhea, 32 trimethoprim-sulfamethoxazole, 55, 65
psychosis, substance-induced, 111 rhomboid birefringent crystals, 66 stenotic lesion, right carotid, 85 trochanteric bursa tenderness, 59
pulmonary artery catheterization, 37 rib fractures, 35, 95 sterile pyuria, 103 Tropheryma whipplei, 32
pulmonary artery hypertension, 44 rifampin, 47, 53, 56 steroid myopathy, 66 tubal ligation, 70
pulmonary diseases rifaximin, 33 Streptococcus pneumoniae, 37, 106 tuberculin skin test (TST), 47
answers right (dormant) hand clumsiness, 35 stroke, 11, 14, 41, 79 tuberculosis, 47
critical care medicine, 41 right ventricular heave, 38 ST-segment depression, 14, 17, 21, 22 tubular adenomas, 27
diffuse lung disease, 4143 rituximab, 91 subepithelial collagen band (colon), 27 tubulovillous adenoma, 27
occupational lung disease, 4143 rosacea, 117, 118f, 120 substance abuse, 95, 96, 98 tularemia, 53
signs, symptoms, chest radiography, rosiglitazone, 74, 75 substance-induced psychosis, 111 Tumor lysis syndrome, 103
4344 Roux-en-Y gastric bypass, 70, 75 suicidal ideation, 109 tumor necrosis factor (TNF) therapy, 61, 65
vascular disease, embolism, sulfonylurea medications, 74 2-block claudication, 16
hypertension, 43 S sumatriptan, 114 2-pillow orthopnea, 11
questions saddle pulmonary embolism, 35 sunitinib, 99 type B aortic dissection, 1516
critical care medicine, 3536 salmeterol, 44, 80 superior mesenteric artery embolus, 33 type 2 diabetes mellitus, 9, 45, 50, 80, 98,
diffuse lung disease, 3638 sarcoidosis, 41 superior vena cava syndrome, 12, 20 99, 109, 125
occupational lung disease, 3638 Sarcoptes scabiei var hominis, 117f, 120 swallowing, fluctuating difficulties, 113
signs, symptoms, chest radiography, schizophrenia, 109 symptomatic carotid disease, 16 U
3940 seborrheic dermatitis, 49 syncope, 910, 13, 35 ulcerative colitis, 27
vascular disease, embolism, secondary amenorrhea, 135 post-exertion, 12 upper respiratory tract infection, 105
hypertension, 3839 second heart sound splitting, 10 syndrome of antidiuretic hormone urinary tract infection, 55
pulmonary edema, 13 sedentary lifestyle, 9 (SIADH), 76, 102 urinary urgency incontinence, 133
pulmonary embolism, 3536, 86 synovitis, 60, 85 urine, tea-colored, 97

INDEX 145
US Preventive Services Task Force, screening vasculitis, 83, 103 supplementation, 70, 72, 76 Western blot test, 48
recommendations, 134 answers, 64 vitamin E supplement, 71, 101 Whipple disease, 28, 32
uveitis, 31, 61, 65 questions, 5960 voriconazole, 55 Wolf-Parkinson-White syndrome, 18
venous Doppler ultrasonography, 99 womens health
V ventilator-associated pneumonia, 45 W answers, 134137
vaccinations, 124, 126, 134, 135 ventilator dyssynchrony, 35 walking difficulties, 113 questions, 125127
vaginal irritation, 126, 135136 ventricular thickening, 87 warfarin, 9, 11, 23, 86, 90, 123
valproate sodium, 111 verapamil, 53 efficacy trial, 127128 X
Valsalva maneuver, 11, 12, 19 visceral arteriovenous treatment number, 128 xanthelasmas, 29, 33
vancomycin, 49 malformation, 44 warm autoimmune hemolytic anemia
vascular disease, embolism, hypertension viscosupplementation, 65 (WAIHA), 90 Z
answers, 43 vitamin B12 water deficit estimation, 93, 102 zidovudine, 54
questions, 3839 deficiency, 83, 89 weakness, generalized, 94 ziprasidone, 111
vascular medicine supplementation, 70, 75 Wegener granulomatosis, 59, 60, 64 zoonoses
answers, 2224 vitamin D weight loss, 123 answers, 5253
questions, 1517 deficiency, 76 Wernicke encephalopathy, 75 questions, 4546

146 INDEX

Potrebbero piacerti anche